Location via proxy:   [ UP ]  
[Report a bug]   [Manage cookies]                
ALGEBRA BASICA Texto-Gua Convocatoria IDU-99 Universidad de Murcia Angel del Ro Mateos Juan Jacobo Simon Pinero Alberto del Valle Robles Septiembre de 2001 2   A Ignacia, Angel, Presentacion, Rafael, Diotila, Julia, Maruchi y Alvaro 2 Contenidos Introduccion 1 El anillo de los numeros enteros 1.1 1.2 1.3 1.4 1.5 1.6 1.7 1.8 1.9 1.10 Axiomas y propiedades basicas . . . . . . . . . . . . . . . Divisibilidad e ideales . . . . . . . . . . . . . . . . . . . . Maximo comun divisor y mnimo comun multiplo . . . . . Algoritmo de Euclides y ecuaciones diofanticas . . . . . . El Teorema Fundamental de la Aritmetica . . . . . . . . . Congruencias . . . . . . . . . . . . . . . . . . . . . . . . . Ecuaciones de congruencias Teorema Chino de los Restos Teoremas de Euler y Fermat . . . . . . . . . . . . . . . . . Apendice: criptografa de clave publica . . . . . . . . . . . Problemas . . . . . . . . . . . . . . . . . . . . . . . . . . . 2 Anillos 2.1 2.2 2.3 2.4 2.5 2.6 2.7 2.8 2.9 2.10 Operaciones . . . . . . . . . . . . . . . . . . . . . Grupos abelianos y anillos . . . . . . . . . . . . . Subanillos . . . . . . . . . . . . . . . . . . . . . . Ideales y anillos cociente . . . . . . . . . . . . . . Operaciones con subanillos e ideales . . . . . . . Homomorsmos . . . . . . . . . . . . . . . . . . . Isomorsmos y Teoremas de Isomorfa . . . . . . Cuerpos y dominios ideales maximales y primos El cuerpo de fracciones de un dominio . . . . . . Problemas . . . . . . . . . . . . . . . . . . . . . . . . . . . . . . . . . . . . . . . . . . . . . . . . . . . . . . . . . . . . . . . . . . . . . . . . . . . . . . . . . . . . . . . . . . . . . . . . . . . . . . . . . . . . . . . . . . . . . . . . . . . . . . . . . . . . . . . . . . . . . . . . . . . . . . . . . . . . . . . . . . . . . . . . . . . . . . . . . . . . . . . . . . . . . . . . . . . . . . . . . . . . . . . . . . . . . . . . . . . . . . . . . . . . . . . . . . . . . . . . . . . . . . . . . . . . . . . . . . . . . . . . . . . . . . . . . . . . . . . . . . . . . . . . . . . . . . . . . . . . . . . . . . . . . . . . . . . . . . . . . . . . . . . . . . . . . . . . . . . . . . . . . . . . . . . . . . . . . . . . . . . . . . . . . . . . . . . . . . . . . . . . . . . . . . . . . . . . . . . . . . . . . . . . . . . . . . . . . . . . . . . . . . . . . . . . . . . . . . . . . . . . . . . . . . . . . . . . . . . . . . . . . . . . . . . . . . . . . . . . . . . . . . . . . . . . . . . . . . . . . . . . . . . . . . . . . . Deniciones y propiedades basicas . . . . . . . . . . . . Propiedad Universal . . . . . . . . . . . . . . . . . . . . Races de polinomios . . . . . . . . . . . . . . . . . . . . Existencia de races Teorema Fundamental del A lgebra Factorizacion unica en anillos de polinomios . . . . . . . Factorizacion e irreducibilidad de polinomios . . . . . . Polinomios en varias indeterminadas . . . . . . . . . . . Problemas . . . . . . . . . . . . . . . . . . . . . . . . . . . . . . . . . . . . . . . . . . . . . . . . . . . . . . . . . . . . . . . . . . . . . . . . . . . . . . . . . . . . . . . . . . . . . . . . . . . . . . . . . . . . . . . . . . . . . . . . . . . . . . . . . . . . . . . . . . . . . . . . . . . . . . . . . . . . . . . . . . . . . . . . . . Divisibilidad . . . . . . . . . . . . . . . Dominios de ideales principales . . . . Dominios eucldeos . . . . . . . . . . . Dominios de factorizacion unica . . . . Aplicaciones de la factorizacion unica . Problemas . . . . . . . . . . . . . . . . . . . . . . . . . . . . 4 Anillos de polinomios 4.1 4.2 4.3 4.4 4.5 4.6 4.7 4.8 . . . . . . . . . . . . . . . . . . . . 3 Divisibilidad y factorizacion en dominios 3.1 3.2 3.3 3.4 3.5 3.6 . . . . . . . . . . 3 . . . . . . . . . . . . . . . . . . . . . . . . 5 9 10 13 16 18 20 22 25 27 29 30 35 36 37 40 42 45 47 49 52 55 58 65 66 70 71 75 78 83 87 88 90 93 95 98 99 104 107 CONTENIDOS 4 5 Grupos 5.1 5.2 5.3 5.4 5.5 5.6 5.7 5.8 5.9 5.10 Grupos . . . . . . . . . . . . . . . . . . . Ejemplos . . . . . . . . . . . . . . . . . . Subgrupos . . . . . . . . . . . . . . . . . . Operaciones con subgrupos . . . . . . . . Clases laterales y Teorema de Lagrange . Subgrupos normales y grupos cociente . . Homomorsmos y Teoremas de Isomorfa O rdenes de elementos y grupos cclicos . . Conjugacion y Ecuacion de Clases . . . . Problemas . . . . . . . . . . . . . . . . . . . . . . . . . . . . . . . . . . . . . . . . . . . . . . . . . . . . . . . . . . . . . . . . . . . . . . . . . . . . . . . . . . . . . . . . . . . . . . . . . . . . . . . . . . . . . . . . . . . . . . . . . . . . . . . . . . . . . . . . . . . . . . . . . . . . . . . . . . . . . . . . . . . . . . . . . . . . . . . . . . . . . . . . . . . . . . . . . . . . . . . . . . . . . . . . . . . . . . . . . . . . . . . . . . . . . . . . . . . . . . . . . . . . . . . . . . . . . . . . . . . . . . . . . . . . . . . . . . . . . . 6 Grupos de permutaciones 113 114 116 123 125 128 130 134 136 138 142 149 6.1 Ciclos y trasposiciones . . . . . . . . . . . . . . . . . . . . . . . . . . . . . . . . . . . . . 150 6.2 Grupos alternados . . . . . . . . . . . . . . . . . . . . . . . . . . . . . . . . . . . . . . . 153 6.3 Problemas . . . . . . . . . . . . . . . . . . . . . . . . . . . . . . . . . . . . . . . . . . . . 157 7 Grupos abelianos nitamente generados 7.1 7.2 7.3 7.4 7.5 7.6 7.7 Sumas directas . . . . . . . . . . . . . . . . Grupos abelianos libres . . . . . . . . . . . Grupos de torsion y libres de torsion . . . . Grupos indescomponibles y p-grupos . . . . Descomposiciones primarias e invariantes . Presentaciones por generadores y relaciones Problemas . . . . . . . . . . . . . . . . . . . . . . . . . . . . . . . . . . . . . . . . . . . . . . . . . . . . . . . . . . . . . . . . . . . . . . . . . . . . . . . . . . . . . . . . . . . . . . . . . . . . . . . . . . . . . . . . . . . . . . . . . . . . . . . . . . . . . . . . . . . . . . . . . . . . . . . . . . . . . . . . . . . . . . . . . . . . . . . . . . . . . . . . . . . . . . . . . . Accion de un grupo sobre un conjunto . . . . O rbitas y estabilizadores . . . . . . . . . . . . Teorema de Cauchy y p-grupos . . . . . . . . Los Teoremas de Sylow . . . . . . . . . . . . Productos directo y semidirecto de subgrupos Grupos de orden bajo . . . . . . . . . . . . . Problemas . . . . . . . . . . . . . . . . . . . . . . . . . . . . . . . . . . . . . . . . . . . . . . . . . . . . . . . . . . . . . . . . . . . . . . . . . . . . . . . . . . . . . . . . . . . . . . . . . . . . . . . . . . . . . . . . . . . . . . . . . . . . . . . . . . . . . . . . . . . . . . . . . . . . . . . . . . . . . . . . . . . . . . . . . . . . . . . . . . . . . . . . . . . . El subgrupo derivado . . . . . . . . . . . . . . . . La serie derivada grupos resolubles . . . . . . . . Series de composicion grupos de longitud nita . Problemas . . . . . . . . . . . . . . . . . . . . . . . . . . . . . . . . . . . . . . . . . . . . . . . . . . . . . . . . . . . . . . . . . . . . . . . . . . . . . . . . . . . . . . . . . . . . . . . . . . . . . . . . . . . . . . . . . . . . . . . . . . . . . . . . . . . . . . . . . . . . . . . . . . . . . . . . . . . . . . . . . . . . . . . . . . . . . . . . . . . . . . . . . . . . . . . . . . . . . . . . . . . . . . . . . . . . . . . . . . . . . . . . . . . . . . . . . . . . . . . . . . . . . . . . . . . . . . . . . . . . . . . . . . . . . . . . . . . . . . . . . . . . . . . . . . . . . . . . . . . . . . . . . . . . 8 Estructura de los grupos nitos 8.1 8.2 8.3 8.4 8.5 8.6 8.7 9 Series normales 9.1 9.2 9.3 9.4 10 Formas canonicas de matrices 10.1 10.2 10.3 10.4 10.5 10.6 10.7 10.8 10.9 Representaciones matriciales de endomorsmos Subespacios invariantes . . . . . . . . . . . . . Endomorsmos indescomponibles . . . . . . . . Descomposicion primaria . . . . . . . . . . . . . Forma Canonica de Jordan . . . . . . . . . . . Calculo efectivo . . . . . . . . . . . . . . . . . . Matrices reales de Jordan . . . . . . . . . . . . Aplicaciones . . . . . . . . . . . . . . . . . . . . Problemas . . . . . . . . . . . . . . . . . . . . . Bibliografa Indice terminologico Smbolos usados frecuentemente . . . . . . . . . 161 162 164 168 171 174 179 184 187 188 189 192 193 196 199 201 205 206 207 210 216 221 222 225 227 233 237 240 243 246 250 253 254 261 Introduccion Presentacion Este texto-gua corresponde con el curso de A lgebra Basica, una asignatura obligatoria dentro del plan de estudios vigente de la Licenciatura en Matematicas en la Facultad de Matematicas de la Universidad de Murcia. Se trata de una asignatura anual de primer curso con una carga docente de 12 creditos, 7'5 teoricos y 4'5 practicos. En este contexto, el A lgebra Basica tiene una caracterstica que la hace muy particular. Con toda seguridad se trata de la asignatura del primer curso de la Licenciatura de Matematicas que contiene conceptos mas novedosos para los alumnos. Lo normal es que el estudiante de este curso ni siquiera haya odo hablar de un alto porcentaje de los conceptos que aqu se introducen. Ademas, la intuicion geometrica no resulta tan util como en otras asignaturas del primer curso. En muchas universidades espa~nolas, una buena parte de los contenidos de esta asignatura son impartidos en el segundo curso de la licenciatura. Por esta causa, los libros de texto en castellano que abarcan los temas de la asignatura suelen presuponer una destreza en el manejo de los conceptos abstractos que no podemos esperar en los alumnos recien llegados a la universidad. Esto es lo que nos ha hecho creer en la utilidad de este texto-gua, que hemos escrito pensando en alumnos de un primer curso de la Licenciatura en Matematicas. Objetivos El objetivo generico de la asignatura es la adquisicion de capacidad de comprension y manejo de conceptos abstractos, as como el desarrollo de la capacidad de analisis y el rigor en la comprension de demostraciones y la resolucion de problemas. Mas concretamente, en esta asignatura se pretende que el alumno adquiera destreza en la manipulacion de los objetos algebraicos mas basicos: anillos, grupos, polinomios, etc. Estos objetos seran una herramienta fundamental en muchas de las asignaturas de la licenciatura, especialmente de las del area de algebra. Los metodos que se aprenderan en este curso dejaran al alumno a las puertas de la Teora de Galois y la Teora de Numeros, entre otras. Temario La organizacion en bloques tematicos es la siguiente: Primera parte: Anillos (48 horas) { Enteros (12 horas)  Captulo 1: El anillo de los numeros enteros. { Anillos (24 horas)  Captulo 2: Anillos.  Captulo 3: Divisibilidad y factorizacion en dominios. { Polinomios (12 horas)  Captulo 4: Anillos de polinomios. 5 CONTENIDOS 6 Segunda parte: Grupos (58 horas) { Grupos (22 horas)  Captulo 5: Grupos.  Captulo 6: Grupos de permutaciones. { Estructura de los grupos (36 horas)  Captulo 7: Grupos abelianos nitamente generados.  Captulo 8: Estructura de los grupos nitos.  Captulo 9: Series normales. Tercera parte: Formas canonicas (14 horas) { Formas canonicas (14 horas)  Captulo 10: Formas canonicas de endomorsmos. Estructura y uso del texto gua El texto-gua esta dividido en tres partes. Las dos primeras, mas extensas, se dedican al estudio de los anillos y los grupos, y en una tercera se estudian las formas canonicas de matrices. Suponemos que el alumno esta familiarizado con el lenguaje y las propiedades elementales de Logica y Teora de Conjuntos intuitiva, aunque no necesariamente de Logica Formal y Teora de Conjuntos avanzada. Mas concretamente, suponemos que el alumno conoce y maneja las operaciones conjuntistas elementales el concepto de aplicacion entre conjuntos los conceptos de relacion binaria, de equivalencia y de orden los numeros naturales, enteros, racionales, reales y complejos y el Principio de Induccion. Excepto por los prerrequisitos que acabamos de comentar, el texto es autocontenido es decir, los conceptos, metodos y resultados necesarios son presentados siguiendo una secuencia logica completa. Por tanto, una lectura del principio al nal proporciona metodos y resultados que pueden ser utilizados posteriormente en el texto. Por supuesto, el orden elegido no es el unico posible. De hecho, es habitual en muchos textos de algebra empezar por grupos y acabar con anillos. Una tal lectura del texto es posible aunque llevara consigo problemas en la comprension de algunas demostraciones aunque no necesariamente de los conceptos. El ultimo captulo (sobre formas canonicas de endomorsmos) podra ser ledo despues del captulo cuarto (sobre polinomios), y posiblemente esto sera mas logico desde un punto de vista conceptual. Sin embargo, hemos preferido dejarlo para el nal ya que los metodos utilizados en el captulo siete (sobre grupos abelianos) son muy similares a los que se usan en el ultimo y mas faciles de asimilar en el contexto de grupos abelianos. Hay dos excepciones a la armacion de que el texto es autocontenido. La primera se reere a la demostracion del Teorema Fundamental del A lgebra, en la que adelantamos resultados que el alumno vera, en este mismo curso, en las asignaturas de Analisis Matematico y Topologa. La segunda se reere a las propiedades elementales de las expresiones matriciales de aplicaciones lineales, que el alumno ya conocera por la asignatura de A lgebra Lineal cuando estudie el ultimo captulo. Es bien sabido que las matematicas solo se asimilancorrectamente comprendiendo las demostraciones de los resultados y resolviendo ejercicios. Por tanto, recomendamos al lector que lea el texto provisto de papel y lapiz para ir realizando los calculos y argumentos necesarios que le garanticen que esta comprendiendo todo lo que se arma. Para incentivar esta actitud crtica hemos incluido en lugares claves del texto preguntas o ejercicios. La mayora de los ejercicios incluidos en el texto son utilizados posteriormente en demostraciones es decir, estos ejercicios deberan ser considerados como lemas o proposiciones que se dejan al lector porque se consideran accesibles y ponen de maniesto el grado de comprension que en ese momento se tiene. Por tanto, recomendamos encarecidamente su resolucion en el momento en que sean propuestos. Cada captulo termina con una amplia lista de problemas y ejercicios cuya resolucion es la mejor garanta de exito. Hemos procurado incluir problemas de \todos los niveles", con el n de que el estudiante se ejercite gradualmente sin frustrarse y que nalmente se alcance el nivel de dicultad habitualmente requerido en los examenes. Algunos problemas especialmente difciles, o que desarrollan conceptos que no estan entre los contenidos fundamentales de la asignatura, han sido marcados con el smbolo *]. No hay separacion entre problemas teoricos y practicos, simplemente porque es difcil CONTENIDOS 7  marcar la frontera entre unos y otros, al menos en el Algebra Basica y, ademas, a nuestro entender, esta separacion resultara inutil. Hemos procurado presentarlos en orden de dicultad ascendente, pero por supuesto, la dicultad es un concepto subjetivo, as que hay que tomarlo con reservas. En la seccion dedicada a la evaluacion haremos mas comentarios. Evaluacion En general, en un examen de matematicas los estudiantes se encuentran con dos grandes tipos de preguntas: aquellas donde se pide que el estudiante reproduzca o aplique directamente algun resultado visto en clase y aquellas donde se reeja el conocimiento operativo de los resultados vistos en clase y sus interconexiones. Las primeras se suelen llamar preguntas teoricas y las segundas practicas, aunque muchas de estas tienen un componente teorico importante. Las preguntas teoricas esconden poco misterio. En ellas, se pretende comprobar que se hayan asimilado ciertas tecnicas y resultados que se consideran especialmente importantes. Una condicion necesaria (pero de ninguna manera suciente) para estar verdaderamente preparado para un examen es ser capaz de reproducir la demostracion de cualquier resultado visto en clase. Las preguntas practicas son ejercicios y problemas cuya resolucion requiere la manipulacion de los resultados y tecnicas vistos en clase. Hay diversos subtipos en funcion de la cantidad de resultados y tecnicas que se ven involucradas. El abanico se abrira con aquellas preguntas que se pueden resolver aplicando directamente un resultado o una tecnica vista en clase, y se cerrara con aquellas donde la respuesta requiere mezclar diversas tecnicas vistas en clase teniendo incluso que modicar los argumentos originales o, mas aun, fabricar algunos argumentos elementales. Vamos a ver algunos ejemplos. Como muestra de preguntas teoricas tenemos: Septiembre 1992: >Cuales de las siguientes armaciones son verdaderas? Razonar la respuesta: 1. 2. 3. 4. 5. Todo ideal maximal es primo. Todo ideal primo es maximal. Todo ideal primo de un DFU es maximal. Todo DIP es DFU. Todo DFU es DIP. Febrero 1999: Demuestra el Teorema de Cauchy. Diciembre 1998: Dar un contraejemplo al recproco del Teorema de Lagrange. Como se ve, no hay misterio. Casi se puede decir en que pagina de los apuntes esta la respuesta. Como ejemplos de preguntas practicas tenemos las siguientes, en orden de dicultad: Julio 1999 Sea f el endomorsmo de R 3 cuya matriz en cierta base es 0 2 ;2 11 A = @ 0 1 0A ;1 2 0 (a) Determinar su polinomio mnimo, sus factores invariantes, su forma canonica racional y su forma canonica de Jordan. (b) Hallar una matriz invertible P tal que P ;1AP sea la forma canonica de Jordan. Julio 1999 Sea G un grupo y sean H K subgrupos de G. (a) Demostrar que, para cualesquiera dos clases laterales Hx y Ky de G, la interseccion Hx \ Ky verica una de dos: o es vaca, o es una clase lateral de G modulo H \ K. (b) Deducir que si H y K tienen ambos ndice nito en G entonces H \ K tambien tiene ndice nito en G. CONTENIDOS 8 Julio 1997 Sea G un grupo nito de orden n que actua sobre s mismo mediante traslaciones por la izquierda. Sabemos que dicha accion da lugar a un homomorsmo L : G ! Sn . Se pide: 1. Probar que si g 2 G es un elemento de orden m, entonces la paridad de la permutacion L (g) n. coincide con la paridad del entero (m ; 1) m 2. Probar que si existe un g 2 G tal que la permutacion L (g) es impar, entonces G posee un ZZ subgrupo normal de ndice dos. 3. Probar que si P es un 2-subgrupo de Sylow del grupo alternado A5 , entonces P es isomorfo al 4-grupo de Klein (es decir, isomorfo a 2  2). Diciembre 1992 Sea K un cuerpo y sean P Q 2 KX Y ] polinomios no nulos. Probar que si P y Q son coprimos entonces el siguiente conjunto es nito f(a b) : P(a b) = Q(a b) = 0g : Los argumentos para responder a este tipo de preguntas pueden variar de una persona a otra. No pretendemos decir como \hay" que responder, sino solo ilustrar la clasicacion anterior. Para responder a la primera pregunta hay que hacer los calculos. Es directa. La segunda pregunta es casi monotematica. Hay que manipular y reproducir las tecnicas empleadas en los resultados sobre el Teorema de Lagrange y propiedades de las clases laterales. Para obligar al estudiante a repetir tecnicas y manipulaciones se elimina la hipotesis de que el grupo sea nito. La tercera es del tipo que involucra diversos temas, de los que hay que usar propiedades y tambien reproducir tecnicas. Los temas involucrados en este caso son: Teorema de Cayley, clases laterales y Teorema de Lagrange, permutaciones, grupos alternados y clasicacion de grupos nitos. Para resolver la ultima pregunta se utiliza el cuerpo de cocientes, K(X), del anillo de polinomios KX] para construir un dominio de ideales principales K(X)Y ] donde aplicar el Lema de Bezout. Luego, es necesario fabricar un argumento dentro de este dominio y aplicar que un polinomio en una indeterminada sobre un cuerpo tiene un numero nito de races. A menudo, en este tipo de problemas se da una indicacion. En este caso se dio la siguiente: Utilizar el Lema de Bezout en K(X)Y ]. Como se ve, presentarse a un examen con solamente un \panorama" de los resultados vistos en clase es como jugar a la lotera (o peor, porque el precio de la matrcula es superior al de un billete de lotera de navidad). Para presentarse a un examen con opciones a un buen resultado, ademas de saber resolver los problemas mas tpicos y de conocer y saber manipular las tecnicas de demostracion involucradas en los resultados vistos en clase, es necesario tener una vision general que permita hacer interconexiones entre los temas. De hecho, este ultimo es uno de los grandes objetivos de esta asignatura. Agradecimientos Durante la elaboracion de este texto gua nos hemos beneciado de muchas conversaciones con nuestros compa~neros del area de A lgebra de la Universidad de Murcia (e incluso de sus notas de clase), cuya experiencia previa como profesores de la asignatura A lgebra Basica y de su predecesora, el A lgebra I del antiguo plan de estudios, esta sin duda reejada en el texto. Captulo 1 El anillo de los numeros enteros Partiendo de nociones basicas sobre numeros naturales y numeros enteros, se demuestran los principales teoremas de la aritmetica. Introduccion Todos tenemos una nocion intuitiva de los numeros enteros (: : :  ;3 ;2 ;1 0 12 3 :: :), y manejamos con soltura algunas de sus propiedades basicas, que a menudo asumimos como verdades indiscutibles. Sin embargo, estas propiedades no son independientes es decir, algunas de ellas se pueden deducir a partir de otras. En este captulo vamos a introducir los numeros enteros de forma axiomatica: Deniremos el conjunto de los numeros enteros enunciando una lista de propiedades elementales, llamadas axiomas, que entenderemos como verdades asumidas (y que por tanto no requieren demostracion). A partir de ellas, y usando las reglas de la logica, iremos deduciendo otras propiedades importantes. Nos centraremos en los aspectos multiplicativos, hasta llegar al Teorema Fundamental de la Aritmetica: todo numero entero se factoriza, de modo esencialmente unico, como producto de numeros primos. Entre las herramientas que usaremos, las mas importantes seran: la posibilidad de cancelar enteros no nulos en productos, la posibilidad de dividir con resto por enteros no nulos, y la propiedad que tiene todo numero primo de dividir a alguno de los factores cuando divide a un producto. En la segunda mitad del captulo estudiaremos la aritmetica de congruencias, que se revelara como un concepto de la mayor importancia en diversos momentos del curso. Estableceremos sus propiedades basicas y las usaremos para esbozar una aplicacion a la criptografa de clave publica. Ademas, el proceso de dotar a los \enteros modulo n" de una estructura algebraica a partir de la que ya conocemos en los numeros enteros ilustra la que sera otra de las nociones fundamentales del curso: la formacion de \estructuras cociente". Objetivos del captulo Deducir las propiedades basicas de los numeros enteros a partir de los axiomas. Conocer los conceptos de maximo comun divisor y mnimo comun multiplo de un conjunto de enteros, y relacionarlos con el concepto de ideal. Usar el Algoritmo de Euclides para calcular el maximo comun divisor de dos enteros y la correspondiente identidad de Bezout, y para resolver algunas ecuaciones diofanticas. Conocer las propiedades basicas de los numeros primos. Demostrar el Teorema Fundamental de la Aritmetica, y utilizarlo para obtener otras propiedades de los numeros enteros. Manejar las operaciones con congruencias en particular, el calculo de inversos y de potencias y la resolucion de ciertas ecuaciones. Usar el Teorema Chino de los Restos para resolver sistemas de ecuaciones de congruencias. 9  CAPITULO 1. EL ANILLO DE LOS NUMEROS ENTEROS 10 Desarrollo de los contenidos 1.1 Axiomas y propiedades basicas Z Z Se llaman numeros enteros los elementos de un conjunto que denotamos por de forma que se verican los siguientes axiomas1: Axiomas de operaciones. Existen dos operaciones en , o sea, dos aplicaciones Z ZZ ZZ Z ZZ Z Z Z + ;! (a b) 7! a + b   ;! (a b) 7! a b = ab  y llamadas, respectivamente, suma y producto, que verican: Asociativa: Para todo a b c 2 , a + (b + c) = (a + b) + c y a(bc) = (ab)c. Conmutativa: Para todo a b 2 , a + b = b + a y ab = ba. Distributiva: Para todo a b c 2 , a(b + c) = ab + ac. Neutros: Existen dos numeros enteros 0 6= 1 tales que a + 0 = a y a1 = a, para todo a 2 . Opuestos: Para cada a 2 existe a0 2 tal que a + a0 = 0. Regularidad: Si a y b son dos numeros enteros tales que ab = 0, entonces a = 0 o b = 0. Z Z Axiomas de orden. Existe una relacion binaria en (si a esta relacionado con b por la relacion ZZ ZZ \ " escribimos a b) para la que se verican: Reexiva: Para todo a 2 , a a. Antisimetrica: Si a b 2 verican a b y b Transitiva: Si a b c 2 satisfacen a b y b Dicotoma: Dados a b 2 se tiene o bien a Buena Ordenacion: Todo subconjunto S de mnimo (o primer elemento). Es decir, si S 6= a, entonces a = b. c, entonces a c. b o bien b a. no vaco y acotado inferiormente tiene un y existe c 2 tal que c s para cada s 2 S, entonces existe m 2 S tal que m s para todo s 2 S. Compatibilidad del orden con las operaciones : Si a y b son dos numeros enteros tales que a b entonces a + c b + c, para todo c 2 . Ademas, si 0 c, entonces ac bc. Z Z Z A partir de estos axiomas podemos empezar a demostrar propiedades del conjunto de los numeros enteros. Empezamos con las que afectan a las operaciones suma y producto: Z Proposicion 1.1.1 En se verican las siguientes propiedades: ZZ ZZ Z Z Z 1. (Unicidad de los neutros) Solo hay un entero 0 tal que 0 + a = a para todo a 2 . Y solo hay un entero 1 tal que 1a = a para todo a 2 . Z 2. (Unicidad de los opuestos) Para cada a 2 existe un unico a0 2 tal que a + a0 = 0. Este unico elemento se llama el opuesto de a y se denota por ;a la suma b + (;a) se denota por b ; a. 3. (Cancelacion en sumas) Dados a b c 2 , la igualdad a + b = a + c implica b = c. 4. (Multiplicacion por cero) Para cada a 2 se verica a0 = 0. 5. (Reglas de signos) Dados a b 2 se verican -(-a) = a, a(-b) = (-a)b = -(ab) y (-a)(-b) = ab. 6. (Cancelacion en productos) Dados a b c 2 con a 6= 0, la igualdad ab = ac implica b = c. 1 El origen de la notacion Zes la palabra alemana Zahl que signica numero.  1.1. AXIOMAS Y PROPIEDADES BASICAS 11 Demostracion. 1. Supongamos que 0 y 00 satisfacen la propiedad. Entonces 00 = 0 + 00 = 00 + 0 = 0. La unicidad del neutro para el producto se demuestra de forma similar. 2. Sabemos que existe un entero a0 con esa propiedad, y si otro entero a00 la verica entonces a00 = 0 + a00 = (a0 + a) + a00 = a0 + (a + a00 ) = a0 + 0 = a0. 3. No hay mas que sumar ;a en ambos miembros. 4. Basta cancelar en la igualdad 0 + a0 = a0 = a(0 + 0) = a0 + a0. 5. La primera igualdad es consecuencia de 2. Para la segunda, como ab + a(;b) = a((b + (;b)) = a0 = 0, deducimos que a(;b) es el opuesto de ab. La igualdad (;a)b = ;(ab) se obtiene de modo similar, y usando esas dos igualdades y el hecho de que ;(;x) = x se deduce que (;a)(;b) = ab. 6. De ab = ac se obtiene a(b ; c) = ab ; ac = 0 (usando 5), y como a 6= 0, deducimos por el Axioma de Regularidad que b ; c = 0, de donde b = c (sumando c en ambos miembros). Las siguientes propiedades que consideraremos tienen que ver con el orden es conveniente introducir la terminologa y la notacion siguientes: Emplearemos del modo usual los smbolos , < y > y la terminologa \menor o igual", \mayor o igual", \estrictamente mayor", etc. Con esta notacion, el axioma de dicotoma se puede reenunciar como una tricotoma: Dados a b 2 , se verica exactamente una de las siguientes condiciones: Z a < b N N Z a=b o a > b: Z Z Z Los numeros enteros mayores o iguales que 0 se llaman numeros naturales, y el conjunto que forman se denota por . Los numeros enteros estrictamente mayores que 0 se llaman enteros positivos, y el conjunto que forman se denota por +. Los numeros enteros estrictamente menores que 0 se llaman enteros negativos, y el conjunto que forman se denota por ;. En smbolos: = fa 2 : a 0g + = fa 2 : a > 0g Z Z Z ; = fa 2 : a < 0g Las siguientes propiedades sobre el orden tienen demostraciones sencillas a partir de los axiomas (principalmente el de compatibilidad) y de las propiedades ya demostradas, y las dejamos como ejercicio para el lector. <Cuidado! no se puede utilizar nada que no sea una axioma o no haya sido previamente demostrado. Z Ejercicio 1.1.2 Demostrar que se verican las siguientes propiedades para a b c d 2 . 1. 2. 3. 4. 5. 6. 7. 8. 9. a b si y solo si a + c b + c. a < b si y solo si a + c < b + c. Si c > 0 entonces a b si y solo si ac bc. Si c > 0 entonces a < b si y solo si ac < bc. Si a b y c d entonces a + c b + d, y esta desigualdad es estricta si lo es alguna de las Z iniciales. Dados a b c d 2 + con a b y c d, se tiene ac alguna de las iniciales. a 2 + si y solo si ;a 2 ;. a < b si y solo si b ; a 2 +, si y solo si a ; b 2 ; a < b si y solo si ;a > ;b. Z ZZ bd, y esta desigualdad es estricta si lo es Z Z Proposicion 1.1.3 Se verican las siguientes propiedades para a b c 2 . 1. Si a b y c < 0 entonces ac bc. 2. Si a 6= 0 entonces a2 > 0. En particular, 1 > 0 y ;1 < 0.  CAPITULO 1. EL ANILLO DE LOS NUMEROS ENTEROS Z 12 Z Z 3. El 1 es el mnimo de + es decir, 1 n para cada n 2 +. 4. No hay enteros entre a y a + 1 es decir, no existe ningun n 2 tal que a < n < a + 1. 5. a < b si y solo si a + 1 b. Demostracion. 1. Si c < 0 entonces ;c > 0 (Ejercicio 1.1.2), y el Axioma de Compatibilidad nos dice que a(;c) b(;c), o sea ;(ac) ;(bc). Finalmente, ac bc, por el Ejercicio 1.1.2. 2. Por el Axioma de Dicotoma, se tiene o bien a > 0 o bien a < 0 el resultado se deduce entonces del Axioma de Compatibilidad (en el primer caso) o del apartado 1 (en el segundo). Como 1 = 12, deducimos que 1 > 0, y entonces ;1 < 0, por el Ejercicio 1.1.2. 3. Por su denicion, + esta acotado inferiormente por el 0, y no es vaco pues 1 2 + entonces + tiene un elemento mnimo m por el Axioma de Buena Ordenacion. Entonces m 1, de donde m2 m por compatibilidad pero m2 2 + por el apartado anterior, y en consecuencia m2 = m por la minimalidad de m cancelando m en esa igualdad obtenemos nalmente m = 1, como queramos ver. 4. Una desigualdad a < n < a + 1 implicara 0 < n ; a < 1, contra el apartado anterior. 5. Si a < b entonces b ; a 2 + y por lo tanto 1 b ; a, de donde a + 1 b. La implicacion recproca es facil usando que 0 < 1 y la transitividad. Z Z Z Z Z Ejercicio 1.1.4 Demostrar que: 1. Ninguna sucesion estrictamente decreciente de enteros esta acotada inferiormente. En particular, no existen sucesiones estrictamente decrecientes de enteros positivos. 2. Todo subconjunto S de no vaco y acotado superiormente tiene un maximo. Es decir, si S 6= y existe c 2 tal que c s para cada s 2 S , entonces existe m 2 S tal que m s para todo s 2 S . 3. Ninguna sucesion estrictamente creciente de enteros esta acotada superiormente. Z Z N Teorema 1.1.5 (Principio de Induccion) Sea S un subconjunto de n 2 S se verica que n + 1 2 S . Entonces S = . ZZ N tal que 0 2 S y para todo N N N Demostracion. Procederemos por reduccion al absurdo. Supongamos que S 6= . Entonces X = n S es un subconjunto de no vaco y acotado inferiormente por el 0, por lo que tiene un primer elemento a, y de hecho a 2 + puesto que 0 2 S. Entonces a 1, lo que implica que a ; 1 2 ademas, a ; 1 2 S, pues de lo contrario se tendra a ; 1 2 X, en contra de la eleccion de a. Pero ahora la hipotesis del teorema nos dice que a = (a ; 1) + 1 2 S, una contradiccion. Z Ejercicio 1.1.6 Demostrar las siguientes variantes del Principio de Induccion (en ambas, b es un entero jo y S es un subconjunto de ): 1. Si b 2 S , y si n 2 S implica n+1 2 S (o lo que es lo mismo, si n ; 1 2 S implica n 2 S ), entonces S contiene a todos los enteros mayores o iguales que b. 2. Si b 2 S , y si la condicion \a 2 S cuando b a < n" implica que n 2 S , entonces S contiene a todos los enteros mayores o iguales que b. Concluimos esta seccion con una ultima propiedad basica de los numeros enteros: la posibilidad de NN N Z dividir con resto. Se dene el valor absoluto de un entero a como  jaj = ;aa sisi aa 262 Del apartado 7 del Ejercicio 1.1.2 se deduce que jaj 2 , para todo a 2 . Ejercicio 1.1.7 Demostrar que para toda pareja de numeros enteros a y b se verica la igualdad: jabj = jaj jbj: 1.2. DIVISIBILIDAD E IDEALES 13 Teorema 1.1.8 (Division con Resto) Para toda pareja de numeros enteros a y b con b 6= 0 existen otros dos numeros enteros q y r tales que a = bq + r y 0 r < jbj: Estos enteros q y r son unicos y se les llama, respectivamente, el cociente y el resto de la division de a entre b. N Z Z N Demostracion. Como b 6= 0, el conjunto S = fa ; bn : n 2 g contiene una sucesion estrictamente creciente (>por que?). Por tanto no tiene una cota superior (>por que?) y en consecuencia S \ 6= (>por que?). Sea r el mnimo de S \ y sea q 2 tal que a ; bq = r. Observese que r ; jbj = a ; bq ; jbj = a ; b(q  1) 2 S: De la minimalidad de r se deduce que r ; jbj < 0 y, por tanto r < jbj. Esto prueba la existencia de q y r satisfaciendo la propiedad requerida. Veamos la unicidad. Supongamos que a = bq + r = bq1 + r1 con 0 r r1 < jbj. Entonces jbj > jr1 ; rj = jb(q1 ; q)j = jbj jq1 ; qj (>por que?). De donde se deduce que q1 = q (>por que?) y, por tanto, r = r1 . Ejercicio 1.1.9 Dar demostraciones rigurosas de todas las armaciones que se han acompa~nado de un \>por que?" en la demostracion anterior. 1.2 Divisibilidad e ideales De nicion 1.2.1 Sean a y b dos numeros enteros. Diremos que a divide a b, o que a es un divisor de b, o que b es multiplo de a, si existe un entero n tal que b = an. En tal caso escribiremos a j b. De la propiedad de cancelacion en productos se deduce que si a 6= 0 y a j b, entonces existe un unico elemento n 2 tal que b = an. Denotaremos este elemento n por ab o por b=a. El lector puede demostrar algunas propiedades basicas de la relacion de divisibilidad. Ejercicio 1.2.2 Dados a b c d 2 se verica: 1. (Reexiva) a j a. 2. (Transitiva) Si a j b y b j c, entonces a j c. 3. (Casi Antisimetrica) Si a j b y b j a, entonces b = a (o sea, jbj = jaj). 4. El 0 es multiplo de cualquier entero, y solo es divisor de s mismo. 5. El 1 es divisor de cualquier entero, y solo es multiplo de 1 y de ;1 (para demostrar esto ultimo, Z 6. 7. 8. 9. 10. Z Z usa las propiedades del orden en ). a j b si y solo si a j ;b, si y solo si ;a j b, si y solo si ;a j ;b. Si a j b y a j c, entonces a j rb + sc para cualesquiera r s 2 (en particular, a j b + c, a j b ; c y a j rb para cualquier r 2 ). Si a j b y c j d, entonces ac j bd. Si 0 6= r 2 entonces: a j b si y solo si ar j br. Si a j b, entonces jaj jbj. Z Z Z El apartado 7 del Ejercicio 1.2.2 sugiere las siguientes deniciones, que seran muy utiles para trabajar con cuestiones de divisibilidad.  CAPITULO 1. EL ANILLO DE LOS NUMEROS ENTEROS 14 Z De nicion 1.2.3 Una combinacion lineal con coecientes enteros (o una combinacion -lineal) de los enteros a1 : : : an es un entero de la forma Z Z r1 a1 + + rn an donde cada ri 2 . Los enteros ri son los coecientes de la combinacion lineal. Un subconjunto I de es un ideal si no es vaco y si, dados a b 2 I , cualquier combinacion lineal suya ra + sb sigue siendo un elemento de I . Por induccion, se ve facilmente que cualquier combinacion lineal de un numero nito de elementos de un ideal I sigue siendo un elemento de I. Por otra parte, en la denicion de ideal, la condicion I 6= se puede sustituir por la condicion 0 2 I, tomando combinaciones lineales con coecientes nulos. Proposicion 1.2.4 Para un subconjunto no vaco I de , las condiciones siguientes son equivalentes: 1. I es un ideal. 2. Si a b 2 I , entonces a+b 2 I y ra 2 I para cada r 2 (I es cerrado para sumas y para multiplos). 3. Si a b 2 I entonces a ; b 2 I (I es cerrado para restas). Demostracion. Es claro que las dos primeras condiciones son equivalentes e implican la tercera. Por ultimo, si asumimos 3, es elemental ver, por este orden, que 0 2 I, que si a 2 I entonces ;a 2 I, y que si a b 2 I entonces a + b 2 I. Ahora es facil ver, por induccion, que si a 2 I entonces ra 2 I (y por lo tanto ;ra 2 I) para cada r 2 , con lo que se tiene 2. Z Z N Veamos algunos ejemplos de ideales. Dejamos al lector que demuestre las armaciones que siguen. ZZ Z Ejemplos 1.2.5 Ideales. 1. y f0g son dos ideales de . 2. Si a 2 , el conjunto (a) formado por los multiplos de a es un ideal de . Este ideal se llama ideal principal generado por a. Z 3. Mas generalmente, si a1 : : :  an 2 , entonces el conjunto (a1  : : :  an) = fk1a1 + + knan : k1 : : :  kn 2 g es un ideal de , llamado ideal generado por a1 : : :  an. 4. La interseccion de una familia de ideales es un ideal. 5. Si I y J son dos ideales de , entonces I + J = fa + b : a 2 I b 2 J g es un ideal de . Mas generalmente, si I1  : : : Ir son ideales de , entonces I1 + + Ir = fa1 + + ar : cada ai 2 Ii g es un ideal de . Observese que, con esta notacion, se tiene (a1  : : : an) = (a1 ) + + (an): Z Z Z Z Z Z Z Z Z (1.2.1) Z Z Z Z Sea S un subconjunto de . Como es un ideal de , entonces existe al menos un ideal de que contiene a S. Acabamos de mencionar que la interseccion de una familia de ideales de es otro ideal de . En particular, la interseccion I de todos los ideales que contienen a S es un ideal que esta contenido en todos los ideales que contienen a S. Es decir, I es el menor ideal que contiene a S, donde el sentido de menor se reere a la relacion de inclusion. Por esta razon dicho ideal se llama ideal generado por S. Observese que en los ejemplos anteriores tambien hemos llamado ideal generado por a1 : : :  an al conjunto (1.2.1). Vamos a ver que la utilizacion del mismo nombre esta justicada: 1.2. DIVISIBILIDAD E IDEALES Z 15 Proposicion 1.2.6 Sea S un conjunto de numeros naturales entonces la interseccion de los ideales de que contienen a S coincide con el conjunto (S) = f n X i=1 Z Z Z kiai : ki 2 ai 2 S g de todas las combinaciones lineales de familias nitas de elementos de S . Z Demostracion. En los comentarios realizados antes de la proposicion se ha visto que la interseccion de los ideales de que contienen a S es el menor ideal de que contiene a S. Por tanto basta demostrar que el conjunto (S) del enunciado tambien es el menor ideal de que contiene a S es decir, que se verican las tres siguientes condiciones: S  (S). (S) es un ideal de . Si I es un ideal de que contiene a S, entonces (S)  I. El lector puede demostrar que efectivamente estas condiciones se verican. ZZ Ejercicio 1.2.7 Demostrar que si I y J son conjuntos de numeros enteros, entonces (I)+(J) = (I  J). En particular, si I y J son ideales, entonces I + J es el menor ideal que contiene a I y a J . La relacion de divisibilidad se puede expresar de forma sencilla en terminos de ideales: Proposicion 1.2.8 Las siguientes condiciones son equivalentes para dos numeros enteros a y b. 1. a j b. 2. b 2 (a). 3. (b)  (a). Demostracion. La equivalencia entre 1 y 2 viene de la denicion de ideal principal. Como (b) es el menor ideal de que contiene a b, se tiene (b)  (a) precisamente si b 2 (a). Z Usando este resultado y las propiedades del Ejercicio 1.2.2, los ejercicios siguientes son faciles: Z Ejercicio 1.2.9 Las siguientes condiciones son equivalentes para un numero entero a: 1. (a) = . 2. a j 1. 3. a = 1 o a = ;1 (o sea, jaj = 1). Ejercicio 1.2.10 Las siguientes condiciones son equivalentes para dos numeros enteros a y b: 1. (a) = (b). 2. a j b y b j a. 3. a = b o a = ;b (o sea, jaj = jbj). Z Nuestros primeros ejemplos de ideales de fueron los ideales principales terminaremos esta seccion viendo que no hay otros. Z Z Teorema 1.2.11 Todos los ideales de son principales. Es decir, si I es un ideal de , entonces existe un entero a tal que I = (a).  CAPITULO 1. EL ANILLO DE LOS NUMEROS ENTEROS 16 Z Z Demostracion. Sea I un ideal de . Si I = f0g = (0), entonces I es principal. Supongamos que I= 6 f0g. Entonces I contiene un elemento x diferente de 0, y ademas ;x = (;1)x 2 I, por lo que I contiene algun entero positivo. Sea a el menor entero positivo que pertenece a I (el mnimo de I \ +). Veamos que I = (a). Como (a) es el menor ideal que contiene a a, se tiene que (a)  I. Recprocamente, si x 2 I, existen q r 2 tales que x = aq + r y 0 r < jaj = a (Teorema 1.1.8). Como a 2 I y x 2 I, deducimos que r = x ; aq 2 I, y de la minimalidad de a se deduce que r no es positivo. Por tanto r = 0, y en consecuencia x = aq 2 (a), como queramos ver. Z Corolario 1.2.12 Si S es un conjunto no vaco de numeros enteros y d es un numero entero, las condiciones siguientes son equivalentes: 1. (d) = (S) es decir, el ideal generado por S coincide con el ideal principal generado por d. 2. d j s para cada s 2 S y d es una combinacion lineal de elementos de S . Demostracion. (d)  (S) si y solo si d 2 (S), si y solo si d es combinacion lineal de elementos de S. Por otra parte, (S)  (d) si y solo si s 2 (d) para cada s 2 S (pues (S) es el menor ideal que contiene a cada s 2 S), si y solo si d divide a cada s 2 S. Esto demuestra el resultado. 1.3 Maximo comun divisor y mnimo comun multiplo De nicion 1.3.1 Sea S un conjunto de numeros enteros y sean d y m numeros enteros. Se dice que d es un maximo comun divisor de S si se verican las dos condiciones siguientes: 1. d divide a todos los elementos de S . 2. Si x es un entero que divide a todos los elementos de S , entonces x divide a d. (Es facil ver que estas dos condiciones equivalen a una sola: Un entero x divide a todos los elementos de S si y solo si x divide a d). Diremos que m es mnimo comun multiplo de S si se verican las dos condiciones siguientes: 1. m es multiplo de todos los elementos de S . 2. Si x es un entero que es multiplo de todos los elementos de S , entonces x es multiplo de m. (Equivalentemente: Un entero x es multiplo de cada s 2 S si y solo si x es multiplo de m). Escribiremos d = mcd(S) para indicar que d es un maximo comun divisor de S, y m = mcm(S) si m es un mnimo comun multiplo de S. Si S = fa1  : : :  ang, entonces utilizaremos la notacion: mcd(a1  : : :  an) = mcd(S) y mcm(a1 : : :  an) = mcm(S): Hay que tener cuidado con esta notacion, ya que es ambigua. Por ejemplo, a = mcd(S) y b = mcd(S), no implica que a = b. En efecto, si a es maximo comun divisor de S, entonces ;a tambien es maximo comun divisor de S. Una propiedad similar se verica para el mnimo comun multiplo. Sin embargo, el siguiente ejercicio muestra los lmites de esta ambig#uedad. Ejercicio 1.3.2 Sean a y b dos numeros enteros y S un conjunto de numeros enteros. Demostrar que si a y b son ambos maximos comunes divisores de S , entonces a = b. Demostrar que la misma propiedad se verica para el mnimo comun multiplo. Z Observese que todava no hemos demostrado que cualquier subconjunto de tenga un maximo comun divisor ni un mnimo comun multiplo. Usando el concepto de ideal podemos demostrar no solo la existencia, sino tambien otras propiedades importantes del maximo comun divisor y el mnimo comun multiplo.   DIVISOR Y MINIMO COMUN  MULTIPLO  1.3. MAXIMO COMUN Z 17 Proposicion 1.3.3 Sea S un conjunto de numeros enteros y sean d m 2 . Entonces: 1. Las siguientes condiciones son equivalentes: (a) d = mcd(S). (b) (d) = (S). (c) d j s para cada s 2 S y existen s1  : : : sn 2 S y r1  : : : rn 2 tales que d = r1s1 + 2. Las siguientes condiciones son equivalentes: (a) m = mcm(S). (b) (m) = \s2S (s). 3. En particular, S tiene maximo comun divisor y mnimo comun multiplo. Z +rn sn . Demostracion. De la Proposicion 1.2.8, se deduce que d = mcd(S) precisamente si se verican las dos siguientes condiciones: S  (d). Si x es un entero tal que S  (x), entonces (d)  (x). En otras palabras, d = mcd(S) precisamente si (d) es el menor ideal que contiene a S es decir, (d) = (S). Esto demuestra la equivalencia de (a) y (b) en 1, y la equivalencia de (b) y (c) no es mas que el Corolario 1.2.12. Dejamos que el lector demuestre 2. Finalmente, 3 es consecuencia del hecho de que todo ideal de es principal (Teorema 1.2.11). Z El siguiente corolario es inmediato. Corolario 1.3.4 Dados enteros a1 : : : an y d, se verica: d = mcd(a1  : : :  an) si y solo si d divide a cada ai y existen r1  : : :  rn 2 tales que r1a1 + + rnan = d: Esta ultima expresion se conoce como una identidad de Bezout para a1  : : : an. Los conjuntos cuyo maximo comun divisor es 1 son especialmente importantes, y por eso destacamos aqu el siguiente caso particular del corolario anterior: Z Corolario 1.3.5 (Lema de Bezout) Las siguientes condiciones son equivalentes para los numeros enteros a1 : : :  an: 1. Los unicos divisores comunes a todos los ai son 1 y ;1. 2. mcd(a1  : : :  an) = 1. 3. (a1 : : :  an) = . 4. Existen r1 : : :  rn 2 tales que r1a1 + + rnan = 1. Diremos que los numeros enteros a1  : : :  an son coprimos o primos entre s cuando veriquen las ZZ condiciones equivalentes del Corolario 1.3.5. Usando adecuadamente las identidades de Bezout se pueden demostrar facilmente numerosas propiedades: Proposicion 1.3.6 Se verican las siguientes propiedades sobre numeros enteros: 1. 2. 3. 4. Si mcd(a1  : : : an) = d entonces mcd( ad1  : : : adn ) = 1. mcd(a b) = 1 y mcd(a c) = 1 si y solo si mcd(a bc) = 1. Si mcd(a b) = 1 y a j bc, entonces a j c. Si mcd(a b) = 1, a j c y b j c, entonces ab j c. 18  CAPITULO 1. EL ANILLO DE LOS NUMEROS ENTEROS Demostracion. 1. Si mcd(a1 : : : an) = d entonces d = r1a1 + +rnan para ciertos enteros r1 : : : rn. Z Dividiendo por d en ambos miembros y aplicando el Lema de Bezout (1.3.5) se obtiene el resultado deseado. 2. Si mcd(a bc) = 1, del Lema de Bezout se deduce que existen u v 2 tales que au + bcv = 1. De nuevo aplicando el Lema de Bezout se obtiene que mcd(a b) = 1 = mcd(a c). Supongamos ahora que mcd(a b) = 1 y mcd(a c) = 1. Por el Lema de Bezout (1.3.5), existen enteros r s t u tales que ra + sb = 1 y ta + uc = 1. Multiplicando ambas expresiones se obtiene (rat + ruc + sbt)a + (su)(bc) = 1 y por lo tanto mcd(a bc) = 1. 3. Por las hipotesis existen enteros r s t tales que 1 = ra + sb y bc = at, y multiplicando por c la primera expresion se obtiene c = rac + sbc = rac + sat = (rc + st)a, por lo que a j c. 4. Como b j c, existe un entero t tal que c = bt, y ahora el apartado 3, aplicado a las otras dos hipotesis, nos dice que a divide a t, por lo que ab divide a tb = c. Ejercicio 1.3.7 Dar ejemplos que muestren que los dos ultimos apartados del resultado anterior son falsos si a y b no son coprimos. 1.4 Algoritmo de Euclides y ecuaciones diofanticas En esta seccion resolvemos dos problemas practicos: comenzamos mostrando un metodo para calcular el maximo comun divisor de dos numeros enteros y la correspondiente identidad de Bezout, y aplicamos entonces esto a la resolucion de cierto tipo de ecuaciones. El primer metodo es conocido con el nombre de Algoritmo de Euclides para el Calculo del Maximo Comun Divisor y esta basado en el siguiente ejercicio. Ejercicio 1.4.1 Sean a y b dos numeros enteros. Si a = bq + r, entonces mcd(a b) = mcd(b r). Sean a y b dos numeros enteros cuyo maximo comun divisor queremos calcular. Si a = 0 entonces mcd(a b) = b. Por tanto podemos suponer que a y b son distintos de 0. Si a se cambia por su opuesto el maximo comun divisor de a y b no vara. Por tanto, podemos suponer que a y b son positivos. Supongamos que b a. Construimos una tabla como la siguiente: a = r;1 b = r0 r1 r2 : : : rn = d rn+1 = 0 q1 q2 q3 : : : qn+1 donde qi y ri son, respectivamente, el cociente y el resto de dividir ri;2 por ri;1, y denotamos por d al ultimo resto no nulo. Observese que algun ri tiene que ser 0 ya que la sucesion r;1  r0 r1 r2 : : : esta formada por enteros no negativos y es estrictamente decreciente (Ejercicio 1.1.4). Entonces mcd(a b) = mcd(r;1  r0) = mcd(r0  r1) = mcd(r1 r2) = = mcd(rn;1 rn) = mcd(rn 0) = rn = d: De hecho, el metodo anterior sirve para calcular dos enteros s y t tales que d = sa + tb, donde d = rn = mcd(a b). El metodo consiste en escribir d = rn en funcion de a y b utilizando las ecuaciones ri = ri;2 ; qiri;1 \de derecha a izquierda", y lo ilustramos con el siguiente ejemplo: Z Ejemplo 1.4.2 Algoritmo de Euclides en . Sean a = 18444 y b = 1632. Entonces construimos la tabla a = 18444 b = 1632 492 156 24 12 0 11 3 3 6 2 Luego mcd(a b) = 12 y 12 = 156 ; 6 24 = 156 ; 6 (492 ; 3 156) = ;6 492 + 19 156 = ;6 492 + 19 (b ; 3 492) = 19 b ; 63 492 = 19 b ; 63 (a ; 11 b) = ;63 a + 712 b:  1.4. ALGORITMO DE EUCLIDES Y ECUACIONES DIOFANTICAS 19 Una vez que sabemos como calcular el maximo comun divisor, podemos calcular el mnimo comun multiplo utilizando la siguiente proposicion. Proposicion 1.4.3 Si a y b son dos numeros enteros, entonces ab = mcd(a b)mcm(a b): O mas rigurosamente, si d es un maximo comun divisor de a y b, entonces ab=d es un mnimo comun multiplo de a y b. Demostracion. Sean d = mcd(a b), a1 = da , b1 = db y m = da1b1. Basta con demostrar xque m = mcm(a b). Claramente m es multiplo de a y de b. Sea x un multiplo de a y de b. Entonces d es multiplo de a1 y de b1. Por la Proposicion 1.3.6, xd es multiplo de a1 b1 y, por tanto, x es multiplo de da1b1 = m. A continuacion damos una aplicacion de los resultados vistos hasta ahora. Se trata de la resolucion de las ecuaciones diofanticas del tipo aX + bY = c (1.4.2) donde a b c son numeros enteros con a y b no nulos y X e Y son incognitas. El adjetivo \diofantica" (en honor del matematico griego Diofanto de Alejandra) signica que solo nos interesan las soluciones de dicha ecuacion que sean numeros enteros. Por lo tanto, una solucion de la ecuacion diofantica (1.4.2) es un par ordenado de numeros enteros (x y) tal que ax +by = c. Comenzaremos estudiando la existencia de soluciones, para pasar despues a su calculo. En lo que sigue hacemos d = mcd(a b), a0 = ad y b0 = db , y tomamos enteros r s tales que ra + sb = d. Proposicion 1.4.4 Con lac notacion anterior, la ecuacion (1.4.2) tiene soluciones enteras si y solo si d j c. En este caso, si c0 = d , las soluciones de (1.4.2) son las mismas que las de a0 X + b0 Y = c0 : (1.4.3) (Notese que la ecuacion inicial siempre tiene solucion en dos casos especiales: cuando a y b son coprimos y cuando c = 0.) Z Demostracion. Si (x y) es una solucion de (1.4.2) entonces d j ax + by = c. Recprocamente, si existe c0 2 tal que c = c0 d, entonces (rc0  sc0 ) es una solucion de (1.4.2). Esto prueba el \si y solo si", y el resto es claro pues d = 6 0 y por lo tanto podemos cancelarlo. Supongamos ahora que (1.4.2) tiene solucion, y veamos como calcular todas sus soluciones. De hecho calcularemos las de (1.4.3), que es mas sencilla. La demostracion anterior nos dice que una solucion \particular" es (x0 y0 ) = (rc0  sc0), pero puede haber otras. Si (x y) es otra solucion de (1.4.3) entonces, restando las igualdades a0 x + b0 y = c0 y a0 x0 + b0y0 = c0, se obtiene a0(x ; x0) + b0 (y ; y0 ) = 0: (1.4.4) Esto implica que b0 divide a a0 (x ; x0), y como mcd(a0  b0) = 1 (Proposicion 1.3.6) deducimos que b0 divide a x ; x0, y por lo tanto existe  2 tal que x ; x0 = b0 . Sustituyendo en (1.4.4), cancelando b0 y despejando deducimos que y ; y0 = ;a0 , de modo que (x y) = (x0 + b0 y0 ; a0): (1.4.5) Como es inmediato comprobar que todos los pares de esa forma (con  2 ) son soluciones de (1.4.3), deducimos que estas son todas las soluciones de nuestra ecuacion. En resumen, para resolver la ecuacion diofantica (1.4.2) procedemos de la siguiente forma (el lector puede estudiar como se simplican los pasos en los dos casos especiales mcd(a b) = 1 o c = 0): 1. Calculamos d = mcd(a b), por ejemplo usando el Algoritmo de Euclides. 2. Si d no divide a c, deducimos que la ecuacion no tiene solucion y acabamos. Z Z  CAPITULO 1. EL ANILLO DE LOS NUMEROS ENTEROS 20 3. Si d divide a c, calculamos una solucion particular como sigue: Ponemos c0 = dc y buscamos, usando el Algoritmo de Euclides, enteros r s tales que ar + bs = d entonces (x0 y0 ) = (rc0 sc0 ) es una solucion de (1.4.2). 4. Si a0 = da y b0 = db , las soluciones de (1.4.2) son entonces todos los pares (x y) = (x0 +b0  y0 ; a0 ), donde  es un entero arbitrario. Ejemplos 1.4.5 Ecuaciones Diofanticas. 1. Considerese la ecuacion diofantica 18X +15Y = 14. En este caso, como mcd(18 15) = 3 no divide a 14, concluimos que la ecuacion no tiene solucion. 2. Considerese la ecuacion diofantica 1717X + 2121Y = 0, que tiene al menos la solucion trivial (X0  Y0) = (0 0). Para calcular el resto de soluciones, dividimos la ecuacion inicial por mcd(1717 2121) = 101 y obtenemos la ecuacion equivalente 17X + 21Y = 0. Como sus coecientes son coprimos, la solucion general de la ecuacion es (X Y ) = (21 ;17) Z ( 2 ): 3. Considerese ahora la ecuacion diofantica 100X + 35Y = 20. En este caso, como mcd(100 35) = 5 divide a 20, sabemos que hay soluciones, y que estas son las mismas que las de la ecuacion 20X + 7Y = 4. Los coecientes de esta, 20 y 7, son coprimos, y una identidad de Bezout para ellos es 20 (;1) + 7 3 = 1. As, (X0  Y0) = (;4 12) es una solucion particular. Finalmente, la solucion general es (X Y ) = (;4 + 7 12 ; 20) ( 2 ): Z 1.5 El Teorema Fundamental de la Aritmetica En esta seccion vamos a demostrar el Teorema Fundamental de la Aritmetica que asegura que todo entero se puede escribir de forma esencialmente unica como producto de numeros primos. Por supuesto tenemos que empezar deniendo que signica ser primo. Si a es un entero, entonces 1, ;1, a y ;a son divisores de a. Proposicion 1.5.1 Las siguientes condiciones son equivalentes para un entero p distinto de 0, 1 y ;1. 1. Los unicos divisores de p son 1, ;1, p y ;p. 2. Si a y b son enteros tales que p = ab, entonces a = 1 o b = 1. 3. Si I es un ideal de tal que (p)  I , entonces I = (p) o I = . 4. Si a y b son enteros tales que p j ab, entonces p j a o p j b. Z Z 5. Si a y b son enteros que no son multiplos de p, entonces ab tampoco es multiplo de p. Demostracion. La equivalencia entre 1 y 2 es evidente. Tambien es evidente que las condiciones 4 y 5 son equivalentes. La equivalencia entre 1 y 3 es una consecuencia inmediata del Teorema 1.2.11, la Proposicion 1.2.8 y el Ejercicio 1.2.9. 1 implica 4. Supongamos que p satisface 1 y sean a y b enteros tales que p j ab. Si p no divide a a entonces a y p no tienen mas divisores comunes que 1 y por lo tanto son coprimos entonces p j b por la Proposicion 1.3.6. 4 implica 1. Supongamos que p satisface 4 y sea a un divisor de p. Si p = ab entonces p j a o p j b. En el primer caso (p) = (a), y del Ejercicio 1.2.10 se deduce que a = p. En el segundo caso b = pt para cierto entero t entonces 1 = ta y del Ejercicio 1.2.9 se deduce que a = 1. De nicion 1.5.2 Un numero entero p se dice primo si es distinto de 0, 1 y ;1 y satisface las condiciones de la Proposicion 1.5.1. 1.5. EL TEOREMA FUNDAMENTAL DE LA ARITME TICA 21 Ejercicio 1.5.3 Si p es un entero primo y p j a1 an (para ciertos enteros a1  : : : an), demostrar que p divide a ai para cierto ndice i. Teorema 1.5.4 (Teorema Fundamental de la Aritmetica) Sea a un numero entero diferente de 0 y 1. Entonces: 1. a es producto de numeros primos. 2. Si a = p1 pn = q1 qm , con p1 : : :  pn q1 : : :  qm primos, entonces n = m y podemos reordenar los qi de modo que se tenga qi = pi para cada ndice i. Demostracion. Para demostrar 1, podemos suponer que a 2 (>por que?). Por reduccion al absurdo suponemos que a es el menor numero natural mayor que 1 que no es producto de primos. Entonces a no puede ser primo, y por tanto a = bc para dos enteros b y c, ambos diferentes de 1 y ;1. Cambiando el signo si hace falta podemos suponer que b y c son numeros positivos y por tanto son ambos estrictamente menores que a. Por la minimalidad de a, se tiene que tanto b como c son producto de numeros primos. Por tanto a es producto de numeros primos, lo que induce una contradiccion. Esto prueba 1. Supongamos que a = p1 pn = q1 qm , como en 2. Podemos suponer que n m. Razonamos por induccion sobre n. Si n = 1, entonces a es primo y, por tanto, todos los qi menos uno son iguales a 1 o ;1. Como ninguno de los qi puede ser 1 ni ;1, se deduce que m = 1 y p1 = q1. Supongamos que el resultado es cierto para menos de n primos pi . Entonces pn divide a a = q1 qm . Como pn es primo, divide a algun qi, y reordenando los qi si es necesario podemos suponer que pn divide a qm . Como qm es primo, se tiene que qm = pn , y por tanto pan = p1 pn;1 = q1 : : :(qm;1 ). Por hipotesis de induccion se tiene n ; 1 = m ; 1 (luego n = m) y podemos reordenar los qi de manera que se tenga qi = pi para todo i = 1 2 : : :  n ; 1, y por supuesto tambien para i = n. Ejercicio 1.5.5 Dados enteros a y b, demostrar que: 1. Si jbj > 1 entonces existe un entero primo p que divide a b. 2. a y b son coprimos si y solo si no tienen ningun divisor primo comun (es decir, si no existe ningun entero primo p tal que p j a y p j b. Sea a un numero entero no nulo. Una factorizacion prima de a es una expresion del tipo a = p1 1 pnn (1.5.6) donde n 0, los pi son primos positivos distintos, y los exponentes i son enteros no negativos (cuando n = 0 se obtiene un \producto vaco" que, por convenio, vale 1). Observese que la denicion permite la aparicion de exponentes nulos es decir, de \primos cticios" en la descomposicion de a si esto no ocurre (si los i son positivos) decimos que (1.5.6) es una factorizacion prima irredundante de a. Con esta terminologa, el Teorema Fundamental de la Aritmetica puede reenunciarse como sigue: Teorema 1.5.6 (Teorema Fundamental de la Aritmetica) Todo entero no nulo tiene una facto- rizacion prima irredundante que es unica salvo el orden. El exponente i de la expresion (1.5.6) se llama multiplicidad de pi (o de ;pi ) en a. Claramente, un primo p divide a a precisamente si la multiplicidad de p en a no es 0. Mas generalmente tenemos el siguiente criterio de divisibilidad. Ejercicio 1.5.7 Sean a y b dos numeros enteros. Demostrar que a divide a b precisamente si para todo primo p la multiplicidad de p en a es menor o igual que la multiplicidad de p en b. De donde se deduce facilmente: Ejercicio 1.5.8 Sean a = p1 1 pnn y b = p1 1 pnn factorizaciones primas de los numeros enteros a y b (siempre podemos encontrar dos factorizaciones en las que aparecen los mismos primos, a~nadiendo con exponente 0 los que sean necesarios). Demostrar que 1 1 ) mcd(a b) = pmin( 1 max(1 1 )   ) pmin( n n n y mcm(a b) = p1 n n ) : pmax( n  CAPITULO 1. EL ANILLO DE LOS NUMEROS ENTEROS 22 Una consecuencia elemental pero importante del Teorema Fundamental de la Aritmetica es que cada entero diferente de 0 y de 1 tiene (al menos) un divisor primo. Esto nos permite demostrar el siguiente Teorema de Euclides: Teorema 1.5.9 (Euclides) El conjunto de enteros primos es innito. Demostracion. Supongamos por reduccion al absurdo que el conjunto de enteros primos es nito y sean p1  : : :  pn todos los primos positivos. Sea a = p1 pn + 1. Por el Ejercicio 1.5.5, a es divisible por algun primo positivo, que debe ser uno de los pi. Entonces pi divide a a ; p1 pn = 1, de lo que se deriva una contradiccion. Terminamos esta seccion con una consecuencia del Teorema Fundamental de la Aritmetica de la que haremos uso mas adelante. En ella usaremos el concepto de numero racional, que el lector conocera al menos de forma intuitiva. De forma natural, es posible construir los numeros racionales de forma rigurosa a partir de los numeros enteros. Veremos una construccion mas general en la Seccion 2.9. Proposicion 1.5.10 Si a es un entero que no es un cuadrado de un numero entero (es decir, la ecuacion a = x2 no tiene ninguna solucion entera), entonces tampoco es el cuadrado de un numero racional (es decir, la ecuacion a = x2 no tiene ninguna solucion racional). Z Demostracion. Vamos a probar que, si 0 6= a 2 es el cuadrado de un numero racional, entonces es el cuadrado de un numero entero, lo que obviamente equivale al enunciado. Supongamos pues que existen enteros no nulos b y c tales que a = (b=c)2 , o sea, ac2 = b2 . Sabemos que existen enteros primos p1  : : :  pr distintos dos a dos y enteros no negativos 1 : : :  r  1  : : :  r  1 : : :  r tales que a = p1 1 pr r  b = p1 1 pr r y c = p11 pr r : La unicidad del Teorema Fundamental de la Aritmetica aplicada a la igualdad ac2 = b2 nos dice que, para cada i = 1 : : :  r, se tiene i + 2 i = 2 i , de donde 2( i ; i ) = i 0 y por tanto i i . Esto implica que c divide a b y en consecuencia a es el cuadrado del numero entero b=c. 1.6 Congruencias Dados tres numeros enteros a, b y n, la expresion a  b mod n se lee \a es congruente con b modulo n" y signica n j b ; a. En otras palabras a  b mod n precisamente si b ; a 2 (n): Dejaremos que el lector demuestre las propiedades mas elementales de la relacion de congruencia. Ejercicio 1.6.1 Demostrar que se verican la siguientes propiedades (todas las variables representan numeros enteros): 1. a  a mod n. 2. Si a  b mod n, entonces b  a mod n. 3. Si a  b mod n y b  c mod n, entonces a  c mod n. 4. a  b mod 0 precisamente si a = b. 5. a  b mod 1 para cualquier par de enteros a b. 6. a  b mod n precisamente si a  b mod (;n). 7. a  0 mod n precisamente si n j a. 8. Si r es el resto de dividir a entre n, entonces a  r mod n. 1.6. CONGRUENCIAS 9. 10. 11. 12. 23 Sea n 6= 0. Entonces a  b mod n si y solo si a y b dan el mismo resto al dividirlos entre n. Si a  b mod n y c  d mod n, entonces a + c  b + d mod n y ac  bd mod n. Si a  b mod n y m j n, entonces a  b mod m. Las dos congruencias a  b mod n y a  b mod m equivalen a la congruencia a  b mod t, donde t = mcm(n m). Fijemos un entero n. Las tres primeras propiedades del Ejercicio 1.6.1 muestran que la relacion binaria \ser congruente modulo n" es una relacion de equivalencia. La propiedad 6 muestra que n y ;n denen la misma relacion de equivalencia, y por lo tanto no perdemos generalidad si suponemos que n no es negativo. Esta relacion induce una particion de en clases de equivalencia. Denotaremos con a]n (o con a], si n esta claro por el contexto) la clase de equivalencia que contiene a a. O sea a]n = fb 2 : a  b mod ng = fb 2 : n j b ; ag = fa + tn : t 2 g En vista de esta ultima expresion, a veces se escribe a + (n) en vez de a]n. El conjunto de las clases de equivalencia se denota =(n) o n, y en el denimos una suma y un producto mediante las reglas: a] + b] = a + b] y a] b] = ab]: Observacion 1.6.2 Es conveniente observar que para sumar (o multiplicar) dos elementos A y B de n hacemos lo siguiente: 1. Elegimos a 2 A y b 2 B. 2. Calculamos a + b (o ab). 3. Denimos A + B (o AB) como la clase de equivalencia de a + b (o de ab). Hemos hecho la anterior observacion para que quede claro que, en principio, el valor de la suma A + B podra depender de los elementos a 2 A y b 2 B elegidos. Es decir, si a a0 2 A y b b0 2 B, entonces, en principio, podra ocurrir que a + b y a0 + b0 no estuvieran en la misma clase y, por tanto, a + b] y a0 + b0 ] seran distintos. En tal caso la denicion de A + B que hemos dado sera ambigua. Las mismas observaciones podran hacerse sobre la denicion del producto. Sin embargo, la propiedad 10 del Ejercicio 1.6.1 nos muestra que esto no ocurre, con lo que podemos garantizar que la suma y el producto de n estan bien denidas. Observese que si n = 0, entonces cada clase de equivalencia tiene un unico elemento. Por tanto, podemos identicar 0 con y cada a 2 con a]0. Supongamos que n > 0. Por las propiedades 8 y 9 del Ejercicio 1.6.1, cada numero entero esta en una de las siguientes clases de equivalencia: 0] 1] : : :  n ; 1] y cada dos de estas clases de equivalencia son diferentes (>por que?). Luego n tiene n elementos y n = f0] 1]: ::  n ; 1]g: Casi todas las propiedades de la suma y el producto de numeros enteros tambien se verican en n: Ejercicio 1.6.3 Sea n un numero entero. Demostrar: 1. Asociativa: Dados r s t 2 n, se tiene r + (s + t) = (r + s) + t y r(st) = (rs)t. 2. Conmutativa: Dados r s 2 n, se tiene r + s = s + r y rs = sr. 3. Distributiva: Dados r s t 2 n, se tiene r(s + t) = rs + rt. 4. Neutro: Para todo r 2 n se verican r + 0] = r y r1] = r. Ademas 0] y 1] son los unicos Z ZZ Z Z Z Z Z Z Z ZZ ZZ Z Z ZZ Z Z Z elementos de n que satisfacen esta propiedad. 5. Opuesto: Para todo a 2 , a]+ ;a] = 0]. Ademas, ;a] es el unico elemento de n que verica esta propiedad. 24  CAPITULO 1. EL ANILLO DE LOS NUMEROS ENTEROS Z Z Sin embargo, en general, la propiedad de regularidad no se verica en n. Por ejemplo 2]42]4 = 0]4. Se dice que x 2 n es un divisor de cero, o que es singular, si existe un y 6= 0], tal que xy = 0]. En caso contrario (es decir, si xy = 0] implica y = 0]) decimos que x es regular. Se dice que x es invertible si existe y 2 n tal que xy = 1]. Z Ejercicio 1.6.4 Demostrar que si un elemento x de Z Z ZZ Z n es invertible, entonces existe un unico y 2 n tal que xy = 1]n. Dicho elemento se llama inverso de x y se denota x;1. Z Las deniciones anteriores pueden aplicarse a los elementos de , y en este caso sabemos que 0 es el unico divisor de cero y que los unicos elementos invertibles son 1 y ;1. En particular, en hay elementos regulares que no son invertibles (cualquiera distinto de 0 y de 1). Esto no ocurre en n (cuando n 6= 0), como muestra la siguiente proposicion. Z Proposicion 1.6.5 Sea 0 6= n 2 . Las siguientes condiciones son equivalentes para todo a 2 . 1. a]n es invertible. Z 2. a]n es cancelable es decir, si a]ny = a]nz con y z 2 n entonces y = z . 3. a]n es regular. 4. mcd(a n) = 1. Demostracion. Pongamos x = a]n, y en general r] = r]n. 1 implica 2. Si x es invertible y se verica xy = xz entonces, multiplicando ambos miembros por x;1, deducimos que y = z. 2 implica 3. Si xy = 0 entonces xy = x0], y cancelando x deducimos que y = 0]. 3 implica 4. Veamos que si un entero d divide a a y a n entonces d j 1. Poniendo a0 = da y n0 = nd tenemos xn0] = an0] = a0dn0] = a0n] = 0] y entonces por hipotesis n0] = 0] es decir, n0 d = n j n0 y por lo tanto d j 1. 4 implica 1. Supongamos ahora que a y n son coprimos. Por el Lema de Bezout existen enteros r s tales que ra + sn = 1, y entonces ra  1 mod n, o sea r]a] = 1, por lo que a] es invertible en n. Z Z La demostracion de la ultima implicacion nos dice como se calcula en la practica un inverso de a] en n (cuando existe): basta con encontrar una identidad de Bezout ra + sn = 1, y entonces r] = a];1. ZZ Ejemplo 1.6.6 Inversos en n. Consideremos el elemento 7] en 26. Aplicando el Algoritmo de Euclides a 7 y 26 obtenemos la tabla: 26 7 5 2 1 0 3 1 2 2 que nos dice que mcd(7 26) = 1 por lo tanto 7] es invertible, y como ademas 1 = = = = = deducimos que 7];1 = ;11] = 15] en Z 26. 5;2 2 5 ; 2 (7 ; 5 1) 3 5;2 7 3 (26 ; 3 7) ; 2 7 3 26 ; 11 7 Z Cuando n es peque~no, hay una forma rudimentaria de calcular el inverso de a] en n que suele ser mas rapida que el Algoritmo de Euclides. Se trata simplemente de ir multiplicando a] por 1] 2] :: : hasta que el producto valga 1]. Por cierto, el inverso de n ; 1] es precisamente n ; 1] (>por que?). 1.7. ECUACIONES DE CONGRUENCIAS TEOREMA CHINO DE LOS RESTOS 25 1.7 Ecuaciones de congruencias Teorema Chino de los Restos Una ecuacion lineal de congruencias es una ecuacion del tipo aX  c mod n (1.7.7) con n 6= 0. Una solucion de (1.7.7) es un numero entero x tal que ax  c mod n. La ecuacion (1.7.7) es equivalente a la ecuacion Z a]nX = c]n: (1.7.8) Esto quiere decir que x 2 es una solucion de la ecuacion (1.7.7) precisamente si x]n es una solucion de la ecuacion (1.7.8). Vamos a ver como resolver ecuaciones lineales de congruencias. Empezaremos considerando el caso mas sencillo, que se da cuando a]n es invertible. Entonces la unica solucion de la ecuacion (1.7.8) es a];n 1c]n (es decir, se obtiene \dividiendo por a]n"). En otras palabras, si a];n 1c]n = b]n, entonces las soluciones de la ecuacion (1.7.7) son los numeros enteros de la forma Z b + n ( 2 ): Por ejemplo, para resolver la ecuacion 7X  3 mod 26 observamos que 7] = 7]26 es invertible y su inverso es 15] = 15]26. Como 15]3] = 45] = 19], las soluciones de la ecuacion son los numeros enteros de la forma Z 19 + 26 ( 2 ): Observese que, en este caso, todas las soluciones son congruentes (entre s) modulo n. Este hecho se suele expresar diciendo que (1.7.7) tiene solucion unica modulo n. El caso general (a]n no es necesariamente invertible) se reduce al anterior gracias al siguiente resultado (comparese con la Proposicion 1.4.4). Proposicion 1.7.1 Sean a y n enteros con n 6= 0, y sean d = mcd(a n), a0 = ad y n0 = nd . Entonces la ecuacion (1.7.7) tiene solucion si y solo si d j c. En este caso, si c0 = dc , las soluciones de (1.7.7) son las mismas que las de a0 X  c0 mod n0 (1.7.9) y por tanto (1.7.7) tiene solucion unica modulo n0. Demostracion. El \si y solo si" se tiene porque ambas condiciones equivalen a la existencia de enteros x y tales que c = ax + ny. El resto es claro pues podemos cancelar d = 6 0. En resumen, para resolver la ecuacion (1.7.7) seguiremos los siguientes pasos, en los que mantenemos la notacion anterior: 1. Calculamos d = mcd(a n). 2. Si d no divide a c, concluimos que la ecuacion no tiene solucion y acabamos. Z 3. Si d divide a c entonces las soluciones de (1.7.7) son las de (1.7.9) es decir, las de la forma b0 + n0 con b0]n = a0];n 1c0]n . 0 0 0 ( 2 )  CAPITULO 1. EL ANILLO DE LOS NUMEROS ENTEROS 26 Ejemplo 1.7.2 Ecuacion lineal de congruencias. Vamos a resolver la ecuacion lineal de congruencias 6x  9 mod 15 Como mcd(6 15) = 3 divide a 9, la ecuacion tiene solucion y es equivalente a la ecuacion 2x  3 mod 5: Como 2];5 1 = 3]5 y 3]53]5 = 4]5, las soluciones de esta ecuacion son de la forma 4 + 5 Z ( 2 ): En la segunda parte de esta seccion vamos a ver como se resuelve un sistema de ecuaciones lineales de congruencias del tipo X  c1 mod n1 X  c2 mod n2 (1.7.10) .. . X  ck mod nk donde los ni son coprimos dos a dos es decir, mcd(ni  nj ) = 1 para todo i 6= j. Comencemos con el siguiente lema: Lema 1.7.3 Si los enteros n1 : : : nk son coprimos dos a dos, entonces su mnimo comun multiplo coincide con su producto n = n1 nk . Demostracion. Cuando k = 2, el resultado es una consecuencia inmediata de la Proposicion 1.4.3, y el caso general se demuestra facilmente por induccion usando la "asociatividad del mnimo comun multiplo", que a su vez es consecuencia de la Proposicion 1.3.3. Teorema 1.7.4 (Teorema Chino de los Restos) Sean n1  : : :  nk enteros coprimos dos a dos, y sea n = n1 nk . Entonces la aplicacion Z Z Z n x]n f ;! n1   nk 7! (x]n1  : : :  x]nk ) (1.7.11) es biyectiva. (De hecho, usando la terminologa que introduciremos mas tarde, esta aplicacion es un isomorsmo de anillos ver el Teorema 2.7.10.) ZZ Z Demostracion. Primero hay que asegurarse de que f esta bien denida (su valor no depende del representante de x]n que tomemos ver la Observacion 1.6.2), pero eso es muy facil y lo dejamos como ejercicio para el lector. Para ver que f es biyectiva, como los conjuntos n y n1   nk tienen el mismo cardinal (vease el Problema 7), basta con demostrar que f es inyectiva. Sean pues x]n e y]n con la misma imagen por f esto signica que x  y mod ni para cada i = 1 2 : : : k, de modo que x ; y es un multiplo comun de los ni y por lo tanto es multiplo de su mnimo comun multiplo, que por el lema es n en otras palabras, x  y mod n, y esto prueba que f es inyectiva. Aunque el teorema, tal como lo hemos enunciado, no parece tener nada que ver con sistemas de ecuaciones, una lectura atenta del mismo nos permite deducir que el sistema (1.7.10) tiene solucion unica modulo n cuando los ni son coprimos dos a dos. En efecto, la suprayectividad de f se traduce en que, dados enteros arbitrarios c1  : : : ck, existe un entero x con f(x]n) = (c1]n1  : : : ck]nk ), y esta igualdad signica exactamente que x es una solucion del sistema. Por otra parte, es claro que la inyectividad de f se traduce en la unicidad modulo n de esta solucion. Por lo tanto, se tiene: Teorema 1.7.5 Sean n1 : : : nk enteros coprimos dos a dos y sean c1  : : : ck enteros arbitrarios. Entonces el sistema (1.7.10) tiene solucion unica modulo n = n1 nk . 1.8. TEOREMAS DE EULER Y FERMAT 27 La demostracion que hemos dado del Teorema Chino presenta un problema: en ella se deduce que f es suprayectiva porque es una aplicacion inyectiva entre conjuntos nitos del mismo cardinal, pero dados c1  : : : ck no nos dice como es el elemento x]n tal que f(x]n ) = (c1]n1  : : : ck]nk ). Afortunadamente, hay un metodo para obtener ese x. En el parrafo siguiente vemos como se obtiene x cuando solo hay 2 ecuaciones, y en el ejemplo que le sigue queda claro que para sistemas con mas ecuaciones basta con repetir este metodo. Supongamos pues que k = 2, y sean r1 r2 enteros tales que r1n1 + r2 n2 = 1 entonces se tiene r1 n1  0 mod n1 r2n2  1 mod n1  r1n1  1 mod n2 y r2 n2  0 mod n2 de donde se deduce que el entero x = c2r1n1 + c1r2 n2 verica x  c1 mod n1 x  c2 mod n2  y por tanto x es la solucion buscada. Ejemplo 1.7.6 Sistemas de ecuaciones lineales de congruencias. Vamos a resolver el sistema de ecuaciones lineales de congruencias x  1 mod 3 2x  3 mod 7 3x  12 mod 15 En primer lugar convertimos cada ecuacion en otra ecuacion equivalente, en la que el coeciente de la incognita sea 1 en todas las ecuaciones y en la que los modulos sean coprimos dos a dos (en el sistema dado, 3 y 15 no son coprimos). A la primera ecuacion no hay que hacerle nada. Como 2]7 es invertible y 2];7 1 = 4], la segunda ecuacion es equivalente a x  5 mod 7. Esto no vale para la tercera ecuacion, pues 3]15 no es invertible. Sin embargo esta ecuacion es equivalente a x  4 mod 5 por la Proposicion 1.7.1. Por tanto el sistema original es equivalente a x  1 mod 3 x  5 mod 7 x  4 mod 5 Como (;2)3 + 1 7 = 1, el parrafo anterior nos dice que podemos sustituir las dos primeras ecuaciones por la ecuacion x  5(;2)3 + 1 1 7 = ;23 mod 21, o lo que es lo mismo por x  ;2 mod 21. Nos queda as el sistema x  ;2 mod 21 x  4 mod 5 Finalmente como 1 21 + (;4)5 = 1, nos quedamos con la unica ecuacion x  4 1 21 + (;2)(;4)5 = 124  19 mod 105: Luego las soluciones del sistema son las de la forma: x = 19 + 105t (<compruebalo!). Z NN Z 1.8 Teoremas de Euler y Fermat Z (t 2 ) Z Z Denotaremos por n al conjunto de los elementos invertibles de n. El cardinal de n se denota por (n), y la aplicacion : ! se llama funcion de Euler. Para algunos valores bajos de n, el lector puede dar una descripcion explcita de n (usando la Proposicion 1.6.5) y deducir el valor de (n). Por otra parte, es elemental ver que, si p es un entero primo positivo, entonces (p) = p ; 1. La funcion de Euler es util a la hora de hacer calculos con congruencias en los que aparecen potencias, como veremos en los ejemplos que siguen. El resultado fundamental para esto es el siguiente:  CAPITULO 1. EL ANILLO DE LOS NUMEROS ENTEROS 28 Z Teorema 1.8.1 (Teorema de Euler) Si mcd(a n) = 1, entonces a(n)  1 mod n: Demostracion. Sean x1] x2] : : : xk] todos los elementos invertibles de n, de modo que k = (n) y n = fx1] x2] : : : xk]g. Como a] es invertible, los elementos a]x1] a]x2] : : : a]xk] estan en n (por las Proposiciones 1.6.5 y 1.3.6) y son distintos entre s (porque a] es cancelable). Por lo tanto, n = fa]x1] a]x2] : : : a]xk]g, y en consecuencia se tiene Z Z Z x1]x2] xk ] = a] x1] a] x1] a] xk ]: Ahora basta con cancelar los xi] para obtener el resultado deseado. Un caso particular del Teorema de Euler (1.8.1) es el siguiente: Corolario 1.8.2 (Teorema Peque~no de Fermat) Si p es un numero primo y p 6 j a, entonces ap;1  1 mod p: Por tanto, para todo numero entero x se verica xp  x mod p: Estos teoremas sirven para resolver situaciones como las del siguiente ejemplo: Ejemplos 1.8.3 Aplicaciones de los Teoremas de Euler y Fermat. 1. Vamos a calcular el resto que se obtiene al dividir 347231 entre 5, lo que equivale a encontrar un entero x entre 0 y 4 tal que x  347231 mod 5. Por una parte tenemos 347  2 mod 5, por otra el Teorema de Fermat nos dice que 24  1 mod 5, y por ultimo dividiendo entre 4 tenemos 231 = 4 57 + 3. Juntando todo esto deducimos que 347231  2457+3  (24 )5723  15723  8  3 mod 5 as que el resto buscado es 3. 2. Para calcular el resto de dividir 2370 por 18 procedemos de modo similar: como (18) = 6 (<compruebalo!), el Teorema de Euler nos dice que 236  1 mod 18, y como 70 = 6 11 + 4 obtenemos 2370  5611+4  (56)11 54  11154  625  13 mod 18 as que el resto buscado es 13. En el apartado 2 del ejemplo anterior hemos necesitado el valor de (18), que se puede calcular de modo directo. Para valores grandes de n es interesante encontrar un modo rapido de calcular (n), y eso es lo que hacemos en el ultimo resultado de este captulo. Proposicion 1.8.4 (Calculo de la funcion de Euler) 1. Si p es un entero primo positivo y  es un entero positivo, entonces (p) = p;1 (p ; 1). 2. Si n y m son enteros coprimos entonces (nm) = (n) (m). 3. Si n = p1 1 pr r es una factorizacion prima irredundante del entero n, entonces    1 (pr ; 1) = n 1 ; p1 1; p : 1 r Demostracion. 1. Podemos describir p eliminando en p = f1] 2] :: : p]g los elementos no invertibles, que son los a] en los que a no es coprimo con p como esto ocurre si y solo si a es multiplo de p, estamos eliminando exactamente p;1 elementos (uno de cada p), por lo que en p quedan exactamente p ; p;1 = p;1 (p ; 1) elementos, y esto es lo que queramos ver. 2. Si n y m son coprimos, el Teorema Chino nos dice que la aplicacion f : nm ! n  m dada por f(x]nm) = (x]n x]m) es una biyeccion. Usando la Proposicion 1.3.6 es facil ver que x]nm esta en nm si y solo si su imagen por f esta en n  m, por lo que f se restringe a una biyeccion entre esos conjuntos. Por lo tanto nm y n  m tienen el mismo cardinal, y ahora el resultado es claro. Ahora 3 se demuestra facilmente por induccion a partir de los apartados anteriores. (n) = p1 1;1 Z Z prr ;1 (p1 ; 1) Z Z ZZ Z Z Z Z ZZ  1.9. APE NDICE: CRIPTOGRAFIA DE CLAVE PUBLICA 29 1.9 Apendice: criptografa de clave publica Algunas de las ideas que hemos desarrollado en este captulo admiten una interesante aplicacion a la criptografa que, obviando algunos detalles tecnicos, desarrollaremos en este apendice. El \sistema criptograco de clave publica" que vamos a describir es el mas utilizado en la actualidad, y recibe el nombre de RSA por las iniciales de sus autores: Rivest, Shamir y Adleman. El sistema funciona as: Nosotros queremos que cualquier usuario del sistema pueda cifrar y enviarnos un mensaje, con un procedimiento que ha de ser publico. Y queremos ser los unicos capaces de descifrarlo, incluso en el caso de que, en el proceso de envo del mensaje, este sea interceptado por alguien. Obviamente, el procedimiento de descifrado debe tener alguna diferencia con el de cifrado. Hacemos lo siguiente: Elegimos dos numeros primos distintos p y q que sean \grandes", y calculamos n = pq y (n) = (p ; 1)(q ; 1). Elegimos entonces un entero k coprimo con (n) y obtenemos una identidad de Bezout 1 = kr + (n)s. Hacemos publicos los valores de n y de k y reservamos en secreto el resto (de hecho, basta con conservar el valor de r). En lo que sigue trabajaremos en n, y asumiremos que las igualdades son modulo n y que cada elemento viene dado por su representante en f0 1 : : :  n ; 1g. Un mensaje puede descomponerse en bloques de un cierto numero de caracteres, y cada bloque puede asociarse unvocamente a un elemento a de n. Este proceso de traduccion de un mensaje a una sucesion de elementos de n (los asociados a cada bloque), y el proceso inverso, son tambien publicos, de modo que podemos asumir que cada a 2 n es un mensaje claro. El problema estriba entonces en cifrar y descifrar elementos de n. Para cifrar a 2 n, simplemente se calcula c = ak (mejor dicho, el resto de la division entera de k a por n). Por tanto, nosotros recibimos el mensaje cifrado c vamos a ver que, para descifrarlo, basta con calcular cr . En efecto, se trata de ver que cr  a mod n y, por el Teorema Chino de los Restos, esto equivale a que la congruencia valga modulo p y modulo q. Comprobaremos solo que cr  a mod p, pues el otro caso es analogo. Ahora hay dos opciones: Si a  0 mod p entonces c = ak  0 mod p y el resultado es claro. En otro caso, el Teorema Peque~no de Fermat nos dice que cr = (ak )r = akr = a1;(n)s = a1;(p;1)(q;1)s = a(ap;1 )(1;q)s = a1(1;q)s = a como queramos comprobar. Los calculos que se han indicado, especialmente las exponenciaciones, pueden ser tremendos si los enteros que intervienen son grandes. Ademas hay mensajes que viajan y pueden ser interceptados. Por tanto, hay que convencerse de dos cosas: Los calculos son factibles. Es decir, un buen ordenador puede hacerlos en poco tiempo. El sistema es seguro. Es decir, si alguien intercepta c = ak , no sera capaz de calcular a en un tiempo razonable. El punto anterior exige que nosotros s seamos capaces de hacer esto, y para eso usamos la clave secreta r. En cuanto al primer punto, diremos que hay algoritmos rapidos2 para tratar enteros grandes, que nos permiten hacer sumas, multiplicaciones y exponenciaciones en un tiempo razonable y tambien es rapido el algoritmo de Euclides, que nos permite hallar identidades de Bezout. En cuanto al segundo punto, observemos lo siguiente: Si alguien intercepta el mensaje c = ak , puede tratar de descifrarlo encontrando nuestra clave secreta r, y en seguida veremos que esto es practicamente imposible si elegimos adecuadamente los primos p y q del principio. Antes de eso comentaremos que podran estudiarse otros metodos para obtener a conociendo c = ak , pero hoy en da no se conoce ninguno. Es decir, no hay algoritmos rapidos para la extraccion de races k-esimas modulo n. Volvamos pues al problema de encontrar r, el inverso de k modulo (n). Para ello solo se necesita conocer (n), porque entonces, como k es publico, el Algoritmo de Euclides proporciona r. Como el interceptor tambien conoce n, hemos de estar seguros de que no puede calcular (n) a partir de n, y por tanto no debe ser capaz de hallar la factorizacion prima de n. El problema es entonces este: >pueden encontrarse primos p y q tales que, en la practica, sea imposible factorizar n = pq sin conocer de antemano esos factores? La respuesta es, hoy da, s. Si se toman dos primos p y q sucientemente grandes, los mejores algoritmos de factorizacion tardaran siglos en hallar p y q a partir de n. Actualmente, no es difcil Z Z Z Z Z Z 2 Ni la idea de algoritmo r apido ni la descripcion de esos algoritmos son difciles, pero se salen de los objetivos de este texto.  CAPITULO 1. EL ANILLO DE LOS NUMEROS ENTEROS 30 generar enteros primos de unas 200 cifras decimales, y los numeros de 400 cifras (como n) son demasiado grandes para los algoritmos de factorizacion actuales. Tal vez un ejemplo sencillo ayude apcomprender esto: Es facil ver a mano que el numero p = 1:607 es primo, pues ningun primo menor que 1:607 = 40008::: lo divide (solo hay que probar con 12 primos). Del mismo modo, 16 divisiones sencillas son sucientes para asegurarse de que q = 3:433 es primo. Pero si nos dan directamente n = 5:516:831, tendramos que hacer unas 400 divisiones de enteros medianamente grandes para encontrar el factor p, lo que llevara mucho tiempo si trabajamos a mano. Esto ilustra la situacion a la que nos referamos en el parrafo anterior: si dos personas usan el mismo algoritmo y los mismos medios tecnicos, una puede fabricar facilmente un numero compuesto de tal forma que a la otra le cueste mucho trabajo encontrar sus factores primos. Terminaremos con algunos comentarios. Los dos primeros se reeren a la eleccion de las claves, y los restantes ponen de maniesto otras ventajas del sistema. No es difcil, si se tiene un buen test de primalidad y un buen ordenador, obtener primos de unas 200 cifras: Se genera aleatoriamente un numero impar m de 200 cifras y se le aplica el test sucesivamente a m, m+2, m+4... (para enteros de este tama~no los mejores tests dan un resultado seguro). Por mucho que tarde en aparecer un primo, la cosa no llevara mas de unos minutos. Los primos p y q no solo han de ser grandes, sino que deben cumplir otras condiciones sin las cuales seran detectados por algunos tests de factorizacion. Por ejemplo: no deben estar muy proximos entre s, no deben ser de ciertos tipos especiales (por ejemplo, de la forma 2n ; 1), y el maximo comun divisor de p ; 1 y q ; 1 no debe ser muy grande. Si se obtienen p y q aleatoriamente, como en el punto anterior, hay que tener muy mala suerte para que no se cumplan estos requisitos. En estos casos nos olvidamos de los primos obtenidos y repetimos el proceso. La clave secreta r no ha de ser compartida con el emisor, no ha de viajar, y esto hace al sistema muy seguro. Los mensajes cifrados pueden hacerse publicos, no hay que enviarlos a escondidas. Si cada usuario U del sistema hace publica su propia clave de cifrado (nU  kU ) y conserva secreta su clave de descifrado rU , se pueden enviar mensajes rmados. Sean V y W dos usuarios del sistema, y sea a 2 nW un mensanje claro. Si V enva a W el mensaje cifrado c = (arV )kW , W puede descifrarlo calculando (crW )kV = a, y ademas puede estar seguro de que V es el emisor (>por que?). Z Z 1.10 Problemas 1. Sean a b c d 2 . Demostrar (a) Si a b y c d, entonces a + c b + d. (b) Si 0 < a < b y 0 < c d, entonces ac < bd. (c) Si a < b < 0 y c d < 0, entonces bd < ac. 2. Demostrar por induccion que se verican las siguientes formulas: P ; (a) (Binomio de Newton) (a + b)n = ni=0 ni ai bn;i: P (b) ni=1 i = n(n2+1) . P n+1) . (c) ni=1 i2 = n(n+1)(2 6 3. Demostrar que, en , la relacion de divisibilidad verica las propiedades reexiva, antisimetrica y transitiva, pero no la dicotoma (en otras palabras, la relacion es un orden parcial pero no es un orden total). 4. Demostrar que el Principio de Induccion y el Axioma de Buena Ordenacion son equivalentes es decir, demostrar el Axioma de Buena Ordenacion a partir de los demas axiomas del conjunto de los enteros y del Principio de Induccion. N 1.10. PROBLEMAS 31 N NN Z N NN N Z 5. Decimos que un conjunto X con una relacion de orden parcial esta bien ordenado si todo subconjunto no vaco tiene un mnimo. Demostrar que el Axioma de Buena Ordenacion es equivalente a que el conjunto , con la relacion de orden inducida por , este bien ordenado. 6. Dado n 2 , denotamos por n al conjunto de los enteros positivos menores o iguales que n es decir, n = fa 2 : 1 a ng (en particular, 0 es el conjunto vaco). Demostrar que las siguientes condiciones son equivalentes para un conjunto A: (a) Para cierto n 2 , existe una aplicacion biyectiva n ! A. (b) Para cierto m 2 , existe una aplicacion suprayectiva m ! A. (c) Para cierto k 2 , existe una aplicacion inyectiva A ! k. (d) Si f : A ! A es una aplicacion inyectiva, entonces f es suprayectiva. (e) Si f : A ! A es una aplicacion suprayectiva, entonces f es inyectiva. (f) Si S es un subconjunto propio de A, no existe ninguna aplicacion inyectiva A ! S. (g) Si S es un subconjunto propio de A, no existe ninguna aplicacion suprayectiva S ! A. (h) Si S es un subconjunto propio de A, no existe ninguna aplicacion biyectiva S ! A. Un conjunto se dice que es nito si satisface las condiciones anteriores. 7. Demostrar que si A es un conjunto nito (ver el Problema 6), entonces el numero n 2 para el que existe una biyeccion A ! n es unico. Dicho numero natural n se llama cardinal de A y lo denotaremos por jAj. En particular j j = 0. Demostrar: (a) Dos conjuntos nitos tienen el mismo cardinal precisamente cuando existe una biyeccion entre ellos. (b) Dados a b 2 con a < b, el conjunto A = fx 2 : a x bg es nito y jAj = b ; a + 1. (c) Si A y B son dos conjuntos nitos, entonces jA  B j = jAj jB j. (d) Si A es nito, entonces el conjunto P (A) formado por los subconjuntos de A es nito y jP (A)j = 2jAj . 8. Calcular el maximo comun divisor y el mnimo comun multiplo de a = 2689 y b = 4001. Encontrar tambien enteros r y s tales que mcd(a b) = ra + sb. >Son unicos estos valores de r y s? 9. Demostrar que, para cualquier n 2 , los enteros 5n + 2 y 12n + 5 son coprimos. 10. Demostrar que cualesquiera dos miembros consecutivos de la sucesion de Fibonacci N NN N N N Z Z Z Z 1 1 2 3 5 8 : : : an = an;1 + an;2 son coprimos. 11. Sean a b c 2 y sean d = mcd(a b) y m = mcm(a b). Demostrar que cd = mcd(ac bc) y cm = mcm(ac bc): 12. Sean S y T dos conjuntos de numeros enteros. Demostrar que mcd(S  T ) = mcd(mcd(S) mcd(T)) 13. Resolver las siguientes ecuaciones diofanticas: (a) 225X + 15Y = 100. (b) 213X + 180Y = 300. (c) 2040X ; 3740Y = 1360. y mcm(S  T) = mcm(mcm(S) mcm(T )):  CAPITULO 1. EL ANILLO DE LOS NUMEROS ENTEROS 32 14. En una clase hay 30 alumnos que cursan dos asignaturas. Se llega al acuerdo siguiente: El que suspenda dos asignaturas pagara 3 euros el que apruebe solo una recibira 2 euros y el que apruebe las dos recibira 4 euros. >Cuantos alumnos han de aprobar dos, una y ninguna asignatura para que no sobre ni falte dinero? >Hay una unica respuesta valida? 15. Un hombre compro una docena de piezas de fruta (naranjas y manzanas) por 99 pesetas. Si una manzana cuesta 3 pesetas mas que una naranja y compro mas manzanas que naranjas, >cuantas manzanas y cuantas naranjas compro? 16. Demostrar que, si S es un conjunto innito de numeros enteros, entonces mcm(S) = 0. 17. Para a b c 2 , demostrar las formulas Z mcm(a mcd(b c)) = mcd(mcm(a b) mcm(a c)) mcd(a mcm(b c)) = mcm(mcd(a b) mcd(a c)): 18. Sean a1 : : :  an numeros enteros y sean c = a1 an, Ai = c=ai , d = mcd(A1  : : :  An) y m = mcm(a1  : : :  an). Demostrar que dm = c. 19. Demostrar que el numero de divisores de un entero no nulo es nito y obtener una formula que proporcione el numero de divisores de un numero entero en funcion de su descomposicion en producto de primos. 20. Se pide, dados los numeros enteros a = 464200 b = 905760 y c = 336435 : (a) Hallar las factorizaciones primas irredundantes de a, b y c. (b) Usarlas para calcular mcd(a b), mcd(a c), mcd(b c), mcm(a b), mcm(a c) y mcm(b c). (c) Calcular el numero de divisores positivos de a, b y c, y encontrar explcitamente los de c (usar el Problema 19). 21. Sean a y b dos enteros coprimos. Demostrar que ab y a + b tambien son coprimos. 22. Sea b un numero natural mayor que 1. Demostrar que para todo numero natural n existe una unica lista de enteros r0  r1 : : :  rk tales que 0 r0 r1 : : :  rk < b 23. 24. 25. 26. 27. rk 6= 0 y n = r0 + r1b + r2 b2 + + rk bk : La sucesion rk  : : :  r2 r1 r0 se llama representacion de n en base b (cuando n = 2 se habla de representacion binaria y cuando n = 10 se habla de representacion decimal, y si no se dice otra cosa usaremos las representaciones decimales). Como aplicacion, calcular las representaciones de 69 en bases 2, 3 y 6. Demostrar los criterios escolares de divisibilidad por 2, 3, 5 y 9: (a) Un numero entero es multiplo de 2 (respectivamente de 5) precisamente si lo es su ultima cifra en base decimal. (b) Un numero entero es multiplo de 3 (respectivamente de 9) precisamente si la suma de sus cifras en base decimal es multiplo de 3 (respectivamente de 9). Dar criterios de divisibilidad por 4, 6, 11, 13 y 101. *] Demostrar que si a1 : : :  an son numeros enteros consecutivos, entonces n divide a exactamente uno de ellos y n! divide a su producto. ; Demostrar que un entero positivo p es primo si y solo si p divide al coeciente binomial pi para cada i = 1 2 : : :  p ; 1. Demostrar que hay innitos numeros primos positivos de la forma 4n ; 1. (Indicacion: Un entero positivo de esa forma ha de tener un divisor primo que tambien sea de esa forma.) 1.10. PROBLEMAS 33 28. Se pide: (a) Si a es un numero entero y p es un divisor primo impar de a2 +1, demostrar que p  1 mod 4. (Indicacion: Poner p = 2t + 1 y usar el Teorema de Fermat para ver que t es par.) (b) Deducir que hay innitos numeros primos positivos de la forma 4n+1. (Indicacion: En caso contrario, considerar a = q!, donde q es el mayor de tales primos.) (c) >Por que no sirve un argumento como el del Problema 27 para demostrar el apartado anterior? 29. Demostrar que un numero primo mayor que 3 es congruente con 1 o con ;1 modulo 6, y deducir que hay innitos numeros primos de la forma 6n ; 1. 30. Sean m y n numeros enteros y sea k un numero natural. Demostrar que: (a) n ; 1 divide a nk ; 1 mas generalmente, n ; m divide a nk ; mk . (b) Si k es impar, entonces n + 1 divide a nk + 1 mas generalmente, n + m divide a nk + mk . N 31. Demostrar los siguientes enunciados, donde n 2 (usar el Problema 30): (a) Si el entero3 2n ; 1 es primo, entonces n es primo. (b) Si el entero4 2n + 1 es primo, entonces n = 0 o n es una potencia de 2. 32. Para cada n 2 , sea Fn = 22n + 1. Demostrar que se tiene 33. 34. 35. 36. 37. 38. N Z Fn+1 = 2 + F0F1 Fn y deducir que los Fn son coprimos dos a dos y que por tanto existen innitos enteros primos. Dados a b 2 tales que 3 j a2 + b2, demostrar que 3 j a y 3 j b. Hallar todas las soluciones enteras de la ecuacion x2  1 mod 4. Demostrar que la ecuacion x2 + y2  3 mod 4 no tiene soluciones enteras. Demostrar que 13 divide a 42n+1 + 3n+2 para todo n 2 . Demostrar que 4 no divide a n2 + 1 ni a n2 + 2 para ningun numero entero n. Resolver cada una de las ecuaciones lineales de congruencias siguientes: 3x  14 mod 17 2x ; 1  1 mod 5 3x + 6  0 mod 12 33x  9 mod 1128 6x  3 mod 35 3x  13 mod 18 7x  4 mod 10 33x  9 mod 128 N 39. Determinar los numeros enteros entre 400 y 500 que dan resto 5 al dividirlos por 6 y dan resto 2 al dividirlos por 11. 40. Resolver los siguientes sistemas de congruencias: 8 x  3 mod 4 9 8 7x  28 mod 35 9 < = < = x  4 mod 25 6x  18 mod 42 : x  5 mod 9  : 5x  3 mod 12  Z 41. Demostrar que si a, b y n son enteros tales que a  b mod n entonces mcd(a n) = mcd(b n). 42. Sean a b 2 n. Demostrar que ab es invertible precisamente si lo son a y b. 3 Los enteros de la forma M = 2p ; 1 con p primo se llaman n umeros de Mersenne. No es cierto que todos ellos sean p primos (>cual es el menor primo p tal que Mp no es primo?). Por otra parte, los mayores numeros primos que se conocen son de este tipo. 4 Los enteros de la forma F = 22m + 1 se llaman n umeros de Fermat. No es difcil comprobar que Fm es primo para m m = 1 2 3 4 y que F5 es divisible por 641. De hecho, para m  5, todos los Fm que se han analizado son compuestos, pero no existe una demostracion de la conjetura que este hecho sugiere. 34  CAPITULO 1. EL ANILLO DE LOS NUMEROS ENTEROS Z 43. Si n 6= 0 y d = mcd(a n) divide a c, demostrar que la ecuacion aX  c mod n tiene exactamente d soluciones en n. 44. Si p es un entero positivo primo, demostrar que, para enteros arbitrarios a y b, se tiene (a + b)p  ap + bp mod p: 45. Sean a, b y c enteros tales que a2 + b2 = c2 . Demostrar que: (a) Al menos uno de los enteros a, b o c es multiplo de 3. (b) Al menos uno de los enteros a, b o c es multiplo de 5. (c) El producto abc es multiplo de 60. (Indicacion: Trabajar modulo 8 para ver que abc es multiplo de 4.) 46. Sean a k l n 2 . Demostrar que las condiciones Z mcd(a n) = 1 mcd(k (n)) = 1 y kl  1 mod (n) implican que akl  a mod n, y usar este resultado para resolver la ecuacion x7  5 mod 64. 47. *] Sea n un entero positivo. Demostrar que 48. 49. 50. 51. 52. Z N n= X (d) donde d recorre el conjunto de los divisores positivos de n. (Indicacion: Para cada d, considerar el conjunto Xd = f nd x]n : x]d 2 dg.) Demostrar que, si a m n 2 y m 6= n, entonces mcd(a2m + 1 a2n + 1) vale 1 si a es par y 2 si a es impar. Calcular el ultimo dgito decimal de 3400. Hallar el resto de las siguientes divisiones enteras: (a) 132231 entre 7. (b) 246218 entre 11. (c) 2113143 entre 12. (d) 347231 entre 35. Para cada uno de los valores n = 2 3 : : :  30, se pide: (a) Calcular (n) usando la formula de la Proposicion 1.8.4. (b) Describir explcitamente n y comprobar que tiene (n) elementos. Determinar todos los enteros n tales que (n) = 4, y todos los enteros n tales que (n) = 8. Bibliografa del captulo Z Allenby 1], Clark 9], Cohn 10], Delgado-Fuertes-Xambo 12], Dorronsoro-Hernandez 13], Jacobson 23]. Captulo 2 Anillos Se estudia la estructura de anillo y se presentan sus ejemplos y propiedades basicas. Introduccion Algunas de las propiedades que verican la suma y el producto de numeros enteros son compartidas por otros \sistemas" formados por un conjunto con dos operaciones: clases de restos, polinomios, matrices, funciones... Estos sistemas, y otros muchos que veremos, se engloban en el concepto abstracto de anillo, a cuyo estudio dedicamos este captulo. Los primeros problemas que abordaremos son los que se plantean al estudiar la mayora de las estructuras abstractas en Matematicas. Para un anillo A, describiremos los subconjuntos de A que, con las operaciones de A, siguen siendo anillos (subanillos). Tambien consideraremos las relaciones de equivalencia en A que son compatibles con sus operaciones, lo que dara lugar a los conceptos de ideal y de anillo cociente. Por ultimo, estudiaremos las aplicaciones entre anillos que conservan la estructura (homomorsmos de anillos) y las entenderemos como el modo natural de establecer relaciones entre anillos, hasta interpretar los anillos isomorfos como anillos \esencialmente iguales". En las ultimas secciones estudiamos dos tipos de anillos con ciertas propiedades especiales: los dominios y los cuerpos. Sus prototipos son los anillos de numeros enteros y racionales, respectivamente, y veremos como, imitando la construccion de a partir de , a cada dominio se le puede asociar un cuerpo con el que se relaciona estrechamente. Q Z Objetivos del captulo Conocer los axiomas que denen un anillo, las propiedades basicas de las operaciones en un anillo arbitrario y los principales ejemplos de anillos. Conocer y manejar los conceptos de subanillo e ideal saber identicar el ideal generado por un subconjunto y saber encontrar y manejar sistemas generadores de ideales. Conocer el concepto de anillo cociente y la relacion entre los ideales de un anillo y los de sus cocientes (Teorema de la Correspondencia). Conocer las propiedades basicas de los homomorsmos de anillos, las nociones de nucleo e imagen y los Teoremas de Isomorfa. Manejar las nociones de elemento cancelable e invertible, las de dominio y cuerpo, y las relaciones de estas con los ideales primos y maximales. Entender la construccion del cuerpo de fracciones de un dominio y la relacion entre ambos. 35 CAPITULO 2. ANILLOS 36 Desarrollo de los contenidos 2.1 Operaciones En las matematicas elementales se consideran y estudian las \operaciones aritmeticas" primero con numeros naturales, luego con enteros y con racionales, mas tarde con reales o complejos, o con vectores del plano o del espacio, por ejemplo. Entre estas operaciones, las mas simples son la suma y el producto, cuyas deniciones dependen del conjunto que estemos considerando. Por ejemplo, el producto de dos numeros naturales n y m puede denirse como el resultado de sumar n consigo mismo m veces. El producto de dos numeros reales (positivos) x e y tiene un signicado mas geometrico: puede interpretarse como el valor del area de un rectangulo cuyos lados tienen longitud x e y. Analogamente, la suma de numeros naturales no tiene el mismo signicado que la suma de vectores o la de numeros complejos. Lo que resulta de la discusion anterior es que, si uno quiere abstraer una idea general de \operacion entre los elementos de un conjunto", no es posible atender al signicado de cada operacion particular. Lo que hay de comun entre todas las operaciones que hemos considerado antes es algo formal, que se reeja en la denicion que sigue: De nicion 2.1.1 Una operacion1 o ley de composicion interna en un conjunto A es una aplicacion del producto cartesiano A  A en A. Usualmente, una operacion se denota por un smbolo como , +, , , \, , etcetera, y entonces la imagen del par (a b) 2 A  A se denota por a  b, a + b, a b, etcetera. De hecho, cuando se usa la notacion es costumbre omitir el punto y se suele escribir ab en lugar de a b. La mayora de los smbolos propuestos en el parrafo anterior sugeriran al lector ejemplos de operaciones que conoce. Algunas \operaciones" familiares pueden no serlo en el sentido de la denicion precedente. Por ejemplo, la resta de numeros naturales puede no ser un numero natural, de modo que la resta no es una ley de composicion interna en (s podramos verla como una \ley de composicion externa"  ! ). Por razones analogas, la division no es una operacion en ni en , aunque s lo es en n f0g. La denicion de operacion es demasiado general, y en la practica nos interesaran solo las operaciones que veriquen ciertas propiedades que las hagan manejables. Pasamos pues a considerar ciertas propiedades que puede vericar una operacion. De nicion 2.1.2 Sea  una operacion en un conjunto A. Decimos que: La operacion es asociativa si a  (b  c) = (a  b)  c para cualesquiera a b c 2 A. La operacion es conmutativa si a  b = b  a para cualesquiera a b 2 A. El elemento e 2 A es un elemento neutro si a  e = a = e  a para cada a 2 A. Si existe un elemento neutro e, el elemento b 2 A es simetrico2 de a 2 A si a  b = e = b  a. El elemento a 2 A es cancelable si cualquiera de las relaciones a  b = a  c o b  a = c  a (con b c 2 A) implica que b = c. QN N Z N Z Q Es claro que, cuando se cumple la propiedad conmutativa, las deniciones de elemento neutro, elemento simetrico y elemento cancelable se simplican. Ejemplos 2.1.3 Operaciones. ZN Z 1. La suma usual es una operacion en cada uno de los conjuntos , y +. En los tres casos es asociativa y conmutativa, solo hay elemento neutro en los dos primeros y el 1 solo tiene elemento simetrico en el primer caso. Otra operacion que podemos considerar en esos conjuntos es el producto usual. >Que propiedades tiene en cada uno de ellos? Este ejemplo y otros muchos que vendran muestran que, en general, las propiedades de una operacion dependen fuertemente del conjunto ambiente en el que la estemos considerando. Deberamos decir operacion binaria porque se aplica a pares de elementos, pero no necesitaremos hablar de otro tipo de operaciones y nos ahorraremos la repeticion del adjetivo. 2 En contextos m as concretos usaremos elemento opuesto y elemento inverso en lugar de elemento simetrico. 1 2.2. GRUPOS ABELIANOS Y ANILLOS 37 2. Si A es el conjunto de todas las funciones reales de variable real, la composicion de funciones es una operacion en A que es asociativa y tiene un elemento neutro e dado por e(x) = x, pero en general no es conmutativa por ejemplo, si f y g son las funciones denidas por f(x) = x2 y g(x) = x + 1 entonces (f  g)(x) = x2 + 2x + 1, mientras que (g  f)(x) = x2 + 1. 3. En el conjunto + la operacion  dada por a  b = ab no es asociativa ni conmutativa ni tiene elemento neutro (aunque s tiene lo que podramos llamar un neutro por la derecha: el 1 verica a  1 = a para cualquier a, pero no 1  a = a). Z Ejercicio 2.1.4 Para un conjunto A con una operacion asociativa  con un elemento neutro e, demostrar que se verican las siguientes propiedades (ver la demostracion de la Proposicion 1.1.1): 1. (Unicidad del neutro) El neutro es unico mas aun, si e verica e  a = a para cada a 2 A y f verica a  f = a para cada a 2 A entonces e = f . 2. (Unicidad de los simetricos) Si a 2 A tiene elemento simetrico, este es unico mas aun, si b 2 A verica a  b = e y c 2 A verica c  a = a entonces b = c. 3. Todo elemento que tenga un simetrico es cancelable. En virtud de este resultado, cuando existan, hablaremos de el elemento neutro de la operacion y de el elemento simetrico de a, y no solo de un elemento neutro o un elemento simetrico de a. Tambien es claro que, si b es el simetrico de a, entonces a es el simetrico de b. En este parrafo suponemos que  es una operacion asociativa en el conjunto A. Dados tres elementos a b c 2 A podemos escribir a  b  c sin ambig#uedad, en el sentido de que es indiferente cual de los dos  actue primero. Cuando tenemos cuatro elementos, la asociatividad nos dice que a  ((b  c)  d) = a  (b  (c  d)) = (a  b)  (c  d) = ((a  b)  c)  d = (a  (b  c))  d y por lo tanto tampoco en este caso hay ambig#uedad al escribir a  b  c  d sin parentesis (comprueba que no hay otras opciones para el orden en el que actuan los tres ). Esta propiedad de asociatividad generalizada es cierta para cualquier conjunto nito de elementos, y la asumiremos sin demostracion. As, dados elementos a1  : : : an en A, escribiremos a1   an sin ambig#uedad y se vericaran relaciones como a1   an = (a1   an;1)  an = a1  (a2   an ): Cuando ademas la operacion sea conmutativa, entonces se tiene una conmutatividad generalizada que nos permite, en una expresion como la anterior, reordenar los ai de cualquier forma sin alterar el resultado de la operacion. 2.2 Grupos abelianos y anillos De nicion 2.2.1 Un grupo abeliano es un par3 (A +) formado por un conjunto no vaco A y una operacion + en A que es asociativa, conmutativa, con elemento neutro y tal que cada elemento de A tiene un simetrico (y por lo tanto es cancelable). Al neutro le llamaremos cero y lo denotaremos por 0. Al simetrico de a le llamaremos su opuesto y lo denotaremos por ;a (se tiene por lo tanto ;(;a) = a). Escribiremos ademas a ; b en lugar de a + (;b). Z Ejemplos 2.2.2 Grupos abelianos. Q ZN R ZC 1. Los conjuntos de numeros enteros, de numeros racionales, de numeros reales y de numeros complejos son grupos abelianos con la suma usual. Tambien lo es el conjunto 2 de los numeros enteros pares, y en general cualquier ideal de . 2. La suma de numeros naturales no convierte a en un grupo abeliano >por que? 3 Cuando no haya riesgo de confusion con la operacion diremos simplemente que A es un grupo abeliano. CAPITULO 2. ANILLOS 38 3. Un conjunto con un unico elemento es un grupo abeliano con la unica operacion posible, y se llama grupo trivial. Observese que hay muchos grupos triviales (uno por cada conjunto unitario) pero todos son esencialmente iguales. 4. La operacion de un grupo abeliano no siempre es una suma en el sentido usual. Por ejemplo, el conjunto  de los numeros racionales no nulos con el producto usual es un grupo abeliano (con neutro 1), y lo mismo ocurre con los conjuntos n denidos en la Seccion 1.8. Nos referiremos a estos grupos como el grupo multiplicativo de los racionales no nulos o de n. De nicion 2.2.3 Un anillo (conmutativo y con identidad) es una terna4 (A + ) formada por un conjunto no vaco A y dos operaciones + y en A la primera llamada usualmente suma y la segunda producto o multiplicacion, que verican: 1. A es un grupo abeliano con la suma (cuyo neutro denotamos por 0). 2. El producto es asociativo y conmutativo y tiene un elemento neutro al que llamaremos uno o identidad, y que denotaremos por 1. Q Z Z Se pueden denir anillos \no conmutativos" o \sin identidad" excluyendo las correspondientes propiedades para el producto en la denicion. En este curso asumiremos que los anillos son conmutativos y con identidad y avisaremos cuando esto no ocurra. Por ejemplo, en el Captulo 10 trabajaremos con anillos no conmutativos. 3. Se verica la siguiente propiedad distributiva que relaciona las dos operaciones: Dados a b c 2 A se verica a (b + c) = (a b) + (a c): En general, no se asume que cada elemento a de A tenga simetrico para el producto. Cuando lo tiene, lo denotamos por a;1 (de modo que aa;1 = 1 y (a;1);1 = a), le llamamos el inverso de a y decimos que a es invertible o una unidad en A (del Ejercicio 2.1.4 se deduce que si a es invertible entonces es cancelable en productos es decir, ab = ac implica b = c). Denotaremos por A al conjunto de todas las unidades de A. Usualmente escribiremos ab en vez de a b. Ademas asumiremos que, en ausencia de parentesis, los productos se realizan antes que las sumas (y que las restas). As, por ejemplo, la propiedad distributiva se reescribe como a(b + c) = ab + ac. Si A es conmutativo y b es invertible, escribiremos a veces a=b o ab en lugar de ab;1. Esta notacion no se debe utilizar con anillos no conmutativos pues en ese caso ab;1 y b;1a pueden ser diferentes. De los axiomas de anillo se pueden deducir algunas propiedades elementales: Ejercicio 2.2.4 Sea A un anillo y sean a b c 2 A. Demostrar que se verican las siguientes propiedades: 1. 0a = 0 = a0. 2. a(;b) = (;a)b = ;(ab). 3. a(b ; c) = ab ; ac. 4. ab es invertible precisamente si a y b son invertibles. En tal caso (ab);1 = a;1 b;1. 5. A es un grupo abeliano con el producto de A. Dados un anillo A, un elemento a 2 A y un entero positivo n, la notacion na (respectivamente an ) representa el resultado de sumar (respectivamente multiplicar) a consigo mismo n veces, y si n = 0 hacemos5 0a = 0 y a0 = 1. Mas rigurosamente, a partir de estas ultimas igualdades se denen na y an de forma recurrente poniendo (n + 1)a = a + na y an+1 = aan para n 0. Por ultimo, si n 1 se dene (;n)a = ;(na), y si ademas a es invertible se dene a;n = (a;1 )n . Cuando no haya riesgo de confusion con las operaciones diremos simplemente que A es un anillo. En general, si una operacion en un conjunto tiene neutro, se asume que \operar el conjunto vaco" da como resultado el neutro. As, con las notaciones usuales, la \suma de ningun elemento" se interpreta como el cero y el "producto de ningun elemento" se interpreta como el uno. Esta convencion tiene importantes ventajas para la notacion que se pueden observar ya en el ejercicio que sigue. 4 5 2.2. GRUPOS ABELIANOS Y ANILLOS Z 39 Ejercicio 2.2.5 Dados un anillo A, elementos a b 2 A y enteros m n 2 , se verican las siguientes propiedades: 1. n(a + b) = na + nb. 2. (n + m)a = na + ma. 3. Si n 0 entonces (ab)n = an bn. 4. Si n m 0 entonces an+m = an am . 5. Si a y b son invertibles, las dos propiedades anteriores valen tambien para exponentes negativos. ZQ R C Q Z Z Z Z Z Ejemplos 2.2.6 Anillos. 1. Los conjuntos , , y son anillos con la suma y el producto usuales. Notese que todo elemento no nulo es invertible en , pero en solo hay dos elementos invertibles en general, las propiedades de un elemento dependeran del anillo en el que lo estemos considerando. 2. El conjunto 2 de los numeros enteros pares es un \anillo sin identidad". 3. Las operaciones denidas en la Seccion 1.6 dotan a n de una estructura de anillo. Observese que 0 es el propio y que la notacion n para el conjunto de las unidades de n es consistente con la que se empleo anteriormente . 4. Sean A y B dos anillos. Entonces el producto cartesiano A  B tiene una estructura de anillo con las operaciones (a1 b1) + (a2  b2) = (a1 + a2 b1 + b2) y (a1  b1) (a2  b2) = (a1 a2  b1 b2 ) (se dice que las operaciones en A  B se denen componente a componente). Observese que A  B es conmutativo precisamente si lo son A y B, y que esta construccion se puede generalizar a productos cartesianos de cualquier familia (nita o no) de anillos. 5. Dados un anillo A y un conjunto X, el conjunto AX de las aplicaciones de X en A es un anillo con las siguientes operaciones: (f + g)(x) = f(x) + g(x) (f g)(x) = f(x) g(x): >Cual es la relacion entre este ejemplo y el anterior? 6. Dado un anillo A, un polinomio en la indeterminada X con coecientes en A es6 una expresion del tipo P = P (X) = a0 + a1 X + a2 X 2 + + anX n donde n es un numero entero no negativo y ai 2 A para todo i. Para cada i, ai se llama coeciente de grado i de P, a0 se llama coeciente independiente de P y, si an 6= 0, entonces n es el grado de P y an es su coeciente principal. Dos polinomios son iguales si y solo si lo son coeciente a coeciente. Denotaremos por AX] al conjunto de los polinomios en la indeterminada X con coecientes en A. Utilizando la estructura de anillo de A se puede dotar a AX] de una estructura de anillo deniendo la suma y el producto de la forma usual: (a0 + a1 X + a2 X 2 + ) + (b0 + b1 X + b2 X 2 + ) = c0 + c1X + c2X 2 +  donde cada cn = an + bn, y (a0 + a1 X + a2 X 2 + ) (b0 + b1 X + b2 X 2 + ) = d0 + d1 X + d2X 2 +  P donde cada dn = a0 bn + a1bn;1 + + an;1b1 + an b0 = ni=0 ai bn;i (si un coeciente no aparece en la expresion de un polinomio se considera que vale 0). Z 6 Z Por ahora no daremos deniciones rigurosas sobre polinomios estas vendran en el Captulo 4. CAPITULO 2. ANILLOS 40 7. Dado un anillo A, denotamos por AX]] el conjunto de las sucesiones (a0  a1 a2 : : :) de elementos de A. En AX]] consideramos la suma y el producto dados por (a0  a1 a2 : : :) + (b0  b1 b2 : : :) = (a0 + b0  a1 + b1 a2 + b2 : : :) (a0 a1 a2 : : :)(b0  b1 b2 : : :) = (a0 b0 a0b1 + a1 b0 a0b2 + a1b1 + a2 b0 : : :): Observese la similitud con la denicion de las operaciones el anillo de polinomios de hecho, P a X i .enCon el elemento (a0  a1 a2 : : :) se suele denotar por 1 estas operaciones, AX]] es un i i=0 anillo llamado el anillo de series de potencias con coecientes en A. 8. Sean A un anillo y n un numero natural. Denotaremos por Mn (A) el conjunto de las matrices cuadradas de dimension n con coecientes en A. Es decir un elemento de Mn(A) tiene la forma 0 a a ::: a 1 11 12 1n B a21 a22 : : : a2n C B @ : : : : : : : : : : : : CA an1 an2 : : : ann con aij 2 A, para todo 1 i j n. Con el n escribir menos, una matriz como la anterior la representaremos como (aij )1ij n o simplemente como (aij ). Denimos la suma y el producto de matrices como (aij ) + (bij ) = (aij + bij ) (aij ) (bij ) = (ai1b1j + + ainbnj ): El lector reconocera la suma y producto de matrices del algebra lineal y no debe tener problemas para demostrar que Mn (A) es un anillo cuando A lo es. Observese que Mn (A) solo es conmutativo si n = 1 y A es conmutativo. R 9. Sea V un espacio vectorial no trivial sobre un cuerpo K. El lector que no este familiarizado con el concepto de cuerpo puede pensar que K = . Sea A el conjunto de los endomorsmos de V . Entonces A tiene una estructura de anillo dada por las operaciones (f + g)(x) = f(x) + g(x) (fg)(x) = f(g(x)): Es decir, el producto en A es la composicion de endomorsmos. Este anillo solo es conmutativo si la dimension de V es 1. 2.3 Subanillos Si A es un conjunto con una operacion  y B es un subconjunto de A, decimos que B es cerrado para la operacion  si se tiene x  y 2 B cuando x y 2 B. Esto implica que  : A  A ! A se restringe a una aplicacion  : B  B ! B, y por lo tanto podemos considerar a  como una operacion en B que se dice inducida por la operacion en A. De nicion 2.3.1 Si (A + ) es un anillo, un subanillo de A es un subconjunto B de A cerrado para ambas operaciones, que contiene al 1 y tal que (B + ) es un anillo. La siguiente proposicion nos dice como comprobar si un subconjunto es un un subanillo. Proposicion 2.3.2 Las condiciones siguientes son equivalentes para un subconjunto B de un anillo A: 1. B es un subanillo de A. 2. B contiene al 1 y es cerrado para sumas, productos y opuestos. 3. B contiene al 1 y es cerrado para restas y productos. 2.3. SUBANILLOS 41 Demostracion. 1 implica 2. Si B es un subanillo de A es evidente que B contiene al 1 y es cerrado para sumas y productos. Por otro lado, como B es un anillo, cada elemento b 2 B tiene un opuesto. Por la unicidad del elemento simetrico (Ejercicio 2.1.4), este opuesto ha de ser el de A, con lo que B es cerrado para opuestos. 2 implica 3 es evidente. 3 implica 1. Sea B un subconjunto de A que contiene al uno y es cerrado para restas y productos. Entonces 0 = 1 ; 1 2 B, con lo que si b 2 B, entonces ;b = 0 ; b 2 B es decir, B es cerrado para opuestos. Si a b 2 B, entonces ;b 2 B y, por tanto, a + b = a ; (;b) 2 B es decir, B es cerrado para sumas. Ahora es evidente que B es un subanillo de A. Ejemplos 2.3.3 Subanillos. ZQ R C 1. Cada uno de los anillos , , y es un subanillo de los posteriores. 2. Todo anillo A es un subanillo de s mismo, al que llamamos impropio por oposicion al resto de subanillos, que se dicen propios. 3. Si A es un anillo, el subconjunto f0g es cerrado para sumas, productos y opuestos. Sin embargo, no contiene al 1 (si A 6= 0), con lo que no es un subanillo de A. 4. Si A es un anillo, el conjunto 1 = fn1 : n 2 g de los multiplos enteros de 1 es un subanillo de A contenido en cualquier otro subanillo de A es decir, 1 es el menor subanillo de A, y se conoce como el subanillo primo de A. Es claro que y los n son sus propios subanillos primos, y por lo tanto no tienen subanillos propios. 5. Si A y B son anillos y B 6= 0 entonces A  0 = f(a 0) j a 2 Ag es cerrado para sumas y productos pero no es un subanillo de A  B (>por que?). 6. Dado un numero entero m, los conjuntos p p p p  m] = fa + b m : a b 2 g y  m] = fa + b m : a b 2 g son subanillos de . Si ademas m es positivo, entonces ambos son subanillos de . Si m es el cuadrado de un numero entero entonces esos conjuntos coinciden con y con , respectivamente, por lo que el ejemplo carece de interes. Cuando m no es el cuadrado de un entero (por ejemplo m = 2 o m = ;1) entonces, por la Proposicion 1.5.10, tampoco es el cuadrado de unpnumero racional, de manera que, en cualquiera de los dos anillos descritos, la igualdad a + b m = 0 implica que a = 0 y b = 0, y por lo tanto la igualdad a + bpm = c + dpm implica que a = c y b = d. p Un caso particular es el anillo i], donde i = ;1, llamado el anillo de los enteros de Gauss. Podemos visualizar i] dentro del plano complejo como el conjunto de los vertices de un enlosado del plano complejo por losas cuadradas de lado 1, como muestra el siguiente esquema: Z Z Z ZC Z Z Z Q QR Z Q Z Z Z Mas generalmente, si m < 0, entonces podemos visualizar pm] como el conjunto de vertices de un enlosado p con una base de longitud 1 y una altura de p del plano complejo por losas rectangulares longitud ;m. Por ejemplo, una porcion de  ;5] esta representada por los siguientes puntos del plano complejo: Z CAPITULO 2. ANILLOS 42 { { { { { { { { { 7. Todo anillo A puede verse como un subanillo del anillo de polinomios AX] si identicamos los elementos de A con los polinomios constantes (del tipo P = a0). 8. Sea A un anillo y X un conjunto. Entonces la diagonal B = ff 2 AX : f(x) = f(y) para todo x y 2 X g (es decir, el conjunto de las aplicaciones constantes de X en A) es un subanillo de AX . 2.4 Ideales y anillos cociente La nocion de ideal que vimos en el Captulo 1 para numeros enteros se puede generalizar a un anillo arbitrario. De nicion 2.4.1 Sea A un anillo. Una combinacion lineal con coecientes en A (o una combinacion A-lineal) de los elementos a1  : : : an de A es un elemento de A de la forma r1 a1 + + rn an donde cada ri 2 A. Los enteros ri son los coecientes de la combinacion lineal. Un subconjunto I de A es un ideal si no es vaco y si, dados a b 2 I , cualquier combinacion A-lineal suya ra + sb esta en I . Como ocurre con los ideales de , en la denicion podemos sustituir la condicion I 6= por la condicion 0 2 I, y cualquier combinacion lineal de un numero nito de elementos de un ideal I sigue siendo un elemento de I. Z Ejemplos 2.4.2 Ideales. 1. Si A es un anillo, el conjunto Z bA = (b) = fba : a 2 Ag es un ideal de A llamado ideal principal generado por b. Ya vimos en el Captulo 1 que todos los ideales de son de esta forma. Esto no es cierto en general, como pronto veremos. Observese que bA es el menor ideal de A que contiene a b. Observese tambien que 1A = A y que 0A = f0g, con lo que estos dos son ideales principales de A llamados respectivamente ideal impropio (en oposicion a ideales propios, para los demas) e ideal cero o trivial. El ideal trivial f0g lo representaremos a partir de ahora por 0. 2. Mas generalmente, si T es un subconjunto de un anillo, entonces el conjunto n X (T) = f i=1 ai ti : n 2 es un ideal, llamado ideal generado por T . Z i 2 A ti 2 X g + a 2.4. IDEALES Y ANILLOS COCIENTE 43 Z 3. Si A y B son dos anillos entonces A  0 = f(a 0) : a 2 Ag es un ideal de A  B. 4. Sea X] el anillo de los polinomios con coecientes enteros. Es facil ver que el ideal generado por el elemento X puede describirse como el de los polinomios sin coeciente independiente es decir, Z I = (X) = fa0 + a1 X + : : : + an X n 2 X] : a0 = 0g: Tambien es sencillo ver que el conjunto Z ZZ J = fa0 + a1X + : : : + anX n 2 X] : a0 2 2 g de los polinomios con coeciente independiente par es un ideal de X]. En el Ejemplo 2.5.9 veremos que este ideal no es principal. De nicion 2.4.3 Sea I un ideal de un anillo A. Decimos que dos elementos a b 2 A son congruentes modulo I , y escribimos a  b mod I , si su diferencia esta en I o sea: a  b mod I , b ; a 2 I: >A que te recuerda esto? Ejercicio 2.4.4 Dados un anillo A, un ideal I de A y elementos a b c d 2 A, demostrar que: 1. a  a mod I 2. Si a  b mod I , entonces b  a mod I . 3. Si a  b mod I y b  c mod I , entonces a  c mod I . 4. a  b mod (0) precisamente si a = b. Del Ejercicio 2.4.4 se deduce que la relacion \ser congruente modulo I" es una relacion de equivalencia en A y, por tanto, las clases de equivalencia por esta relacion denen una particion de A. La clase de equivalencia que contiene a un elemento a 2 A es a + I = fa + x : x 2 I g (en particular 0 + I = I), de modo que a + I = b + I , a  b mod I (en particular a + I = 0 + I , a 2 I). El conjunto de las clases de equivalencia se denota A=I = AI = fa + I : a 2 Ag: Ejercicio 2.4.5 Sea A un anillo con un ideal I . Las operaciones suma y producto en A=I dadas por (a + I) + (b + I) = (a + b) + I (a + I) (b + I) = (ab) + I estan bien denidas (ver la Observacion 1.6.2) y dotan a A=I de una estructura de anillo con neutro 0 + I y unidad 1 + I . Este anillo se llama anillo cociente de A modulo I . Al hacer el cociente de un anillo A por un ideal I, elementos que eran distintos en A \pasan a ser iguales" en el cociente (estrictamente hablando, son sus clases de equivalencia las que se hacen iguales) en particular, los elementos de I \se hacen cero". En muchas de las ocasiones en las que se construyen estructuras cociente eso es precisamente lo que se busca, identicar entre s o anular ciertos elementos. Z Ejemplos 2.4.6 Anillos cociente. Z 1. El anillo cociente del anillo por el ideal (n) es el anillo n de las clases de restos modulo n. CAPITULO 2. ANILLOS 44 Z Z 2. Para analizar las soluciones enteras de la ecuacion x2 ; 15y2 = 2 podemos considerarla en 5, donde toma la forma mas sencilla x2 = 2 (<anulamos ese ;15 que complicaba la ecuacion!) ahora es elemental ver que esta ecuacion no tiene soluciones en 5 y deducir que la ecuacion inicial no tiene soluciones enteras. 3. A=0 es el propio anillo A, mientras que A=A = 0. Z Z Z 4. Consideremos el ideal (X) del anillo de polinomios X] (ver los Ejemplos 2.4.2). Como todo polinomio es congruente modulo (X) con su coeciente independiente (visto como polinomio), no es difcil convencerse de que X]=(X) y son, en esencia, el mismo anillo (mas tarde precisaremos este comentario viendo que son isomorfos). 5. Sean A y B anillos e I = A  0. Como (a b)  (0 b) mod I, los anillos (A  B)=I y B son esencialemente iguales (isomorfos). Ejercicio 2.4.7 Demostrar que un elemento b de un anillo A es invertible si y solo si (b) = A, y deducir que las siguientes condiciones son equivalentes para un ideal I de A. 1. I es impropio es decir, I = A. 2. I contiene a la identidad de A es decir, 1 2 I . 3. I contiene una unidad de A es decir, I \ A 6= . El siguiente resultado describe los ideales de un anillo cociente. Emplearemos la siguiente notacion: si A es un anillo e I es un ideal suyo, : A ! A=I denotara la aplicacion que lleva cada elemento de A a su clase de equivalencia es decir, (a) = a + I. La imagen por de un subconjunto J de A es (J) = fa + I : a 2 J g: Si J contiene a I, denotaremos este conjunto por J=I. La preimagen por de un subconjunto X de A=I es ;1 (X) = fa 2 A : a + I 2 X g: Teorema 2.4.8 (Teorema de la Correspondencia) Si I es un ideal de un anillo A, las asignaciones J 7! J=I y X 7! ;1(X) son biyecciones (una inversa de la otra) que conservan la inclusion entre el conjunto de los ideales de A que contienen a I y el conjunto de todos los ideales de A=I . Demostracion. Hay que comprobar los siguientes puntos, cosa que el lector podra hacer como ejercicio: Si J es un ideal de A que contiene a I entonces J=I es un ideal de A=I y ;1 (J=I) = J. Si X es un ideal de A=I entonces ;1 (X) es un ideal de A que contiene a I y ;1(X)=I = X. Si J  K son ideales de A que contienen a I entonces J=I  K=I. Si X  Y son ideales de A=I entonces ;1 (X)  ;1 (Y ). ZZ Z ZZ Z ZZ Ejercicio 2.4.9 Si n es un entero positivo, demostrar que los ideales de =n son precisamente los de la forma m =n , donde m es un divisor positivo de n, y ademas m =n esta contenido en m0 =n si y solo si m0 divide a m. 2.5. OPERACIONES CON SUBANILLOS E IDEALES 45 2.5 Operaciones con subanillos e ideales Ejercicio 2.5.1 Si A es un anillo, demostrar que la interseccion de cualquier familia de subanillos (respectivamente de ideales) de A es un subanillo (respectivamente un ideal) de A. En general, no ocurre lo mismo con las uniones. Z El Ejercicio 2.5.1 nos va a permitir repetir lo que hicimos al denir el ideal de generado por un subconjunto. De nicion 2.5.2 Sea A un anillo y sea X un subconjunto de A. Llamamos subanillo de A generado por X a la interseccion B de todos los subanillos de A que contienen a X (>podemos asegurar que al menos existe uno?). Es claro que B es el menor subanillo de A que contiene a X es decir, B es un subanillo de A y todos los subanillos de A que contienen a X tambien contienen a B . De modo analogo, se dene el ideal de A generado por X que, siguiendo la notacion del Captulo 1, se denota (X). Ejercicio 2.5.3 Demostrar que las dos deniciones de ideal generado (ver los Ejemplos 2.4.2 y la Denicion 2.5.2) coinciden. Ejemplos 2.5.4 Subanillos e ideales generados por conjuntos. 1. Si A es un anillo, el ideal generado por es el ideal trivial, y el subanillo generado por es el subanillo primo (ver los Ejemplos 2.3.3). C Z 2. Si b es un elemento de un anillo A, el ideal generado por fbg es bA = (b) (ver los Ejemplos 2.4.2). p p 3. Si m es un entero que no es un cuadrado, el subanillo de generado por m es el anillo  m]. 4. Si X es un subanillo (respectivamente, un ideal) de A entonces el subanillo (respectivamente, el ideal) que genera es el propio X. Ejercicio 2.5.5 Demostrar que el conjunto I = fa + bi : a  b mod 2g Z es el ideal principal de i] generado por el elemento 1+i. (Indicacion: Usar la igualdad 2 = (1+i)(1 ; i).) Ya hemos observado que, en general, la union de ideales no es un ideal, as que es razonable considerar los ideales generados por esas uniones. Comenzamos con un ejercicio que resuelve el caso nito, y pasaremos luego al caso general. Ejercicio 2.5.6 Sean I y J ideales de un anillo A. Demostrar que el ideal de A generado por la union I  J es I + J = fx + y : x 2 I y 2 J g: Generalizar este resultado considerando un numero nito de ideales B1  : : : Bn. Un primer intento de generalizar la primera parte del ejercicio anterior al caso de familias innitas de ideales puede llevarnos a la consideracion de sumas innitas, que no estan denidas. Esto se resuelve facilmente como sigue: Diremos que una propiedad se cumple para casi todos los elementos de un conjunto X si se cumple para todos los elementos de X, salvo para una cantidad nita. Por ejemplo, si fax : x 2 X g es una familia de elementos de un grupo abeliano, diremos que ax = 0 para casi todo x 2 X (o que casi todos los ax son nulos) P si el conjunto fx 2 X : ax 6= 0g es nito. En este caso tiene sentido considerar la suma innita x2X ax interpretada como la suma de los elementos ax distintos de cero. CAPITULO 2. ANILLOS 46 Proposicion 2.5.7 Sea fIt : t 2 T g una familia de ideales de un anillo A. Entonces el subconjunto denido por X t2T It = f X t2T xt : xt 2 It para cada t 2 T y xt = 0 para casi todo t 2 T g (llamado la suma de los ideales fIt : t 2 T g) coincide con el ideal generado por la union t2T It es decir, es el menor ideal de A que contiene a cada uno de los It . Con la notacion anterior X t2T It = (t2T It ): Demostracion. Se trata de ver que el conjunto denido es el menor ideal de A que contiene a t2T It es decir, hay que comprobar tres cosas: P 1. t2T It es un ideal de A. P 2. t2T It contiene a t2T It . P 3. Cualquier ideal de A que contenga a t2T It debe contener a t2T It . Las tres demostraciones son elementales y se dejan a cargo del lector. Ejercicio 2.5.8 Si A es un anillo y X es un subconjunto, demostrar que el ideal generado por X es P x2X xA es decir, el ideal formado por todas las combinaciones A-lineales de elementos de Z X. Ejemplo 2.5.9 Un ideal que no es principal. El ideal J de X] descrito en los Ejemplos 2.4.2, consistente en los polinomios con coeciente independiente par, esta generado por el conjunto f2 X g es decir, J = (2 X). Vamos a ver que, de hecho, J no es un ideal principal. Para ello, llegaremos a una contradiccion suponiendo que existe un polinomio p 2 X] tal que J = (p). En efecto, en ese caso se tendra p 2 J y por tanto p = 2a0 +a1 X +a2 X 2 + para ciertos enteros ai. Pero tambien se tendra 2 2 (p) y X 2 (p), lo que signica que existen enteros bi y ci tales que Z 2 = p(b0 + b1X + X = p(c0 + c1 X + ) = 2a0b0 + (2a0 b1 + a1b0 )X + ) = 2a0c0 + (2a0 c1 + a1 c0)X + Comparando terminos independientes en las igualdad obtenemos a0 b0 = 1, de donde a = 1, y a0 c0 = 0, de donde c0 = 0. Comparando ahora los coecientes de X en la segunda igualdad obtenemos 1 = 2a0c1 + a1c0 = 2a0 c1, lo que nos da la contradiccion buscada. Terminamos la seccion considerando los productos de ideales. Dados dos ideales I y J de un anillo A, parece natural denir su producto como el conjunto T de todos los elementos de la forma ab con a 2 I y b 2 J. Sin embargo este conjunto no es, en general, cerrado para sumas. Por ejemplo, si A = X] e I = J consiste en los polinomios con termino independiente par, entonces los polinomios 2(2 + X) = 4 + 2X y X 2 estan en T, pero su suma 4 + 2X + X 2 no esta en T , pues una igualdad Z 4 + 2X + X 2 = (2a0 + a1 X + a2X 2 + )(2b0 + b1X + b2X 2 + ) = 4(a0b0 ) + 2(a0b1 + a1 b0)X + (2a0b2 + a1 b1 + 2a2b0 )X 2 + implicara a0 = b0 = 1 y por tanto a1 + b1 = 1, por lo que a1b1 sera par, y en consecuencia lo sera 1 = 2a0b2 + a1b1 + 2a2 b0, lo que supone una contradiccion. Por tanto hay que ser algo mas sutil en la denicion del producto de ideales. De nicion 2.5.10 Sea A un anillo con ideales I y J . El producto de ambos, denotado IJ , es el ideal de A generado por los elementos de la forma ab con a 2 I y b 2 J . 2.6. HOMOMORFISMOS 47 Ejercicio 2.5.11 Sean I J K I1 : : : Ir ideales de un anillo A. Demostrar: 1. El ideal producto IJ consiste en las sumas nitas de elementos de la forma ab con a 2 I y b 2 J . Es decir, n X IJ = f i=1 ai bj : n 2 Z i 2 I bi 2 J g: + a 2. Si I = (a) es un ideal principal, entonces IJ = fab : b 2 J g. En particular, si I = (a) y J = (b) son principales, entonces IJ = (ab) es principal. 3. Se verica IJ  I \ J , y la igualdad se da si I + J = A. 4. El producto de ideales es asociativo es decir, I(JK) = (IJ)K . Esto permite denir sin ambiguedad el producto de un numero nito de ideales, I1 Ir , y se verica I1 n X Ir = f i=1 ai1 air : n 2 Z ij 2 Ij g: + a 2.6 Homomorsmos En Matematicas se suele llamar homomorsmo entre dos objetos A y B con una estructura a una aplicacion f : A ! B que conserva la estructura. Por supuesto, la frase es bastante ambigua porque no hemos dicho que signica la palabra \estructura" ni la expresion \conservar la estructura", y en cada caso hemos de dar una denicion explcita. De nicion 2.6.1 Sean A y B dos anillos. Un homomorsmo de anillos entre A y B es una aplicacion f : A ! B que conserva las operaciones y la unidad es decir, que satisface f(x + y) = f(x) + f(y) f(x y) = f(x) f(y) para cada par de elementos x y 2 A y f(1) = 1: Un endomorsmo de A es un homomorsmo de un anillo de A en s mismo. En la denicion anterior hemos usado el mismo smbolo para las operaciones y los neutros en los dos anillos que intervienen. Por ejemplo, para calcular f(x + y) primero hay que sumar x con y en A y luego aplicarle f al resultado, mientras que en f(x) + f(y) primero hay que calcular las imagenes de x e y por f y luego hay que sumar estas en B. Usualmente el contexto hace evidente a que operacion o a que neutro nos referimos en cada caso, as que mantendremos estos abusos de notacion y dejaremos que el lector analice cada caso. Analogamente, las unidades de la ecuacion f(1) = 1 estan en dos anillos probablemente diferentes y por tanto son objetos distintos, que sin embargo denotamos igual. La condicion f(x + y) = f(x) + f(y) suele leerse como la imagen de la suma es la suma de las imagenes, o tambien como f conserva sumas. Del mismo modo se habla de aplicaciones que conservan productos o que conservan identidades. A continuacion establecemos ciertas propiedades elementales de los homomorsmos de anillos. Demostramos algunas y dejamos el resto como ejercicio para el lector. Proposicion 2.6.2 Sea f : A ! B un homomorsmo de anillos. Entonces se verican las siguientes propiedades para a b a1 : : : an 2 A: 1. (f conserva ceros) f(0) = 0. 2. (f conserva opuestos) f(;a) = ;f(a). 3. (f conserva restas) f(a ; b) = f(a) ; f(b). 4. (f conserva sumas nitas) f(a1 + + an ) = f(a1 ) + + f(an ). 5. (f conserva multiplos enteros) Si n 2 entonces f(na) = nf(a). Z CAPITULO 2. ANILLOS 48 (f conserva inversos) Si a es invertible, entonces f(a) es invertible y f(a);1 = f(a;1 ). (f conserva productos nitos) f(a1 an) = f(a1 ) f(an ). Si A1 es un subanillo de A, entonces f(A1 ) es un subanillo de B . Si B1 es un subanillo de B , entonces f ;1 (B1 ) es un subanillo de A. Si X es un ideal de B , entonces f ;1 (X) es un ideal de A. Si I es un ideal de A y f es suprayectiva, entonces f(I) es un ideal de B . Demostracion. Para ver 1 basta con aplicar la propiedad de cancelacion a la igualdad 0 + f(0) = f(0 + 0) = f(0) + f(0). 2 se tiene porque f(a) + f(;a) = f(a + (;a)) = f(0) = 0, y entonces 3 es claro. 4 se demuestra por induccion el caso n = 2 no es mas que la denicion de homomorsmo y el caso general se reduce a este notando que a1 + + an = (a1 + + an;1) + an. 6. 7. 8. 9. 10. 11. Ejercicio 2.6.3 Comprobar que la hipotesis de suprayectividad en el apartado 11 de la Proposicion 2.6.2 no es superua es decir, dar un ejemplo de un homomorsmo de anillos f : A ! B y un ideal I de A, tal que f(I) no es un ideal de B . Observacion 2.6.4 En la Proposicion 2.6.2 hemos visto que la conservacion de sumas implica la con- servacion del neutro para la suma, pero no hemos podido adaptar la demostracion al caso de productos (>por que?) de hecho, en seguida veremos ejemplos de aplicaciones entre anillos que conservan sumas y productos pero no identidades. Ejemplos 2.6.5 Homomorsmos de anillos. 1. Si A y B son anillos, la aplicacion f : A ! B dada por f(a) = 0 para cada a 2 A no es un homomorsmo de anillos salvo que B = 0. Este homomorsmo se llama homomorsmo cero u homomorsmo trivial. Observese que no hay ningun homomorsmo 0 ! B, salvo que B sea 0. He aqu otro ejemplo menos trivial de anillos entre los que no hay homomorsmos: la existencia de un homomorsmo de anillos f : 2 ! 3 nos llevara al absurdo 0]3 = f(0]2 ) = f(1]2 + 1]2) = f(1]2 ) + f(1]2 ) = 1]3 + 1]3 = 2]3: ZZ 2. Sea A un un anillo con un subanillo B. Entonces la aplicacion de inclusion u : B ,! A, dada por u(b) = b, es un homomorsmo. En particular, la aplicacion identidad 1A : A ! A es un homomorsmo. 3. Sea A un anillo con un ideal I. Entonces la proyeccion (o proyeccion canonica) : A ! A=I, dada por (a) = a+I, es un homomorsmo. Observese que parte de la demostracion del Teorema de la Correspondencia puede verse como un caso particular de la Proposicion 2.6.2 aplicada a la proyeccion . 4. Si A es un anillo, la aplicacion  : ! A dada por (n) = n1 (es decir, la aplicacion consistente en multiplicar por 1) es un homomorsmo de anillos. De hecho, es el unico homomorsmo de anillos f : ! A (>por que es el unico?). 5. Si A y B son anillos, la aplicacion pA : A  B ! A dada por pA (a b) = a es un homomorsmo llamado proyeccion en la primera coordenada, y de modo analogo se tiene una proyeccion en la segunda coordenada. Dado un producto arbitrario de anillos, debe estar claro como se generaliza este ejemplo para denir la proyeccion en cada coordenada. 6. Dados a b 2 , el conjugado del numero complejo z = a + bi es z% = a ; bi, y la aplicacion conjugacion ! dada por z 7! z% es un homomorsmo de anillos. p Analogamente, si d es unpentero quepno sea un cuadrado entonces el conjugado p p a ; b pd de a +pb d (elementos de  d] o de  d]) esta bien denido y la conjugacion  d] !  d] o p  d] !  d] es un homomorsmo de anillos. Z Z Z CR C Z Q Z Q Q 2.7. ISOMORFISMOS Y TEOREMAS DE ISOMORFIA 49 7. Sea A un anillo y sea b 2 A. Entonces la aplicacion AX] P = a0 + a1X + Sb ;! A 7! P(b) = a0 + a1b + + an X n + an bn es un homomorsmo de anillos llamado homomorsmo de sustitucion en b. En particular, el homomorsmo de sustitucion en 0 lleva cada polinomio a su coeciente independiente. Hemos visto que, si f : A ! B es un homomorsmo de anillos, entonces f(A) es un subanillo de B, y es evidente que f es suprayectivo precisamente cuando f(A) = B. Mas generalmente, podemos decir que cuanto mayor es f(A) mas cerca esta f de ser suprayectivo. En el otro extremo, f ;1 (0) es un ideal de A que nos va a servir para determinar si f es o no inyectivo. De nicion 2.6.6 Sea f : A ! B un homomorsmo de anillos llamamos imagen y nucleo de f , respectivamente, a los conjuntos Im f = f(A) = ff(a) : a 2 Ag Ker f = f ;1 (0) = fa 2 A : f(a) = 0g (la notacion para el nucleo procede de la voz germanica Kernel). En general Im f es un subanillo de B y Ker f es un ideal de A. Proposicion 2.6.7 Un homomorsmo de anillos f : A ! B es inyectivo precisamente si Ker f = 0. Demostracion. Si f es inyectivo, entonces f ;1 (a) tiene a lo sumo un elemento, para todo a 2 A. En particular Ker f = f ;1 (0) tiene exactamente un elemento, a saber 0. Recprocamente, supongamos que Ker f = 0 y sean a b 2 A tales que a 6= b. Entonces f(a) ; f(b) = f(a ; b) 6= 0 es decir, f(a) 6= f(b), y por tanto f es inyectiva. 2.7 Isomorsmos y Teoremas de Isomorfa Ejercicio 2.7.1 Demostrar que, si f : A ! B y g : B ! C son homomorsmos de anillos, entonces su composicion g  f : A ! C es tambien un homomorsmo. Ademas se verican los siguientes enunciados (que de hecho son ciertos para aplicaciones entre conjuntos, no solo para homomorsmos de anillos): 1. Si f y g son ambas inyectivas entonces g  f es inyectiva. 2. Si f y g son ambas suprayectivas entonces g  f es suprayectiva. 3. Si f y g son ambas biyectivas entonces g  f es biyectiva. 4. Si g  f es inyectiva entonces f es inyectiva. 5. Si g  f es suprayectiva entonces g es suprayectiva. De nicion 2.7.2 Un homomorsmo de anillos f : A ! B que sea biyectivo se llama un isomorsmo de anillos. Un automorsmo es un endomorsmo biyectivo. Si existe un isomorsmo f : A ! B , se  B. dice que los anillos A y B son isomorfos, situacion que se denota por A = Conforme vayamos estudiando propiedades de los anillos y de sus elementos, veremos que los isomorsmos conservan esas propiedades en un sentido que estara claro en cada caso. Como consecuencia, dos anillos isomorfos tendran las mismas propiedades y deberan ser considerados como esencialmente iguales. Proposicion 2.7.3 Si f : A ! B es un isomorsmo de anillos, entonces la aplicacion inversa f ;1 : B ! A tambien lo es. En consecuencia, la relacion ser isomorfos es una relacion de equivalencia en la clase de todos los anillos. CAPITULO 2. ANILLOS 50 Demostracion. Sean x y 2 B entonces f(f ;1 (x + y)) = x + y = f(f ;1 (x)) + f(f ;1 (y)) = f(f ;1 (x) + f ;1 (y)) y como f es inyectiva esto implica que f ;1 (x + y) = f ;1 (x) + f ;1 (y). De igual modo se ve que f ;1 conserva productos e identidades. Como ademas f ;1 es biyectiva, deducimos que es un isomorsmo. Como las identidades son isomorsmos, la relacion de isomorfa es reexiva, mientras que es simetrica por la primera parte de esta proposicion y es transitiva por el Ejercicio 2.7.1. En el Captulo 1 hemos introducido los numeros enteros de forma axiomatica. Es decir, por denicion, los numeros enteros forman un objeto que satisface una lista de axiomas. >Cuantos conjuntos de numeros enteros hay? Es decir, >cuantos objetos hay que satisfagan los axiomas de la Seccion 1.1? En realidad hay muchos. En efecto, si f : A ! es una aplicacion biyectiva y denimos en A las operaciones a + b = f ;1 (f(a) + f(b)) ab = f ;1 (f(a) f(b)) y el orden a b , f(a) f(b) entonces es facil ver que A satisface los axiomas de la Seccion 1.1 y por tanto es un conjunto de numeros enteros. En tal caso, >que nos permite hablar de el conjunto de los numeros enteros utilizando el artculo determinado? En este momento debe estar claro que la respuesta a esta pregunta es que todos los conjuntos que satisfacen los axiomas de la Seccion 1.1 son isomorfos. Esto es consecuencia del apartado 4 del Ejemplo 2.6.5. E ste es un modo tpico de trabajo con el metodo axiomatico: se dene un objeto matematico en funcion de sus propiedades basicas y eventualmente se prueba que dicho objeto es \esencialmente unico" es decir, que todos los objetos que satisfacen dichas propiedades son isomorfos. Veremos otro ejemplo de este modo de actuar al nal de la Seccion 2.8. Los siguientes resultados establecen la existencia de ciertos isomorsmos de anillos que usaremos con frecuencia. Z Teorema 2.7.4 (Primer Teorema de Isomorfa) Sea f : A ! B un homomorsmo de anillos. Entonces existe un unico isomorsmo de anillos f% : A=Ker f ! Im f que hace conmutativo el diagrama A p ? A=Ker f f - f% - Im f B 6i es decir, i  f%  p = f , donde i es la inclusion y p es la proyeccion. En particular A  Ker f = Im f: % + K) = f(x) Demostracion. Sean K = Ker f e I = Im f. La aplicacion f% : A=K ! I dada por f(x esta bien denida (no depende de representantes) pues si x+K = y+K entonces x ; y 2 K y por lo tanto f(x) ; f(y) = f(x ; y) = 0 es decir, f(x) = f(y). Ademas es elemental ver que es un homomorsmo de anillos y que es suprayectiva. Para ver que es inyectiva usamos la Proposicion 2.6.7: si x + K esta % + K) = f(x), de modo que x 2 K y as x + K = 0 + K. Es decir en el nucleo de f% entonces 0 = f(x % % Ker f = 0 y por lo tanto f es inyectiva. En conclusion, f% es un isomorsmo, y hace conmutativo el diagrama porque, para cada x 2 K, se tiene % + K) = f(x): i(f%(p(x))) = f(x En cuanto a la unicidad, supongamos que otro homomorsmo fb : A=K ! I verica i  fb  p = f b + K) = i(f(p(x))) b % + K), y por lo tanto fb = f.% entonces para cada x 2 K se tiene f(x = f(x) = f(x 2.7. ISOMORFISMOS Y TEOREMAS DE ISOMORFIA 51 Teorema 2.7.5 (Segundo Teorema de Isomorfa) Sea A un anillo y sean I y J dos ideales tales que I  J . Entonces J=I es un ideal de A=I y existe un isomorsmo de anillos A=I  A: = J=I J Demostracion. Por el Teorema de la Correspondencia 2.4.8, J=I es un ideal de A=I. Sea f : A=I ! A=J la aplicacion denida por f(a + I) = a + J. Es elemental ver que f esta bien denida, que es un homomorsmo suprayectivo de anillos y que Ker f = J=I. Entonces el isomorsmo buscado se obtiene aplicando el Primer Teorema de Isomorfa a f. Teorema 2.7.6 (Tercer Teorema de Isomorfa) Sea A un anillo con un subanillo B y un ideal I . Entonces: 1. B \ I es un ideal de B . 2. B + I es un subanillo de A que contiene a I como ideal. 3. Se tiene un isomorsmo de anillos B  = B +I. B \I I Demostracion. Los dos primeros apartados se dejan como ejercicio. En cuanto al ultimo, sea f : B ! A=I la composicion de la inclusion j : B ! A con la proyeccion p : A ! A=I. Es claro que Ker f = B \ I y que Im f = (B + I)=I, por lo que el resultado se sigue del Primer Teorema de Isomorfa. Ejemplos 2.7.7 Aplicaciones del Primer Teorema de Isomorfa. 1. Si A y B son anillos, el homomorsmo A  B ! A de proyeccion en la primera componente es suprayectivo y tiene nucleo I = 0  B, por lo que A0BB  = A. 2. Si A es un anillo, el homomorsmo f : AX] ! A de sustitucion en 0 (dado por a0 +a1 X+ 7! a0 ) es suprayectivo y tiene por nucleo el ideal (X) generado por X (consistente en los polinomios con  A, como ya habamos observado en los coeciente independiente nulo), de modo que AX]=(X) = Ejemplos 2.4.6. 3. Sean A un anillo e I un ideal de A. Para cada a 2 A, sea a = a + I. La aplicacion f : AX] ! (A=I)X] dada por f(a0 + a1 X + + an X n ) = a0 + a1 X + + an X n es un homomorsmo suprayectivo de anillos cuyo nucleo es IX] = fa0 + a1X + + an X n : ai 2 I g. Del Primer Teorema de Isomorfa se deduce que (A=I)X]  = AX]=IX]. Z Z Z De nicion 2.7.8 Sea A un anillo, y recordemos que si n 2 + escribimos n1 = 1 + + 1 (n veces). Si existe n 2 + tal que n1 = 0, denimos la caracterstica de A como el menor n 2 + que verica Z tal igualdad. Si no existe un tal n, decimos que la caracterstica de A es 0. Proposicion 2.7.9 Sea A un anillo A y sea f : ! A el unico homomorsmo de anillos (dado por f(n) = n1). Para un numero natural n las condiciones siguientes son equivalentes: Z 1. n es la caracterstica de A. 2. n es el nucleo de f . Z Z 3. El subanillo primo de A es isomorfo a n (recuerdese que 4. A contiene un subanillo isomorfo a n. Z ZZ 0= y 1 = 0). CAPITULO 2. ANILLOS 52 Demostracion. La equivalencia entre 1 y 2 se deja como ejercicio para el lector, y es obvio que 3 implica 4. 2 implica 3. Se obtiene aplicando el Primer Teorema de Isomorfa y observando que Im f es el subanillo primo de A. 4 implica 2. Si B es un subanillo de A y g : n ! B es un isomorsmo, considerando la proyeccion : ! n y la inclusion u : B ,! A se obtiene un homomorsmo de anillos u  g  : ! A que debe coincidir con f por su unicidad (Ejemplos 2.6.5). Como u  g es inyectiva, es elemental ver que Ker f = n . ZZ Z Z Z Terminamos esta seccion generalizando el Teorema Chino de los Restos (1.7.4) a anillos arbitrarios. Teorema 2.7.10 (Teorema Chino de los Restos) Sea A un anillo y sean I1  : : :  In ideales de A tales que Ii + Ij = A para todo i 6= j . Entonces I1 \ \ In = I1 In . Ademas A A A I1 \ \ In = I1   In : Demostracion. Razonamos por induccion sobre n, empezando con el caso n = 2. La hipotesis I1 + I2 = A nos dice que existen x1 2 I1 y x2 2 I2 tales que x1 + x2 = 1, y entonces para cada a 2 I1 \ I2 se tiene a = ax1 + ax2 2 I1 I2 , de modo que I1 \ I2  I1 I2 , y la otra inclusion es clara. Claramente la aplicacion f : A ! A=I1  A=I2 dada por f(a) = (a + I1  a + I2 ) es un homomorsmo de anillos con nucleo I1 \ I2 , y es suprayectiva pues, dado un elemento arbitrario (a1 + I1 a2 + I2 ) de A=I1  A=I2 , el elemento a = a1x2 + a2 x1 verica f(a) = (a1 + I1  a2 + I2 ). Ahora el resultado se obtiene aplicando el Primer Teorema de Isomorfa. En el caso general (n > 2) basta ver que las hipotesis implican que (I1 \ \ In;1 ) + In = A, pues entonces la hipotesis de induccion nos da I1 \ \ In;1 \ In = (I1 \ \ In;1)In = I1 In;1In y A A A A A = nA;1  IA  ;1Ii ) \ In  I1 \ \ In = (\ni=1 \i=1 Ii n = I1   In;1  In : Para ver que (I1 \ \ In;1) + In = A notemos que, para cada i n ; 1, existen ai 2 Ii y bi 2 In tales que 1 = ai + bi, y multiplicando todas esas expresiones se obtiene ;1(ai + bi ) = a1 an;1 + b 1 = &ni=1 donde b engloba a todos los sumandos que se obtendran desarrollando los productos (excepto a1 an;1) y esta en In porque en cada sumando hay al menos un factor del ideal In. Como ademas a1 an;1 2 I1 \ \ In;1, deducimos que 1 2 (I1 \ \ In;1)+ In y as (I1 \ \ In;1)+In = A, como queramos ver. 2.8 Cuerpos y dominios ideales maximales y primos En esta seccion suponemos que, en todos los anillos que aparecen, el 1 es distinto del 0. De nicion 2.8.1 Un elemento a de un anillo A se dice regular si la relacion ab = ac con b c 2 A implica que b = c es decir, si a es cancelable para el producto en el sentido de la Denicion 2.1.2. Claramente, el 0 nunca es cancelable7 . Un cuerpo es un anillo en el que todos los elementos no nulos son invertibles, y un dominio (o dominio de integridad) es un anillo en el que todos los elementos no nulos son regulares. Como todo elemento invertible es regular (Ejercicio 2.1.4), tenemos: Proposicion 2.8.2 Todo cuerpo es un dominio. 7 Observese la importancia de la hipotesis 1 6= 0 en este caso. 2.8. CUERPOS Y DOMINIOS IDEALES MAXIMALES Y PRIMOS 53 Otras propiedades que se demuestran facilmente quedan recogidas en el siguiente ejercicio: Ejercicio 2.8.3 Si A es un anillo, demostrar que: 1. Las condiciones siguientes son equivalentes (usar el Ejercicio 2.4.7, la Proposicion 2.6.7 y las proyecciones A ! A=I ): (a) A es un cuerpo. (b) Los unicos ideales de A son 0 y A. (c) Todo homomorsmo de anillos A ! B es inyectivo. 2. Un elemento a 2 A es regular si y solo si la relacion ab = 0 con b 2 A implica b = 0 (por este motivo, los elementos no regulares se suelen llamar divisores de cero). 3. A es un dominio si y solo si, para cualesquiera a b 2 A no nulos, se tiene ab 6= 0. 4. Todo subanillo de un dominio es un dominio. 5. La caracterstica de un dominio es cero o un numero primo. QR C Ejemplos 2.8.4 Dominios y cuerpos. Z ZQ 1. Los anillos , y son cuerpos y es un dominio que no es un cuerpo (aunque es subanillo de un cuerpo). 2. Para n 2, el anillo n es un dominio si y solo si es un cuerpo, si y solo si n es primo (>por que?). pm] es un dominio (subanillo de ) que no es un cuerpo 3. Si m 2 no es un cuadrado entonces  p (el 2 no tiene inverso). Sin embargo,  m] s que es un cuerpo de hecho, si a + bpm 6= 0, p p entonces q =p (a + b m)(a ; b m) =p a2 ; b2m es un numero racional no nulo (>por que?) y aq;1 ; bq;1 m es el inverso de a + b m. 4. Un producto de anillos A  B nunca es un dominio, pues (1 0)(0 1) = (0 0). 5. Los anillos de polinomios no son cuerpos, pues la indeterminada genera un ideal propio y no nulo. Por otra parte, AX] es un dominio si y solo si lo es A. Una implicacion es clara, pues A es un subanillo de AX]. La otra se sigue del siguiente resultado, interesante en s mismo, que el lector puede intentar demostrar (vease el Ejercicio 4.1.2): Si A es un dominio y P Q son polinomios de AX] de grados n y m, entonces el grado del producto PQ es n + m. Z Z De nicion 2.8.5 Sean A un anillo e I un ideal propio de A. C Se dice que I es maximal si no esta contenido en ningun ideal propio (excepto en s mismo). Se dice que I es primo si, para todo a b 2 A, la relacion ab 2 I implica a 2 I o b 2 I . Z De la Proposicion 1.5.1 se deduce que los ideales maximales de son los ideales principales generados por numeros primos, y sus ideales primos son los anteriores junto con el ideal trivial 0. Proposicion 2.8.6 Sean A un anillo e I un ideal propio de A. Entonces: 1. I es maximal precisamente si A=I es un cuerpo. 2. I es primo precisamente si A=I es un dominio. 3. Si I es maximal entonces es primo. 4. A es un cuerpo precisamente si el ideal 0 es maximal. 5. A es un dominio precisamente si el ideal 0 es primo. CAPITULO 2. ANILLOS 54 Demostracion. El apartado 1 es consecuencia inmediata del primer apartado del Ejercicio 2.8.3 y del Teorema de la Correspondencia 2.4.8. Para ver 2, supongamos que I es primo y sean a + I b + I dos elementos no nulos de A=I entonces a b 62 I y por lo tanto ab 62 I, luego (a + I)(b + I) = ab + I = 6 0 y en consecuencia A=I es un dominio. El recproco se demuestra usando la misma idea, y el resto de apartados se deducen de estos dos y de la Proposicion 2.8.2. A continuacion estudiamos algunas propiedades de los ideales primos y maximales. Ejercicio 2.8.7 Si I es un ideal propio del anillo A, las biyecciones del Teorema de la Correspondencia llevan ideales maximales (respectivamente primos) de A que contienen a I a ideales maximales (respectivamente primos) de A=I , y viceversa. (Indicacion: Usar la Proposicion 2.8.6 y el Segundo Teorema de Isomorfa.) Ejercicio 2.8.8 Sea f : A ! B un homomorsmo de anillos. Demostrar que: 1. Si p es un ideal primo de B , entonces f ;1 (p) es un ideal primo de A. 2. En general, no se verica el resultado analogo para ideales maximales. (Indicacion: Considerar la inclusion de en .) ZQ La siguiente proposicion proporciona nueva una caracterizacion de los ideales primos. Proposicion 2.8.9 Un ideal P de un anillo A es primo precisamente si para cada dos ideales I y J de A, si IJ  P entonces I  P o J  P . Demostracion. Supongamos que P no es primo. Entonces existen a b 2 A n P tales que ab 2 P, de modo que el ideal (a)(b) = (ab) esta incluido en P mientras que (a) y (b) no lo estan. Recprocamente, supongamos que P es un ideal primo y que I y J son ideales de A no contenidos en P . Entonces existen a 2 I n P y b 2 J n P , por lo que ab 2 IJ n P y as IJ 6 P. De la Proposicion 2.8.9 y de la inclusion IJ  I \ J se deduce: Corolario 2.8.10 Sea P un ideal primo de un anillo A y sean I y J ideales de A tales que I \ J  P . Entonces I  P o J  P . El recproco del Corolario 2.8.10 no se verica. Por ejemplo, el ideal (4) de no es primo. Sin embargo para cada dos ideales I y J de tales que I \ J  (4) se verica que I  (4) o J  (4). Z Z Terminamos la seccion con un resultado que se sale de los objetivos de este curso, pero que incluimos aqu por su importancia en Teora de Anillos. Se trata del hecho, nada evidente, de que todo anillo posee ideales maximales. Para demostrar este tipo de resultados \de existencia" suele ser util el siguiente Lema de Zorn, que es un resultado fundamental de Teora de Conjuntos. Por desgracia, su demostracion requiere herramientas de las que carecemos en este momento. Necesitamos ademas una denicion previa. Un conjunto ordenado (A ) se dice que es inductivo si toda cadena (o subconjunto linealmente ordenado) B de A posee una cota superior en A. Teorema 2.8.11 (Lema de Zorn) Todo conjunto ordenado no vaco e inductivo tiene un elemento maximal. Teorema 2.8.12 Todo ideal propio J de un anillo A esta contenido en un ideal maximal de A. En particular, tomando J = 0, todo anillo tiene al menos un ideal maximal. Demostracion. Sea X el conjunto de los ideales propios de A que contienen a J, ordenado por inclusion. Como X no es vaco (J es un elemento suyo), si vemos que es inductivo el Lema de Zorn nos dira que X posee un elemento maximal, que claramente sera el ideal postulado en el enunciado. Sea pues fIt gt2T una cadena de X, y sea I = t2T It . Vamos a demostrar que I esta en X, lo que acabara demostracion ya que I es claramente una cota superior del conjunto fIt gt2T . Se trata pues de ver que I es un ideal propio de A que contiene a J. Es obvio que I contiene a J, y tambien es claro que 1 62 I pues los It son ideales propios. Solo queda pues ver que I es un ideal. Claramente I 6= , y si a a0 2 I entonces existen t t0 2 T tales que a 2 It y a0 2 It . Como fItgt2T esta linealmente ordenado, podemos suponer que It  It, y entonces cualquier combinacion lineal de a y a0 esta en It y por tanto en I. Esto prueba que I es un ideal y termina la demostracion. 0 0 2.9. EL CUERPO DE FRACCIONES DE UN DOMINIO 55 2.9 El cuerpo de fracciones de un dominio En esta seccion vamos a ver que todo dominio D es un subanillo de un cuerpo. De hecho existe un cuerpo que, en cierto sentido, es el menor cuerpo que contiene a D. Dicho cuerpo es unico salvo isomorsmos y se llama el cuerpo de fracciones de D. Comenzaremos con la construccion de ese cuerpo, que es una traduccion literal de la construccion de a partir de , y analizaremos entonces sus propiedades. En realidad, la construccion del cuerpo de fracciones es parte de una construccion mas general, que presentaremos al nal de la seccion en una serie de ejercicios. En esta seccion, D representara un dominio. Un subanillo de un anillo A que sea un cuerpo se llama un subcuerpo de A, y un homomorsmo de anillos entre dos cuerpos (que ha de ser inyectivo por el Ejercicio 2.8.3) se llama homomorsmo de cuerpos, y tambien extension de cuerpos. Q Z La idea de la construccion es la de formar un cuerpo Q(D) cuyos elementos sean \fracciones" del tipo a=b con a b 2 D y b 6= 0. De este modo, D estara contenido en Q(D) (identicando cada elemento a de D con la fraccion a=1), y los elementos no nulos de Q(D) seran invertibles, pues b=a sera el inverso de a=b. Por supuesto, hay que denir con mas rigor las fracciones y hay que dotar a Q(D) de una estructura de cuerpo. El primer problema que se presenta, si pensamos en el caso D = y Q(D) = , es el hecho de que dos fracciones aparentemente distintas pueden representar el mismo elemento, como en el caso 10=15 = 2=3. Esto se resuelve identicando ciertas fracciones mediante una relacion de equivalencia, y este sera el primer paso en nuestra construccion. Sean S = D n f0g y X = D  S. Denimos en X la relacion binaria Z Q (a1  s1 )  (a2  s2) , a1 s2 = a2 s1 que, como el lector comprobara facilmente, es una relacion de equivalencia. La clase de equivalencia de (a s) se denota por a=s o por as , y el conjunto cociente X=  (es decir, el conjunto de las clases de equivalencia para esa relacion) por Q(D). Dotamos a Q(D) de una estructura de anillo con las siguientes operaciones: a1 + a2 = a1s2 + a2 s1 a1 a2 = a1a2 : (2.9.1) s1 s2 s1 s2 s1 s2 s1 s2 Hay que asegurarse de que esas deniciones no dependen de los representantes elegidos para cada fraccion. Es decir, si a1=s1 = b1=t1 y a2 =s2 = b2=t2 , hay que comprobar que se obtiene la misma suma y el mismo producto si aplicamos las formulas a a1 =s1 y a2 =s2 que si se las aplicamos a b1 =t1 y b2=t2. Las igualdades anteriores signican que a1 t1 = b1s1 y a2 t2 = b2s2 , de donde (a1 s2 + a2 s1 )(t1 t2) = a1s2 t1 t2 + a2 s1 t1t2 = b1 s2 s1 t2 + b2 s1 t1 s2 = (b1 t2 + b2t1 )(s1 s2 ) y por tanto a1 ss21+sa22 s1 = b1 t2t1+tb22 t1 . Esto demuestra que la suma esta bien denida, y con el producto se procede de modo analogo. Ejercicio 2.9.1 Dados a b s t 2 D con s t 6= 0, demostrar que: 1. 2. 3. 4. 5. El neutro para la suma es 0=1. Ademas, la igualdad a=s = 0=1 se verica si y solo si a = 0. El neutro para el producto es 1=1. Ademas, la igualdad a=s = 1=1 se verica si y solo si a = s. Se tiene at=st = a=s. La igualdad a=s = b=s se verica si y solo si a = b. La denicion de suma se simplica cuando hay \denominador comun": a=s + b=s = (a + b)=s. Usando adecuadamente el Ejercicio 2.9.1, la comprobacion de que Q(D) es un cuerpo es rutinaria. Demostramos como ejemplo la propiedad distributiva, y dejamos el resto para el lector:     a b1 + b2 = a b1t2 + b2t1 = ab1t2 + ab2 t1 = ab1t2 + ab2 t1 = ab1 + ab2 = a b1 + a b2 : s t1 t2 s t1t2 st1 t2 st1t2 st1 t2 st1 st2 s t1 s t2 CAPITULO 2. ANILLOS 56 De nicion 2.9.2 El cuerpo Q(D) se llama cuerpo de fracciones o cuerpo de cocientes del dominio D. Ejemplos 2.9.3 Cuerpos de fracciones. Q Z 1. Obviamente, es el cuerpo de fracciones de . 2. Supongamos que un anillo de polinomios AX] es un dominio (lo que ocurre precisamente si A es un dominio por los Ejemplos 2.8.4). Su cuerpo de fracciones se suele denotar por A(X) y se llama el cuerpo de las funciones racionales sobre A. Sus elementos son fracciones del tipo P=Q con P Q 2 AX], que se suman y se multiplican de forma natural. Usando el Ejercicio 2.9.1, es sencillo ver que la aplicacion u : D ! Q(D) dada por u(a) = a=1 es un homomorsmo inyectivo de anillos, lo que nos permite ver a D como un subanillo de Q(D) si identicamos cada elemento a de D con la fraccion a=1 de Q(D). El par (Q(D) u) verica una interesante propiedad: Proposicion 2.9.4 Sean D un dominio, Q(D) su cuerpo de fracciones y u : D ! Q(D) la aplicacion dada por u(a) = a=1. Entonces: 1. (Propiedad Universal del Cuerpo de Fracciones) Para toda pareja (K f) formada por un cuerpo K y un homomorsmo inyectivo de anillos f : D ! K , existe una unica extension de cuerpos f% : Q(D) ! K tal que f%  u = f . Se dice que f% completa de modo unico el diagrama D u ? Q(D) f qK 1 f 2. Si dos extensiones de cuerpos g h : Q(D) ! K coinciden sobre D entonces son iguales. Es decir, si g  u = h  u entonces g = h. 3. Q(D) esta determinado salvo isomorsmos por la Propiedad Universal. Explcitamente: supongamos que existen un cuerpo F y un homomorsmo inyectivo de anillos v : D ! F tales que, para todo cuerpo K y todo homomorsmo inyectivo de anillos f : D ! K , existe una unica extension de cuerpos f% : F ! K tal que f%  v = f . Entonces existe un isomorsmo : F ! Q(D) tal que  v = u. Demostracion. 1. Sea f : D ! K como en el enunciado. Si f% : Q(D) ! K es una extension de cuerpos tal que f%  u = f entonces, para todo a=s 2 Q(D), se verica ;1 ) = (f%  u)(a)(f%  u)(s);1 = f(a)f(s);1 : % % f(a=s) = f(u(a)u(s) Esto prueba que la unica extension de cuerpos f% : Q(D) ! K que puede satisfacer f%  u = f tiene % que venir dada por f(a=s) = f(a)f(s);1 . Solo falta comprobar que f% esta bien denido y es un homomorsmo. Si a1=s1 = a2=s2 entonces a1s2 = a2s1 , luego f(a1 )f(s2 ) = f(a2 )f(s1 ) y, por tanto, f(a1 )f(s1 );1 = f(a2 )f(s2 );1 . Esto prueba que f% esta bien denido. Dejaremos que el lector compruebe que es efectivamente un homomorsmo. 2. Si ponemos f = g  u = h  u : D ! K, los homomorsmos g y h completan el diagrama del apartado 1. Por la unicidad se tiene g = h. 3. Sea v : D ! F como en el enunciado. Aplicando 1 encontramos una extension v% : Q(D) ! F tal que v%  u = v, y aplicando la hipotesis de 3 encontramos una extension u% : F ! Q(D) tal que u%  v = u. Entonces la composicion u%  v% : Q(D) ! Q(D) verica (%u  v%)  u = u%  v = u, y por 2 se obtiene u%  v% = 1Q(D) . En particular u% es suprayectiva, y como es inyectiva por ser una extension de cuerpos, = u% es el isomorsmo que buscamos. La Propiedad Universal permite armar que Q(D) es \el menor cuerpo que contiene a D" en un sentido que se hace explcito en el siguiente resultado: 2.9. EL CUERPO DE FRACCIONES DE UN DOMINIO 57 Proposicion 2.9.5 Sea D un dominio. Si K es un cuerpo y f : D ! K es un homomorsmo inyectivo de anillos, entonces K contiene un subcuerpo isomorfo a Q(D). Demostracion. Por la propiedad universal del cuerpo de fracciones existe una extension de cuerpos f% : Q(D) ! K, y como f% es inyectiva, Im f% es un subcuerpo de K isomorfo a Q(D). Z Z C Z Z Q Q Q Z Q Z Ejemplo 2.9.6 El cuerpo de fracciones de pm]. Z p Sea m un numero entero que no es un cuadrado, y sea f :  m] ! la inclusion. Si f%pes como % en la demostracion de la Proposicion 2.9.5, entonces pm Im f es el cuerpo de fracciones p de  m]. Un a + b elemento generico de Im f% es de la forma x = c+dpm , con a b c d 2 y c + d m 6= 0. Si ponemos p p t = (c + d m)(c ; d m) 6= 0 entonces t = c2 ; d2m 2 , y as p (a + bpm)(c ; dpm) = r + spm = r + s pm = x = ca ++ dbpm m t t t t p y el otro contenido es donde r s 2 , y por tanto x 2 pm]. Esto demuestra que Im f%   m], p p at+bspm . claro, pues un elemento generico as + bt m de p m] se reescribe como st En conclusion, el cuerpo de fracciones de  m] es pm]. Z Un interesante corolario de la Proposicion 2.9.5 es el siguiente: Corolario 2.9.7 Todo cuerpo K posee un subcuerpo K 0 , llamado el subcuerpo primo de K , que esta Q contenido en cualquier otro subcuerpo de K (es decir, K 0 es \el menor subcuerpo de K "). Si la caracterstica de K es un entero primo p, entonces K 0 es isomorfo a p en caso contrario K 0 es isomorfo a . Z Demostracion. Si la caracterstica es un primo p entonces el subanillo primo de K (isomorfo a p) es Z ZQ Z Q ya un cuerpo, y contiene a cualquier subcuerpo (de hecho, a cualquier subanillo) de K. En otro caso, al ser K un cuerpo, la caracterstica es cero es decir, el homomorsmo de anillos f : ! K es inyectivo. El cuerpo de fracciones de es , y la extension de cuerpos f% : ! K que nos % da la Propiedad Universal viene dada por f(n=m) = f(n)f(m);1 . Como f% es inyectivo, K 0 = Im f% es un subcuerpo de K isomorfo a , y ahora basta ver que K 0 esta contenido en cualquier subcuerpo F de K. Dado un tal F , se tiene f(m) 2 F para cada m 2 , y si m 6= 0 entonces f(m) 6= 0 y f(m);1 2 F. % Por tanto, para cada n=m 2 se tiene f(n=m) = f(n)f(m);1 2 F , lo que demuestra que K 0  F. Q Q Como ya hemos dicho, la construccion del cuerpo de fracciones es parte de una construccion mas general: la del anillo de fracciones por un subconjunto multiplicativo. Presentaremos a continuacion este concepto, dejando los detalles a cargo del lector. Para mostrar otro enfoque (que puede adoptarse tambien para denir el cuerpo de fracciones), en lugar de construir primero el anillo de fracciones y demostrar despues que verica cierta propiedad universal, lo que hacemos es denir el anillo de fracciones como el que verica cierta propiedad, y despues lo construimos. De nicion 2.9.8 Un subconjunto multiplicativo de un anillo A es un subconjunto S cerrado para el producto y que contiene al 1 pero no al 0 es decir, 1 2 S , 0 62 S y si a b 2 S entonces ab 2 S . Ejercicio 2.9.9 Comprobar que los siguientes son subconjuntos multiplicativos de A. 1. El conjunto A de las unidades de A. 2. El conjunto de los elementos regulares de A. 3. El conjunto A n P , donde P es un ideal primo de A. N 4. El conjunto de los elementos no nulos de un dominio. 5. El conjunto fan : n 2 g de las potencias de un elemento regular a de A. CAPITULO 2. ANILLOS 58 De nicion 2.9.10 Sea S un subconjunto multiplicativo de un anillo A. Un anillo de fracciones de A por S es una pareja (B j) formada por un anillo B y un homomorsmo de anillos j : A ! B tal que j(S)  B  (es decir, j(s) es invertible en B para todo s 2 S ) y tal que se verica la siguiente propiedad: Si (C f) es una pareja formada por un anillo C y un homomorsmo de anillos f : A ! C tal que f(S)  C  , entonces existe un unico homomorsmo de anillos f% : B ! C tal que f%  j = f . Por ejemplo, si S = A entonces el par (A i), donde i : A ! A es la identidad, es un anillo de fracciones de A por S. Y si D es un dominio entonces un anillo de fracciones de D por S = D n f0g es lo mismo que un cuerpo de fracciones de D. Ejercicio 2.9.11 Demostrar que si (B j) y (B 0  j 0) son dos anillos de fracciones de A por S, entonces existe un unico isomorsmo de anillos f : B ! B 0 tal que f  j = j 0 . Ejercicio 2.9.12 Sea A un anillo y sea S un subconjunto multiplicativo de A. 1. Demostrar que la siguiente es una relacion de equivalencia en X = A  S : (a1  s1)  (a2 s2 ) , existe t 2 S tal que ta1s2 = ta2s1 : Denotamos por a=s la clase de equivalencia de (a s) 2 X y por AS ;1 el conjunto de las clases de equivalencia. 2. Demostrar que las formulas (2.9.1) proporcionan operaciones bien denidas en AS ;1 que lo dotan de una estructura de anillo. 3. Demostrar que la aplicacion j : A ! AS ;1 dada por j(a) = a=1 es un homomorsmo de anillos y que j(s) es invertible en AS ;1 para todo s 2 S . 4. Demostrar que (AS ;1  j) es un anillo de fracciones de A por S . 2.10 Problemas Z 1. Decir cuales de los siguientes conjuntos son anillos con las operaciones que se describen: (a) El conjunto de los numeros enteros con las siguientes operaciones: a  b = a + b + 1 a  b = ab + a + b: (b) El conjunto 2X de los subconjuntos de un conjunto X con las operaciones de union e interseccion. >Y si se cambian los papeles de forma que la interseccion represente la suma y la union el producto? (c) El mismo conjunto 2X , tomando como suma la diferencia simetrica, A+B = (A n B)  (B n A), y como producto la interseccion. (d) El conjunto Rde las funciones reales de variable real con las operaciones: (f + g)(x) = f(x) + g(x) y (fg)(x) = f(x)g(x) (8x 2 ): R R R (e) El conjunto Ranterior con la misma suma y el producto dado por la composicion de funciones. >Y si limitamos el conjunto a las funciones que conserven la suma? 2. Indicar si son ciertas, en general, las siguientes relaciones en un anillo A (no necesariamente conmutativo): (a) a2 ; b2 = (a + b)(a ; b). (b) (a + b)3 = a3 + aba + ba2 + b2a + a2b + ab2 + bab + b3 . (c) am an = am+n . (d) (ab)m = am bm . 2.10. PROBLEMAS 59 3. Sea f : A ! B un isomorsmo de anillos. Demostrar que: (a) Un elemento a 2 A es cancelable en A si y solo si f(a) es cancelable en B. (b) Un elemento a 2 A es invertible en A si y solo si f(a) es invertible en B. (c) Un subconjunto I de A es un subanillo de A si y solo si f(I) es un subanillo de B. (d) Un subconjunto I de A es un ideal de A si y solo si f(I) es un ideal de B. (e) Un ideal I de A es principal si y solo si el ideal f(I) de B es principal. (f) Un ideal I de A es maximal si y solo si el ideal f(I) de B es maximal. (g) Un ideal I de A es primo si y solo si el ideal f(I) de B es primo. (h) Un subconjunto X de A genera el ideal I si y solo si el subconjunto f(X) de B genera el ideal f(I). (i) A y B tienen la misma caracterstica. (j) A es un cuerpo si y solo si B es un cuerpo. (k) A es un dominio si y solo si B es un dominio. 4. Calcular las unidades del anillo i]. 5. Sean a y b dos elementos de un anillo. Demostrar que ab es un divisor de cero precisamente si a o b es un divisor de cero. 6. Sea A un anillo nito. Demostrar que todo elemento de A es o divisor de cero o unidad. Deducir que todo dominio nito es un cuerpo. (Indicacion: Considerar la aplicacion x 7! ax.) 7. Demostrar que todo anillo con tres elementos es un cuerpo. 8. Calcular los divisores de cero y las unidades del anillo Rdel apartado 1d del Problema 1. 9. Encontrar todos los divisores de cero en los anillos: 4 10    . 10. Un elemento e de un anillo A es idempotente si verica e2 = e. Demostrar que la suma y el producto de A convierten al subconjunto eA = fea : a 2 Ag en un anillo. >Cual es su elemento identidad? >Es eA un subanillo de A? 11. Decimos que d 2 es libre de cuadrados si p2 no divide a d para ningun numero primo p (en particular 1 es libre de p Demostrar que para todo m 2 existe un d 2 libre de p cuadrados). cuadrados tal que  m] =  d]. >Ocurre lo mismo si cambiamos por ? Z RZ Z Z Z R R Z Q Q ZQ CR R Q C Z p ZZ Z Q Z Z R Z 12. Sea d 2 librepde cuadrados y sea  = 1+2 d . Demostrar que ] = fa + b : a b 2 g es un subanillo de  d] precisamente si d  1 mod 4. 13. En cada apartado se describen un anillo A y un subconjunto B. Decir cuales de los subconjuntos son subanillos: (a) A es el cuerpo y B = i consiste en los numeros de la forma ri con r 2 . (b) A es el anillo Rdel Problema 1d y B es el subconjunto de las funciones continuas. (c) A es el cuerpo racional , q es un entero libre de cuadrados y B es el subconjunto (q) de los numeros a=b con mcd(q b) = 1. >Que ocurre si q no es libre de cuadrados? (d) A es el anillo RX] de polinomios con coecientes en un anillo R, y B es el conjunto de los polinomios de grado menor o igual que n, donde n 2 +. (e) En el cuerpo A = consideramos, dado un entero primo p, una raz p-esima primitiva de la unidad  (es decir,  p = 1 y  n 6= 1 para n = 1 : : :  p ; 1 por ejemplo,  = e2i=p = cos(2 =p) + sen (2 =p)i). B se dene como el subconjunto B = ] = fa0 + a1 + a2 2 + Z Z + ap;1  p;1 : a0 a1 : : :  ap;1 2 g: CAPITULO 2. ANILLOS 60 (f) A es el anillo (no conmutativo) Mn (R) de matrices sobre un anillo R y B consiste en las matrices diagonales de A, o sea las matrices en las que todos los elementos que no esten en la diagonal son nulos. (g) A = Mn(R) es como en el apartado anterior y B es el conjunto de las matrices triangulares superiores es decir, las matrices de la forma 0 a a ::: a 1 BB 011 a1222 : : : a12nn :CC @ ::: ::: ::: ::: A 0 0 : : : ann (h) A es el anillo de los endomorsmos de un espacio vectorial V y B es el conjunto ff 2 A : f(W)  W g 14. 15. 16. 17. 18. 19. de los endomorsmos que dejan invariante W, donde W es un subespacio de V . Sean A un anillo y a 2 A. Demostrar que el subanillo de A generado por a esta formado por los elementos de la forma n0 1+n1a+n2 a2 + +nk ak donde k es un entero no negativo y n0 : : :  nk son enteros. >Como son los elementos del subanillo generado por un conjunto arbitrario? Sea A un anillo y sea  una relacion de equivalencia en A tal que, para cualesquiera a a0 b b0 2 A, se verica a  a0  b  b0 ) a + b  a0 + b0 ab  a0b0 (se dice entonces que la relacion es \compatible con las operaciones"). Si I es la clase de equivalencia que contiene al 0, demostrar que I es un ideal de A y que la relacion  es la que induce I es decir, que a  b si y solo si a ; b 2 I. En otras palabras, dar una relacion de equivalencia en A compatible con sus operaciones equivale a dar un ideal de A. Demostrar que el subconjunto de i] formado por los elementos a + bi con 2a  b mod 5 es un ideal principal. Sean A y B dos anillos. Describir los ideales de A  B en funcion de los ideales de A y de B. Determinar todos los ideales de 12  18. Si I, J y K son ideales de un anillo A, demostrar que: (a) I(J \ K)  IJ \ IK. (b) IJ = JI. (c) I(JK) = (IJ)K. (d) I(J + K) = IJ + IK. (e) IA = I. Sea A un anillo. Para cada subconjunto X de A se dene su anulador como AnuX = fa 2 A : ax = 0 para cada x 2 X g. Demostrar las siguientes propiedades para subconjuntos X e Y de A: (a) AnuX es un ideal de A. (b) AnuX = Anu(X). Es decir, el anulador de un conjunto coincide con el anulador del ideal que genera dicho conjunto. (c) Si X  Y , entonces AnuY  AnuX. (d) X  AnuAnuX. (e) AnuAnuAnuX = AnuX. Z ZZ 2.10. PROBLEMAS 61 20. *] Demostrar que las siguientes condiciones son equivalentes para un anillo A. (a) Todo ideal de A es nitamente generado es decir, para todo ideal I existen a1 : : :  an tales que I = (a1  : : :  an). (b) Toda cadena creciente de ideales de A se estaciona es decir, dado ideales I1  I2  de A, existe n 2 tal que In = In+h para todo h 2 . (c) Todo conjunto no vaco de ideales de A tiene un elemento maximal con respecto a la inclusion. Un anillo que verique estas condiciones se llama anillo noetheriano (Indicacion: Aplicar el Lema de Zorn, observando que la union de una cadena creciente de ideales es un ideal.) 21. Demostrar que si f : A ! B es un homomorsmo suprayectivo de anillos y todos los ideales del anillo A son principales entonces todos los ideales de B son principales. 22. Sea a 2 . >Que se deduce al aplicar el Primer Teorema de Isomorfa al homomorsmo X] ! , dado por P (X) 7! P (a)? >Y que se deduce al aplicarlo al homomorsmo X] ! , dado por P(X) 7! P(i)? 23. Sea f : A ! B un homomorsmo suprayectivo de anillos. Demostrar que existe una correspondencia biunvoca, que conserva la inclusion, entre el conjunto de los ideales de B y los ideales de A que contienen a Ker f. 24. Demostrar el recproco del Teorema Chino de los Restos para anillos Q es decir, probar que si I1  : : :  In son ideales de un anillo A tales que la aplicacion f : A ! ni=1 A=Ii , dada por f(a) = (a + I1 : : :  a + In ) es suprayectiva, entonces Ii + Ij = (1), para todo i 6= j. 25. Sean A1  : : : An anillos. Demostrar que la caracterstica del anillo producto A1   An es el mnimo comun multiplo de las caractersticas de los Ai . 26. Sea A un anillo cuya caracterstica es un numero primo p. Demostrar que la aplicacion x 7! xpn es un endomorsmo de A para todo n 2 . 27. Demostrar que, si K es un cuerpo nito con un subcuerpo F , entonces el cardinal de K es una potencia del cardinal de F. (Indicacion: Considerar K como espacio vectorial sobre F ). Deducir que: (a) El cardinal de cualquier cuerpo nito es una potencia de un numero primo. (Indicacion: Considerar el subanillo primo de K.) (b) Si K es un cuerpo nito con un subcuerpo F, entonces existen un numero primo p y enteros positivos n y m tales que n j m, jF j = pn y jK j = pm . 28. Determinar los automorsmos de que cumplen f(x) = x, para todo x 2 . 29. *] Demostrar que el unico automorsmo de es la identidad. (Indicacion: Un automorsmo de debe jar los numeros racionales y conservar el orden.) 30. Sea A un anillo de caracterstica n y sea m un numero entero. >Cuantos homomorsmos de anillos m ! A existen? >Cuantos homomorsmos de anillos n ! m existen? 31. Determinar los ideales de n. >Cuales de ellos son primos o maximales? Dar una formula, en funcion de la descomposicion de n en producto de primos para el numero de ideales de n. 32. >Es 3X]=(X 2 + 1) un cuerpo? 33. *] Se considera el anillo AX]] de series de potencias con coecientes en un anillo A. Se pide: P a X i es una unidad de AX]] precisamente si a es una unidad de A. (a) Demostrar que 1 0 i=1 i (b) Si A es un cuerpo, demostrar que todo ideal de AX]] es de la forma (X n ) para algun n 2 . (c) Demostrar que AX]] es un dominio precisamente si A es un dominio. (d) Identicar los ideales maximales de AX]] en funcion de los ideales maximales de A. N N R R CR R N R Z C R Z Z R ZZ Z N CAPITULO 2. ANILLOS 62 34. *] Sea A un anillo. Un elemento a 2 A es nilpotente si existe un entero n 1 tal que an = 0. Demostrar que: (a) Si x 2 A es nilpotente y u 2 A es una unidad, entonces u + x es una unidad. (b) El conjunto de los elementos nilpotentes de A es un ideal de A que coincide con la interseccion de todos los ideales primos de A. (c) Si I es un ideal, entonces Rad I = fa 2 A : an 2 I para algun n 2 g es un ideal de A que se llama radical de I, y que coincide con la interseccion de todos los ideales primos de A que contienen a I. >Como se relaciona esto con el apartado anterior? (d) Calcular Rad (n) para n 2 . (Indicacion: Considerar la factorizacion prima de n.) 35. Demostrar que si p es un ideal primo de A, entonces pX] es un ideal primo de AX]. >Puede ser pX] maximal? 36. Sea A un anillo. Demostrar que el conjunto S de los divisores de cero de A contiene un ideal primo de A. (Indicacion: Aplicar el Lema de Zorn al conjunto de los ideales contenidos en S.) 37. Demostrar que si p es un ideal maximal (respectivamente primo) de A, entonces N Z p + (X) = fa0 + a1X + 38. 39. 40. 41. 42. 43. 2 AX] : a0 2 pg es un ideal maximal (respectivamente primo) de AX]. *] Sea I un ideal de A y sean p1  : : :  pn ideales primos de A. Demostrar que si I  ni=1 pi, entonces I  pi , para algun i = 1 : : :  n. (Indicacion: Razonar por induccion sobre n.) Demostrar que las siguientes condiciones son equivalentes para un anillo A. (a) A tiene un unico ideal maximal. (b) A tiene un ideal propio I que contiene todos los elementos no invertibles de A. (c) El conjunto de los elementos no invertibles de A es un ideal. (d) Para todo a b 2 A, si a + b es invertible, entonces a o b es invertible. Un anillo que satisface las condiciones anteriores se dice que es local. Demostrar que los siguientes anillos son locales: (a) pn, donde p es primo y n 0. (b) A=mn , donde A es cualquier anillo, m es un ideal maximal y n 2 +. (c) (p), donde p es primo (ver el Problema 13). (d) *] KX]], donde K es un cuerpo. Demostrar que si f : A ! B es un homomorsmo suprayectivo de anillos y A es local, entonces B tambien es local. *] Un ideal I de un anillo A se dice que es primario si I 6= A y para todo x y 2 A tales que xy 2 I, se verica que x 2 I o existe n 2 + tal que yn 2 I. Demostrar: (a) Si I es un ideal primario, entonces Rad (I) es un ideal primo. (b) Si I 6= A, entonces I es primario precisamente si todo divisor de cero de A=I es nilpotente. (c) Encontrar los ideales primarios de . Sea D un dominio y sea Q su cuerpo de fracciones. Demostrar que: (a) Si D0 es un subanillo de D con cuerpo de fracciones Q0, entonces Q contiene un subcuerpo isomorfo a Q0 . (b) Si A es un subanillo de Q que contiene a D, entonces Q es un cuerpo de cocientes de A. Z Z Z Z Z 2.10. PROBLEMAS 63 44. *] Sean A un anillo, S un subconjunto multiplicativo de A, y (AS ;1  j) el anillo de cocientes de A por S. Demostrar: (a) as = 10 precisamente si existe un t 2 S tal que ta = 0. (b) AS ;1 es un dominio si lo es A. (c) La aplicacion j es inyectiva precisamente si S no contiene divisores de cero. (d) Si todo elemento de S es invertible, entonces j es un isomorsmo. (e) Si I es un ideal de A, entonces IS ;1 = f as : a 2 I s 2 S g es un ideal de AS ;1 . Si I es principal, entonces lo es IS ;1 . (f) Si I es un ideal de A, entonces IS ;1 = AS ;1 precisamente si I \ S 6= . (g) Si J es un ideal de AS ;1 , entonces existe un ideal I de A tal que J = AS ;1 . (h) Si p es un ideal primo de A tal que S \ p = , entonces pS ;1 es un ideal primo de AS ;1 . (i) La aplicacion p 7! pS ;1 dene una correspondencia biunvoca entre los ideales primos de A que no intersecan a S y los ideales primos de AS ;1 cuya inversa es la aplicacion dada por q 7! j ;1 (q). (j) Sea T un subconjunto multiplicativo de AS ;1 . Probar que T% = fa 2 A : as 2 T para algun s 2 S g es un subconjunto multiplicativo de A y que AT%;1  = (AS ;1 )T ;1. (k) Sea I un ideal de A tal que S \ I = = S \ (1 + I). Sea : A ! A=I el homomorsmo proyeccion. Demostrar que (S) es un subconjunto multiplicativo de A=I y que (A=I) (S);1 es isomorfo a (AS ;1 )=(IS ;1 ). 45. Sea q = p1 : : :pn un entero libre de cuadrados (los pi son primos distintos), y sea S el conjunto de los enteros de la forma p1 1 pr r con cada i 2 . (a) Demostrar que S es un subconjunto multiplicativo de y que S ;1 es el anillo (q) del Problema 13c. (b) *] Deducir del Problema 44 que (q) es un dominio cuyos ideales son todos principales. 46. *] Sean A un anillo y p un ideal primo de A. Sea S = A n p. El anillo S ;1 A se denota Ap y se llama el anillo localizado de A en p. Analogamente, si I es un ideal de A, entonces S ;1 I se denota Ip . Demostrar que todos los ideales propios de Ap estan contenidos en pp . Concluir que Ap es un anillo local cuyo unico ideal maximal es pp . 47. *] Demostrar que si S = fX n : n 2 g, donde X es la indeterminada de un anillo de polinomios A = BX], entonces AS ;1 es isomorfo al anillo Z Z BX X ;1 ] = fbnX n + bn+1X n+1 + con la suma y el producto naturales. Bibliografa del captulo N Z Z Z Z + bm X m : n m 2 y bi 2 B 8ig Allenby 1], Atiyah-Macdonald 5], Delgado-Fuertes-Xambo 12], Dorronsoro-Hernandez 13], HartleyHawks 19]. 64 CAPITULO 2. ANILLOS Captulo 3 Divisibilidad y factorizacion en dominios Se estudian diversas condiciones de divisibilidad y factorizacion que pueden vericarse en un dominio, se establecen las relaciones entre ellas y se presentan algunas aplicaciones a la Teora de Numeros. Introduccion Z En el Captulo 1 vimos algunas propiedades basicas relativas a la divisibilidad en el anillo de los numeros enteros. Entre ellas la division entera (y con ella el algoritmo de Euclides), el hecho de que todos los ideales de son principales, y el Teorema Fundamental de la Aritmetica, que asegura que todo numero entero se puede escribir, de modo esencialmente unico, como producto de numeros primos. Estas tres propiedades se pueden estudiar en abstracto, como veremos en este captulo, y son fundamentales para entender la divisibilidad en los anillos que las satisfagan. Por ejemplo, si un anillo satisface una propiedad analoga al Teorema Fundamental de la Aritmetica, entonces podremos usar las descomposiciones en productos de primos para determinar si un elemento divide a otro, o para calcular el maximo comun divisor y el mnimo comun multiplo de cualquier conjunto de elementos. Por otra parte, como veremos, las tres propiedades no son independientes. Comenzaremos deniendo las nociones generales relativas a la divisibilidad entre los elementos de un dominio y estableciendo sus primeras propiedades, en particular la relacion con los ideales principales esto nos llevara al estudio de la divisibilidad en los dominios de ideales principales. Mas tarde, consideraremos anillos en los que hay una funcion eucldea (un modo razonable de \dividir con resto") y observaremos que estos son dominios de ideales principales en los que se puede aplicar el algoritmo de Euclides. En tercer lugar consideraremos los dominios en los que todo elemento es producto de primos, llamados dominios de factorizacion unica, y demostraremos que todo dominio de ideales principales es de este tipo. Ademas de y de los anillos de polinomios sobre cuerpos, los principales ejemplos del captulo seran ciertos subanillos del cuerpo complejo , que en muchos casos son necesarios para demostrar resultados clasicos sobre numeros enteros. El captulo terminara con varios ejemplos de esta situacion, en los que usaremos las propiedades del anillo i] de los enteros de Gauss. Z Objetivos del captulo C Z Conocer las nociones basicas sobre divisibilidad en dominios (la relacion `ser asociado", elementos primos e irreducibles, maximo comun divisor y mnimo comun multiplo), y su traduccion en terminos de ideales principales. Conocer algunos ejemplos de dominios eucldeos, y saber usar el algoritmo de Euclides. Conocer las distintas caracterizaciones de los dominios de factorizacion unica, y saber usar las factorizaciones en irreducibles para deducir cuestiones sobre divisibilidad. 65 66  EN DOMINIOS CAPITULO 3. DIVISIBILIDAD Y FACTORIZACION Conocer la relacion entre los tres tipos de dominios estudiados: los dominios eucldeos son dominios de ideales principales, y estos son dominios de factorizacion unica. Manejar cuestiones de divisibilidad y factorizacion en subanillos de del tipo pm], con m 2 , y ser capaz de aplicarlas para resolver ciertos problemas clasicos sobre numeros enteros. C Z Z Desarrollo de los contenidos 3.1 Divisibilidad De nicion 3.1.1 Sea A un anillo y sean a b 2 A. Se dice que a divide a b en A, o que a es un divisor de b en A, o que b es un multiplo de a en A, si existe c 2 A tal que b = ac. Para indicar que a divide a b en A escribiremos a j b en A. Si el anillo A esta claro por el contexto escribiremos simplemente a j b. Q Z Observese que la nocion de divisibilidad depende del anillo. Por ejemplo, si a es un entero diferente de 0, entonces a divide a todos los numeros enteros en , pero no necesariamente en . Ejercicio 3.1.2 Sea A un anillo y sean a b c 2 A. Demostrar que se verican las siguientes propiedades: 1. (Reexiva) a j a. 2. (Transitiva) Si a j b y b j c, entonces a j c. 3. a j 0 y 1 j a. 4. 0 j a si y solo si a = 0. 5. a j 1 si y solo si a es una unidad en este caso a j x para todo x 2 A (es decir, las unidades dividen a cualquier elemento). 6. Si a j b y a j c entonces a j rb + sc para cualesquiera r s 2 A (y en particular a j b + c, a j b ; c y a j rb para cualquier r 2 A). Mas generalmente, si a divide a ciertos elementos, entonces divide a cualquier combinacion lineal suya con coecientes en A. 7. Si A es un dominio, c 6= 0 y ac j bc, entonces a j b. De nicion 3.1.3 Dos elementos a y b de un anillo A se dice que son asociados si se dividen mutuamente es decir, si a j b y b j a. Cuando no este claro por el contexto en que anillo estamos trabajando, hablaremos de elementos asociados en A. Por ejemplo, una unidad es lo mismo que un elemento asociado a 1. Es elemental ver que \ser asociados" es una relacion de equivalencia en A, y que dos elementos son asociados si y solo si tienen los mismos divisores, si y solo si tienen los mismos multiplos. Por lo tanto, al estudiar cuestiones de divisibilidad, un elemento tendra las mismas propiedades que sus asociados. La siguiente caracterizacion de la relacion \ser asociado" en un dominio sera importante (y por motivos como este pronto empezaremos a suponer sistematicamente que los anillos que aparecen son dominios): Ejercicio 3.1.4 Si D es un dominio, demostrar que dos elementos a b 2 D son asociados si y solo si existe una unidad u 2 D tal que b = au (si D no es un dominio, se verica el \si"). p Ejercicio 3.1.5 Consideremos elp anillo  m], donde p p m 2 no es un cuadrado, y recordemos que el conjugado de x = a + b m 2  m] es x% = a ; b m. Denimos la norma de x como el entero p p N(x) = x x% = (a + b m)(a ; b m) = a2 ; mb2 : p Esta nocion es util porque permite traducir problemas de divisibilidad en  m] a problemas de divisi- Z Z Z Z bilidad entre enteros, como se aprecia en las siguientes propiedades elementales. Demostrar que: 3.1. DIVISIBILIDAD 67 Z 1. N(x) = 0 si y solo si x = 0. 2. La norma es multiplicativa es decir, N(xy) = N(x)N(y) para cualesquiera x y 2 pm] (recuerdese p p que la conjugacion es un homomorsmo de anillos  m] !  m]). Z p Z Z 3. x 2  m] si y solo si N(x) = 1, y en este caso x;1 = N(x)%x (es decir, x;1 = x%, donde el signo es el de N(x)). p 4. Si x j y en  m] entonces N(x) j N(y) en . p 5. Si x e y son asociados en  m] entonces N(x) = N(y). p p 6. Si x j y en  m] y N(x) = N(y) entonces x e y son asociados en  m]. 7. i] = f1 ;1 i ;ig. Z Z Z Z Z Z Z p 8. Si m ;2 entonces  m] = f1 ;1g. Observaciones 3.1.6 p Z p 1. Si m < 0, entonces el conjugado z% = a ; b m de z = a + b m es su conjugado complejo en p el sentido usual. Por tanto, N(z) = z%z = jz j2, lo que implica que los elementos invertibles de  m] son los que pertenecen a la circunferencia de centro 0 y radio 1. Esto ilustra los dos ultimos apartados del ejercicio anterior. p 2. Si m 2 entonces  m] tiene una innidad de elementos invertibles, pero no es facil de mostrarlo. O sea, la Ecuacion de Pell x2 ; my2 = 1 tiene innitas soluciones enteras. Este es un profundo resultado que se vera en el curso de Introduccion a la Teora de Numeros y que implica que las ecuaciones diofanticas del tipo x2 ; my2 = n p (con m 2 y n 2 )ptienen innitas soluciones o ninguna. S es facil ver, por ejemplo, que en  2] el elemento 1 + 2 es invertible, y por lo tanto lo son todas sus potencias, que son distintas entre s (>por que?). p 3. En general un elemento de  m] no es asociado de su conjugado, aunque el hecho de que la conjugacion sea un isomorsmo nos permitira relacionar algunas de sus propiedades (por ejemplo, usando nociones que vamos a denir en seguida, x es irreducible o primo si y solo si lo es x%). Z Z Z Z Z Z Del Ejercicio 3.1.4 se deduce que, si en un dominio D se conocen todas las unidades, se conocen automaticamente todos los asociados de un elemento. Por ejemplo, como  = f1 ;1g, los asociados de a en son el propio a y ;a. El conocimiento de D tambien es util para encontrar un conjunto de representantes de los elementos no nulos de D para la relacion \ser asociados" es decir, un subconjunto D0 de D tal que cada elemento no nulo de D es asociado a un unico elemento de D0 . Por ejemplo: ZZ Z Ejercicio 3.1.7 Con la notacion anterior, demostrar que: 1. Si D = , podemos tomar D0 = +. 2. Si D = i] entonces podemos tomar como D0 el conjunto de los enteros de Gauss de la forma a + bi, con a > 0 y b 0 (es decir, los del primer cuadrante, incluido el semieje real positivo pero no el eje imaginario). Z p 3. Encontrar un conjunto de representantes salvo asociados para  m] con m ;2.  EN DOMINIOS CAPITULO 3. DIVISIBILIDAD Y FACTORIZACION 68 4. Si K es un cuerpo y D = KX], entonces D consiste en los polinomios constantes no nulos (es decir, D = K  = K n f0g), y podemos tomar como D0 el conjunto de los polinomios con coeciente principal 1. Sabemos que cualquier elemento a de un anillo A es divisible por sus asociados y por las unidades de A, y que si a divide a uno de los elementos b o c entonces divide a su producto bc. A continuacion estudiamos los elementos que verican \los recprocos" de estas propiedades. A menudo consideraremos elementos a de un anillo A que no son cero ni unidades, lo que sintetizaremos en la forma 0 6= a 2 A n A . De nicion 3.1.8 Diremos que un elemento a del anillo A es irreducible si 0 6= a 2 A n A y la relacion a = bc en A implica que b 2 A o c 2 A (y por lo tanto que uno de los dos es asociado de a). Diremos que a es primo si 0 6= a 2 A n A y la relacion a j bc en A implica que a j b o a j c. Ambas nociones dependen del anillo ambiente, y si este no esta claro por el contexto hablaremos de irreducibles y primos \en A". Ejercicio 3.1.9 Si D es un dominio, demostrar que: 1. Un elemento 0 6= a 2 D n D es irreducible si y solo si sus unicos divisores son sus asociados y las unidades1 . 2. Si dos elementos son asociados, entonces uno es irreducible (respectivamente primo) si y solo si lo es el otro. p 3. En  m], un elemento es irreducible (respectivamente primo) si y solo si lo es su conjugado. Ejemplos 3.1.10 Irreducibles y primos. Z Z Z Z Z Z 1. En ambos conceptos coinciden con la nocion usual de entero primo (Proposicion 1.5.1). p 2. Supongamos que m 2 no es un cuadrado. Entonces, en  m], los elementos de norma prima p sonpirreducibles es decir, si x 2  m] y N(x) es un entero primo, entonces x es irreducible en  m]. En efecto, es claro que x no es cero ni invertible, y si x = yz, entonces N(x) = N(y)N(z), con lo que N(y) = 1 o N(z) = 1, y por tanto y o z es una unidad de pm]. El recproco no es cierto. Por ejemplo, 3 es irreducible en i] (>por que?), aunque su norma no es un entero primo. 3. El polinomio X 2 +1 es irreducible en X] pero no en X], donde se tiene X 2 +1 = (X+i)(X ; i). 4. El polinomio 2X + 2 es irreducible en X] pero no en X], donde se tiene 2X + 2 = 2(X + 1). Acabamos de ver que un elemento de es irreducible si y solo si es primo. Solo una parte es cierta en general (la demostracion se deja como ejercicio). Proposicion 3.1.11 En un dominio A todo elemento primo es irreducible. El recproco no se verica en general, como muestra el siguiente ejemplo. Z ZZ Z Z RQ Z CZ Z Ejemplo 3.1.12 p Irreducible no implica primo. En el anillo  ;5] hay elementos irreducibles que no son primos. Comencemos observando que los cuadrados en 5 son 0]p5 y 1]5, y que por lo tanto la congruencia a2  2 mod 5 no tiene solucion. Esto implica que en  ;5] no hay elementos cuya norma valga p p 2, 3 o 12 (>por que? observese que en  ;5] todas las normas son positivas). Sea ahora x 2  ;5] con N(x) = 4 si y j x entonces N(y) j N(x) = 4 y por lo tanto N(y) vale 1, 2 o 4 en el primer caso y es una unidad, el segundo es Z imposible y en el tercero y es asociado de x (>por que?), y en consecuencia x es irreducible. De igual modo se ve que los elementos con norma 6 o 9 son irreducibles, y en particular lo son todos los factores de la igualdad p p 2 3 = (1 + ;5)(1 ; ;5): p p Pero ninguno de ellospes primo: por ejemplo de la igualdad se deduce que 2 j (1 + ;5)(1 ; ;5), y p es claro que 2 6 j (1 + ;5) y 2 6 j (1 ; ;5). 1 Si D no es un dominio se verica el \solo si". En A = Z6 el \si" falla para a = 4]6 . 3.1. DIVISIBILIDAD 69 Z Las deniciones de maximo comun divisor y mnimo comun multiplo, as como sus propiedades elementales, son analogas a las de y las resumimos en el siguiente ejercicio/denicion: Ejercicio 3.1.13 Sean A un anillo, S un subconjunto de A y a b d m 2 A. Demostrar que: 1. Las condiciones siguientes son equivalentes: (a) m es multiplo de cada elemento de S , y si un elemento x 2 A es multiplo de cada elemento de S entonces x es multiplo de m. (b) Un elemento x 2 A es multiplo de cada elemento de S si y solo si es multiplo de m. En este caso se dice que m es un mnimo comun multiplo de S , y otro elemento es un mnimo comun multiplo de S si y solo si es asociado de m. Escribiremos m = mcm(S), entendiendo que tal elemento (si existe) es unico salvo asociados. Asimismo, hablaremos de el mnimo comun multiplo de S , con el mismo signicado de unicidad salvo asociados. 2. Las condiciones siguientes son equivalentes: (a) d divide a cada elemento de S , y si un elemento x 2 A divide a cada elemento de S entonces x divide a d. (b) Un elemento x 2 A divide a cada elemento de S si y solo si divide a d. En este caso se dice que d es un maximo comun divisor de S , y otro elemento es un maximo comun divisor de S si y solo si es asociado de d. Escribiremos d = mcd(S), entendiendo que tal elemento (si existe) es unico salvo asociados. 3. Si d es un divisor comun de los elementos de S y ademas es combinacion lineal de elementos de S es decir, existen elementos s1  : : :  sn 2 S y a1  : : :  an 2 A tales que d = a1s1 + + an sn  (3.1.1) entonces d = mcd(S), y se dice que esta expresion es una identidad de Bezout para S . Si existe una identidad de Bezout (3.1.1) con d = 1 entonces mcd(S) = 1. 4. Se verica 1 = mcd(S) si y solo si los unicos divisores comunes de los elementos de S son las unidades de A. En este caso decimos que los elementos de S son coprimos. Si para cada par de elementos distintos a b 2 S se verica mcd(a b) = 1, decimos que los elementos de S son coprimos dos a dos. 5. a j b si y solo si a = mcd(a b), si y solo si b = mcm(a b). 6. En particular, 1 = mcd(a 1), mcd(a 0) = a = mcm(a 1) y 0 = mcm(a 0). 7. Si a es irreducible entonces mcd(a b) = 1 si y solo si a 6 j b. Ejercicio 3.1.14 Demostrar que, para cualquier entero m, dos enteros a y b son coprimos en pm] Z si y solo si son coprimos en . Z Z Z Z El siguiente ejemplo muestra ciertos fenomenos que no ocurran en en particular, la existencia de un maximo comun divisor o un mnimo comun multiplo no esta garantizada en un dominio arbitrario. Ejemplo 3.1.15 Patologas en p;5]. p En este ejemplo trabajamos en el anillo  ;5] y hacemos uso de las armaciones del Ejemplo 3.1.12. Se verican: p 1. Los elementos 2 y 1 + ;5 son coprimos, pues si x es un divisor comun entonces su norma divide a 4 y a 6, luego divide a 2 y por lo tanto vale p 1, de manera que x es unapunidad. Sin embargo no hay una identidad de Bezout para f2 1 + ;5g, pues la igualdad en  ;5] p p p 1 = 2(a + b ;5) + (1 + ;5)(c + d ;5) equivale al par de igualdades en 2a + c ; 5d = 1 2b + c + d = 0 que reducidas modulo 2 nos llevan al absurdo c + d  1 mod 2 y c + d  0 mod 2. Z Z 70  EN DOMINIOS CAPITULO 3. DIVISIBILIDAD Y FACTORIZACION p 2. Aunque acabamos de ver que 2 y 1+ ;5 tienen mpaximo comun divisor, no tienen mnimo comun un multiplo. Supongamos que existe x = mcm(2 1 + ;5) entonces N(x) sera un multiplo p;com de 4 y 6, y por lo tanto sera un m u ltiplo de 12. Por otra parte los elementos 6 y 2(1 + 5) son p multiplos comunes de 2 y de 1 + ;5, luego x los divide y por p lo tanto N(x) divide a 36 y a 24, por lo que divide a 12. En conclusion, si existe mcm(2 1 + ;5) su norma es 12, pero ya vimos que esto es imposible. p 3. Veamos por ultimo que no existe x = mcd(6 2(1 + ;5)). Sipexistiera, N(x) dividira a 36 p ya 24 y por lo tanto dividira a 12 pero ademas, como 2 y 1 + ;5 dividen a 6 y a 2(1 + ;5), deberan dividir a x y as N(x) sera un multiplo comun de 4 y 6 y por lo tanto sera un multiplo de 12, lo que nos llevara a la situacion imposible N(x) = 12. 3.2 Dominios de ideales principales Comenzamos esta seccion observando que, sobre un dominio, todas las nociones de divisibilidad que hemos presentado pueden reenunciarse en terminos de los ideales principales generados por los elementos involucrados. Ejercicio 3.2.1 Sea D un dominio con elementos a b d m y un subconjunto S . Demostrar que se verican las siguientes propiedades: 1. a = 0 precisamente si (a) = 0. 2. a 2 D precisamente si (a) = D. 3. a j b precisamente si (b)  (a) (o si b 2 (a)). 4. a y b son asociados precisamente si (a) = (b). 5. a es primo precisamente si (a) es un ideal primo no nulo de D. 6. a es irreducible precisamente si (a) es maximal entre los ideales principales propios no nulos de D es decir, a 6= 0 y (a)  (b)  D implica (a) = (b). 7. d = mcd(S) precisamente si (d) es mnimo entre los ideales principales que contienen a S (o al ideal generado por S ). En particular, si (S) es un ideal principal entonces cualquier generador suyo es un maximo comun divisor de S , y ademas existe una identidad de Bezout para S . 8. m = mcm(S) si y solo si (m) = \s2S (s). En consecuencia, mcm(S) existe si y solo si el ideal \s2S (s) es principal, y entonces cualquier generador de \s2S (s) es un mnimo comun multiplo de S . En vista de estos resultados, las nociones sobre divisibilidad se manejaran facilmente en dominios en los que todos los ideales son principales. De nicion 3.2.2 Un dominio de ideales principales, o DIP (PID, en la literatura en ingles), es un dominio en el que todos los ideales son principales. Proposicion 3.2.3 Si D es un DIP y 0 6= a 2 D n D , las siguientes condiciones son equivalentes: 1. a es irreducible. 2. (a) es un ideal maximal. 3. A=(a) es un cuerpo. 4. a es primo. 5. (a) es un ideal primo. 6. A=(a) es un dominio. 3.3. DOMINIOS EUCLIDEOS 71 Demostracion. La equivalencia entre 1, 2 y 3 es consecuencia del Ejercicio 3.2.1 y de la Proposicion 2.8.6, y lo mismo puede decirse de la equivalencia entre 4, 5 y 6. Tambien de la Proposicion 2.8.6 se deduce que 2 implica 5. Finalmente, 4 implica 1 por la Proposicion 3.1.11. Z Z Ejemplos 3.2.4 Dominios de ideales principales. Z 1. es un DIP por el Teorema 1.2.11. 2. El anillo de polinomios X] no es un DIP por el Ejemplo 2.5.9. p 3.  ;5] no es un DIP, pues contiene p elementos irreducibles que no son primos. Tambi p en podemos observar que el ideal (6 2(1 + ;5) no es principal, pues no existe mcd(6 2(1 + ;5). Vease el Ejemplo 3.1.15. Veamos otra consecuencia inmediata del Ejercicio 3.2.1. Proposicion 3.2.5 Sea D un DIP y sea S un subconjunto de D. Entonces 1. S tiene un mnimo comun multiplo (cualquier generador de \s2S (s)). 2. S tiene un maximo comun divisor d (cualquier generador del ideal que genera S ) y ademas existe una identidad de Bezout para S . 3. El elemento d es un maximo comun divisor de a1 : : :  an si y solo si d j ai para cada i = 1 : : :  n y existen r1 : : :  rn 2 D tales que r1a1 + + rnan = d: 4. Los elementos a1 : : :  an son coprimos si y solo si existen r1 : : :  rn 2 D tales que r1 a1 + + rnan = 1: Ejercicio 3.2.6 Si D es un DIP, demostrar que para cualesquiera a b c 2 D se verican las siguientes propiedades (ver las Proposiciones 1.3.6 y 1.4.3): 1. Si d = mcd(a b), entonces mcd( da  db ) = 1. 2. Si mcd(a b) = mcd(a c) = 1, entonces mcd(a bc) = 1. 3. Si mcd(a b) = 1 y a j bc, entonces a j c. 4. Si mcd(a b) = 1, a j c y b j c, entonces ab j c. 5. ab y mcd(a b)mcma b] son asociados. 3.3 Dominios eucldeos En esta seccion nos ocuparemos del concepto de division con resto. En realidad existen varias posibles generalizaciones de dicho concepto. Todas ellas tienen consecuencias similares, y nosotros elegiremos la mas general. De nicion 3.3.1 Sea D un dominio. Una funcion eucldea en D es una aplicacion  : D n f0g ! que cumple las siguientes condiciones: (DE1) Si a b 2 D n f0g verican a j b entonces (a) (b). N (DE2) Dados a b 2 D con b 6= 0, existen q r 2 D tales que a = bq + r y o bien r = 0 o bien (r) < (b). Un dominio eucldeo es un dominio que admite una funcion eucldea.  EN DOMINIOS CAPITULO 3. DIVISIBILIDAD Y FACTORIZACION 72 La posibilidad de \dividir con resto" en un dominio eucldeo nos permite demostrar propiedades como las que siguen: Proposicion 3.3.2 Sea  una funcion eucldea en un dominio D. Entonces las siguientes condiciones son equivalentes para a 2 D: 1. a es una unidad de D. 2. (a) = (1). 3. (a) (x), para todo x 2 D n f0g. Demostracion. 1 implica 2. Por hipotesis, a es asociado de 1, y por (DE1) se tiene (1) = (a). 2 implica 3. Si x 2 D n f0g entonces 1 j x, y por (DE1) se tiene (a) = (1) (x). 3 implica 1. Por (DE2) existen q r 2 D tales que 1 = aq +r y o bien r = 0 o bien (r) < (a) como la hipotesis excluye la segunda posibilidad, debe ser r = 0 y as 1 = aq, por lo que a es invertible. Teorema 3.3.3 Todo dominio eucldeo D es un dominio de ideales principales. Demostracion. La demostracion es esencialmente igual a la del Teorema 1.2.11. Por tanto, mencionaremos sus lneas maestras y dejaremos los detalles para el lector. Sea  una funcion eucldea en D y sea I un ideal no nulo de D. Sea a un elemento no nulo de I tal que (a) es mnimo entre los elementos no nulos de I. Entonces I = (a). Z Z C El recproco de este teorema es falso, pero no facil encontrar un DIP que no sea un dominio p;es 19 1+ 2 eucldeo.p Un tal ejemplo es el subanillo  2 ] de formado por los elementos de la forma a + b 1+ 2;19 con a b 2 . Como consecuencia del Teorema 3.3.3 y la Proposicion 3.2.5, todo subconjunto de un dominio eucldeo tiene maximo comun divisor y mnimo comun multiplo. De hecho, el Algoritmo de Euclides sirve en cualquier dominio eucldeo D para calcular el maximo comun divisor de dos elementos de D y la correspondiente identidad de Bezout, lo que a su vez permite resolver ecuaciones del tipo ax + by = c en D. Las demostraciones de estos hechos son meras adaptaciones de las que hicimos en el Captulo 1 para el anillo de los numeros enteros y se dejan a cargo del lector. En el resto de esta seccion vemos algunos ejemplos de dominios eucldeos y de como se usa en ellos el Algoritmo de Euclides. N Ejercicio 3.3.4 Demostrar las armaciones siguientes. 1. Si D es un dominio y  : D ! es una aplicacion que conserva productos y verica (x) = 0 si y solo si x = 0, entonces  verica (DE1) (estrictamente hablando, es su restriccion a D n f0g la Z que lo verica). p 2. En particular, si D =  m] y  se dene como el valor absoluto de la norma (o como la propia norma, cuando m es negativo), entonces  satisface la condicion (DE1). 3. Si ademas, dados a b 2 D con b 6= 0, existen q r 2 D con a = bq + r y (r) < (b) entonces  es una funcion eucldea en D. ZN Z Z Ejemplos 3.3.5 Funciones eucldeas y dominios eucldeos. 1. La aplicacion valor absoluto  ! es una funcion eucldea en (Teorema 1.1.8). En realidad, hay una sutil diferencia entre la division entera y este hecho: Dados a b 2 , los elementos q y r postulados en (DE2) no son unicos por ejemplo 42 = 5 8 + 2 = 5 9 + (;3). 2 Una demostraci on de este hecho que solo usa tecnicas elementales puede consultarse en Oscar A. Campoli, \A Principal Ideal Domain That Is Not Euclidean Domain", The Teaching of Mathematics (Noviembre 1988), 868-871]. 3.3. DOMINIOS EUCLIDEOS 73 2. Un ejemplo trivial de anillo eucldeo es un cuerpo K, pues entonces la aplicacion que asocia 1 a todo elemento de K  es una funcion eucldea. 3. En el anillo i] de los enteros de Gauss la norma N : i] ! (dada por N(a + bi) = a2 + b2 ) es una funcion eucldea. Por el Ejercicio 3.3.4 basta ver que, dados a b 2 i] con b 6= 0, existen q r 2 i] con a = bq + r y N(r) < N(b). Para ello, observemos primero que la norma puede extenderse a una funcion N : i] ! denida tambien por N(a + bi) = a2 + b2 y que conserva productos. Como i] es el cuerpo de cocientes de i] (Ejemplo 2.9.6), existen Q1 Q2 2 tales que ab = Q1 + Q2i. Aproximando los Qi por el entero mas proximo encontramos q1 q2 2 y R1 R2 2 con Qi = qi + Ri y jRij 12 para cada i = 1 2. Entonces se tiene Z Q Z Z Q Q Q Z Z N Z QZ a = b(Q1 + Q2 i) = b(q1 + q2 i) + b(R1 + R2i) de modo que podemos tomar a q = q1 + q2i como cociente y a r = b(R1 + R2i) como resto, pues esta en i] (vale a ; bq) y verica N(r) = N(b(R1 + R2i)) = N(b)N(R1 + R2i) = N(b)(R21 + R22 ) N(b)( 41 + 14 ) < N(b): El siguiente diagrama ilustra el proceso: 3i 2i  i i -2 -1 0 1 2 x y = Q1 + Q2i q = q1 + q2 i 3 ;i Z Z ;2i 4. Una sencilla ver que la norma es una funcion eucldea p adaptacion del argumento anterior permite 1 + 2 1 , que sigue siendo menor que 1), pero esa en  ;2] (el 14 + 41 anterior se transforma en 4 4 p adaptacion ya no funciona con  ;3] de hecho, este no es un dominio eucldeo (ni con la norma ni con ninguna otra posible funcion eucldea) porque ni siquiera espun DIP: contiene elementos p irreducibles que no son primos (considerese la igualdad 2 2 = (1 + ;3)(1 ; ;3)). Z Ejemplo 3.3.6 Algoritmo de Euclides en i]. Z Vamos a calcular el maximo comun divisor de a = 2 + 2i y b = ;1 + 7i en i] utilizando el Algoritmo de Euclides. Como N(a) = 8 y N(b) = 50, dividimos b entre a siguiendo el proceso explicado en el Ejemplo 3.3.5. Como b ;1 + 7i (;1 + 7i)(2 ; 2i) = 12 + 16i = 3 + 2i a = 2 + 2i = 8 8 2 tomamos q = 1 + 2i y r = b ; aq = 1 ; i. Es decir, empezamos construyendo la tabla b = ;1 + 7i a = 2 + 2i 1 ; i 1 + 2i Dividimos ahora a entre r = 1 ; i. a = 2 + 2i = (2 + 2i)(1 + i) = 4i = 2i: r 1;i 2 2  EN DOMINIOS CAPITULO 3. DIVISIBILIDAD Y FACTORIZACION 74 Por tanto, completamos la tabla anterior: b = ;1 + 7i a = 2 + 2i 1 ; i 0 1 + 2i 2i y deducimos que mcd(2 + 2i ;1 + 7i) = 1 ; i. El lector puede ahora encontrar la correspondiente identidad de Bezout, as como las soluciones en i] de la ecuacion ax + by = 5 + 3i. Observese que no hay unicidad en la division eucldea en i]. Por ejemplo, tras obtener b=a = (3=2) + 2i, podamos haber elegido como primer cociente q0 = 2 + 2i, lo que nos habra dado un resto r0 = b ; aq0 = ;1 ; i, y como r0 divide a a obtendramos mcd(2 + 2i ;1 + 7i) = ;1 ; i. >Atenta esto contra la unicidad del maximo comun divisor? Otro ejemplo de dominios eucldeos lo constituyen los anillos de polinomios en una indeterminada con coecientes en un cuerpo, como se demuestra a continuacion. En la Proposicion 4.1.4 y el Lema 4.3.1 se a~nadiran algunos matices al resultado que sigue. Proposicion 3.3.7 Si K es un cuerpo, entonces el anillo de polinomios KX] es un dominio eucldeo. De hecho, la aplicacion gr : KX] n f0g ! que asocia a cada polinomio f su grado gr(f) es una Z Z N funcion eucldea. Demostracion. Sabemos (Ejemplos 2.8.4) que gr(fg) = gr(f) + gr(g), de donde se deduce (DE1). En cuanto a (DE2), jemos 0 = 6 g 2 KX] con grado m = gr(g) y coeciente principal b =6 0 (invertible en K). Dado f 2 KX] vamos a ver, por induccion en n = gr(f), que existen q r 2 KX] con f = gq + r y r = 0 o gr(r) < m. Si n < m podemos tomar q = 0 y r = f. Supongamos pues que n m y que la propiedad se verica si f se sustituye por un polinomio de grado menor. Si a es el termino principal de f, es claro que el polinomio f1 = f ; ab;1X n;m g 2 KX] tiene grado menor que el de f. Por hipotesis de induccion existen q1 r 2 KX] tales que f1 = gq1 + r y r = 0 o gr(r) < m. Entonces f = g(q1 + ab;1X n;m ) + r, lo que termina la demostracion. Q Q Ejemplo 3.3.8 Algoritmo de Euclides en X]. Vamos a resolver la siguiente ecuacion con incognitas U y V en el anillo de polinomios X]: (X 4 ; 1)U + (X 2 ; 2X + 1)V = X 2 ; 1: Para ello calculamos el maximo comun divisor de X 4 + 1 y X 2 ; 2X + 1: X 4 ; 1 X 2 ; 2X + 1 4X ; 4 0 X 2 + 2X + 3 14 (X ; 1) Por tanto mcd(X 4 ; 1 X 2 ; 2X + 1) = X ; 1 (el \asociado comodo" de 4X ; 4), y como X ; 1 divide a X 2 ; 1 deducimos que la ecuacion tiene solucion. Para encontrar una de ellas, utilizamos los calculos del Algoritmo de Euclides para expresar el maximo comun divisor en funcion de los coecientes: 2 + 3 (X 2 ; 2X + 1): X ; 1 = 41 (4X ; 4) = 14 (X 4 ; 1) ; X + 2X 4 Multiplicando por XX2;;11 = X + 1 se obtiene X 2 ; 1 = (X + 1)  X 4 ; 1 X 2 + 2X + 3  2 ; (X ; 2X + 1)  X+1 4 4  de donde se deduce que (U V ) = 4  ;(X +1)(X4 +2X +3) es una solucion particular de nuestra ecuacion. Como XX4;;11 = X 3 + X 2 + X + 1 y X 2X;2;X1 +1 = X ; 1, la solucion general es: 2 U = X4+1 + (X ; 1)P V = ; (X +1)(X42 +2X +3) ; (X 3 + X 2 + X + 1)P Q (P 2 X]):  UNICA  3.4. DOMINIOS DE FACTORIZACION 75 3.4 Dominios de factorizacion unica De nicion 3.4.1 Sea D un dominio. Una factorizacion en producto de irreducibles de un elemento a de D es una expresion del tipo N a = up1 pn donde n 2 , u es una unidad de D y p1 : : :  pn son irreducibles de D (se admite la posibilidad de que sea n = 0, en cuyo caso la factorizacion se reduce a a = u). Diremos que D es un dominio de factorizacion si todo elemento no nulo de D admite una factorizacion en producto de irreducibles. Dos factorizaciones de a 2 D en producto de irreducibles se dice que son equivalentes si solo se diferencian en el orden y en asociados. Dicho con mas rigor, las factorizaciones N N a = up1 pn = vq1 qm (con u v 2 D y el resto de factores irreducibles) son equivalentes si n = m y existe una permutacion  de n (una biyeccion de n = f1 2 : : :  ng en s mismo) tal que pi y q (i) son asociados para cada i = 1 : : :  n. Diremos que D es un dominio de factorizacion unica o DFU (UFD, en ingles) si es un dominio de factorizacion en el que, para cada 0 6= a 2 D, todas las factorizaciones de a son equivalentes. Comenzamos observando que, sobre un DFU, los elementos irreducibles coinciden con los primos, por lo que podemos hablar indistintamente de factorizaciones en irreducibles o factorizaciones en primos. Lema 3.4.2 Si D es un DFU, entonces todo elemento irreducible de D es primo. Demostracion. Sea p 2 D irreducible, y sean a b t 2 D tales que pt = ab. Se trata de ver que p j a o p j b. Si t = up1 pn , a = vq1 qm y b = wr1 rk son factorizaciones en irreducibles (con u v w 2 D ), entonces se tiene upp1 pn = (vw)q1 qm r1 rk  y por la unicidad de la factorizacion p es asociado de algun qi (y entonces p j a) o de algun ri (y entonces p j b). Sea D un dominio y sea D0 un conjunto de representantes de los elementos no nulos de D para la relacion \ser asociados" (ver el Ejercicio 3.1.7). El conjunto P de los elementos de D0 que son irreducibles es entonces un conjunto de representantes de los elementos irreducibles de D para la relacion \ser asociados" es decir, todo irreducible de D es asociado a un unico elemento de P (>por que?). Por ejemplo, usando el Ejercicio 3.1.7, sabemos que: Si D = entonces podemos tomar como P el conjunto de los enteros primos positivos. ZZ Z Si D = i] entonces podemos tomar como P el conjunto de los enteros de Gauss irreducibles del primer cuadrante (incluyendo el eje real positivo pero no el imaginario). De los puntos de i] que aparecen en el siguiente graco, se han se~nalado los que estan en P . 4 3 2 1 0 1 2 3 4 Si K es un cuerpo y D = KX], podemos tomar como P el conjunto de los polinomios irreducibles con coeciente principal 1.  EN DOMINIOS CAPITULO 3. DIVISIBILIDAD Y FACTORIZACION 76 Si D es un DFU y P es como en el parrafo anterior, entonces todo elemento no nulo a de D tiene una unica factorizacion de la forma Y a = u pp N p 2P donde u es una unidad de D, p 2 para todo p 2 P y p = 0 para casi todo p 2 P. Llamaremos a esta la factorizacion prima de a o la factorizacion de a en irreducibles de P. En muchos casos nos interesara escribir solo los factores distintos de 1 (es decir, con exponente no nulo), y en ese caso tendremos una factorizacion prima irredundante. En otras ocasiones, como veremos en seguida, es conveniente dejar que aparezcan \factores cticios" con exponente 0. La gran ventaja de que un dominio sea un DFU es que la nocion de divisibilidad, y por tanto todas las que dependen de ella, se pueden expresar en terminos de las factorizaciones primas. En particular se pueden calcular el maximo comun divisor y el mnimo comun multiplo de un conjunto a partir de esas factorizaciones. Dejaremos que el lector demuestre estos hechos, que se encuentran resumidos en el siguiente ejercicio. Ejercicio 3.4.3 Sean D un DFU y P un conjunto de representantes de los irreducibles de D por la relacion de equivalencia \ser asociados". Sean a=u Y p2P pp y b=v Y p2P pp las factorizaciones de a y b en irreducibles de P . Demostrar: 1. a j b precisamente si p p , para todo p 2 P. 2. El numero de divisores de a es nito, salvo asociados. Es decir, existe un conjunto nito F tal que todos los divisores de a son asociados de un elemento de F . 3. Obtener una formula para calcular el numero de divisores de a, salvo asociados, en terminos de los exponentes p. Q Q 5. mcd(a b) = p2P pmin(p p ) . 4. mcm(a b) = p2P pmax(p p ) . 6. a y b son coprimos si y solo si no tienen ningun divisor primo comun. Las formulas de los apartados 4 y 5 del Ejercicio 3.4.3 se generalizan del modo obvio al caso de familias arbitrarias de elementos, con un precaucion en el caso del calculo del mnimo comun multiplo de familias innitas: Si no existe alguno de los maximos que intervienen, ese mnimo comun multiplo vale 0. En particular, todo subconjunto de un DFU tiene un maximo comun divisor y un mnimo comun multiplo. Sin embargo, no es cierto en general que existan identidades de Bezout. Por ejemplo, en Captulo 4 demostraremos que D = X] es un DFU, y como 2 y X son irreducibles no asociados en D, deducimos que son coprimos es decir, mcd(2 X) = 1. Sin embargo 1 62 (2 X) (ver el Ejemplo 2.5.9). La unicidad en las factorizaciones permite demostrar resultados como este: Z Proposicion 3.4.4 Sea D un DFU y sean a b c 2 D, con a y b coprimos. Demostrar que: 1. a + b y ab son coprimos. 2. mcd(a c) = mcd(a bc). 3. Si a j bc entonces a j c. 4. Si a j c y b j c entonces ab j c. 5. Si ab = cn (con n 2) entonces existe x 2 D tal que a es asociado de xn.  UNICA  3.4. DOMINIOS DE FACTORIZACION 77 Demostracion. Usaremos repetidamente el Ejercicio 3.4.3, y jaremos las siguientes factorizaciones en elementos de un conjunto de representantes de irreducibles P: a=u Y p2P pp  b=v Y p 2P pp y c=w Y p2P pp : Como a y b son coprimos, para cada p 2 P se tiene p = 0 o p = 0 (tal vez ambos). 1. Si existiera un divisor primo comun p 2 P de a+b y de ab, entonces p dividira a las combinaciones lineales a(a + b) ; ab = a2 y b(a + b) ; ab = b2 , y por tanto p sera un divisor comun de a y de b, lo cual es imposible. Q 2. Como bc = vw p2P pp +p , basta ver que min(p  p ) = min(p  p + p ) para cada p 2 P, lo cual es obvio pues p = 0 o p = 0. 3. Es consecuencia de 2, pues a j bc implica a = mcd(a bc) = mcd(a c) y por tanto a j c. 4. Para cada p 2 P se tiene p p y p p por hipotesis, y como p = 0 o p = 0 se tiene p + p p . En consecuencia ab j c. 5. La igualdad ab = cn nos da la igualdad entre factorizaciones primas uv Y p2P pp +p = w Y p2P pnp  por lo que p + p = n p para cada p 2 P. Si p 6= 0 entonces p = 0 y por tanto p = n p . En consecuencia se tiene 0 1 Y p @ Y p An =u p =u p  p2Pp 6=0 Q y el resultado se obtiene tomando x = p2Pp 6=0  p2Pp 6=0 p p . Proposicion 3.4.5 Para un dominio D, las condiciones siguientes son equivalentes: 1. D es un dominio de factorizacion unica. 2. D es un dominio de factorizacion en el que todo elemento irreducible es primo. Demostracion. 1 implica 2. Por la denicion de DFU y por el Lema 3.4.2. 2 implica 1. Por hipotesis, todo elemento no nulo de D se factoriza como un producto de primos, y podemos demostrar la unicidad de tales factorizaciones adaptando la demostracion del Teorema Fundamental de la Aritmetica (1.5.4). En efecto, sean up1 pn = vq1 qm , con pi y qi irreducibles para todo i, y u v 2 D . Suponemos que n m y razonamos por induccion sobre n. Si n = 0 entonces m = 0, ya que los divisores de unidades son unidades, y no hay nada que demostrar. Supongamos que n > 0 y, la hipotesis de induccion. Por hipotesis, pn es primo, luego divide a algun qi y de hecho son asociados (>por que?) ademas, reordenando si es necesario, podemos suponer que i = m. Es decir, existe una unidad w tal que qm = wpn . Entonces up1 N pn;1 = (vw)q1 qm;1 : Por hipotesis de induccion se tiene n;1 = m;1 (luego n = m) y existe una biyeccion  : f1 : : :  n;1g ! f1 : : :  n ; 1g tal que pi y q (i) son asociados para cada i = 1 : : :  n ; 1. Ahora es evidente que  se extiende a una permutacion  de n tal que pi y q (i) son asociados para cada i = 1 : : :  n, y por lo tanto las factorizaciones iniciales son equivalentes. Z Pero no hemos dado todava ningun ejemplo de DFU, que no sea . Vamos a terminar esta seccion demostrando que todo DIP es un DFU. Esto, junto con los ejemplos de la seccion anterior, nos proporciona ejemplos de dominios de factorizacion unica. El recproco del teorema que sigue es falso: en el siguiente captulo veremos que X] es un DFU que no es un DIP. Z  EN DOMINIOS CAPITULO 3. DIVISIBILIDAD Y FACTORIZACION 78 Teorema 3.4.6 Todo dominio de ideales principales D es un dominio de factorizacion unica. Demostracion. Por las Proposiciones 3.4.5 y 3.2.3, basta con demostrar que D es un dominio de factorizacion. Por reduccion al absurdo suponemos que D no lo es, y vamos a construir, por recurrencia, una sucesion a1  a2 : : : de elementos de D que no admiten factorizacion y tales que (a1 )  (a2)  es una cadena estrictamente creciente de ideales de D. Para el primer paso simplemente elegimos un elemento arbitrario a1 de D que no admita factorizacion en irreducibles. Supongamos ahora que hemos elegido a1 : : :  an satisfaciendo las condiciones requeridas. Entonces an no es irreducible, luego existen x y 2 D n D tales que an = xy. Como an no es producto de irreducibles, al menos uno de los factores x o y (digamos que x) no es producto de irreducibles. Entonces, poniendo an+1 = x, tenemos (an )  (an+1 ) con la inclusion estricta porque y no es una unidad. + (ai ) (dejamos que el lector Una vez construida la sucesion (ai ), tomamos I = (a1  a2 : : :) = i2Z compruebe la igualdad anterior). Como D es un DIP, existe x 2 D tal que I = (x) en particular x 2 I = i2Z + (ai ) y por tanto existe un  ndice i tal que x 2 (ai ) como es claro que ai 2 (x), se tiene (ai ) = (x) = I y por lo tanto (ai ) = (ai+1 ), en contra de la construccion realizada. Este absurdo concluye la demostracion. 3.5 Aplicaciones de la factorizacion unica El hecho de que un dominio sea un DFU suele tener muchas aplicaciones en Teora de Numeros. En esta seccion vamos a ver tres de ellas: La determinacion de todos los numeros naturales que son suma de  dos cuadrados, la descripcion de los triangulos rectangulos con lados enteros, y la solucion del Ultimo Teorema de Fermat para el exponente 4. Consideraremos el dominio de los numeros enteros y el dominio i] de los enteros de Gauss, y en el camino daremos una descripcion precisa de los elementos irreducibles del segundo. Z Z Sumas de cuadrados en Z e irreducibles en Z i] Comencemos preguntandonos que numeros naturales son suma de dos cuadrados. El lector puede analizar los casos peque~nos y tratar de sacar conclusiones. Quizas no se haga evidente una regla general, pero si nos jamos en los numeros primos parece que solo el 2 y los que son congruentes con 1 modulo 4 son suma de dos cuadrados. De modo que puede ser interesante atacar primero el \caso primo", pero esto solo nos servira para el caso general si probamos que la condicion es multiplicativa. Esto es lo que hacemos en el primer lema, que ademas nos indica el papel que va a representar i] en este problema. Z Lema 3.5.1 ZZ Z Z 1. Un entero es la suma de dos cuadrados en si y solo si es la norma de un elemento de i] es decir, si es de la forma N(x) = x%x con x 2 i]. 2. Si los enteros n1  : : :  nr se expresan como suma de dos cuadrados en , lo mismo le ocurre a su producto n1 nr . Z Demostracion. El apartado 1 es claro. En 2, para cada i = 1 : : :  r existe xi 2 i] tal que ni = N(xi), y entonces, poniendo x = x1 xr , se tiene n1 nr = N(x1 ) lo que demuestra la armacion. N(xr ) = N(x1 xr ) = N(x) Para abordar el caso primo comenzamos con un resultado clasico sobre congruencias que usaremos como herramienta en la siguiente demostracion y que deducimos aqu de un resultado mas general: Lema 3.5.2 (Teorema de Wilson) Si K es un cuerpo nito entonces el producto de todos sus elementos no nulos vale ;1. En particular, si p es un entero positivo primo, se verica la congruencia (p ; 1)!  ;1 mod p .  UNICA  3.5. APLICACIONES DE LA FACTORIZACION 79 Demostracion. En K  = K n f0g denimos la relacion dada por a  b si a = b o a = b;1, que es claramente de equivalencia. En K  hay dos clases de equivalencia (solo una si la caracterstica de K es 2) que constan de un solo elemento: f1g y f;1g, y el resto de clases consta exactamente de dos elementos a y a;1 (no puede haber mas de dos elementos en cada clase por la unicidad del inverso, y hay dos porque a = a;1 implicara a2 = 1, o sea (a+1)(a ; 1) = a2 ; 1 = 0, y por lo tanto sera a = ;1 o a = 1). Como estas clases forman una particion de K  , al hacer el producto los pares equivalentes se van simplicando y solo sobrevive el ;1. (Si la caracterstica de K es 2 entonces 1 = ;1 y la demostracion Z sigue valiendo). El caso particular se obtiene considerando el cuerpo nito p. Z Proposicion 3.5.3 (Fermat) Para un entero positivo primo p, son equivalentes las condiciones: 1. p es suma de dos cuadrados en . 2. p 6 3 mod 4 3. p = 2 o p  1 mod 4. 4. La ecuacion X 2  ;1 mod p tiene soluciones enteras (o la ecuacion X 2 = ;1 las tiene en p). 5. p no es irreducible (ni primo) en i]. En este caso, la expresion de p como suma de dos cuadrados, p = a2 + b2, es unica salvo el orden y el signo de a y b. Z Z Demostracion. 1 implica 2. Como 02  22  0 mod 4 y 12  32  1 mod 4, una suma de dos cuadrados no puede ser congruente con 3 modulo 4. 2 implica 3. Un primo positivo o es 2 o es impar, y en este caso solo puede vale 1 o 3 modulo 4, pero la ultima posibilidad esta excluida por la hipotesis. 3 implica 4. Si p = 2, el 1 es solucion. Si p = 4t+1, usando el Teorema de Wilson (3.5.2) y operando modulo p, se tiene ;1  (p ; 1)!  (4t)! = 1 2 (2t ; 1)(2t)(2t + 1)(2t + 2) (4t ; 1)(4t)  1 2 (2t ; 1)(2t)(;(2t))(;(2t ; 1)) (;2)(;1) = (;1)2t((2t)!)2 = ((2t)!)2 de modo que (2t)! es solucion. 4 implica 5. Si existe n 2 con n2  ;1 mod p entonces p j n2 +1 en y por lo tanto p j (n+i)(n ; i) en i]. Como es claro que p no divide a n + i ni a n ; i en i], deducimos que p no es primo en i]. 5 implica 1. Por hipotesis existen x y 2 i], no unidades, con p = xy por lo tanto se tiene, en , p2 = N(p) = N(xy) = N(x)N(y), y como N(x) N(y) > 1 por no ser unidades, deducimos que N(x) = N(y) = p. En particular, si x = a + bi con a b 2 , se tiene p = a2 + b2. En cuanto a la unicidad, si p = a2 + b2 = c2 + d2, con a b c d 2 , entonces los elementos a  bi y c  di son irreducibles en i] por tener norma prima, de modo que p = (a +bi)(a ; bi) = (c +di)(c ; di) son factorizaciones en irreducibles de p. Por la unicidad de la factorizacion se deduce que c + di es asociado de a + bi o de a ; bi, y en consecuencia c + di es igual a uno de estos: a + bi ;a ; bi b ; ai ;b + ai a ; bi ;a + bi b + ai ;b ; ai lo que prueba la armacion del enunciado. La demostracion del caso general requiere el siguiente resultado, bien interesante por s mismo. Z Z Z Z Z Z Z Z Z Z Proposicion 3.5.4 Un entero de Gauss es irreducible si y solo si es de una de estas dos formas: Tipo A: p o pi, donde p es un entero positivo primo y p  3 mod 4. Tipo B: a + bi con a2 + b2 primo en . En este caso a2 + b2 6 3 mod 4. Z Z En consecuencia, un conjunto de representantes salvo asociados de los irreducibles de i] esta formado por los enteros primos positivos congruentes con 3 modulo 4 y por los elementos de norma prima del primer cuadrante.  EN DOMINIOS CAPITULO 3. DIVISIBILIDAD Y FACTORIZACION Z 80 Z Z Z Z Demostracion. Los elementos del tipo A son irreducibles en i] por la Proposicion 3.5.3, y los del tipo B lo son por tener norma prima (Ejemplos 3.1.10) y esta no es congruente con 3 modulo 4 por la Proposicion 3.5.3. Se trata de ver que no hay mas irreducibles en i] sea pues x = a + bi uno de ellos. Si a = 0 o b = 0 entonces x es asociado de un entero positivo, que obviamente debe ser primo y entonces debe ser congruente con 3 modulo 4 por la Proposicion 3.5.3 por consiguiente, x es del tipo A en este caso. En el otro caso, como x% tambien es irreducible en i] (>por que?), a2 + b2 = x%x es una factorizacion en irreducibles en i]. Si el entero a2 + b2 se factorizase en tendramos dos factorizaciones en irreducibles no equivalentes (>por que?) en i], que es un DFU. Deducimos que a2 + b2 es un entero primo y por lo tanto x es del tipo B. Z ZZ Z Z Ejemplo 3.5.5 Factorizaciones en i]. Z Z Z Factorizar en i] un elemento  que este en no es mucho mas difcil que factorizarlo en : Primero se factoriza  en los factores primos que no sean congruentes con 3 modulo 4 siguen siendo primos en i] y los otros son el producto de dos primos de i]. Por ejemplo, 252 = 22 32 7 = (1 + i)(1 ; i)]2 32 7 = (1 + i)2 (1 ; i)2 32 7 es la factorizacion prima de 252, y 46631 = 211 13 17 = 211 (22 + 32) (12 + 42) = 211(2 + 3i)(2 ; 3i)(1 + 4i)(1 ; 4i) es la factorizacion de 46631. Las factorizaciones de elementos de tambien son utiles para factorizar en i] elementos que no estan en . En efecto, dado  2 i] podemos factorizar N() =  % en i], y en esa factorizacion estaran todos los divisores primos de . Por ejemplo, si  = 5 + i entonces  % = 52 + 12 = 26 = 2 13 = (1 + i)(1 ; i)(3 + 2i)(3 ; 2i) y dividiendo  entre 1 + i se obtiene la factorizacion prima  = (1 + i)(3 ; 2i). Veamos un caso mas complicado, por ejemplo  = 33 + 4i. Entonces  % = 332 + 42 = 1105 = 5 13 17 = (2 + i)(2 ; i)(3 + 2i)(3 ; 2i)(4 + i)(4 ; i): De cada par de factores conjugados, uno divide a  y el otro a % . Dividiendo con resto se obtiene  14;5i 2+i = 14 ; 5i, que no es divisible por 3 + 2i pero s por su conjugado, de hecho 3;2i = 4 ; i, lo que nos da la factorizacion  = (2 + i)(3 ; 2i)(4 ; i): La Proposicion 3.5.4 nos permite, por n, caracterizar los enteros que son suma de dos cuadrados. Observese que no podemos decir nada (sencillo) sobre la unicidad, pues se tienen situaciones como 25 = 02 + 52 = 32 + 42, 169 = 02 + 132 = 52 + 122 o 1125 = 62 + 332 = 152 + 302. Teorema 3.5.6 Un entero positivo n es suma de dos cuadrados si y solo cada entero primo positivo congruente con 3 modulo 4 tiene multiplicidad par en n. Es decir, si en su descomposicion prima Z ZZ ZZ irredundante (con primos positivos), los primos congruentes con 3 modulo 4 tienen exponente par. Demostracion. Si, al factorizar n como producto de enteros primos positivos, los que son congruentes con 3 modulo 4 aparecen con exponente par, podemos agruparlos y escribir n = m2 p1 pr con cada pi en las condiciones de la Proposicion 3.5.3. Entonces n es suma de dos cuadrados por el Lema 3.5.1 (<porque m2 = m2 + 02 !). Recprocamente, si n es suma de dos cuadrados, entonces n = N(x) con x 2 i] (Lema 3.5.1). Por la Proposicion 3.5.4, la factorizacion en irreducibles de x en i]podemos ponerla como x = p1 pr z1 zs con cada pi del tipo A, que podemos suponer en , y cada zi del tipo B. Entonces n = N(x) = p21 p2r N(z1 ) N(zs ) y los N(zi ) son enteros primos no congruentes con 3 modulo 4. ZZ Z  UNICA  3.5. APLICACIONES DE LA FACTORIZACION 81 Ternas pitagoricas De nicion 3.5.7 Una terna (a b c) de enteros positivos tales que a2 +b2 = c2 se llama terna pitagorica. Los enteros a y b son los catetos de la terna, y c es su hipotenusa. La terna es primitiva si sus catetos son coprimos (es decir, mcd(a b) = 1) y b es par. Las siguientes ternas son pitagoricas, y solo las 3 primeras son primitivas: (3 4 5) (5 12 13) (55 48 73) (24 7 25) (45 24 51) (80 18 82): Por el Teorema de Pitagoras, cada terna pitagorica se corresponde con un triangulo rectangulo tal que la longitud de sus tres lados (con respecto a cierta unidad de medida) es un entero. El objetivo en este apartado es la descripcion de todas las ternas pitagoricas, y en consecuencia la de todos esos triangulos3. Lema 3.5.8 Toda terna pitagorica se obtiene a partir de una primitiva, multiplicando cada componente de esta por el mismo entero positivo y, si es necesario, cambiando el orden de sus catetos. Es decir, dada cualquier terna pitagorica (a b c), existen una terna pitagorica primitiva (a0  b0 c0) y un entero positivo t tales que a = ta0 b = tb0 c = tc0 o a = tb0 b = ta0  c = tc0 : Demostracion. Comencemos observando que, si (a b c) es una terna pitagorica arbitraria, entonces a y b no pueden ser simultaneamente impares, pues en ese caso la igualdad a2 + b2 = c2 implicara c2  2 mod 4, lo cual es imposible. Si t es el maximo comun divisor positivo de a y b, entonces es obvio que (a=t b=t c=t) es una terna pitagorica con catetos coprimos, y por el parrafo anterior uno (y solo uno) de ellos ha de ser par. Cambiando el orden de los catetos si es necesario, obtenemos la terna primitiva del enunciado. Las ternas primitivas asociadas a las que hemos mostrado anteriormente son: (3 4 5) (5 12 13) (55 48 73) (7 24 25) (15 8 17) (9 40 41): En el siguiente teorema describimos todas las ternas pitagoricas primitivas. Combinandolo con el Lema 3.5.8, tendremos descritas todas las ternas pitagoricas. Teorema 3.5.9 Si r > s > 0 son enteros coprimos y r 6 s mod 2, entonces (r2 ; s2  2rs r2 + s2 ) es una terna pitagorica primitiva, y toda terna pitagorica primitiva es de esa forma. Demostracion. Dados r y s en esas condiciones, se comprueba operando que la terna dada es pitagorica, y es obvio que su segundo cateto es par. Supongamos que un primo p divide a r2 ; s2 y a 2rs. La condicion r 6 s mod 2 implica que r2 ; s2 es impar, y por tanto p es un primo impar. De la condicion p j 2rs se deduce entonces que p j r o p j s. La primera opcion implica p j r2 , por lo que p j s2 (pues p j r2 ; s2 ) y en consecuencia p j s, lo cual es imposible porque r y s son coprimos. De igual modo se descarta la segunda opcion, y en consecuencia r2 ; s2 y 2rs son coprimos, por lo que la terna pitagorica (r2 ; s2  2rs r2 + s2 ) es primitiva. Veamos ahora que toda terna pitagorica primitiva (a b c) tiene la forma requerida. Comenzamos demostrando que los elementos a + bi y a ; bi son coprimos en i]. Supongamos que 2 i] es un divisor primo comun de a + bi y a ; bi. Entonces divide a la suma y a la diferencia, de donde j 2a y j 2b. Como mcd(a b) = 1 en i] (>por que?), debe ser j 2 y por tanto es asociado de 1 + i (>por que?). Pero esto implica que a  b mod 2 (Ejercicio 2.5.5), lo que contradice el hecho de que la terna (a b c) sea primitiva. Como en i] se tiene (a + bi)(a ; bi) = a2 + b2 = c2 , el parrafo anterior y la Proposicion 3.4.4 Z Z Z Z Z implican que a + bi es asociado de (r + si)2 para cierto elemento r + si de i]. Sustituyendo r + si por Se han encontrado listas de ternas pitagoricas en tablas babilonicas datadas entre los a~nos 1900 y 1600 antes de Cristo. Parece ser que Pitagoras visito Babilonia antes de establecerse en el sur de lo que hoy es Italia, donde nacio el \grupo de los Pitagoricos". 3  EN DOMINIOS CAPITULO 3. DIVISIBILIDAD Y FACTORIZACION 82 su opuesto, si hace falta, podemos asumir que r 0, y de hecho r > 0 pues de lo contrario a o b sera 0. Las opciones son a + bi = (r + si)2 = (r2 ; s2 ) + 2rsi] o a + bi = i(r + si)2 = 2rs ; (r2 ; s2 )i] y como b es par y a no lo es, se tiene una de las primeras. La opcion a + bi = (r + si)2 equivale a a = r2 ; s2  b = 2rs con s > 0 (pues b > 0) y r > s (pues a > 0). La opcion a + bi = ;(r + si)2 equivale a a = (;s)2 ; r2  b = 2(;s)r Z con ;s > 0 (pues b > 0) y ;s > r (pues a > 0). Ademas r y s son coprimos en , pues un divisor primo comun dividira claramente a los dos catetos. Finalmente, se tiene r 6 s mod 2, pues de lo contrario a sera par. En consecuencia (a b c) tiene la forma requerida. Es muy facil ver que la asignacion (r s) 7! (r2 ; s2  2rs r2 + s2 ) es inyectiva si r y s son positivos. Por tanto, esa asignacion dene una biyeccion entre el conjunto de los pares (r s) que satisfacen las condiciones del Teorema 3.5.9, y el conjunto de todas las ternas pitagoricas primitivas. Tambien es elemental ver que la biyeccion inversa viene dada por (a b c) 7! r a+c ! b 2  p2(a + c) : As, las ternas pitagoricas primitivas mostradas mas arriba se corresponden, en el mismo orden, con (2 1) (3 2) (8 3) (4 3) (4 1) (5 4):  El Ultimo Teorema de Fermat (exponente 4) La historia de este teorema es bien conocida. Las ternas pitagoricas son las soluciones enteras positivas de la ecuacion X 2 +Y 2 = Z 2 . Fermat, hacia 1637, armo sin demostrarlo que, para cualquier exponente n 3, la ecuacion Xn + Y n = Zn no tiene soluciones enteras no triviales (es decir, con XY Z 6= 0 es elemental ver que esto equivale a que no tenga soluciones positivas es decir, con X Y Z 2 +). Los esfuerzos realizados durante siglos por demostrar este resultado contribuyeron enormemente al desarrollo de la Teora de Numeros. Por citar un ejemplo, en algunas supuestas demostraciones se cometio el error de asumir que en cualquier subanillo de hay factorizacion unica, lo que dio origen al concepto abstracto de DFU. Finalmente, en la ultima decada del siglo XX, Andrew Wiles (con la ayuda de R. Taylor) demostro el teorema combinando diversas tecnicas que los citados esfuerzos haban \generado", y que han tenido aplicaciones en muchos otros contextos. En este apartado demostraremos el U ltimo Teorema de Fermat para el exponente n = 4. De hecho, veremos algo un poco mas fuerte: la ecuacion C Z X4 + Y 4 = Z2 (3.5.2) no posee soluciones positivas. La estrategia que emplearemos se debe al propio Fermat y se conoce como el \Metodo del Descenso Innito". Vamos a demostrar, usando los resultados sobre ternas pitagoricas, que a partir de una supuesta solucion positiva (a b c) de la ecuacion, se obtiene otra solucion positiva (A B C) con C < c. Repitiendo el proceso encontraramos una sucesion estrictamente decreciente innita de enteros positivos, lo cual es absurdo. Supongamos pues que (a b c) es una solucion positiva de (3.5.2) es decir, que a4 + b4 = c2. Si d = mcd(a b) > 1 entonces d2 j c y (A B C) = ( ad  db  dc2 ) es una solucion positiva de (3.5.2) con C < c, que es lo que buscamos. Podemos pues suponer que a y b son coprimos, e intercambiando a con b si es 3.6. PROBLEMAS 83 necesario podemos suponer que a es impar. Por tanto, (a2  b2 c) es una terna pitagorica primitiva, y por el Teorema 3.5.9 existen enteros r > s > 0, coprimos y de distinta paridad, tales que a2 = r2 ; s2  b2 = 2rs c = r2 + s2 : Considerando la primera de esas igualdades modulo 4 se deduce que r es impar y s es par. Ademas, todo divisor primo de s divide a b2 = 2rs y por tanto a b, por lo que a y s son coprimos. As, (a s r) es una terna pitagorica primitiva, y por consiguiente existen enteros coprimos u > v > 0 tales que a = u2 ; v2  s = 2uv Z r = u2 + v 2 : Se tiene pues b2 = 4ruv, y la Proposicion 3.4.4 nos dice que r, u y v son asociados de cuadrados en , y de hecho son cuadrados por ser positivos. Poniendo r =pC 2, u = A2 y v = B 2 con A B C > 0 se obtiene una solucion positiva (A B C) de (3.5.2) con C = r r2 < c, que es lo que buscamos. 3.6 Problemas 1. Sea f : A ! B un isomorsmo de anillos, y sean a a0 2 A. Demostrar (hasta estar seguro de que se va a saber demostrar el resto) los siguientes apartados: (a) a j a0 en A si y solo si f(a) j f(a0 ) en B. (b) a y a0 son asociados en A si y solo si f(a) y f(a0 ) son asociados en B. (c) d es el maximo comun divisor en A del subconjunto S si y solo si f(d) es el maximo comun divisor en B del subconjunto f(S). (d) d es el mnimo comun multiplo en A del subconjunto S si y solo si f(d) es el mnimo comun multiplo en B del subconjunto f(S). (e) Un elemento a 2 A es irreducible en A si y solo si f(a) es irreducible en B. (f) Un elemento a 2 A es primo en A si y solo si f(a) es primo en B. (g) Los elementos a y a0 de A son coprimos si y solo si los elementos f(a) y f(a0 ) de B son coprimos. (h) A es un DIP si y solo si B es un DIP. (i) A es un dominio eucldeo si y solo si B es un dominio eucldeo. (j) A es un dominio de factorizacion si y solo si B es un dominio de factorizacion. (k) A es un DFU si y solo si B es un DFU. 2. Calcular el cociente y el resto de las siguientes divisiones: (a) X 5 ; 1 entre X 2 + 3X, en 5X]. (b) X 7 + X 6 + X 5 + X 4 + X 3 + X 2 + X + 1 entre X 3 + X 2 + X + 1 en 2. 3. Calcular el maximo comun divisor y el mnimo comun multiplo de: (a) X 4 + 3X 3 + 4X 2 + 2X y X 5 + X 2 en X]. (b) 16 + 17i y 3 + 2i en i]. (c) 20 + 17i y 4 + 5i en i]. 4. Hallar el inverso de X 2 + X + 1 + (X 5 ; 2) en X]=(X 5 ; 2). Como aplicacion, racionalizar la siguiente expresion (es decir, transformarla en otra equivalente sin races en el denominador): Z ZZ R p5 Z Q p5 2p5; 3 : 4+ 2+1  EN DOMINIOS CAPITULO 3. DIVISIBILIDAD Y FACTORIZACION 84 5. Si D es un dominio y a b c 2 D, demostrar que: (a) Si existe d = mcd(a b) entonces existe mcd(a + bc b) y vale d. (b) Si existe mcm(a b) entonces existe mcd(a b). >Es cierto el recproco? (c) Si existen d = mcd(a b) y t = mcd(ca cb) entonces t es asociado de cd. (d) Si existe el maximo comun divisor de cualquier par de elementos de D entonces existe el mnimo comun multiplo de cualquier par de elementos de D. >Es cierto el recproco? 6. Sea D un dominio y supongamos que existe una aplicacion  : D ! que verica las tres propiedades siguientes: (ab) = (a)(b) para cualesquiera a b 2 D. (a) = 0 si y solo si a = 0. (a) = 1 si y solo si a es invertible. Demostrar que D es un dominio de factorizacion (no necesariamente unica), y deducir que pm] lo es para cualquier m 2 . p p 7. Factorizar el elemento 5 + 3 como producto de irreducibles en  3]. 8. Mostrar mediante ejemplos que las propiedades de la Proposicion 3.4.4 (excepto la primera) no son ciertas en general sobre un dominio de factorizacion. 9. Demostrar que si f : A ! B es un homomorsmo suprayectivo entre dominios de ideales principales, entonces f es un isomorsmo o B es un cuerpo. 10. Sea  una funcion eucldea en D tal que (xy) = (x)(y) para todo x y 2 D n f0g. Demostrar, para a 2 D, que si (a) es un numero primo diferente de (1), entonces a es irreducible en D. >Es cierto el recproco? N Z Z Z Z p Z 11. Sea ! = ;1+2 ;3 una raz cubica de 1 y sea D = !] = fa + b! : a b 2 g. Demostrar que D es un subanillo del anillo de numeros complejos y que (a + b!) = a2 ; ab + b2 es una funcion eucldea en D. Calcular las unidades de D. (Indicacion: Representar los elementos de D en el plano.) 12. *] Demostrar las armaciones siguientes: (a) Sean D un dominio, K su cuerpo de cocientes y  : K  !  una aplicacion que conserva productos y tal que (D n f0g)  . Entonces la restriccion de  a D n f0g es una funcion eucldea en D precisamente si para todo x 2 K n D existe y 2 D tal que (x ; y) < 1. (b) Sea m p un entero negativo libre de cuadrados y sea  : ppm] ! la aplicacion dada por (a + b m) = a2 ; mb2 . Entonces la restriccion de  a  m] n f0g es una funcion eucldea precisamente si m = ;1 o m = ;2. p (c) Si ademas m  1 mod 4, entonces la restriccion de  a  1+p2 m ] nf0g es una funcion eucldea precisamente si m = ;3, ;7 o ;11. >Prueba esto que  1+ 2;19 ] no es un dominio eucldeo? >Por que hemos hecho esta pregunta? p p (d) Las restricciones de  a  2] y a  3] son normas eucldeas. 13. >Para que valores de a b 2 son asociados en i] los elementos a + bi y a ; bi? 14. Determinar todos los irreducibles de i] con norma 5. 15. Encontrar todos los divisores de X 4 + 3X 3 + 4X 2 + X + 4 en 5X]. 16. En i], encontrar todos los pares de elementos que pueden aparecer como cociente y resto para la division de 1 + 2i entre 1 + i. Repetir para la division de 11 + 7i entre 2 + 5i. N Z ZZ Z Z Z Q QZ Q ZZ Z Z 3.6. PROBLEMAS 85 17. Encontrar todas las soluciones (X Y ) en i] de cada una de las ecuaciones siguientes: (a) (2 + 2i)X + (1 ; 7i)Y = 4 ; 6i. (b) (4 + 5i)X + (20 + 17i)Y = 0. (c) (11 ; 7i)X + (14 ; 3i)Y = 3 ; 4i. p p p 18. Calcular en  ;2] el maximo comun divisor de los elementos  = 5 ; 2 ;2 y = 1 + 5 ;2, as como la correspondiente identidad de Bezout. 19. Sea D un DIP, sea 0 6= a 2 D n D y sea a = pn1 1 pnr r la factorizacion prima irredundante de a en D. Demostrar que existe un isomorsmo de anillos D  D D (a) = (pn1 1 )   (pnr r ) : Z Z 20. Se pide, para los siguientes elementos de i]: 1 = 3 + i 2 = 4430 3 = 4437 4 = 4 + 3i 5 = 75 + 28i: (a) Expresar cada i como producto de irreducibles de i]. (b) Calcular el maximo comun divisor y el mnimo comun multiplo de cada dos de ellos. i] (c) Describir todos los ideales primos de cada uno de los anillos Z i . >Cuales de ellos son maximales? i] (d) Calcular la caracterstica de cada uno de los anillos Z i . (Indicacion: Usar el Problema 19 de este captulo, y el Problema 25 del Captulo 2.) 21. Dados a b 2 , demostrar que: (a) a + bi es multiplo de 1 + i si y solo si a  b mod 2. (b) a + bi es coprimo con 2 en i] si y solo si a 6 b mod 2. (c) Si a 6 b mod 2 entonces en i] se tiene mcd(a b) = mcd(a + bi a ; bi). (d) Si a y b son coprimos en y b es par entonces a + bi y a ; bi son coprimos en i]. 22. Demostrar que si A es un DIP y 0 6= b 2 A, entonces el numero de ideales de A que contienen a b es nito. 23. Sea A un DIP. (a) Probar que todo ideal propio de A se puede poner de forma unica, salvo el orden, como producto de ideales maximales. (b) Demostrar que un ideal q de A es primario precisamente si es de la forma (pn ) para algun irreducible p de A y algun numero natural n. (c) Demostrar que todo ideal propio I de A se puede poner de forma unica, salvo el orden, como un producto I = Q1 Qr en el que cada Qi es un ideal primario y, para cada i 6= j, se tiene Qi + Qj = A. 24. *] Un anillo en el que todos los ideales tienen un sistema generador nito se dice que es noetheriano. Demostrar que todo dominio noetheriano es un dominio de factorizacion. (Indicacion: Adaptar la demostracion del Teorema 3.4.6.) 25. Sea m 6= 1 un entero libre de cuadrados y sea R = pm] se pide: (a) Usando la igualdad (m + pm)(m ; pm) = m(m ; 1), demostrar que 2 no es primo en R. (b) Si m ;3, demostrar que 2 es irreducible en R y deducir que R no es un DFU. (c) Si m es un multiplo de 5, demostrar que 2 es irreducible en R y deducir que R no es DFU. (d) Si m  1 mod 4, demostrar que 2 es irreducible en R y deducir que R no es DFU. Z Z ZZ Z Z Z  EN DOMINIOS CAPITULO 3. DIVISIBILIDAD Y FACTORIZACION 86 ZZZ Z p (e) Encontrar dos factorizaciones de 4 esencialmente distintas en  ;3]. p (f) Encontrar dos factorizaciones de 6 esencialmente distintas en ;6]. p (g) Encontrar dos factorizaciones de 4 esencialmente distintas en  5]. p (h) Encontrar dos factorizaciones de 6 esencialmente distintas en  10]. p p 26. En el problema p 12 se ha visto que (a + b 2) = ja2 ; 2b2 j es una norma eucldea en  2], y por tanto p  p2] es un DFU.p>Contradice p a esta armacion el hecho de que se tenga la igualdad (5 + 2)(2 ; 2) = (11 ; 7 2)(2 + 2) con los cuatro factores irreducibles? 27. Sea q = p1 pr un entero libre de cuadrados (los pi son primos distintos). Por el Problema 45 del Captulo 2 sabemos que el subanillo de Z Z Q Z (q) = fa=b : mcd(q b) = 1g es un DIP, y por tanto es un DFU. Aqu vamos a describir la factorizacion de sus elementos. Demostrar que: (a) Un elemento a=b de (q) es invertible si y solo si mcd(q a) = 1. (b) Cada elemento no nulo de (q) se expresa de modo unico en la forma up1 1 pr r , donde u es invertible y cada i 2 . (c) Salvo asociados, p1 : : :  pn son los unicos elementos irreducibles de (q), y son todos ellos primos. Por tanto (q) es un DFU. (d) Como en (q) solo hay un numero nito de primos salvo asociados, un intento de adaptar la demostracion del Teorema de Euclides (1.5.9) a este caso debe fallar en algun paso. >En cual lo hace? (e) Si q es primo (es decir, si r = 1) entonces (q) es un dominio eucldeo y todo ideal no nulo de (q) es el ideal principal generado por q para cierto  2 . (Indicacion: Usa la factorizacion del apartado (b) para denir una funcion eucldea.) Encontrar todas las formas posibles de expresar 169 como suma de dos cuadrados de enteros positivos. Lo mismo para 1125 y para 2197. Fermat escribio: \>Puede encontrarse un numero entero cuyo cuadrado, p distinto de 25, cuando se aumente en 2 sea un cubo?". Vamos a trabajar en el DFU  ;2] para responder a esta pregunta. p p (a) Demuestra que, para cualquier entero impar y, los elementos y+ ;2 e y ; ;2 son coprimos p en  ;2]. p (b) Demuestra que si los enteros x y verican xp3 = y2 + 2 entonces el elemento y + ;2 es un cubo (no solo el asociado de un cubo) en  ;2]. (c) Demuestra que las unicas soluciones enteras de la ecuacion 3a2b ; 2b3 = 1 son b = 1 y a = 1. (d) Responde a la pregunta de Fermat. *] Sea D un dominio. Demostrar que el ideal (X 2  XY Y 2) del anillo de polinomios en dos indeterminadas DX Y ] no es principal. >Tiene un conjunto de generadores con dos elementos? Determinar los ideales de A = KX Y ]=(X 2 XY Y 2), siendo K un cuerpo y demostrar que A tiene ideales que no son principales. Demostrar el recproco del Teorema de Wilson: Si un entero p 2 verica (p ; 1)!  ;1 mod p entonces p es primo. ZZ N Z Z Z 28. 29. Z 30. 31. Bibliografa del captulo Z Z N Z Z Allenby 1], Clark 9], Delgado-Fuertes-Xambo 12], Dorronsoro-Hernandez 13], Jacobson 23], Sharpe 32]. Captulo 4 Anillos de polinomios Se estudian con detenimiento los anillos de polinomios, con especial dedicacion a los anillos de polinomios en una indeterminada con coecientes en un dominio de factorizacion unica o en un cuerpo. Introduccion En los captulos anteriores hemos usado los anillos de polinomios de modo informal para manejar algunos ejemplos. En este comenzamos con una denicion rigurosa del anillo de polinomios en una indeterminada sobre un anillo arbitrario, y observamos como el concepto de grado y la Propiedad Universal nos permiten obtener numerosas propiedades elementales de estos anillos. A continuacion estudiamos las races de un polinomio con coecientes en un dominio D, viendo que son importantes a la hora de factorizar el polinomio, que siempre existen (en cierto cuerpo que contiene a D) y que su numero esta acotado por el grado del polinomio. Entre los resultados elementales habremos visto que el anillo de polinomios AX] es un dominio eucldeo, o un dominio de ideales principales, si y solo si A es un cuerpo. Es natural preguntarse cuando es AX] un dominio de factorizacion unica, y buena parte del captulo la dedicamos a demostrar que esto ocurre precisamente cuando lo es A. Abordamos a continuacion el problema practico de factorizar polinomios con coecientes en un dominio de factorizacion unica (o en su cuerpo de fracciones, pues ambas factorizaciones se relacionan estrechamente), para lo que es necesario establecer criterios que permitan conrmar la irreducibilidad de un polinomio. El captulo termina con la denicion de los anillos de polinomios en un numero nito de indeterminadas y con el analisis de sus propiedades elementales, algunas de las cuales generalizan las correspondientes propiedades en el caso de una indeterminada. Objetivos del captulo Conocer la denicion formal de los anillos de polinomios en una indeterminada y alcanzar uidez en el manejo de sus operaciones, especialmente del producto y de la division con resto. Conocer y manejar las propiedades del grado y la Propiedad Universal. En particular, manejar los homomorsmos de sustitucion y comprender la diferencia entre polinomio y funcion polinomica. Interpretar las races de un polinomio f en terminos de los factores lineales de f, y saber calcular su multiplicidad. Dado un dominio de factorizacion unica D con cuerpo de fracciones K, conocer la relacion entre las factorizaciones de un polinomio en DX] y en KX]. Manejar metodos de factorizacion y criterios de irreducibilidad para polinomios sobre cuerpos y sobre dominios de factorizacion unica. Conocer la denicion formal y las propiedades basicas de los anillos de polinomios en un numero nito de indeterminadas. 87 CAPITULO 4. ANILLOS DE POLINOMIOS 88 Desarrollo de los contenidos 4.1 Deniciones y propiedades basicas Existen diversas formas equivalentes de denir los anillos de polinomios. Ya vimos una, poco rigurosa, en los Ejemplos 2.2.6. Una alternativa mas formal es la siguiente: Sea A un anillo. Un polinomio en una indeterminada con coecientes en A es una sucesion p = (pn)n2N= (p0  p1 p2 : : :) con cada pn en A y tal que pn = 0 para casi todo n 2 , lo que equivale a que exista n0 2 tal que pn = 0 para cada n n0 . Estas sucesiones se conocen como sucesiones casi nulas. El elemento pi 2 A se conoce como el coeciente de grado i del polinomio p, y p0 es el coeciente independiente de p. De la denicion se deduce la siguiente propiedad, obvia pero fundamental: Dos polinomios p = (pn )n2Ny q = (qn)n2Nson iguales precisamente si lo son coeciente a coeciente es decir, si pn = qn para cada n2 . Por ahora, llamaremos P (A) al conjunto de todos los polinomios en una indeterminada con coecientes en A. Vamos a denir en P (A) dos operaciones que lo convertiran en un anillo. Dados dos polinomios p = (pn )n2Ny q = (qn)n2N, la suma p + q se dene componente a componente es decir p + q = (pn + qn)n2N: La denicion del producto es mas compleja: el producto de p y q es un nuevo polinomio pq = r = (rn)n2N cuyo coeciente de grado n (para cada n 2 ) viene dado por N N rn = X i+j =n pi qj = n X i=0 N pi qn;i = p0qn + p1 qn;1 + N + pn;1q1 + pn q0: Antes de comprobar que estas operaciones hacen de P(A) un anillo, vamos a introducir una notacion que nos devolvera al concepto habitual de polinomio que hemos usado en los captulos previos. De esta forma habremos resuelto los problemas del rigor en la denicion de los anillos de polinomios sin perder la facilidad de manejo que aporta una notacion adecuada y tpica. Dado a 2 A, tambien denotaremos por a al polinomio a = (a 0 0 : : :) y le llamaremos polinomio constante. La aplicacion u : A ! P (A) dada por a 7! a = (a 0 0 : : :) es claramente inyectiva y nos permite identicar al anillo A con el subconjunto de P(A) formado por los polinomios constantes. Es decir, podemos interpretar u como una inclusion. Llamaremos indeterminada de P (A) al elemento X = (0 1 0 0: ::): Usando la denicion del producto y la notacion usual de exponentes, es facil ver que se tiene X 2 = (0 0 1 0 : : :) X 3 = (0 0 0 1 0 : : :) y as sucesivamente. Por tanto, si a 2 A y n 2 , tenemos aX n = (0 0 : : :  0 a 0 : ::) donde a aparece en el lugar correspondiente al subndice n (recuerdese que el primer subndice es el 0). A los elementos de la forma aX n se les llama monomios (de grado n). Para un polinomio arbitrario p = (pn )n2N2 P (A) se tiene p = (p0 p1 p2 : : :) = (p0 0 0 : : :) + (0 p1 0 : : :) + (0 0 p2 : : :) + = pP0 + p1 X + p2X 2 + = n2Npn X n  N  4.1. DEFINICIONES Y PROPIEDADES BASICAS 89 donde las sumas tienen sentido porque casi todos los sumandos son nulos. Como p esta determinado por sus coecientes, tenemos: Lema 4.1.1 Todo polinomio en una indeterminada con coecientes en A se escribe de modo unico como suma de monomios. A partir de ahora olvidaremos la notacion P(A) y denotaremos por AX] el conjunto de los polinomios en una indeterminada con coecientes en A. Con frecuencia usaremos la notacion p(X) para un polinomio, para destacar que nombre le estamos dando a la indeterminada (por supuesto, la eleccion del smbolo X es meramentePconvencional). Dado un polinomio p = i2NpiX i en AX], existe m 2 tal que pm+1 = pm+2 = = 0, luego p puede escribirse como un suma nita de monomios p= m X i=0 N piX i = p0 + p1 X + p2 X 2 + + pm;1 X m;1 + pm X m : Podemos pues escribir un polinomio de AX] como suma innita o nita de monomios, y usaremos en cada caso la notacion que mas nos convenga. En las sumas innitas entenderemos que casi todos los coecientes son nulos y en las nitas podremos omitir los sumandos con coeciente 0. Con la notacion de sumas de monomios, las operaciones en AX] pueden reenunciarse como X pn X n + n2N X qn X n = n2N X (pn + qn)X n n2N y ( X pn X n ) ( n2N X qnX n ) = n2N X X ( n2Ni+j =n pi qj )X n : El lector podra ahora comprobar que estas operaciones dotan a AX] de una estructura de anillo conmutativo (el anillo de polinomios en una indeterminada con coecientes en A) cuyos elementos nulo e identidad son los de A, vistos como polinomios constantes. Como ejemplo, desarrollamos P p X ia, continuaci o n la comprobaci o n de la asociatividad del producto. Dados tres polinomios p = i2N i P P q = j 2Nqj X j y r = k2Nrk X k , se tiene P P P  p(qr) = ( i2Npi X i ) n2N( j +k=n qj rk)X n P P P = m2N i+n=m pi ( j +k=n qj rk ) X m P P = m2N i+j +k=m piqj rk X m  y de modo analogo se obtiene la misma expresion para (pq)r, lo que prueba la asociatividad. Hablaremos en consecuencia de AX] como del anillo de polinomios en una indeterminada con coecientes en A. Es facil verPque A es un subanillo de AX]. Sea A un anillo y sea p = i2Npi X i 2 AX] un polinomio no nulo de AX]. Entonces, por denicion de polinomio, el conjunto fi 2 : pi 6= 0g no es vaco y esta acotado superiormente. Por tanto ese conjunto tiene un maximo, al que llamamos grado del polinomio p y denotamos por gr(p). Es decir, N N gr(p) = maxfi 2 : pi 6= 0g: El coeciente de mayor grado, pgr(p) , se conoce como el coeciente principal de p, y diremos que p es monico si su coeciente principal es 1. Por convenio, consideramos que el polinomio 0 tiene grado ;1 y coeciente principal 0. Es claro que los polinomios de grado 0 son precisamente los polinomios constantes no nulos. A veces llamaremos lineales a los polinomios de grado 1, cuadraticos a los de grado 2, cubicos a los de grado 3, etcetera. Ejercicio 4.1.2 Sean p = a0 + a1X + con an = 6 0 6= bm . Demostrar que: + an X n y q = b0 + b1X + + bm X m polinomios de AX], 1. gr(p + q) max(gr(p) gr(q)), con la desigualdad estricta si y solo si n = m y an + bm = 0. 2. gr(pq) gr(p) + gr(q), con igualdad si y solo si anbm 6= 0. CAPITULO 4. ANILLOS DE POLINOMIOS 90 3. Si an es regular (por ejemplo, si p es monico, o si A es un dominio), entonces se tiene gr(pq) = gr(p) + gr(q): 4. Las desigualdades de los apartados 1 y 2 pueden ser estrictas (buscar un ejemplo cuando A = Una consecuencia inmediata del Ejercicio 4.1.2 es: Z 6). Corolario 4.1.3 Un anillo de polinomios AX] es un dominio si y solo si lo es el anillo de coecientes A. En este caso se tiene AX] = A es decir, los polinomios invertibles de AX] son los polinomios constantes invertibles en A. En particular, los polinomios invertibles sobre un cuerpo son exactamente los de grado 0, y AX] nunca es un cuerpo. En vista de este resultado, es natural preguntarse que condiciones debera cumplir un anillo A para que el anillo de polinomios AX] sea un dominio eucldeo, un dominio de ideales principales o un dominio de factorizacion unica. Las dos primeras cuestiones las podemos resolver ya, y la ultima la estudiaremos en la Seccion 4.5. Proposicion 4.1.4 Para un anillo A, las condiciones siguientes son equivalentes: 1. AX] es un dominio eucldeo. 2. AX] es un dominio de ideales principales. 3. A es un cuerpo. En este caso, un polinomio no constante f 2 AX] es irreducible (o primo) si y solo si no es producto de dos polinomios de grado menor es decir, si una igualdad f = gh en AX] implica que gr(g) = gr(f) (y gr(h) = 0) o gr(h) = gr(f) (y gr(g) = 0). Demostracion. De la Proposicion 3.3.7 se deduce que 3 implica 1, y del Teorema 3.3.3 que 1 implica 2. Es facil ver que, si A es un dominio, el polinomio X es un elemento irreducible de AX]. Por tanto, del isomorsmo A  = AX]=(X) (Ejemplos 2.7.7) y de la Proposicion 3.2.3 se deduce que 2 implica 3. Dejamos que el lector demuestre la armacion sobre los polinomios irreducibles. 4.2 Propiedad Universal Hemos observado que un anillo A es un subanillo del anillo de polinomios AX], y por tanto la inclusion u : A ! AX] es un homomorsmo de anillos. Tambien es claro que el subanillo de AX] generado por A y X es todo AX]. Es decir, la indeterminada X y las constantes de A (las imagenes de u) generan todos los elementos de AX]. El siguiente resultado nos dice que AX] puede caracterizarse por una propiedad en la que solo intervienen X y u. Proposicion 4.2.1 Sea A un anillo y sean AX] y u : A ! AX] los que se acaban de describir. 1. (Propiedad Universal del Anillo de Polinomios, PUAP) Para todo homomorsmo de anillos f : A ! B y todo elemento b de B existe un unico homomorsmo de anillos f% : AX] ! B % = b. Se dice que f% completa de modo unico el diagrama tal que f%  u = f y f(X) A u ? AX] f qB 1 f 2. Si dos homomorsmos de anillos g h : AX] ! B coinciden sobre A y en X entonces son iguales. Es decir, si g  u = h  u y g(X) = h(X) entonces g = h. 4.2. PROPIEDAD UNIVERSAL 91 3. AX] y u estan determinados salvo isomorsmos por la PUAP. Explcitamente: supongamos que existen un homomorsmo de anillos v : A ! P y un elemento T 2 P tales que, para todo homomorsmo de anillos f : A ! B y todo elemento b 2 B , existe un unico homomorsmo de % ) = b. Entonces existe un isomorsmo : AX] ! P anillos f% : P ! B tal que f%  v = f y f(T tal que  u = v y (X) = T . existe un homomorsmo Demostracion. 1. Sean f : A ! B y b 2 B como en el enunciado. Si P % = b, entonces para un polinomio p = n2NpnX n , se tendra f% : AX] ! B tal que f%  u = f y f(X) % = f( % X u(pn )X n ) = X f(pn )bn: f(p) n2N % =P Por tanto, la aplicacion dada por f(p) n2Nf(pn )b n2N n es la unica que puede cumplir tales condiciones. El lector puede ahora comprobar que esta aplicacion f% es un homomorsmo de anillos, y es elemental % = b. ver que satisface f%  u = f y f(X) 2. Si ponemos f = g  u = h  u : A ! B, los homomorsmos g y h completan el diagrama del apartado 1. Por la unicidad se tiene g = h. 3. Sean v : A ! P y T 2 P como en 3. Aplicando 1 y la hipotesis de 3 encontramos homomorsmos v% : AX] ! P y u% : P ! AX] tales que v%  u = v y v%(X) = T u%  v = u y u%(T ) = X: Entonces la composicion u%  v% : AX] ! AX] verica (%u  v%)  u = u%  v = u y (%u  v%)(X) = u%(T) = X y por 2 se obtiene u%  v% = 1AX ] . De modo analogo, y observando que v y T verican una condicion similar a 2, se demuestra que v%  u% = 1P , con lo que v% es el isomorsmo que buscamos. Observacion 4.2.2 La PUAP admite una \version no conmutativa" que necesitaremos en el Captulo 10, y que es la siguiente (la demostracion es analoga a la anterior y se deja a cargo del lector): Sea u : A ! AX] como antes (A conmutativo), y sea B un anillo no necesariamente conmutativo. Sea f : A ! B un homomorsmo de anillos y sea b 2 B un elemento tal que f(a)b = bf(a) para cualquier a 2 A. Entonces existe un unico homomorsmo de anillos f% : AX] ! B tal que f%  u = f y f(X) = b. La utilidad de la PUAP estriba en que, dado un homomorsmo f : A ! B, nos permite crear un homomorsmo AX] ! B que \respeta" a f y que \se comporta bien" sobre un elemento b 2 B que nos interese. Los siguientes ejemplos son aplicaciones de la PUAP a ciertos homomorsmos que aparecen con frecuencia y son importantes tanto en este captulo como en algunos de los siguientes (y en otras muchas situaciones que no estudiaremos aqu). Ejemplos 4.2.3 Aplicaciones de la PUAP. 1. Sean A un subanillo de B y b 2 B. Aplicando la PUAP a la inclusion A ,! B obtenemos un homomorsmo Sb : AX] ! B que es la identidad sobre A (decimos a veces que ja los elementos de A) y tal que Sb (X) = b. Se le llama el homomorsmo de sustitucion (o de evaluacion) en b. Dado p = p(X) 2 AX], escribiremos a menudo p(b) en vez de Sb (X). Podemos describir explcitamente la accion de Sb en un polinomio: X Sb : AX] ! B p(X) = pn X n n2N  Sb (p) = p(b) = X pnbn: n2N 2. Sean A un anillo y a 2 A. Si en el ejemplo anterior tomamos B = AX] y b = X + a, obtenemos un homomorsmo AX] ! AX] dado por p(X) 7! p(X + a): Este homomorsmo es un automorsmo cuyo inverso viene dado por p(X) 7! p(X ; a) (>por que?). CAPITULO 4. ANILLOS DE POLINOMIOS 92 3. Todo homomorsmo de anillos f : A ! B induce un homomorsmo entre los correspondientes anillos de polinomios: Aplicandole la PUAP a la composicion de f con la inclusionPB ,! BX] % = X. Explcitamente, dado p = n2Npn X n obtenemos f% : AX] ! BX] tal que f% jA = f y f(X) en AX], se tiene % = X f(pn )X n : f(p) n2N Es facil ver que, si f es inyectivo o suprayectivo, entonces lo es f% como casos particulares de esta armacion se obtienen los dos ejemplos siguientes: 4. Si A es un subanillo de B entonces AX] es un subanillo de BX]. 5. Si I es un ideal del anillo A, la proyeccion : A ! A=I induce un homomorsmo suprayectivo % : AX] ! (A=I)X]. Si ponemos %a = a + I, el homomorsmo % viene dado explcitamente por %( X pn X n ) = n2N X pn X n : n2N A % se le llama el homomorsmo de reduccion de coecientes modulo I. Su nucleo, que es un ideal de AX], consiste en los polinomios con coecientes en I, y lo denotaremos por IX]. Observese que (A=I)X]  = AX] IX] . Ejercicio 4.2.4 Sea A un subanillo de B y sea Sb : AX] ! B el homomorsmo de sustitucion en cierto elemento b de B . Demostrar que: 1. Im Sb es el subanillo de B generado por A  fbg, y consiste en las \expresiones polinomicas en b con coecientes en A" es decir, en los elementos de la forma n X N i=0 Z C ai bi  donde n 2 y ai 2 A para cada i. Este subanillo se suele denotar por Ab]. p Z p Z Q 2. Si A = , B = y b = m (o b = 1+2 m con m  1 mod 4) para cierto p m 2 , la notacion anterior es compatible con la que se uso anteriormente (es decir,  m] representa el mismo subanillo atendiendo a cualquiera de las dos deniciones). Lo mismo ocurre si se toma A = . Es importante observar que, mientras que dos polinomios coinciden solo si lo hacen coeciente p;7] sea coeciente, en general no ocurre lo mismo con las expresiones polin o micas. Por ejemplo, en Z p tiene, poniendo b = ;7, 2 + 3b ; b2 + 4b3 + b4 = 58 ; 25b: Es conveniente en este punto hacer una precision sobre los polinomios. Hemos denido un polinomio p = p(X) 2 AX] como una sucesion de elementos de A (los coecientes). Sin embargo, el lector puede estar familiarizado con la interpretacion de un polinomio, digamos que con coecientes reales, como una funcion real de variable real. En ese caso, si es un lector inquieto, se habra preguntado ya si es posible hacer una interpretacion de este tipo en general es decir, para cualquier anillo de coecientes A. La respuesta es s, pero con un matiz importante. En efecto, jado un polinomio p 2 AX], podemos considerar la funcion polinomica determinada por p, que se dene como la aplicacion A ! A dada por a 7! p(a) (con la notacion usada para el homomorsmo de sustitucion). El matiz es el siguiente: en contra de lo que ocurre cuando A = , en general dos polinomios distintos pueden dar lugar a la misma funcion polinomica. Esto es claro si A es nito, pues entonces solo hay una cantidad nita de aplicaciones A ! A, mientras que hay una cantidad innita de polinomios en AX]. Como ejemplos concretos, el lector puede analizar lo que ocurre con los polinomios 0 y X + X 2 en 2X] o con los polinomios X 2 y X 4 en 4X] o con los polinomios X y X p en pX], cuando p es un primo positivo. Volveremos sobre esta discusion en el Corolario 4.3.5. Z R Z Z 4.3. RAICES DE POLINOMIOS 93 4.3 Races de polinomios Empezaremos esta seccion con el siguiente lema, que generaliza la Proposicion 3.3.7 y le a~nade una condicion de unicidad recuerdese que consideramos el polinomio cero como un polinomio de grado ;1. Lema 4.3.1 Sea A un anillo y sean f g 2 AX]. Si el coeciente principal de g es invertible en A, entonces existen dos unicos polinomios q r 2 AX] tales que f = gq + r y gr(r) < gr(g). En esta situacion, q y r se llaman cociente y resto de la division de f entre g. Demostracion. La existencia se demuestra como en la Proposicion 3.3.7. En cuanto a la unicidad, supongamos que f = gq1 + r1 = gq2 + r2 con gr(ri ) < gr(g) para cada i = 1 2. Como el termino principal de g es regular, del Ejercicio 4.1.2 se deduce que gr(g) + gr(q1 ; q2) = gr(g(q1 ; q2)) = gr(r2 ; r1 ) maxfgr(r2 ) gr(r1 )g < gr(g): Luego gr(q1 ; q2) < 0 y en consecuencia q1 = q2 , de donde r1 = r2 . Especialmente sencillo es el caso en el que se divide por un polinomio lineal y monico. Proposicion 4.3.2 Sean A un anillo, a 2 A y f 2 AX]. Entonces: 1. (Teorema del Resto) El resto de la division de f entre X ; a es f(a). 2. (Teorema de Runi) f es divisible por X ; a precisamente si f(a) = 0. Demostracion. Dividiendo f entre X ; a tenemos f = q(X ; a) + r con gr(r) < 1, por lo que r es constante y as r = r(a) = f(a) ; q(a)(a ; a) = f(a). Esto demuestra 1, y 2 es entonces inmediato. Diremos que a 2 A es una raz de f 2 AX] si f(a) = 0 (es decir, si X ; a divide a f). Por ejemplo, 0 es raz de f si y solo si f tiene coeciente independiente 0, y cualquier elemento de A es raz del polinomio 0. Fijemos a 2 A. Como, para cada k 2 , el polinomio (X ; a)k es monico de grado k, se tiene gr((X ; a)k q) = k +gr(q) para cada q 2 AX]. Por tanto, para cada f 2 AX] no nulo, existe un mayor m 2 tal que (X ; a)m divide a f. Este entero m, que verica 0 m gr(f), se llama la multiplicidad de a en f. Por el Teorema de Ru)ni, a es raz de f precisamente si m 1. Cuando m = 1 se dice que a es una raz simple de f, y cuando m > 1 se dice que a es una raz multiple de f. Ejercicio 4.3.3 Sean A un anillo y a 2 A. Demostrar que el polinomio X ; a es cancelable en AX], y deducir que m 2 es la multiplicidad de a en f 2 AX] si y solo si existe un polinomio q 2 AX] con f = (X ; a)m q y q(a) 6= 0. Cuando D es un dominio, del Teorema de Ru)ni se deduce que X ; a es primo para cualquier a 2 D. Esto es esencial en la demostracion del siguiente resultado. Proposicion 4.3.4 (Acotacion de races) Sean D un dominio y 0 6= f 2 DX]. Entonces: 1. Si a1  : : :  an 2 D son distintos dos a dos y 1 : : : Pn 1 son enteros con cada (X ; ai)i j f , entonces (X ; a1 )1 (X ; an )n j f . Por tanto ni=1 i gr(f). 2. La suma de las multiplicidades de todas las races de f es menor o igual que gr(f). En particular, el numero de races distintas de f es menor o igual que gr(f). Demostracion. P Es claro que basta con demostrar la primera armacion de 1, cosa que hacemos por induccion en s = ni=1 i con el caso s = 1 evidente. Cuando s > 1, usando la hipotesis (X ; a1 )1 j f y la hipotesis de induccion, sabemos que existen polinomios g y h tales que g(X ; a1 )1 = f = h(X ; a1)1 ;1 (X ; a2 )2 (X ; an)n : Cancelando (X ; a1)1 ;1 y usando el hecho de que X ; a1 es primo y no divide a ningun otro X ; ai (>por que?), deducimos que X ; a1 divide a h, y esto nos da el resultado. N N N CAPITULO 4. ANILLOS DE POLINOMIOS 94 Si D no es un dominio, siempre podemos encontrar un polinomio en DX] para el que falle la acotacion de races (es decir, \con mas races que grado"). En efecto, si 0 6= a b 2 D y ab = 0, entonces aX es un polinomio de grado 1 con al menos 2 races, 0 y b. Otro ejemplo se obtiene considerando el polinomio X 2 ; 1, que tiene 4 races en 8. El siguiente corolario evidente de la Proposicion 4.3.4 se conoce como el principio de las identidades polinomicas. Ya hemos comentado que su segundo apartado falla sobre cualquier anillo nito. Z Corolario 4.3.5 Sea D un dominio, y sean f g 2 DX]. Entonces: 1. Si las funciones polinomicas f g : D ! D coinciden en m puntos, con m > gr(f) y m > gr(g), entonces f = g (como polinomios). 2. Si D es innito entonces dos polinomios distintos denen funciones polinomicas distintas en D. El siguiente concepto es util para calcular multiplicidades: Si A es un anillo, la derivada de P = a0 + a1 X + + an X n 2 AX] se dene como D(P ) = P 0 = a1 + 2a2X + 3a3 X 2 + Z + nanX n;1 : Observese que la derivada no se ha denido a partir de ningun concepto topologico, y por ejemplo no es cierto en general que un polinomio con derivada nula sea constante (considerese por ejemplo X n 2 nX]). Sin embargo, esta derivada formal satisface las mismas propiedades algebraicas que la derivada del Analisis. Ejercicio 4.3.6 Dados a b 2 A y P Q 2 AX], demostrar que: 1. (aP + bQ)0 = aP 0 + bQ0. 2. (PQ)0 = P 0Q + PQ0. 3. (P n)0 = nP n;1P 0 . Proposicion 4.3.7 Una raz a 2 A de P 2 AX] es multiple precisamente si P 0(a) = 0. Demostracion. Si a es raz simple se tiene P = (X ; a)Q para cierto Q 2 AX] con Q(a) 6= 0, por lo que P 0 = Q + (X ; a)Q0 y as P 0(a) = Q(a) = 6 0. Si a es raz multiple se tiene P = (X ; a)2 Q para 0 cierto Q 2 AX], por lo que P = 2(X ; a)Q + (X ; a)2 Q0 y as P 0(a) = 0. N En dominios de caracterstica cero, la idea de la demostracion anterior puede usarse para determinar la multiplicidad de a en P (no solo para decidir si a es simple o multiple). Para ello, necesitamos considerar las derivadas sucesivas de un polinomio: Para cada n 2 se dene la derivada n-esima P (n) de P 2 AX], de forma recurrente, por las formulas: P (0) = P y P (n+1) = (P (n))0 : N Proposicion 4.3.8 Sea D un dominio de caracterstica 0, y sean P 2 DX] y a 2 D. Entonces la multiplicidad de a en P es el menor numero natural m 2 tal que P (m) (a) 6= 0. Demostracion. Haremos induccion en la multiplicidad m de a en P , con el caso m = 0 claro. Si m 1 entonces a es raz de P y por tanto P = (X ; a)Q para cierto Q 2 DX]. Entonces la multiplicidad de a en Q es m ; 1, y por hipotesis de induccion Q(i) (a) = 0 = 6 Q(m;1) (a) para todo i < m ; 1. Ademas, para cada t 1 se tiene P (t) = tQ(t;1) + (X ; a)Q(t): Ahora el lector podra completar facilmente la demostracion. Z La hipotesis sobre la caracterstica de D en la Proposicion 4.3.8 es necesaria. Por ejemplo, si p es un numero primo, K = p y P = X p , entonces P 0 = 0 y as P (n)(0) = 0 para todo n.  4.4. EXISTENCIA DE RAICES TEOREMA FUNDAMENTAL DEL ALGEBRA 95 4.4 Existencia de races Teorema Fundamental del A lgebra No todos los polinomios con coecientes en un anillo A tienen races en A. Por ejemplo, los polinomios de grado 0 no tienen ninguna raz, y un polinomio lineal aX + b (con a 6= 0) tiene una raz en A si y solo si a divide a b. En particular, todo polinomio lineal sobre un cuerpo tiene una raz, pero puede haber polinomios de grado positivo sin races: por ejemplo, X 2 + 1 no tiene races en , y X 3 ; 2 no las tiene en . Ejercicio 4.4.1 Sea K un cuerpo. Demostrar que las siguientes condiciones son equivalentes: 1. Todo polinomio no constante de KX] tiene una raz en K . 2. Los polinomios irreducibles de KX] son precisamente los de grado 1. 3. Todo polinomio no constante de KX] se factoriza como producto de polinomios lineales de KX]. Se dice que un cuerpo K es algebraicamente cerrado cuando verica las condiciones equivalentes del Ejercicio 4.4.1. Es facil encontrar ejemplos de cuerpos que no son algebraicamente cerrados. Por ejemplo, considerando el polinomio X 2 + 1 vemos que no lo son ni , y el polinomio X 2 + X + 1 nos dice que 2 tampoco lo es. Si p 3 es un entero primo entonces p no es algebraicamente cerrado, pues X p;1 + 1 no tiene races en p por el Teorema Peque~no de Fermat (1.8.2). Mas generalmente, ningun cuerpo nito es algebraicamente cerrado (Problema 17). Sin embargo, se tiene: R Q QZ R Z Teorema 4.4.2 (Teorema Fundamental del A lgebra) El cuerpo algebraicamente cerrado. C Z de los numeros complejos es El Teorema Fundamental del A lgebra puede demostrarse usando algunos resultados que no se explicaran hasta el nal del primer curso, en la asignatura de Topologa. En este sentido, esa demostracion se sale de los lmites de la asignatura de A lgebra Basica, pero la incluiremos (al nal de la seccion, para no interrumpir el discurso principal) anticipando esos resultados de Topologa. Del Teorema Fundamental del A lgebra se deduce que todo polinomio no constante con coecientes en , o tiene races en . De modo mas general, es posible demostrar que todo polinomio sobre un cuerpo tiene races \en algun sitio". Recuerdese que una extension de cuerpos no es mas que un homomorsmo de anillos f : K ! Q, donde K y Q son cuerpos. Un tal f es necesariamente inyectivo, y esto permite ver a K, identicado con la imagen de f, como un subcuerpo de Q. Teorema 4.4.3 (Kronecker) Sea K un cuerpo y sea P 2 KX] un polinomio no constante. Entonces existe una extension de cuerpos K ! K1 tal que K1 contiene una raz de P . Demostracion. Como KX] es un DIP (Proposicion 4.1.4) y P no es una unidad, P es divisible por un polinomio irreducible Q. Entonces el anillo cociente K1 = KX]=(Q) es un cuerpo en el que P + (Q) = 0. Entonces f : K ! K1 , la composicion de la inclusion K ,! KX] con la proyeccion KX] ! KX]=(Q), es una extension de cuerpos. Por ultimo,  = X + (Q) 2 K1 es una raz de P, pues si P = a0 + a1X + + anX n entonces P () = a0 + a1(X + (Q)) + a2(X + (Q))2 + + an(X + (Q))n = a0 + a1(X + (Q)) + a2(X 2 + (Q)) + + an(X n + (Q)) = (a0 + a1X + a2X 2 + + an X n ) + (Q) = P + (Q) = 0: ZQ R C Para un cuerpo arbitrario K, diremos que un polinomio no constante P 2 KX] se descompone completamente en K, o que es completamente descomponible en K, si se factoriza como producto de polinomios lineales de KX] es decir, si existen a1 : : : an 2 K (no necesariamente distintos) tales que P = u(X ; a1 ) (X ; an), donde u es el coeciente principal de P y n es el grado de P. El Teorema de Kronecker nos permite demostrar que todo polinomio con coecientes en un cuerpo K se descompone completamente en algun cuerpo que contiene a K como subcuerpo. CAPITULO 4. ANILLOS DE POLINOMIOS 96 Corolario 4.4.4 Sean K un cuerpo y P 2 KX] un polinomio no constante. Entonces existe una extension de cuerpos K ! K 0 tal que P se descompone completamente en K 0 . Demostracion. Razonamos por induccion sobre n = gr(P), con el caso n = 1 trivial. Si n > 1, por el Teorema de Kronecker (4.4.3), existe una extension K ! K1 tal que P tiene una raz a1 en K1, de modo que P = (X ; a1)Q para cierto polinomio Q 2 K1 X]. Por hipotesis de induccion, hay una extension K1 ! K 0 tal que Q = u(X ; a2) (X ; an) para ciertos a2  : : :  an 2 K 0 , y por tanto P = u(X ; a1 ) (X ; an). Componiendo ambas extensiones obtenemos la del enunciado. Observaciones 4.4.5 1. Como todo dominio es subanillo de su cuerpo de fracciones, los enunciados del Teorema de Kronecker (4.4.3) y del Corolario 4.4.4 pueden generalizarse asumiendo que K es un dominio, y sustituyendo las extensiones de cuerpos por homomorsmos de anillos desde K hacia un cuerpo.  2. Por el Teorema Fundamental del Algebra, es un cuerpo que contiene las races de todos los polinomios no constantes de X]. En vista de esto y del Corolario 4.4.4, es natural preguntarse si, dado un cuerpo arbitrario K , existira una extension K ! K% tal que todo polinomio no constante % . La respuesta es armativa, como se vera en el curso de K se descomponga totalmente en KX] de Ecuaciones Algebraicas, y ese resultado sera fundamental para un estudio en profundidad de los cuerpos. C R Demostracion del Teorema Fundamental del A lgebra C Como hemos comentado, el Teorema Fundamental del A lgebra (tambien conocido como Teorema de d'Alembert-Gauss1) arma que el cuerpo de los numeros complejos es algebraicamente cerrado, y en este apartado usaremos algunos resultados basicos de Topologa para dar una demostracion de este hecho basada en una incompleta de Argand, en 1814. Recordando la denicion de cuerpo algebraicamente cerrado, se trata de ver que, dado un polinomio C p(X) = a0 + a1X + a2 X 2 + + an X n de grado n 1 (an 6= 0) con coecientes complejos (ai 2 para cada i = 0 1 : : : n), existe un numero complejo z tal que p(z) = 0. Usaremos propiedades elementales de los numeros complejos, como las desigualdades entre modulos jz1 j ; jz2j jz1 + z2 j jz1 j + jz2j 1 Aunque a menudo se suele armar que la primera demostraci on completa del Teorema Fundamental del A lgebra es debida a Gauss, esto no es completamente cierto. En 1746 d'Alembert dio una demostracion que utilizaba la siguiente armacion de Newton: Sean p 2 C X ] y a 2 C tales que p(a) 6= 0. Entonces en todo crculo cerrado centrado en a hay un z 2 C tal que jp(z)j < jp(a)j. En dicho momento no exista ninguna ninguna demostracion de la armacion de Newton, por lo que la demostracion de d'Alembert no era completa. En 1799, Gauss presento una demostracion del teorema a la que le faltaba un detalle: Gauss considera las curvas Re (P (x)) = 0 e Im (P (x)) = 0, que representan las partes real e imaginaria de p(z) 2 C . Utilizando que P (z) crece en modulo como zn , donde n es el grado de P , Gauss prueba que dichas curvas intersecan a una circunferencia y que las intersecciones de dichas curvas con los puntos de la circunferencia aparecen de forma alterna es decir, moviendonos a lo largo de la circunferencia encontramos un punto de una de las curvas entre cada dos de la otra. Entonces Gauss arma que \sin duda" eso implica que dichas curvas se intersecan dentro del circulo limitado por la circunferencia. Gauss escribe: \Nadie, por lo que yo se, lo ha dudado nunca. Pero si alguien lo desea en otra ocasion intentare dar una demostracion que no dejara dudas". Realmente es difcil negar la armacion de Gauss sin embargo, parece ser que el propio Gauss no estaba completamente satisfecho porque continuo proporcionando otras demostraciones del Teorema Fundamental del A lgebra. En 1850, Puiseux demostro la armacion de Newton que completaba la prueba de D'Alembert. Por otro lado, la armacion \indudable" de Gauss es facilmentedemostrable a partir de las propiedadesbasicas del conceptode continuidad, que no fueron establecidas hasta 1874 por Weierstrass. Sin embargo, exista una demostracion de la armacion de Newton anterior incluso a la demostracion de Gauss. E sta es debida a Wessel en el a~no 1797. Por tanto, antes incluso de que Gauss diera su demostracion casi completa del Teorema Fundamental del A lgebra, exista una demostracion completa uniendo los resultados de d'Alembert y Wessel. La razon de que se asigne a Gauss la primera demostracion completa de dicho teorema es que la demostracion de Wessel permanecio oculta durante muchos a~nos.  4.4. EXISTENCIA DE RAICES TEOREMA FUNDAMENTAL DEL ALGEBRA 97 o el hecho de que todos ellos tienen races m-esimas para cualquier entero m 1 (esto se demuestra considerando la forma polar, o forma modulo-argumento, de un complejo, y aplicando el Teorema de Bolzano al polinomio X m ; r en el intervalo 0 r + 1] para demostrar que todo numero real positivo r tiene una raz m-esima). Tambien emplearemos los conceptos de lmite y continuidad. En particular, el hecho de que toda q funcion continua ! , por ejemplo, z 7! jp(z)j = + p(z)p(z), alcanza su mnimo en cualquier subconjunto cerrado y acotado de , y por tanto en cualquier \bola" fz 2 : jz j rg, donde r es un numero real positivo (Teorema de Weierstrass). El esquema de la demostracion, que desarrollaremos de inmediato, es el siguiente: Comenzamos viendo que la funcion z 7! jp(z)j alcanza su mnimo absoluto en para ello, se demuestra que jp(z)j \se hace grande" fuera de cierta bola fz 2 : jz j rg, y entonces el mnimo que alcanza jp(z)j en esa bola es de hecho un mnimo absoluto en . Bastara entonces ver que ese mnimo vale 0, y esto lo hacemos por reduccion al absurdo: si el mnimo no es 0, construimos una funcion ! cuyo mnimo absoluto vale 1, y sin embargo encontramos un punto en el que la misma funcion vale menos de 1. Vamos con los detalles: C R C CC C C CR Veamos, por induccion en el grado n, que jp(z)j se hace mas grande que cualquier numero real positivo fuera de cierta bola es decir, veamos que: Para cada real k 0, existe un real r 0 tal que jp(z)j > k para cada complejo jz j > r. En efecto, la expresion de p(X) se reesecribe como p(X) = a0 + Xq(X) donde q(X) = a1 + a2 X + + an X n;1 y entonces jp(z)j = jzq(z) + a0j jz j jq(z)j ; ja0 j para cada z 2 : Si n = 1 entonces q = a1 es constante y podemos tomar r = k +ja jaj 0j . En el caso general, la hipotesis de 1 induccion aplicada al polinomio q y a k0 = k+ja0j asegura que existe un real s 0 tal que jq(z)j > k+ja0j cuando jz j > s, y entonces es claro que jp(z)j > a0 cuando jz j > r = maxfs 1g. En particular, tomando k = ja0j, encontramos r 0 con jp(z)j > ja0j cuando jz j > r. Como la funcion jp(z)j es continua, alcanza un mnimo en la bola B = fz 2 : jz j rg es decir, existe z0 2 B tal que jp(z0 )j jp(z)j para cada z 2 B. La misma desigualdad se tiene cuando z 62 B, pues entonces jz j > r y as p(z) > ja0j = p(0) jp(z0)j. En consecuencia, jp(z)j alcanza un mnimo absoluto en z0 es decir, jp(z0 )j jp(z)j para cada z 2 . Es claro que p(X) tiene una raz si y solo si la tiene p(X + z0 ), y este tiene la ventaja de que su modulo alcanza un mnimo absoluto en el 0. Por tanto, sustituyendo p(X) por p(X + z0 ), podemos suponer que z0 = 0, y por tanto que jp(z)j jp(0)j = ja0j para cada z 2 . Si a0 = 0 hemos terminado, claramente, as que se trata de ver que la condicion a0 6= 0 nos lleva a una contradiccion. En este caso, dividir por a0 no va a cambiar el punto en el que se alcanza el mnimo, por lo que podemos suponer que a0 = 1. Excluyendo monomios con coeciente nulo, podemos escribir p(X) = 1 + am X m + am+1 X m+1 + + anX n (con am 6= 0) para cierto entero m con 1 m n. Sea ahora ! una raz m-esima de ;a;m1 (es decir, ! 2 verica !m = ;a;m1 ). Entonces p(!X) = 1 ; X m + (terminos de grado mayor que m) es decir, p(!X) = 1 ; X m + X m h(X) donde h(X) es cierto polinomio con h(0) = 0. Finalmente, vamos a encontrar un numero real t tal que jp(!t)j < 1, lo que nos dara la contradiccion buscada puesto que 1 es el mnimo absoluto de jp(z)j. Consideremos la funcion ! dada por t 7! jh(t)j. Considerando su lmite en x = 0 (que vale 0 por continuidad) encontramos un numero t en el intervalo (0 1) tal que jh(t)j < 1 (haciendo  = 1 en la formulacion usual del lmite). Entonces tambien tm y 1 ; tm estan en el intervalo (0 1), por lo que jp(!t)j j1 ; tm j + jtm h(t)j < 1 ; tm + tm 1 = 1 como queramos ver. C C C C C RR 98 CAPITULO 4. ANILLOS DE POLINOMIOS 4.5 Factorizacion unica en anillos de polinomios En toda esta seccion, excepto mencion expresa de lo contrario, D sera un DFU y K sera su cuerpo de fracciones. Estudiaremos la divisibilidad en el anillo de polinomios DX], hasta demostrar que DX] es tambien un DFU. Comenzamos con un resultado elemental: Ejercicio 4.5.1 Sea D un dominio (no necesariamente un DFU) y sea p 2 D. Demostrar que: 1. p es irreducible en D si y solo si lo es en DX]. 2. Si p es primo en DX] entonces lo es en D. Este ejercicio se usara para demostrar el recproco del resultado principal, y a su vez motiva el siguiente resultado, conocido como Lema de Gauss. Por otra parte, el paso clave en lo que sigue sera la demostracion de que todo irreducible es primo en DX], y podemos interpretar el Lema de Gauss como la solucion del problema para polinomios constantes: Lema 4.5.2 (Lema de Gauss) Para un elemento p 2 D, las condiciones siguientes son equivalentes: 1. p es irreducible (y primo) en D. 2. p es irreducible en DX]. 3. p es primo en DX]. Demostracion. Como D es un DFU, las dos versiones de 1 son equivalentes. 3 implica 2 es cierto en general (Proposicion 3.1.11), y 2 implica 1 se sigue del Ejercicio 4.5.1. Supongamos por tanto que p es primo en D, y veamos que lo es en DX]. Para ello, sean f = a0 + + an X n y g = b0 + + bm X m polinomios de DX] tales que p 6 j f y p 6 j g, y veamos que p 6 j fg. Por hipotesis, existen un menor ndice i tal que p 6 j ai , y un menor ndice j tal que p 6 j bj . El coeciente de grado i + j de fg es ci+j = a0 bi+j + + ai;1 bj +1 + aibj + ai+1bj ;1 + + ai+j b0 y las condiciones dadas implican que p divide a todos los sumandos excepto a ai bj , por lo que p 6 j ci+j y en consecuencia p 6 j fg. En lo que sigue vamos a explotar sistematicamente el hecho de que todo polinomio sobre D puede verse como un polinomio sobre su cuerpo de fracciones K. Para ello necesitamos un lema tecnico en el que usaremos la funcion ' : D n f0g ! que a cada 0 6= a 2 D le asocia el numero '(a) de factores irreducibles en la expresion de a como producto de irreducibles de D, contando repeticiones. Por ejemplo, si D = entonces '(12) = 3 y '(;80) = 5. Es claro que, si a b 2 D n f0g, entonces '(ab) = '(a) + '(b) y '(a) = 0 , a 2 D : Lema 4.5.3 Si a 2 D y f g h 2 DX] verican af = gh 6= 0, entonces existen g1 h1 2 DX] tales que f = g1 h1 gr(g1) = gr(g) gr(h1) = gr(h): Demostracion. Razonamos por induccion en '(a). Si '(a) = 0 podemos tomar g1 = a;1g y h1 = h. Si '(a) > 0, existen p b 2 D tales que a = pb y p es primo. Entonces p j af = gh en DX] y, por el Lema de Gauss (4.5.2), podemos asumir que p j g en DX]. Es decir, existe g% 2 DX] tal que g = p%g, de donde gr(g) = gr(%g ). Cancelando p en la igualdad af = gh obtenemos bf = g%h. Como '(b) = '(a) ; 1 < '(a), la hipotesis de induccion nos dice que existen g1 h1 2 DX] tales que f = g1h1, gr(g1 ) = gr(%g ), y gr(h1) = gr(h), lo que nos da el resultado. N Z Z El siguiente resultado relaciona la irreducibilidad de un polinomio sobre D con su irreducibilidad sobre K. Aunque su recproco es falso en general (piensese en 2X como polinomio sobre ), pronto veremos que es valido con una condicion extra sobre el polinomio (Proposicion 4.6.1).  E IRREDUCIBILIDAD DE POLINOMIOS 4.6. FACTORIZACION 99 Lema 4.5.4 Si f 2 DX] es irreducible en DX], entonces es irreducible (o primo) en KX]. Demostracion. Supongamos que f no es irreducible en KX]. Por la Proposicion 4.1.4, existen G H 2 KX] tales que f = GH gr(G) > 0 gr(H) > 0: Si 0 6= b 2 D es un multiplo comun de los denominadores de los coecientes de G, se tiene g = bG 2 DX], y analogamente existe 0 6= c 2 D tal que h = cH 2 DX]. Aplicando el Lema 4.5.3 a la igualdad (bc)f = gh obtenemos g1 h1 2 DX] tales que f = g1 h1, gr(g1) = gr(g) = gr(G) > 0, y gr(h1 ) = gr(h) = gr(H) > 0, lo que nos da una factorizacion no trivial de f en DX]. Podemos ya demostrar el resultado principal de esta seccion: Teorema 4.5.5 D es un DFU si y solo si lo es DX]. Demostracion. Supongamos primero que DX] es un DFU. Entonces D es un dominio (Corolario 4.1.3), y cada 0 = 6 a 2 D n D es producto de irreducibles de DX], que tendran grado 0 pues lo tiene a. Por el Ejercicio 4.5.1, esa sera una factorizacion de a en irreducibles de D. Del mismo ejercicio se deduce que todo irreducible de D es primo en D, por lo que D es un DFU. Supongamos ahora que D es un DFU y veamos que lo es DX]. Empezaremos demostrando que cada f = a0 + + aX n 2 DX] (con a 6= 0) no invertible es producto de irreducibles, y lo haremos por induccion en n + '(a). Observese que f es invertible si y solo si n + '(a) = 0. El caso n + '(a) = 1 se resuelve facilmente. Supongamos pues que n + '(a) > 1 y que f no es irreducible. Entonces existen g = b0 + + bm X m (b = bm 6= 0) y h = c0 + + ck X k (c = ck 6= 0) en DX], no invertibles, con f = gh. Entonces se tiene 0 < m + '(b) 0 < k + '(c) y n + '(a) = m + k + '(b) + '(c): En consecuencia, podemos aplicar la hipotesis de induccion a g y h, y pegando las factorizaciones as obtenidas conseguimos una factorizacion en irreducibles de f. Por la Proposicion 3.4.5, solo falta demostrar que todo irreducible f de DX] es primo, y por el Lema de Gauss podemos suponer que gr(f) 1. Sean pues g h 2 DX] tales que f j gh en DX], y veamos que f j g o f j h en DX]. Obviamente, f j gh en KX], y como f es primo en KX] por el Lema 4.5.4, podemos asumir que f j g en KX]. Es decir, existe G 2 KX] tal que g = fG, y si vemos que G 2 DX] habremos terminado. Para ello, tomando a 2 D con aG 2 DX] y '(a) mnimo, basta ver que '(a) = 0. Supongamos que '(a) > 0 y sean p b 2 D con a = pb y p primo. Entonces, en DX], se tiene p j ag = f(aG). Como p es primo en DX] (Lema de Gauss) y p 6 j f (pues f es irreducible y gr(f) 1), deducimos que p j aG en DX]. Si g1 2 DX] verica aG = pg1 entonces bG = g1 2 DX], contra la minimalidad de '(a), y esta contradiccion termina la demostracion. Z Del Teorema 4.5.5 se deduce que X] es un DFU pero no un DIP, lo que muestra que el recproco del Teorema 3.4.6 no es cierto. 4.6 Factorizacion e irreducibilidad de polinomios En esta seccion, como en la anterior, D denota un DFU y K su cuerpo de fracciones. El objetivo generico es factorizar polinomios en DX] y en KX], y para ello es necesario disponer de metodos que nos digan cuando un polinomio es irreducible. Como se vera, pocos de los resultados practicos que obtendremos nos dan condiciones necesarias y sucientes para que un polinomio sea irreducible2 . Comenzamos con un resultado que relaciona la irreducibilidad de polinomios en DX] y en KX], y que completa el Lema 4.5.4. Para ello introducimos una nocion que sera muy practica: Diremos que un polinomio f = a0 + + an X n 2 DX] es primitivo si mcd(a0  : : : an) = 1 es decir, si sus unicos divisores de grado 0 son las unidades de DX]. 2 En el Problema31 esbozaremosel llamado algoritmo de Kronecker, que s establece una condici onnecesariay suciente cuando D = Z. CAPITULO 4. ANILLOS DE POLINOMIOS 100 Proposicion 4.6.1 Para un polinomio primitivo f 2 DX], las condiciones siguientes son equivalentes: f es irreducible en DX]. f es irreducible en KX]. Si f = GH con G H 2 KX] entonces gr(G) = 0 o gr(H) = 0. Si f = gh con g h 2 DX] entonces gr(g) = 0 o gr(h) = 0. Demostracion. El Lema 4.5.4 y la Proposicion 4.1.4 aseguran que 1 implica 2 y que 2 implica 3, respectivamente, y es claro que 3 implica 4. Finalmente, como f es primitivo, sus unicos divisores de grado 0 son unidades, por lo que 4 implica 1. 1. 2. 3. 4. Z Z Asumiremos que disponemos de un metodo para factorizar los coecientes de D, y en particular para decidir si son irreducibles o no. Esto es teoricamente posible si D = o D = i] (y tambien lo es en la practica en los casos que se nos presentaran), y nos permite ademas decidir si un polinomio de DX] es o no primitivo. En general, dado un polinomio 0 6= f 2 DX], calcularemos el maximo comun divisor d 6= 0 de sus coecientes y obtendremos f = df1, con f1 2 DX] primitivo. El polinomio constante d es una unidad en KX], mientras que en DX] tiene la misma factorizacion en irreducibles que tenga como elemento de D. En cuanto a f1 , para decidir su irreducibilidad, la Proposicion 4.6.1 nos permite considerarlo como polinomio sobre D o sobre K segun nos convenga. Por tanto, es importante tener criterios de irreducibilidad como los que siguen para polinomios sobre cuerpos. Ejercicio 4.6.2 Sea K un cuerpo y sea f 2 KX]. Demostrar que: 1. Si gr(f) = 1 entonces f es irreducible en KX]. 2. Si gr(f) > 1 y f tiene una raz en K , entonces f no es irreducible en KX]. 3. Si gr(f) = 2 o 3 entonces f es irreducible en KX] si y solo si f no tiene races en K . 4. Si K es algebraicamente cerrado (por ejemplo, si K = ), entonces f es irreducible si y solo si gr(f) = 1. La no existencia de races no garantiza la irreducibilidad de polinomios de grado mayor que 3. Por ejemplo, X 4 + 2X 2 + 1 = (X 2 + 1)2 es reducible en X] pero no tiene races reales. La estrecha relacion entre y nos permite obtener el siguiente criterio de irreducibilidad para polinomios con coecientes reales. Notese que, en la practica, considerar solo polinomios monicos no supone ninguna restriccion. Proposicion 4.6.3 Sea f 2 X] un polinomio monico con coecientes reales. Entonces: 1. Si  es una raz de f en , el conjugado % tambien es una raz de f en , y las multiplicidades de  y % en f son iguales. 2. f es irreducible en X] precisamente si tiene grado 1 o es de la forma X 2 + bX + c con b2 < 4c. Demostracion. El automorsmo de conjugacion en induce un automorsmo X] ! X] que es la identidad sobre X], por lo que f coincide con su imagen por este automorsmo. Q Q Por tanto, si f = ni=1 (X ; i )ki es la factorizacion irredundante de f en X], entonces f = ni=1 (X ; i )ki , de donde se deduce 1. La condicion de 2 es claramente suciente para la irreducibilidad de f en X], y necesaria si grf 2. De 1 y de la igualdad (X ; )(X ; % ) = (X ; r ; si)(X ; r + si) = (X ; r)2 ; (si)2 = X 2 ; 2rX + (r2 + s2 ) (donde  = r + si) se deduce que los polinomios irreducibles de X] tienen grado menor o igual que 2, lo que termina la demostracion. R R RC RC R C C C C R R C C El Ejercicio 4.6.2 pone de maniesto la importancia de encontrar races de un polinomio para decidir si es irreducible. Cuando los coecientes estan en un DFU podemos seleccionar los \candidatos a races":  E IRREDUCIBILIDAD DE POLINOMIOS 4.6. FACTORIZACION 101 Proposicion 4.6.4 Sea D un DFU con cuerpo de fracciones K , y sea f = a0 +a1 X+ +anX n 2 DX] con an = 6 0. Entonces todas las races de f en K son de la forma r=s, donde r j a0 y s j an . Demostracion. Sea t = sr una raz de f con r s 2 D primos entre s. Multiplicando la igualdad f(t) = 0 por sn obtenemos a0 sn + a1rsn;1 + a2 r2sn;2 + + an;1rn;1s + an rn = 0 luego r j a0sn y s j an rn. Como r y s son coprimos, deducimos de la Proposicion 3.4.4 que r j a0 y s j an . Q RZ Q R Q Ejemplos 4.6.5 Factorizaciones de polinomios. Z Q 1. Las posibles races en del polinomio f = 3X 3 + X 2 + X ; 2 son 2, 1, 2=3 y 1=3, y de hecho f(2=3) = 0. Por tanto (X ; 2=3) j f, y as (3X ; 2) j f. Dividiendo se obtiene f = (3X ; 2)(X 2 + X + 1). Como ambos factores son primitivos sobre e irreducibles sobre y sobre , deducimos que la anterior es una factorizacion en irreducibles de f en cualquiera de los anillosp X], X] o X]. La factorizacion en X] es f = (3X ; 2)(X ; !)(X ; !% ), donde ! = ;1+2 ;3 . 2. El polinomio f = 6X 4 + 6X 2 + 18X ; 30 = 3 2 (X 4 + X 2 + 3X ; 5) tiene al 1 por raz, y dividiendo se tiene X 4 +X 2 +3X ; 5 = (X ; 1)(X 3 +X 2 +2X +5). El factor cubico es primitivo y no tiene races en (al sustituir 1 o 5 se obtiene un entero impar), por lo que f = 3 2 (X ; 1)(X 3 + X 2 + 2X + 5) y f = 6(X ; 1)(X 3 + X 2 + 2X + 5) son las factorizaciones de f en X] y en X], respectivamente (en la segunda el 6 no es un factor irreducible, sino una unidad). El polinomio cubico no es irreducible en X] ni en X]. De hecho, un analisis del crecimiento de la funcion polinomica f : ! nos lleva a la conclusion de que f tiene una raz real y dos complejas conjugadas. Para la determinacion explcita de estas races, vease el Problema 39. 3. El polinomio f = X 4 + X 3 + 2X 2 + X + 1 no tiene races racionales, pero esto no implica que sea irreducible sobre . De hecho, se tiene f(i) = 0, y por tanto (X ; i)(X + i) = X 2 + 1 divide a f el otro factor es X 2 + X + 1, por lo que f = (X 2 + 1)(X 2 + X + 1) es una factorizaci p on en irreducibles en X], X] o X], y f = (X ; i)(X + i)(X ; !)(X ; !% ) (con ! = ;1+2 ;3 ) es una factorizacion en X]. 4. Supongamos que el polinomio sin races racionales f = X 4 ; 2X 3 + 6X ; 3 no es irreducible en X]. Por la Proposicion 4.6.1, existen g h 2 X], ambos de grado 1, tales que f = gh. Podemos asumir que g y h son monicos (>por que?), y por tanto no pueden tener grado 1 (>por que?). En consecuencia, ambos tienen grado 2 y por tanto existen a b c d 2 tales que f = (X 2 + aX + b)(X 2 + cX + d). Igualando coecientes, se obtienen las ecuaciones bd = ;3 ad + bc = 6 b + ac + d = 0 a + c = ;2: La primera ecuacion nos da 4 opciones para los valores de b y d. Una de ellas es b = 1 y d = ;3, que sustituida en la segunda ecuacion y combinada con la cuarta nos dice que a = ;2 y c = 0 pero estos valores no satisfacen la tercera ecuacion. De modo similar se ve que las otras opciones tampoco funcionan, lo que signica que no existen tales a b c d 2 y en consecuencia f es irreducible en X], y por tanto tambien en X]. El ultimo ejemplo muestra lo penoso que puede resultar estudiar la irreducibilidad de un polinomio, incluso de grado bajo, con los metodos que hemos desarrollado hasta ahora. El resto de esta seccion lo dedicamos a presentar otros dos criterios de irreducibilidad para polinomios sobre un DFU que son a menudo utiles. En el primero de ellos usaremos los Ejemplos 4.2.3: Un homomorsmo de anillos : A ! B induce otro AX] ! BX] dado por X X f = ai X i 7! f  = (ai )X i : En general se tiene gr(f  ) gr(f), con igualdad si el coeciente principal de f no esta en Ker . Z C Z Q Q Z CQ R Z RR R C Z Q Z Z CAPITULO 4. ANILLOS DE POLINOMIOS 102 Proposicion 4.6.6 (Criterio de Reduccion) Sea : D ! K un homomorsmo de anillos, donde D es un DFU y K es un cuerpo, y sea f un polinomio primitivo de DX]. Si f  es irreducible en KX] y gr(f  ) = gr(f), entonces f es irreducible en DX]. Demostracion. Por la Proposicion 4.6.1 basta ver que, si f = gh con g h 2 DX], entonces gr(g) = 0 o gr(h) = 0. Sean a, b y c los coecientes principales de f, g y h, respectivamente. Entonces a = bc 62 Ker y por tanto b c 62 Ker , por lo que gr(g ) = gr(g) y gr(h ) = gr(h). Como K es un cuerpo y f  es irreducible en KX], la igualdad f  = g h implica que gr(g ) = 0 o gr(h ) = 0, de donde se sigue el resultado. ZZ Z Z Cuando consideramos la proyeccion ! p, el homomorsmo X] ! pX] viene dado por X X f = aiX i 7! f% = a%iX i  Z donde a% es la clase de a en p. Aplicando el Criterio de Reduccion se obtiene: Z Z Z Corolario 4.6.7 Sea p un entero primo y sea f = a0 + + anX n un polinomio primitivo de X]. Si p 6 j an y f% es irreducible en pX], entonces f es irreducible en X]. Ejemplos 4.6.8 Aplicaciones del Criterio de Reduccion. Z Q Z Z ZZ Z Z Z Z Z Z Z Z Z Z Z 1. Reduciendo modulo 2 el polinomio f = 7X 3 + 218X 2 + 121X + 625 obtenemos el polinomio f% = X 3 + X + 1 de 2X], que es irreducible porque no tiene races. Por tanto f es irreducible en X] (y en X]). 2. Reduciendo f = X 4 + 5X + 1 2 X] modulo 2 obtenemos f% = X 4 + X + 1 2 2X]. Como f% no tiene races en 2, si no fuera irreducible se factorizara como producto de dos polinomios irreducibles de grado 2 en 2X]. Pero en 2X] solo hay 4 polinomios de grado 2, y de ellos solo X 2 + X + 1 es irreducible (>por que?). Como f% no es el cuadrado de este, deducimos que f% es irreducible en 2X] y por tanto f es irreducible en X]. 3. Consideremos el polinomio f = X 5 ;X ;1 de X]. Reduciendo modulo 2 obtenemos un polinomio que es divisible por X 2 +X+1, por lo que no podemos aplicar el Criterio de Reduccion. Reduciendo modulo 3 obtenemos f% = X 5 +2X +2 2 3X], que no tiene races. Si no fuera irreducible tendra un factor irreducible de grado 2 es facil ver que los unicos irreducibles monicos de grado 2 de 3X] son X 2 + 1 X 2 + X ; 1 X 2 ; X ; 1: Comprobando que ninguno de ellos divide a f% deducimos que f% es irreducible en 3X], y por tanto f es irreducible en X]. 4. Dado el polinomio f = X 4 + 4X + 1 en X], se tiene f% = (X + 1)4 en 2X] y f% = (X + 2)(X 3 + X 2 + X + 2) en 3X], con el factor cubico irreducible porque no tiene races. Por tanto, no podemos aplicar el Criterio de Reduccion. Sin embargo, la factorizacion en 3X] nos va a permitir demostrar que f es irreducible en X]. En efecto, como f no tiene races en , si no fuera irreducible en X] se tendra f = gh con gr(g) = gr(h) = 2. Esto nos dara, en 3, la factorizacion f% = g%%h con gr(%g ) = gr(%h) = 2, incompatible con la factorizacion en irreducibles (unica salvo asociados) que acabamos de obtener. Z Z Z Z Q Z Veamos nuestro ultimo criterio de irreducibilidad: Proposicion 4.6.9 (Criterio de Eisenstein) Sea D un DFU y sea f = a0 + a1X + + anX n (con an = 6 0) un polinomio primitivo de DX]. Si existe un irreducible p 2 D tal que p j ai para todo i < n y p2 6 j a0  entonces f es irreducible en DX].  E IRREDUCIBILIDAD DE POLINOMIOS 4.6. FACTORIZACION 103 Demostracion. Veamos que, si f = gh en DX], entonces gr(g) = n o gr(h) = n. Pongamos g = b0 + + bm X m y h = c0 + + ck X k , con bm ck 6= 0. Como p2 6 j a0 = b0c0 , entonces p 6 j b0 o p 6 j c0 . Supongamos que se da la segunda opcion. Como f es primitivo se tiene p 6 j g, y por tanto existe i = minfj : p 6 j bj g: P Entonces p no divide a ai = ( ij;=01 bj ci;j ) + bic0 , y por tanto i = n, de modo que gr(g) = n. La opcion p 6 j b0 nos llevara a gr(h) = n, lo que demuestra el resultado. Ejemplos 4.6.10 Aplicaciones del Criterio de Eisenstein. 1. Sean a un entero y p un primo cuya multiplicidad en a es 1. Entonces X n ; a es irreducible. 2. Un argumento similar al del apartado 4 de los Ejemplos 4.6.5 nos permitira ver que el polinomio f = X 4 ; 3X 3 + 6X ; 3 es irreducible en X]. Ahora podemos asegurar lo mismo con menos trabajo aplicando el Criterio de Eisenstein con p = 3. 3. A menudo, el Criterio de Eisenstein se combina con un automorsmo de X] de sustitucion en X+a (Ejemplos 4.2.3). Por ejemplo, el criterio no es aplicable a f(X) = X 4 +4X 3 +10X 2 +12X+7, pero s se puede aplicar (con p = 2) a f(X ; 1) = X 4 +4X 2 +2. Por tanto f(X ; 1) es irreducible, y en consecuencia lo es f(X). 4. Dado un entero n 3, las races en del polinomio X n ; 1 se llaman races n-esimas de la unidad (o de 1). Considerando la interpretacion geometrica de la multiplicacion en , es facil ver que estas races son exactamente los n vertices del n-agono regular inscrito en el crculo unidad de que tiene un vertice en la posicion del 1. Estos numeros complejos son utiles en muy diversas circunstancias. El polinomio X n ; 1 se factoriza como X n ; 1 = (X ; 1)*n (X) donde *n (X) = X n;1 + X n;2 + + X 2 + X + 1: El polinomio *n(X) se conoce como el n-esimo polinomio ciclotomico, y sus races son las races n-esimas de 1 distintas de 1. *n (X) no es en general irreducible sobre (por ejemplo, *4 (X) es divisible por X + 1), pero s lo es cuando n = p es primo. Como en el apartado anterior, esto quedara demostrado si podemos aplicar el Criterio de Eisenstein a *p (X + 1). Ahora bien, *p (X) = (X n ; 1)=(X ; 1), y por tanto  p   p  p p;1 (X + 1) p ; 1 p ; 2 p ; 3 *p (X + 1) = = X + p ; 1 X + p ; 2 X + + 2 X + p: X ; Cuando 1 i < p, el primo p no divide a i! ni a (p ; i)!, y por tanto s divide a pi = i!(pp!; i)! , por lo que podemos aplicar el Criterio de Eisenstein, como queramos. Cerramos la seccion con un ejemplo en el que se combinan casi todos los metodos empleados anteriormente. Ejemplo 4.6.11 Factorizaciones de 5X 4 + 5 en distintos anillos. Vamos a factorizar en irreducibles el polinomio g = 5X 4 + 5 considerando cada uno de los siguientes anillos de coecientes: , , , , 2 y 3. Como g = 5f, donde f = X 4 + 1, podemos factorizar f = X 4 + 1 y a~nadir un 5 a cada una de las factorizaciones que siguen. En todas ellas ese 5 sera una unidad, excepto en la de X], donde sera un factor irreducible mas. En 2X] se tiene X 4 + 1 = (X + 1)4 , luego f es producto de cuatro irreducibles iguales. Como f no tiene races en 3, si no es irreducible en 3X] sera producto de dos irreducibles de grado 2. Como tenemos la lista de estos irreducibles (Ejemplos 4.6.8), podemos dividir f por cada uno de ellos hasta obtener la factorizacion f = (X 2 + X + 2)(X 2 + 2X + 2). La sencillez del polinomio X 4 + 1 nos permite calcular \a mano" sus races complejas. En efecto, la ecuacion X 4 + 1 = 0 se reescribe como (X 2 )2 = ;1, de donde X 2 = i y as X = 1p i . Por tanto 2 1 ; i) f = (X ; 1p+ i )(X ; 1p; i )(X ; ;1p+ i )(X ; ;p 2 2 2 2 Z Z C C C Q Z ZQ R C Z Z Z Z Z CAPITULO 4. ANILLOS DE POLINOMIOS C 104 R es la factorizacion de f como producto de irreducibles en X]. Agrupando las races complejas como en la demostracion de la Proposicion 4.6.3, obtenemos la factorizacion en irreducibles de X]:     p p f = (X ; p1 )2 ; ( pi )2 (X + p1 )2 ; ( pi )2 = (X 2 ; 2X + 1)(X 2 + 2X + 1) 2 2 2 2 Esta factorizacion tambien puede obtenerse directamente, \completando cuadrados" como sigue: Q p p p f = X 4 + 2X 2 + 1 ; 2X 2 = (X 2 + 1)2 ; ( 2X)2 = (X 2 + 1 + 2X)(X 2 + 1 ; 2X): R RQ Z Si f tuviera una factorizacion no trivial en X], ep sta sera tambien una factorizacion no trivial en X]. Como es claro que cualquier asociado de X 2  2X + 1 en X] tiene coecientes irracionales, no existe tal factorizacion y, en consecuencia, f es irreducible en X], y tambien en X] por ser primitivo. Tambien podemos asegurar la irreducibilidad de f aplicando el Criterio de Eisenstein a f(X + 1) = X 4 + 4X 3 + 6X 2 + 4X + 2. En conclusion, g es irreducible en X], mientras que en X] se factoriza como producto de dos irreducibles, g = 5f. Q Z 4.7 Polinomios en varias indeterminadas Dados un anillo A y un entero n 2, denimos el anillo de polinomios en n indeterminadas con coecientes en A, denotado por AX1  : : :  Xn], mediante la formula recurrente AX1  : : :  Xn ] = AX1 : : :  Xn;1]Xn]: Los elementos X1  : : :  Xn de AX1  : : :  Xn] se llaman indeterminadas y los elementos de AX1 : : :  Xn] se llaman polinomios en n indeterminadas. Por induccion a partir del Corolario 4.1.3, de la Proposicion 4.1.4 y del Teorema 4.5.5, se obtiene: Proposicion 4.7.1 Para un anillo A y un entero positivo n se verican: 1. 2. 3. 4. AX1 : : :  Xn ] nunca es un cuerpo. AX1 : : :  Xn ] es un dominio si y solo si lo es A. Si n 2 entonces AX1  : : :  Xn] no es un DIP. AX1 : : :  Xn ] es un DFU si y solo si lo es A. N Si a 2 A e i = (i1  : : : in) 2 n, el elemento aX1i1 tipo i y coeciente a. Xnin de AX1  : : :  Xn ] se llama monomio de Lema 4.7.2 Sean A un anillo y n un entero positivo. Entonces todo elemento p de AX1  : : :  Xn] se escribe de forma unica como suma de monomios de distinto tipo, casi todos con coeciente nulo. Es decir, se tiene una unica expresion p= con pi = 0 para casi todo i = (i1  : : :  in) 2 X N i2Nn n. pi X1i1 Xnin (4.7.1) Demostracion. Aplicamos induccion en n, con el caso n = 1Presuelto por el Lema 4.1.1. Cuando n > 1, un elemento de AX1 : : :  Xn ] es, por denicion, de la forma t2Npt Xnt con cada pt 2 AX1  : : :  Xn;1] y casi todos los pt nulos. Por hipotesis de induccion, cada pt se expresa como pt = X ; (i1 ::: in 1 )2Nn ; (pt)(i1 ::: in 1 ) X1i1 ;1 Xnin;11  ; donde cada (pt )(i1 ::: in 1 ) esta en A y casi todos son nulos. Deniendo pi = (pin )(i1 ::: in i = (i1  : : : in)) tenemos la expresion deseada. ; ;1 ) (para 4.7. POLINOMIOS EN VARIAS INDETERMINADAS 105 N Recprocamente, una expresion como (4.7.1) puede reescribirse como un polinomio en Xn con coeP cientes en AX1 : : :  Xn;1] sin mas que denir cada coeciente como pt = pi X1i1 Xnin;11 , con la suma extendida a todos los i = (i1  : : :  in) 2 n con in = t. Usando esto es sencillo demostrar que estas expresiones son unicas, asumiendo que lo son en AX1  : : :  Xn;1]. ; Usando el Lema 4.7.2 se demuestra: Proposicion 4.7.3 Sean A un anillo, n 1 un entero y u : A ! AX1  : : : Xn] la inclusion. 1. (PUAP en n indeterminadas) Dados un homomorsmo de anillos f : A ! B y n elementos b1  : : :  bn 2 B (no necesariamente distintos) existe un unico homomorsmo de anillos % j ) = bj para cada j = 1 : : : n. f% : AX1  : : :  Xn] ! B tal que f%  u = f y f(X 2. Si dos homomorsmos de anillos g h : AX1  : : :  Xn] ! B coinciden sobre A y en Xj para cada j = 1 : : : n entonces son iguales. 3. La PUAP en n indeterminadas determina AX1  : : :  Xn ] salvo isomorsmos. Explcitamente: supongamos que existen un anillo P con elementos T1  : : : Tn y un homomorsmo de anillos v : A ! P tales que, dados un homomorsmo de anillos f : A ! B y elementos b1  : : :  bn 2 B , % j ) = bj para existe un unico homomorsmo de anillos f% : P ! B tal que f%  v = f y f(T cada j = 1 : : : n. Entonces existe un isomorsmo : AX1 : : : Xn ] ! P tal que  u = v y (Xj ) = Tj para cada j = 1 : : : n. Como en el caso de una indeterminada, se tiene: Ejemplos 4.7.4 Aplicaciones de la PUAP en n indeterminadas. 1. Dados anillos A  B y elementos b1  : : : bn 2 B, existe un homomorsmo S : AX1  : : : Xn] ! B que es la identidad sobre A y tal que S(Xj ) = bj para cada j =P1 : : : n. Dado p 2 AX1  : : : Xn], escribiremos a menudo p(b1  : : : bn) en lugar de S(p). Si p = i2Nn piX1i1 Xnin es la expresion de p como suma de monomios, entonces S(p) = p(b1 : : : bn) = X i2Nn N pi bi11 binn : La imagen de este homomorsmo es el subanillo de B generado por A  fb1 : : : bng. 2. Sea A un anillo y sea  una biyeccion del conjunto n = f1 : : : ng en s mismo con inversa  = ;1 . Si en el ejemplo anterior tomamos B = AX1  : : : Xn] y bj = X (j ) , obtenemos un homomorsmo % : AX1  : : : Xn ] ! AX1  : : : Xn] que \permuta las indeterminadas". Es claro que % es de hecho un automorsmo con inverso %. Usando estos isomorsmos y la denicion de los anillos de polinomios en varias indeterminadas, es facil establecer isomorsmos AX1  : : :  Xn Y1 : : :  Ym ]  = AX1  : : :  Xn ]Y1 : : :  Ym ]  = AY1  : : :  Ym ]X1 : : :  Xn ] por lo que, en la practica, no hay que distinguir entre estos anillos. 3. Todo homomorsmo de anillos f : A ! B induce un homomorsmo f% : AX1  : : : Xn] ! % j ) = Xj para cada j = 1 : : : n. Si BXP 1  : : : Xn] que coincide con f sobre A y verica f(X i i 1 n p = i2Nn pi X1 Xn es la expresion de p como suma de monomios, entonces % = X f(pi )X1i1 Xnin : f(p) i2Nn Q Veamos como pueden usarse las identicaciones del apartado 2 de los Ejemplos 4.7.4. Q Q Q Ejemplo 4.7.5 El Criterio de Eisenstein aplicado a polinomios en dos indeterminadas El polinomio f = X 3 Y + X 2 Y 2 ; X 2 + Y 3 + Y 2 2 XY ] puede considerarse como un polinomio en X]Y], poniendo f = Y 3 + (X 2 + 1)Y 2 + X 3 Y ; X 2 , o como un polinomio en Y ]X], poniendo f = Y X 3 +(Y 2 ; 1)X 2 +(Y 3 +Y 2 ). A esta ultima expresion le podemos aplicar el Criterio de Eisenstein con el polinomio irreducible p = Y + 1 2 Y] para deducir que f es irreducible en X Y]. Q CAPITULO 4. ANILLOS DE POLINOMIOS 106 Por denicion, el grado de un monomio aX1i1 Xnin de AX1 : : :  Xn ] es i1 + + in . El grado gr(p) de un polinomio p 6= 0 de AX1  : : :  Xn ] se dene como el mayor de los grados de los monomios que aparecen con coeciente no nulo en la expresion de p como suma de monomios de distinto tipo. Es claro que, dados dos polinomios p y q, se tiene gr(p + q) maxfgr(p) gr(q)g y gr(pq) gr(p) + gr(q): Sin embargo, no es tan facil como en el caso de una indeterminada ver que, cuando A es un dominio, la segunda desigualdad es de hecho una igualdad. Para esto, y para otras cosas, es interesante considerar el siguiente concepto: Un polinomio p 6= 0 de AX1  : : :  Xn] se dice homogeneo de grado n 0 si es suma de monomios de grado n. Por ejemplo, de los polinomios de X YZ] X 2 Y + Y 3 ; 3XY Z + 6Y Z 2  X 6 + Y 6 + Z 6 + X 3 Y 3 + X 3 Z 3 + Y 3 Z 3 XY Z + X + Y + Z los dos primeros son homogeneos (de grados 3 y 6, respectivamente) y el ultimo no lo es. Proposicion 4.7.6 Dados un anillo A y un entero n 1, todo polinomio de AX1 : : :  Xn] se escribe Z de modo unico como suma de polinomios homogeneos de distintos grados. P i1 Demostraci P on. Si p = i2Nn piX1 i1 i1 ++in =j piX1 hj = buscada. Xnin , es Xnin es la expresion de p como suma de monomios y ponemos claro que p = h0 + h1 + + hk (donde k = gr(p)) es la expresion Corolario 4.7.7 Si D es un dominio y n 1, se tiene gr(pq) = gr(p) + gr(q) para cualesquiera p q 2 DX1  : : :  Xn]. Ademas, los grupos de unidades de DX1  : : :  Xn ] y de D coinciden es decir, DX1  : : :  Xn] = D . Demostracion. Dados polinomios p y q de grados n y m, sean p = h0 +h1 + +hn y q = l0 +l1 + +lm sus expresiones como suma de polinomios homogeneos. Entonces pq = h0l0 + (h0 l1 + h1l0 ) + + (hn;1lm + hn lm;1 ) + hnlm es la expresion de pq como suma de polinomios homogeneos. Como DX1  : : : Xn] es un dominio y hn 6= 0 6= lm , deducimos que hnlm 6= 0, lo que nos da la igualdad deseada. La ultima armacion se demuestra ahora como en el caso de una indeterminada. RR Terminaremos haciendo algunos comentarios sobre las \races" de polinomios en varias indeterminadas, que no se suelen llamar races sino ceros. Para simplicar, consideraremos solo polinomios de grado 2 con coecientes en , y el lector podra imaginar las generalizaciones pertinentes. Un elemento (a b) 2 2 es un cero del polinomio f 2 X Y ] si f(a b) = 0. Denotaremos por V (f) el conjunto de todos los ceros de f. Por ejemplo, V (XY ) consiste en los ejes de 2, y V (X 2 + Y 2 ; 2 ) es la circunferencia de radio  centrada en el origen. Estos ejemplos muestran que el grado de un polinomio en varias indeterminadas no acota en modo alguno el numero de ceros del polinomio, lo que da a estos una gran riqueza geometrica. En efecto, el \dibujo" de las races de un polinomio de X] es muy simple: son unos puntos aislados en la recta real. En X Y ] (y, por supuesto, en tres o mas indeterminadas), los ejemplos anteriores son solo una muestra de las curvas que pueden obtenerse como conjuntos de ceros de un polinomio. De hecho, es claro que V (fg) = V (f)  V (g), y as V (XY (X 2 + Y 2 ; 1)) es la union de los ejes con la circunferencia de radio 1 centrada en el origen. Por otra parte, se suele denotar por V (f g) la interseccion de V (f) con V (g), de modo que V (X ; a Y ; b) = f(a b)g y V (XY X 2 + Y 2 ; 1) = f(1 0) (;1 0) (0 1) (0 ;1)g: Sobre se tiene V (f g) = V (f 2 + g2 ), lo que permite construir polinomios con conjuntos nitos de ceros, como (X ; a)2 + (Y ; b)2 . En general, la complejidad de V (f) aumenta con el grado de f. El estudio de los conjuntos V (f) con gr(f) = 1 (y de sus intersecciones) corresponde a la llamada Geometra Afn, y el caso de grados arbitrarios corresponde a la Geometra Algebraica. R R R R R 4.8. PROBLEMAS 107 4.8 Problemas 1. Sea A un anillo y sean a u 2 A. Demostrar que el homomorsmo AX] ! AX] de sustitucion en uX + a es un automorsmo si y solo si u es invertible en A. 2. Sea P 2 2X]. Demostrar que X ; 1 divide a P precisamente si P tiene un numero par de coecientes no nulos. 3. Justicar la regla de Runi para el calculo del cociente y el resto en la division de p = p0 +p1X + : : : + pnX n entre X ; a. La regla esta representada por la tabla pn pn;1 pn;2 : : : p1 p0 a 0 aqn;1 aqn;2 : : : aq1 aq0 qn;1 qn;2 qn;3 : : : q0 r Z 4. 5. 6. 7. 8. 9. en la que los qi se obtienen, de izquierda a derecha, sumando los dos elementos que estan encima. Entonces q = q0 + q1X + + qn;1X n;1 es el cociente de la division de p entre X ; a, y r es su resto. p >Para que cuerpos es valida la formula usual ( ;b 2ba2 ;4ac ) para el calculo de las races de un polinomio aX 2 + bX + c de grado 2? Q Sea p un entero primo. Demostrar que los polinomios X p ; X y pi=1 (X ; i) de pX] son iguales y deducir una nueva demostracion del Teorema de Wilson: (p ; 1)!  ;1 mod p. (Indicacion: Para la primera parte, considerar las races de ambos polinomios.) Hemos observado que la Proposicion 4.3.4 no se verica para polinomios sobre un anillo que no sea un dominio. Comprobar que en este caso ni siquiera se verica la armacion sobre la nitud del numero de races es decir, dar un ejemplo de un polinomio no nulo en una indeterminada con innitas races. *] Sea A un anillo. Demostrar que si P 2 AX] es un divisor de cero en AX], entonces existe 0 6= a 2 A tal que aP = 0. (Indicacion: Elegir un polinomio Q 6= 0 de grado mnimo entre los que satisfacen PQ = 0 y demostrar por induccion que pi Q = 0, donde p0 p1 : : :  pn son los coecientes de P .) Si K es un cuerpo de caracterstica 0, >que polinomios P 2 KX] verican P 0 = 0? >Y si la caracterstica es un primo p? Sea D un dominio y sea P 2 DX] el polinomio Z Z P = nX n+2 ; (n + 2)X n+1 + (n + 2)X ; n (n 2 +). Demostrar que la multiplicidad de 1 como raz de P es al menos 3, y que es exactamente 3 si la caracterstica de D es 0. (Advertencia: El caso de caracterstica 2 ha de ser considerado aparte.) 10. Demostrar que si 0 6= a 2 K, siendo K un cuerpo de caracterstica 0, entonces X n ; a no tiene races multiples en ningun cuerpo que contenga a K como subcuerpo. >Que se puede armar si K es un cuerpo de caracterstica p, con p primo. 11. Dados dos polinomios P Q 2 AX], se dene su composicion P(Q) de forma natural utilizando la PUAP. Demostrar que se satisface la regla de la cadena para la derivada de la composicion: (P (Q))0 = P 0(Q) Q0. 12. Demostrar la formula de Leibnitz para el calculo de las derivadas sucesivas de un producto de polinomios: n n X (n;i)Q(i) : (PQ)(n) = i P i=0 CAPITULO 4. ANILLOS DE POLINOMIOS 108 13. Sea K un cuerpo de caracterstica 0 y sea P = a0 + a1 X + + an X n 2 KX]. (a) Demostrar que, para cada i 0, se tiene ai = i1! P (i)(0). (b) Demostrar que, para todo b 2 K, se verica la formula de Taylor3 P(X ; b) = n P (i)(b) X i=0 i i! (X ; b) : (c) Deducir del apartado anterior una demostracion alternativa de la Proposicion 4.3.8. 14. Sea K un cuerpo y sean P1  : : :  Pr polinomios de KX]. Demostrar que existe una extension de cuerpos K ! K 0 tal que cada Pi se descompone totalmente en K 0 X]. 15. Sean K un cuerpo, P 2 KX] un polinomio no constante y K1 = KX]=(P). >Cual es la dimension de K1 como espacio vectorial sobre K? Deducir que el cuerpo K 0 construido en el Corolario 4.4.4 tiene dimension nita como espacio vectorial sobre K. 16. Sea K un cuerpo y sean a0  a1 : : :  an 2 K distintos y b0 b1 : : :  bn 2 K. Demostrar que P (X) = 17. n Y (X ; a ) X i br r=0 i6=r ar ; ai es el unico polinomio de KX] de grado n que verica P(ai) = bi para todo i. La formula para P se conoce con el nombre de formula de interpolacion de Lagrange. Demostrar que, si K es un cuerpo, entonces KX] tiene innitos elementos irreducibles. Deducir que: (a) Todo cuerpo algebraicamente cerrado es innito. (b) Si K es nito, entonces en KX] existen polinomios irreducibles de grado arbitrariamente grande (es decir, para cada n 2 +, existe un polinomio irreducible de grado mayor o igual que n). *] Sea K un subcuerpo de K 0 , y sean f g 2 KX]. Demostrar: (a) El maximo comun divisor de f y g en KX] tambien es su maximo comun divisor en K 0 X]. (b) Si f es irreducible en KX] y en K 0 hay una raz comun de f y g, entonces f j g en KX]. *] Sea K un cuerpo, sea f 2 KX]. Demostrar que f tiene una raz doble en alguna extension de K si y solo si f y su derivada f 0 no son coprimos en KX]. (Indicacion: Usar los Problemas 14 y 18.) *] Sea F un cuerpo y K un subcuerpo de F. Se dice que F es algebraico sobre K (o que F es una extension algebraica de K) si todo elemento de F es la raz de un polinomio no nulo de KX]. (a) Demostrar que F es algebraico sobre K precisamente si F no contiene un subanillo isomorfo a KX]. (b) Demostrar que si F no es algebraico sobre K, entonces F contiene un subcuerpo isomorfo al cuerpo de cocientes K(X) de KX]. (c) Demostrar que si F tiene dimension nita como espacio vectorial sobre K, entonces F es algebraico sobre K. >Es cierto que, si D es un DFU y b es un elemento de D, entonces solo hay una cantidad nita de ideales de D que contienen a b? Z 18. 19. 20. 21. En el estudio de funciones reales de variable real, la formula de Taylor se usa para aproximar una funcion f por un polinomio en un entorno de un punto b. Este problema nos dice que, como es de esperar, si f es un polinomio entonces se obtiene una igualdad, y no solo una aproximacion de f . 3 4.8. PROBLEMAS 109 22. Dar un ejemplo de un ideal primo no nulo de un DFU que no sea maximal. 23. Demostrar que toda raz racional de un polinomio monico con coecientes enteros es entera. 24. Sea D un DFU y sea f = ao + a1X + + anX n un polinomio primitivo en DX]. Demostrar que, si existe un irreducible p 2 D tal que p j ai para todo i > 0 25. 26. 27. 28. 29. 30. y p2 6 j an  entonces f es irreducible en DX] (es decir, el Criterio de Eisenstein se puede aplicar \al reves"). Descomponer en factores irreducibles el polinomio X 4 ; 4 en cada uno de los siguientes anillos: X], X], X], 2X] y 3X]. Descomponer los siguientes anillos cociente como producto de anillos \conocidos": (a) X] modulo el ideal principal generado por el polinomio X 3 ; X 2 + X ; 1. (b) X] modulo el ideal principal generado por el polinomio X 3 ; X 2 + X ; 1. (c) X] modulo el ideal principal generado por el polinomio 3X 2 ; 6. Calcular el maximo comun divisor y el mnimo comun multiplo en X] de las siguientes parejas de polinomios: (a) X 3 ; 6X 2 + X + 4 y X 5 ; 6X + 1. (b) X 2 + 1 y X 6 + X 3 + X + 1. (c) 26X 2 ; 104X + 104 y 195X 2 + 65X ; 910. Demostrar que los siguientes polinomios son irreducibles en los anillos que se indican: (a) X 4 + X + 1, 4X 3 ; 3X ; 12 , X 4 + 1, X 6 + X 3 + 1, X 3 + 6X + 3X + 3, X 5 ; 5X + 15 y X 4 + 5X + 12 en X]. (b) X 2 + X + 1 en 2X]. (c) X 2 + Y 2 ; 1 y X 5 Y 3 ; X 3 + XY 2 ; Y 2 + 1 en X Y]. (d) X 4 + X + a con a impar, en X]. (e) X 5 + 3aX 4 ; 4X + 4 con a 2 , en X]. (f) Y 3 + X 2 Y 2 + X 3 Y + X, en DX Y ] donde D es un DFU arbitrario. Factorizar los siguientes polinomios en los anillos que se indican: (a) 3X 4 ; 3X 2 + 6, en X], X], X] y X]. (b) X 3 + 3X 2 + 3X + 4 en 5X]. Decidir cuales de los siguientes polinomios son irreducibles en los anillos que se indican: (a) 2X 2 + 2X + 2 en X], X] y 5X]. (b) X 4 + 2 en 7X] y X]. (c) X 3 ; 18X 2 + 106X ; 203 en X] y X]. (d) X 5 + X + 2 en X], X] y 3X]. (e) X 5 + X ; 2 en X], X] y 3X]. (f) 2X 5 ; 6X 3 + 9X 2 ; 15 en X] y X]. (g) X 4 + 15X 3 + 7 en X]. (h) X n ; p, donde n > 0 y p es un entero primo con p  1 mod 3, en X], X] y 3X]. Q R C Z Z RQ Q QZ QZ Q Z ZQ R C Z Q Z Z Q Z Q RR QQ ZZ Z Q Z Z Q R Q Z CAPITULO 4. ANILLOS DE POLINOMIOS Z Z Z 110 Z Z 31. *] El metodo de Kronecker para factorizar en X] funciona como sigue: Dado 0 6= f 2 X], podemos limitarnos a buscar divisores g de f con gr(g) m, donde m es la parte entera de gr(f)=2. Dado un tal g, para cada a 2 se tiene g(a) j f(a) en . Si jamos enteros a0  : : : am con f(ai ) 6= 0, los posibles valores de cada g(ai ) quedan limitados por la condicion g(ai ) j f(ai ). Combinando esto con la formula de interpolacion de Lagrange (Problema 16) obtenemos un numero nito de candidatos a divisores de f. Si alguno esta en X] y divide a f, tenemos un primer paso en la factorizacion y repetimos el metodo. En caso contrario, f es ya irreducible. Un ejemplo: Si f = X 4 + X + 1, entonces m = 2 y podemos considerar los enteros a0 = ;1, a1 = 0 y a2 = 1. Entonces g(;1) j 1, g(0) j 1 y g(1) j 3, por lo que hay 8 posibilidades para g. Se pide: Calcular estos 8 polinomios, comprobar que ninguno divide a f en X], y deducir que f es irreducible. Esto da idea de lo inecaz que es el metodo si se emplea \a mano". Sin embargo, el metodo es facil de programar, y es ecaz para polinomios \de grado no muy grande y con coecientes no muy grandes". De hecho, el metodo funciona si sustituimos por un DFU innito D (para poder elegir los ai cuando m es grande) con D nito (para que cada f(ai ) tenga un numero nito de divisores) en el que haya un metodo para factorizar elementos. 32. *] En el Problema 27 del Captulo 2 se ha visto que el cardinal de un cuerpo nito K es una potencia de un numero primo (de hecho, una potencia de la caracterstica de K). En este problema, jado un entero primo positivo p, vamos a ver que existen cuerpos4 de cardinal pn para cada n 2 +. (a) Sea K un cuerpo de caracterstica p (entero positivo primo), y sea n 2 +. Demostrar que el n p conjunto de las races en K del polinomio X ; X es un subcuerpo nito de K. (Indicacion: Usar el Problema 26 del Captulo 2.) (b) Deducir que, para cada n 2 +, existe un cuerpo de cardinal pn . 33. *] Construir cuerpos de 4, 8, 16, 9, 27 y 121 elementos. 34. Calcular todos los polinomios monicos irreducibles de grado 4 en KX], cuando K es cada uno de los cuerpos p con p primo menor o igual que 11. >Te atreves con los cuerpos K construidos en el Problema 33? 35. Sea A un anillo. Demostrar que si P 2 AX1 : : :  Xn] tiene grado 1 y uno de los coeciente es una unidad de A, entonces AX1  : : :  Xn]=(P)  = AX1  : : :  Xn;1]. 36. *] Sean K un cuerpo y P = p0 + p1X + + pnX n (con pn 6= 0) un polinomio de KX]. Se llama homogeneizado de P al polinomio de KX Y ] Z Z Z Z Z Z Pb = p0Y n + p1 XY n;1 + + pn;1X n;1Y + pn X n : Demostrar: b 1) = P . (a) Si P 2 KX] entonces P(X b (b) Si P Q 2 KX] entonces PdQ = Pb Q. (c) Si R 2 KX Y ] es homogeneo de grado n y el coeciente de X n en R es diferente de 0, entonces R(X 1) = R. (d) Si R1 R2 2 KX Y ] y R1R2 es homogeneo, entonces R1 y R2 son homogeneos. (e) Si K es algebraicamente cerrado entonces todo polinomio homogeneo en KX Y ] es producto de polinomios homogeneos de grado 1. (f) Escribir Y 3 ; 3Y 2 X + 2X 3 2 X] como producto de polinomios homogeneos de grado 1. \ Q De hecho, salvo isomorsmos, existe un unico cuerpo de cardinal q para cada entero positivo q > 1 que sea potencia de primo. La demostracion de este hecho se vera en la asignatura de tercer curso Ecuaciones Algebraicas. Este unico cuerpo de cardinal q se suele denotar por Fq  en particular, para p primo, se tiene Fp = Zp. 4 4.8. PROBLEMAS 111 37. *] Sea K un cuerpo y sea P 2 KX Y ]. Supongamos que el coeciente principal de P, considerado como polinomio en KX]Y ], no es divisible por X ; 1. Demostrar que, si P (X 1) es irreducible en KX], entonces P (X Y ) es irreducible en KX Y ]. 38. *] Demostrar que si K es un cuerpo y P Q 2 KX Y ] son coprimos, entonces el conjunto V (P) \ V (Q) = f(a b) 2 K 2 : P(a b) = Q(a b) = 0g es nito. (Indicacion: Aplicar el Lema de Bezout en el dominio eucldeo K(X)Y ], donde K(X) es el cuerpo de cocientes de KX].) 39. *] En este problema se van a dar las claves para resolver las ecuaciones de tercer grado sobre un cuerpo K algebraicamente cerrado y de caracterstica distinta de 2 y de 3 (en particular, sobre el cuerpo complejo ). El lector debera comprobar las armaciones que se hacen. (a) Si  2 K son las races del polinomio X 2 +aX +b 2 KX], entonces + = ;a y  = b. p p (b) Sea ! = ;1+2 ;3 una raz en K del polinomio X 2 + X + 1. Entonces 1, ! y !2 = ;1;2 ;3 son las tres races del polinomio X 3 ; 1 es decir, son las tres races cubicas de la unidad. Si u es una raz cubica de 0 6= a 2 K (es decir, una raz del polinomio X 3 ; a), entonces u! y u!2 son las otras dos races cubicas de a. (c) Para resolver una ecuacion polinomica podemos suponer, si perder generalidad, que el polinomio es monico. Sustituyendo X por X ; a, para un a 2 K apropiado, una ecuacion de tercer grado se convierte en una de la forma C X 3 ; qX ; r = 0 (d) (e) (f) (g) (4.8.2) y si es una solucion de (4.8.2) entonces  = ; a es una solucion de la ecuacion original. Por tanto podemos limitarnos a resolver (4.8.2), y como esto es trivial cuando r = 0 o cuando q = 0, supondremos que r q 6= 0. Sustituyendo X por u + v e imponiendo la condicion 3uv + q = 0, se obtiene la ecuacion bicubica (es decir, cuadratica en u3 ) q3 = 0 u6 ; ru3 ; 27 (4.8.3) llamada resolvente de la ecuacion (4.8.2). p Una solucion de (4.8.3) viene dada por una raz cup bica u de (r=2) + (r2 =4) + (q3 =27). Entonces v = ;q=(3u) es una raz cubica de (r=2) ; (r2 =4) + (q3 =27), y por tanto u, u!, u!2 , v, v! y v!2 son las seis soluciones de la resolvente. Las tres soluciones de (4.8.2) son entonces u + v, u! + v!2 y u!2 + v!. Se obtiene as la formula de Cardano para las soluciones de (4.8.2): s r s r r + r2 + q3 + 3 r ; r2 + q3  2 4 27 2 4 27 donde las dos races cubicas han de ser tales que su producto sea ;q=3. (h) Como aplicacion, pueden calcularse las races complejas de los polinomios 3 X 3 ; X 2 ; X + 2 Bibliografa del captulo X 3 + X 2 + 2X + 5 X 6 + X 2 + 3: Allenby 1], Clark 9], Delgado-Fuertes-Xambo 11] y 12], Hungerford 22], Jacobson 23]. 112 CAPITULO 4. ANILLOS DE POLINOMIOS Captulo 5 Grupos Se estudia la estructura de grupo y se presentan sus ejemplos y propiedades basicas. Introduccion Un grupo es un conjunto con una operacion asociativa y con elemento neutro para la que todo elemento tiene un simetrico (inverso). Esta estructura es, en principio, mas sencilla que la de anillo. Sin embargo, no se asume la conmutatividad de la operacion, y esto la convierte en una estructura de naturaleza muy distinta cuando se opera formalmente y cuando se manejan ejemplos. Comenzamos el captulo presentando las propiedades elementales de la operacion en un grupo y mostrando diversos ejemplos que ponen de maniesto que la nocion de grupo aparece de modo natural en muy diversas situaciones. Particularmente importantes seran los grupos simetricos y los grupos diedricos. A continuacion recorremos un camino similar al que seguimos al estudiar la estructura de anillo: Para un grupo G, describiremos los subconjuntos que, con la misma operacion, siguen siendo grupos (subgrupos), y consideraremos las relaciones de equivalencia en G que son compatibles con su operacion, lo que dara lugar a los conceptos de subgrupo normal y de grupo cociente. Tambien estudiaremos los homomorsmos de grupos, hasta interpretar los grupos isomorfos como grupos \esencialmente iguales". En el camino apareceran otras nociones especcas del estudio de los grupos, acompa~nadas de resultados que seran fundamentales: las clases laterales (y el Teorema de Lagrange), el orden de un elemento (y la caracterizacion de los grupos cclicos) y la conjugacion (y la Ecuacion de Clases). Todas ellas son especialmente utiles cuando el grupo considerado es nito, pues permiten obtener informacion usando solo el cardinal del grupo y de algunos de sus subgrupos. De hecho, con la informacion basica de este captulo, describiremos los grupos nitos con un numero primo de elementos, y plantearemos cuestiones sobre la descripcion de otros grupos nitos de cardinal peque~no que iremos resolviendo en los proximos captulos. Objetivos del captulo Conocer la denicion de grupo y sus principales ejemplos, y dominar su aritmetica. Conocer el concepto de subgrupo y los criterios para determinar cuando un subconjunto es subgrupo. Saber identicar el subgrupo generado por un subconjunto, y saber encontrar y manejar sistemas generadores de subgrupos. Conocer los conceptos de clase lateral e ndice, y el Teorema de Lagrange. Conocer el concepto de subgrupo normal N de un grupo G, y los criterios para decidir si un subgrupo es normal. Manejar la operacion del grupo cociente G=N y la relacion entre los subgrupos de G y los de G=N (Teorema de la Correspondencia). 113 CAPITULO 5. GRUPOS 114 Conocer las propiedades basicas de los homomorsmos de grupos, las nociones de nucleo e imagen y los Teoremas de Isomorfa. Conocer la denicion de orden de un elemento, sus distintas interpretaciones y sus propiedades elementales. Saber determinar el orden de un elemento en ejemplos concretos. Conocer la clasicacion de los grupos cclicos y el hecho de que verican el \recproco del Teorema de Lagrange". Conocer el concepto de elementos conjugados, la relacion entre la clase de conjugacion de un elemento a y el centralizador de a, y la Ecuacion de Clases. Desarrollo de los contenidos 5.1 Grupos En el Captulo 2 hemos visto la nocion de grupo abeliano. En muchas ocasiones hay que considerar estructuras que cumplen todas las propiedades de un grupo abeliano excepto la conmutatividad de la operacion son los grupos: De nicion 5.1.1 Un grupo es un par1 (G ) formado por un conjunto G con una operacion  que: Es asociativa: para cualesquiera x y z 2 G, se verica (x  y)  z = x  (y  z). Tiene elemento neutro: existe e 2 G tal que, para todo x 2 G, se verica x  e = x = e  x. Todo elemento x de G tiene un elemento simetrico: existe y 2 G tal que x  y = e = y  x. Si la operacion es ademas conmutativa tenemos la denicion de grupo abeliano. En los captulos precedentes hemos visto numerosos grupos abelianos por ejemplo, si (A + ) es un anillo entonces (A +) es un grupo abeliano y (A  ) tambien lo es. Si el anillo no es conmutativo entonces (A  ) es un grupo que en general no es abeliano: por ejemplo, no lo es si A = Mn ( ) y n > 1 (en cuyo caso A esta formado por las matrices con determinante no nulo). Vamos a posponer la presentacion de los principales ejemplos de grupos hasta la seccion siguiente, y dedicaremos esta a establecer algunas generalidades sobre el concepto. Como vimos en la Seccion 2.2, las condiciones de la denicion de grupo tienen algunas consecuencias inmediatas: La asociatividad garantiza una asociatividad generalizada que permite considerar la accion de la operacion  en un conjunto nito x1 : : :  xn de elementos de G y escribir sin ambig#uedad x1   xn. El elemento neutro es unico. El elemento simetrico de cada elemento es unico. Todo elemento es cancelable por la derecha y por la izquierda: si x  y = x  z o y  x = z  x entonces y = z. En la mayora de las ocasiones usaremos la notacion multiplicativa para un grupo G. Denotaremos la operacion por (y escribiremos ab en vez de a b), el neutro por 1 y el simetrico de x por x;1 (y le llamaremos el inverso de x). La notacion xn (con x 2 G y n 2 ) tendra el signicado habitual: x0 = 1, si n > 0 entonces xn representa el resultado de operar x consigo mismo n veces, y si n < 0 entonces xn = (x;1)jnj . As, cuando digamos \G es un grupo" asumiremos que su operacion es y que su neutro es 1. Por lo general, reservaremos la notacion aditiva para grupos abelianos: en este contexto denotaremos la operacion (conmutativa) por +, el neutro por 0 y el simetrico de x por ;x (y le llamaremos el opuesto de x). En este caso, la notacion xn se sustituye por nx. Algunas propiedades elementales de la operacion en un grupo estan vistas en la Seccion 2.2, pero no esta de mas reenunciarlas aqu. R Z 1 Cuando no existe riesgo de confusion con la operacion diremos simplemente que G es un grupo. 5.1. GRUPOS 115 Ejercicio 5.1.2 Si G es un grupo y e x y x1 : : :  xr 2 G, demostrar: 1. Si e 2 G verica ex = x para cada x 2 G entonces e = 1. Es decir, un neutro por la izquierda es 2. 3. 4. 5. 6. 7. ya el neutro de G. Si x y 2 G verican xy = 1 entonces se tiene tambien yx = 1 y por tanto y = x;1. Es decir, un inverso por la izquierda de x es ya el inverso de x. (x;1);1 = x. (xy);1 = y;1 x;1 mas generalmente (x1 xr );1 = x;r 1 x;1 1 (en general no se tiene necesariamente (xy);1 = x;1y;1 ). Si n m 2 entonces xn+m = xnxm y (xn )m = xnm . La igualdad (xy)2 = x2y2 se verica si y solo si xy = yx (decimos en este caso que x e y conmutan). Si todos los elementos de G verican x2 = 1 entonces G es abeliano. Z El siguiente resultado nos ahorrara trabajo a la hora de vericar que ciertos ejemplos que se nos presentaran son grupos. Lema 5.1.3 Sea G un conjunto con una operacion asociativa ( ) y un elemento e 2 G tales que: 1. e es un neutro por la derecha es decir, xe = x para cada x 2 G. 2. Existen inversos por la derecha es decir, para cada x 2 G existe x^ 2 G tal que x^x = e. Entonces (G ) es un grupo. Demostracion. Observamos primero que hay \cancelacion por la derecha": Si ax = bx entonces a = b, pues a = ae = a(x^x) = (ax)^x = (bx)^x = b(x^x) = be = b: Ahora, para cualquier x, se tiene (ex)^x = e(x^x) = ee = e = x^x y cancelando x^ deducimos que ex = x, de modo que e es un elemento neutro para la operacion. Por ultimo, x^ tambien verica x^x = e, como se ve cancelando en (^xx)^x = x^(x^x) = x^e = x^ = e^x: Proposicion 5.1.4 Sea G un conjunto no vaco con una operacion asociativa ( ). Entonces las siguientes condiciones son equivalentes: 1. (G ) es un grupo. 2. Dados a b 2 G, cada una de las ecuaciones aX = b e Y a = b tiene una unica solucion en G. Demostracion. 1 implica 2. Es claro que X = a;1b e Y = ba;1 son soluciones de las ecuaciones dadas, y son las unicas porque podemos cancelar. 2 implica 1. Como G no es vaco existe a 2 G. Si denotamos por e 2 G a la unica solucion de la ecuacion aX = a, entonces e es un neutro por la derecha, pues dado b 2 G la ecuacion Y a = b tiene una solucion unica Y = y es decir, ya = b, de donde be = (ya)e = y(ae) = ya = b: El hecho de que toda ecuacion de la forma aX = e tenga solucion signica que todo elemento de G tiene inverso por la derecha, y entonces el resultado es consecuencia del Lema 5.1.3. Aunque mas tarde desarrollaremos con detalle la nocion de homomorsmo de grupos, damos aqu la denicion con el objetivo de poder manejar ya el concepto de isomorsmo de grupos. CAPITULO 5. GRUPOS 116 De nicion 5.1.5 Sean (G ) y (H ) dos grupos. Un homomorsmo de grupos entre G y H es una aplicacion f : G ! H que conserva las operaciones es decir, tal que f(a b) = f(a)  f(b) para cualesquiera a b 2 G. Si f es un homomorsmo biyectivo, decimos que es un isomorsmo de grupos, y que G y H son isomorfos. Las nociones de endomorsmo y automorsmo se denen como en el caso de anillos. Ejercicio 5.1.6 Demostrar que, si f : G ! H es un isomorsmo de grupos, entonces la aplicacion inversa f ;1 : H ! G es tambien un isomorsmo de grupos. Como ocurrio con los isomorsmos de anillos, si existe un isomorsmo entre dos grupos estos pueden considerarse como basicamente iguales. Ejercicio 5.1.7 1. Demostrar que, si f1 ag es un grupo (con neutro 1), entonces su operacion viene forzosamente dada por a2 = 1 (en el resto de productos aparece el neutro y, en consecuencia, no es necesario describirlos). En particular, el grupo es abeliano. 2. Demostrar que, si G = f1G gg y H = f1H  hg son grupos (con neutros 1G y 1H ), entonces la aplicacion f : G ! H dada por 1G 7! 1H y g 7! h es un isomorsmo de grupos. Es decir, cualesquiera dos grupos con dos elementos son isomorfos. Se dice que \salvo isomorsmos, solo hay un grupo con dos elementos". Como 2 es uno de ellos (en notacion aditiva la condicion a2 = 1 se traduce en a + a = 0), deducimos que, salvo isomorsmos, 2 es el unico grupo con dos elementos. 3. Demostrar que, si f1 a bg es un grupo con neutro 1, entonces su operacion viene dada por a2 = b ab = ba = 1 b2 = a. En particular, el grupo es abeliano. 4. Demostrar que, si G = f1G g g0g y H = f1H  h h0g son grupos con neutros 1G y 1H , entonces la aplicacion f : G ! H dada por 1G 7! 1H , g 7! h y g0 7! h0 es un isomorsmo de grupos (>hay algun otro isomorsmo entre G y H ?). Deducir que, salvo isomorsmos, 3 es el unico grupo con tres elementos. Z Z Z 5.2 Ejemplos ZC Q Z Q R ZC Z Q Z Z Q R Z Como hemos comentado despues de la Denicion 5.1.1, ya contamos con numerosos ejemplos de grupos abelianos provenientes de anillos. Por ejemplo, los grupos aditivos (  +), ( n +), (  +), ( +), (  +), y los grupos multiplicativos ( n ), (  ), (  ), (   ). Por brevedad, a partir de ahora , n, ... denotaran los correspondientes grupos aditivos, y , n, ... denotaran los correspondientes grupos multiplicativos. Los vectores de un espacio vectorial con la suma proporcionan otro ejemplo de grupo. En esta seccion presentaremos ejemplos de grupos no abelianos, pero antes observemos que hay una manera facil de obtener grupos nuevos a partir de otros, lo que multiplicara el numero de ejemplos a nuestra disposicion. Ejemplo 5.2.1 Producto directo de grupos. Si (G ) y (H ?) son grupos entonces (G  H ) es un grupo, donde la operacion esta denida componente a componente a partir de  y ? es decir, (g h) (g0  h0) = (g  g0  h ? h0 ): Este grupo G  H se llama el producto directo de los grupos G y H, y es abeliano si y solo si lo son G y H. Su neutro es (1G  1H ), donde 1G y 1H son los neutros de G y de H, y el inverso de (g h) es (g;1  h;1). La construccion anterior se generaliza al producto cartesiano de cualquier Q familia de grupos, nita o innita. El producto de una familia de grupos fGi : i 2 I g se denota i2I Gi y si I = n es nito, entonces se denota G1  G2   Gn . N 5.2. EJEMPLOS 117 El ejemplo mas clasico de grupo no abeliano es el siguiente: N De nicion 5.2.2 Sea A un conjunto no vaco. Llamamos permutacion o sustitucion de A a cualquier aplicacion biyectiva f : A ! A, y denotamos el conjunto de todas las permutaciones de A por S(A). Cuando A = k = f1 2 : : :  kg (o, mas generalmente, cuando A es un conjunto nito con k elementos) escribimos Sk en lugar de S(A). La composicion de aplicaciones es una operacion en el conjunto S(A) que lo dota de estructura de grupo: la composicion siempre es asociativa, el elemento neutro es la aplicacion identidad, y el inverso de una aplicacion es su aplicacion inversa en el sentido usual. Llamamos a este grupo el grupo simetrico sobre A a Sk se le suele llamar el grupo simetrico en k elementos. Si A solo tiene 1 elemento entonces S(A) se reduce a la aplicacion identidad y por tanto es el grupo trivial (en el sentido de los Ejemplos 2.2.2). Si A solo tiene 2 elementos, A = fx yg, entonces S(A) consiste en la identidad y en la aplicacion que intercambia x con y, y es claramente un grupo abeliano isomorfo a 2. Si A contiene al menos tres elementos distintos x y z entonces S(A) no es abeliano: sea f 2 S(A) la permutacion que intercambia x con y y deja jos al resto de elementos, y sea g 2 S(A) la permutacion que intercambia x con z y deja jos al resto de elementos. Entonces f(g(x)) = f(z) = z y g(f(x)) = g(y) = y, y por lo tanto f  g 6= g  f. Z Ejercicio 5.2.3 Si X e Y son conjuntos entre los que existe una biyeccion, demostrar que los grupos de permutaciones S(X) y S(Y ) son isomorfos. Ejercicio 5.2.4 Demostrar que Sn tiene n! elementos. Ejemplo 5.2.5 El grupo simetrico sobre tres elementos: S3 . Por el Ejercicio 5.2.4, S3 tiene 6 elementos. Denotemos por e a la permutacion identidad, por  a la que intercambia 1 con 2 y ja 3, y por  la permutacion circular que lleva cada elemento al siguiente, y el ultimo al primero. Daremos la lista de los elementos de S3 escribiendo cada aplicacion biyectiva f : 3 ! 3 en la forma siguiente:  1 2 3  f(1) f(2) f(3) : N N Con esta escritura, los 6 elementos de S3 son los siguientes (recuerdese que, al componer dos aplicaciones, la que actua en primer lugar se escribe a la derecha): 1 2 3 1 2 3 1 2 3 2 e= 1 2 3 = 2 3 1  = 3 1 2 1 2 3 1 2 3   = 2 1 3  = 3 2 1 2  = 11 23 32 Haciendo los calculos directos, que se simplican usando adecuadamente la igualdad  = 2 , podemos construir una \tabla de multiplicar" para el grupo S3 :  e  2   2  e  2   2  e  2   2   2 e  2   2 e  2    2     e 2    2   e 2 2 2       e Observese que, en cada la y en cada columna de la tabla, cada elemento del grupo aparece exactamente una vez. Esto es un hecho cierto en la tabla de cualquier grupo, >por que? Ejercicio 5.2.6 Escribir las tablas de los grupos ZZZ segundo de ellos se conoce como el grupo de Klein. 4 y 2  2, y deducir que no son isomorfos. El CAPITULO 5. GRUPOS 118 Ejemplo 5.2.7 Los giros y simetras del cuadrado: el grupo diedrico D4 . Imaginemos un cuadrado de vertices 1, 2, 3 y 4. Podemos suponer que el cuadrado esta centrado en el origen de un sistema de referencia cartesiano del plano real. 1 2 4 3 Consideremos ahora, en el plano, los giros (en sentido antihorario) alrededor de un punto y las simetras respecto de rectas que llevan el cuadrado sobre s mismo: por ejemplo, si r es un giro de angulo =2 (en radianes) alrededor del origen, entonces el cuadrado se transforma en un cuadrado en la misma posicion, aunque el vertice 1 se haya movido hasta ocupar el punto donde antes estaba el 4, etc. Tales giros y simetras son exactamente ocho: El giro de angulo 0, que deja jo cada punto del cuadrado. Lo denotaremos por e. Los giros de angulos =2, y 3 =2. 1 2 giro =2 2 3 1 4 - 4 3 Las simetras con respecto a los ejes horizontal (y = 0) y vertical (x = 0). 1 2 4 3 1 2 simetra y=0 - 4 3 Las simetras con respecto a las dos diagonales del cuadrado (rectas x = y o x = ;y): estas dejan dos vertices opuestos jos e intercambian los otros dos. 5.2. EJEMPLOS 119 1 2 3 2 4 1 simetra x=y - 4 3 Llamemos D4 a este conjunto. Vamos a dotarlo de una estructura de grupo cuya operacion sera la composicion de movimientos esto es, dados dos movimientos  y en D4 , escribiremos  para designar al movimiento que resulta cuando aplicamos primero y despues . Esta operacion es asociativa, pues podemos interpretarla como una composicion de aplicaciones. Para ver que D4 es un grupo, antes de nada hemos de ver que la operacion es interna, lo que conseguimos calculando directamente la tabla. Claramente, e es el neutro de la operacion. Como en el Ejemplo 5.2.5, vamos a destacar dos elementos r y s a partir de los cuales podremos describir el resto. El elemento r sera el giro de angulo =2, y el elemento s sera la simetra de eje x = y. Esta claro que r2 es el giro de angulo , que r3 el de angulo 3 =2, y que r4 = e. Ademas, rs es la simetra respecto del eje vertical x = 0, r2s es la simetra respecto del eje y = ;x, y r3s es la simetra respecto del eje horizontal y = 0. Como en el Ejemplo 5.2.5, la igualdad sr = r3 s (que se comprueba facilmente) resulta util para construir la tabla: e e e r r r2 r2 r3 r3 s s rs rs r2s r2s r3s r3s r r2 r3 s rs r2 s r3 s r r2 r3 s rs r2 s r3 s r2 r3 e rs r2s r3 s s r3 e r r2s r3s s rs e r r2 r3s s rs r2 s r3s r2 s rs e r3 r2 r s r3 s r2 s r e r3 r2 rs s r3 s r2 r e r3 r2s rs s r3 r2 r e Completada la tabla, vemos que la operacion es interna y que cada elemento tiene un inverso, por lo que D4 es un grupo. Ejemplo 5.2.8 Los giros y simetras del triangulo equilatero: el grupo diedrico D3 . Veamos un ejemplo analogo al anterior, referido ahora al triangulo equilatero. Denotaremos por D3 al siguiente conjunto de movimientos que dejan invariante el triangulo equilatero: la identidad e, los giros centrados en el centro del triangulo de angulos 2 =3 y 4 =3: 2 3 giro 2 =3 - 1 3 2 y las simetras con respecto a las tres bisectrices de los angulos: 1 CAPITULO 5. GRUPOS 120 2 2 simetra x=0 - 1 3 3 1 Como en el ejemplo anterior, consideraremos como operacion en D3 la composicion de movimientos, que es asociativa y tiene a e por elemento neutro. Si r denota el giro de angulo 2 =3 y s denota la simetra de eje vertical, es claro que r2 es el giro de angulo 4 =3 y que r3 = e, y es facil vericar que rs y r2 s son las simetra con respecto a las bisectrices que pasan por 3 y por 1, respectivamente. Observando ademas que sr = r2s, se puede construir la tabla de la operacion: e r r2 s rs r2 s e e r r2 s rs r2 s r r r2 e rs r2s s 2 r r2 e r r2s s rs s s r2 s rs e r2 r rs rs s r2 s r e r2 r2s r2s rs s r2 r e En particular, la operacion es interna y todo elemento tiene un inverso, por lo que D3 es un grupo. Aunque los grupos S3 y D3 se han construido de manera distinta, desde el punto de vista de la Teora de Grupos son esencialmente iguales, como podra comprobar el lector resolviendo el siguiente ejercicio. Ejercicio 5.2.9 Describir un isomorsmo entre S3 y D3 . En vista de los dos ejemplos anteriores, es natural preguntarse si, en general, el conjunto Dn de los giros y simetras de un n-agono regular es un grupo con la composicion. Si tratamos de desarrollar este caso general como los anteriores, nos encontramos con el problema de demostrar que la composicion es una operacion interna en Dn . Esto lo hemos resuelto, para D3 y D4 , dando explcitamente la tabla del grupo, pero este metodo no es razonable en el caso general. Para resolver este problema vamos a dar una descripcion alternativa de Dn , que ademas hara aparecer de modo natural el concepto de subgrupo, al que dedicaremos la proxima seccion. R Ejemplo 5.2.10 Isometras del plano. RR una biyeccion f : 2 ! 2 que conserva la distancia, es decir d(x y) = d(f(x) f(y)) para todo x y 2 2. Claramente la composicion de dos isometras es una isometra y la inversa de una isometra es una isometra, por tanto el conjunto Isom( 2) de las isometras del plano es un grupo. Ejemplos de isometras son los giros alrededor de puntos, Una isometra del plano eucldeo R 2 es R f (B) f (C ) O = f (O) f (A) C A B 5.2. EJEMPLOS 121 las reexiones respecto de rectas, f (B) f (A) f (C ) C O = f (O) B A y las traslaciones. f (O) O R C B f (C ) f (B) f (A) A Si jamos un punto O de 2, entonces las isometras que dejan jo el punto O forman tambien un grupo. Un resultado que se vera en A lgebra Lineal y Geometra Eucldea asegura que las isometras que dejan jo el punto O son precisamente los giros alrededor de O y las reexiones respecto de rectas que pasan por O. Ejemplo 5.2.11 Las isometras de un n-agono regular: el grupo diedrico Dn . Sea V = fv1 v2 : : : vng el conjunto de los vertices de un n-agono regular P del plano real (n 3). Denotamos por O el centro de P, y suponemos que los vertices estan ordenados en sentido horario. Llamamos Dn al conjunto de las isometras del plano que dejan invariante el polgono regular P globalmente (aunque no necesariamente punto a punto). Como la composicion de isometras que dejen P invariante y el inverso de una de ellas tienen la misma propiedad, Dn es un grupo, que llamamos el n-esimo grupo diedrico. Claramente el centro O de P es invariante por todos los elementos de Dn y por tanto los elementos de Dn son o giros alrededor de O o reexiones a traves de rectas que pasan por O. Claramente los unicos giros alrededor de O que dejan jo el polgono son los de angulos que sean multiplos enteros de 2 =n. Por otro lado las rectas de simetra pueden ser de tres tipos: Si n es impar cada recta de simetra pasa por un vertice y el punto medio del lado opuesto. Si n es par, entonces hay dos tipos de rectas de simetra unas pasan por dos vertices opuestos y las otras pasan por los puntos medios de lados opuestos. En cualquier caso Dn esta formado por Las rotaciones (o giros) en torno al centro O de angulos k 2n , con k = 0 1 : : : n ; 1. Para k = 0 obtenemos la aplicacion identidad e, y en general el inverso de la rotacion de angulo k 2n es la rotacion de angulo (n ; k) 2n . Las reexiones respecto de rectas que pasan por O y llevan a V sobre s mismo. Es claro que cada reexion es su propia inversa. Siempre hay exactamente n ejes de simetra. Si n es impar, cada uno va de un vertice al punto medio del lado opuesto y si n es par hay n=2 ejes que unen vertices opuestos, y otros n=2 ejes que unen puntos medios de lados opuestos. Para los casos n = 5 y n = 6, se tienen las siguientes rectas de simetra: CAPITULO 5. GRUPOS 122 1 2 5 1 2 6 3 3 4 5 4 Es claro que las n rotaciones son elementos distintos de Dn , y tambien es claro que las n reexiones son distintas entre s. Ademas, una reexion nunca puede ser una rotacion, pues las primeras invierten el orden de los vertices (de antihorario a horario y viceversa), por ejemplo: 1 2 5 2 3 reexion x=0 - 1 3 5 4 4 mientras que las segundas lo conservan: 1 2 3 4 giro 4 =5 5 - 3 2 4 5 1 Consideremos ahora en Dn los siguientes elementos: R es la rotacion de angulo 2 =n, y S es una reexion, arbitraria pero ja. Es claro que Rn = e, que S 2 = e y que R0 = e R R2 : : : Rn;1 son las n distintas rotaciones de Dn . Por otra parte, los elementos S RS R2S : : : Rn;1S son distintos entre s (podemos cancelar S) y son reexiones (invierten el orden de los vertices), de modo que Dn = fe R R2 : : :  Rn;1 S RS R2S : : :  Rn;1S g: Como Rk S es una reexion, se tiene e = (Rk S)2 = Rk (SRk S), por lo que SRk S = (Rk );1 = Rn;k multiplicando a la derecha por S obtenemos SRk = Rn;kS, y en particular SR = R;1S. Usando adecuadamente las relaciones Rn = e S2 = e y SRk = Rn;kS podemos expresar cualquier producto de elementos de Dn en la forma Rk o Rk S con k 2 f0 1 : : : n ; 1g. Por ejemplo, en D7 se tiene (R5S)R3 = R5 SR3 = R5 R4S = R9 S = R2S 5.3. SUBGRUPOS 123 o (R3S)(R2 S) = R3 (SR2 )S = R3 (R5 S)S = R8S 2 = R: Dejamos como ejercicio para el lector la comprobacion de que, usando solo las relaciones Rn = e = S 2 y SR = R;1S y el hecho de que Dn es un grupo, es posible deducir que Dn = fe R R2 : : :  Rn;1 S RS R2S : : :  Rn;1S g y que SRk = Rn;k S para cada k 2 n. Como acabamos de ver, esto identica totalmente al grupo es decir, al conjunto y a su operacion. Este hecho se expresa diciendo que Dn esta generado por los elementos R y S sujetos a esas tres relaciones2 , y se simboliza por Dn = hR S : Rn = e S 2 = e SRS = R;1 i: Ejercicio 5.2.12 Con la notacion del Ejemplo 5.2.11, supongamos que S es la reexion con respecto a la recta que une el centro O con el vertice vn . Se pide: 1. Comprobar que, para cada i 2 n, se tiene R(vi ) = vi+1 y S(vi ) = vn;i. 2. Determinar cual es el eje de la reexion Rk S . (Indicacion: Si n es impar, el eje pasa por un vertice que depende de k y del inverso de 2 modulo n. Si n es par, el eje pasa por dos vertices si k es par, y por los puntos medios de dos lados si k es impar.) Terminamos la seccion con un ejemplo de naturaleza distinta a los anteriores. En particular, este grupo puede ser innito (de hecho lo es precisamente si el cuerpo K que se considera es innito). Ejemplo 5.2.13 El grupo lineal GLn (K). Sea K un cuerpo. El conjunto de todas las matrices de Mn(K) con determinante no nulo (es decir, invertibles) es un grupo no abeliano (para n 2) con el producto usual de matrices. Su elemento neutro es la matriz identidad y el inverso de una matriz A es su matriz inversa en el sentido usual. Le llamamos el grupo lineal general de matrices n  n sobre K, y lo denotamos por GLn (K). N N 5.3 Subgrupos La denicion de subgrupo de un grupo es similar a la que usamos en el caso de subanillos: De nicion 5.3.1 Sea (G ) un grupo. Un subgrupo de G es un subconjunto H de G cerrado para la operacion y tal que (H ) es un grupo (se dice que la operacion de G induce una estructura de grupo en H ). Consideremos la tabla del grupo D4 construida en el Ejemplo 5.2.7. Si nos jamos en las cuatro primeras las y en las cuatro primeras columnas, observamos que H = f1 r r2 r3g es un subgrupo. En general, este es un metodo poco efectivo de determinar si un subconjunto es o no un subgrupo, y los siguientes resultados constituyen una simplicacion notable de este problema. La demostracion del primero es esencialmente igual que la de la Proposicion 2.3.2. Proposicion 5.3.2 Sea G un grupo y sea H un subconjunto. Las siguientes condiciones son equivalentes: 1. H es subgrupo de G. 2. 1 2 H y H es cerrado para el producto y para inversos (es decir, si a b 2 H entonces ab 2 H y a;1 2 H ). 3. H no es vaco y, si a b 2 H , entonces ab;1 2 H . Decidir cuando un subconjunto nito es un subgrupo es algo mas facil como la mayora de nuestros ejemplos seran de grupos nitos, el resultado que sigue sera de gran utilidad: 2 Existe una denici on rigurosa del concepto de grupo expresado en funcion de generadores y relaciones que no veremos en este curso. CAPITULO 5. GRUPOS 124 Corolario 5.3.3 Sea G un grupo. Si H es un subconjunto nito de G, no vaco y cerrado para el producto, entonces H es subgrupo de G. Demostracion. Por la proposicion anterior, basta ver que H contiene al 1 y es cerrado para inversos. Como H no es vaco, existe un elemento a 2 H. Entonces a a2 : : :  an : : : estan en H por la hipotesis y, como H es nito, deben existir enteros positivos distintos n m con an = am . Podemos suponer que n > m, de modo que r = n ; m > 0. Cancelando se obtiene ar = 1, igualdad de la que deducimos que 1 2 H (por la hipotesis) y que a;1 = ar;1 esta en H. Ejemplos 5.3.4 Subgrupos. 1. Sea G un grupo cualquiera. Entonces f1g y el propio G son subgrupos de G, llamados respectivamente el subgrupo trivial y el subgrupo impropio de G. Los subgrupos distintos de f1g se llaman no triviales y los subgrupos distintos de G se llaman propios. 2. Los ideales de un anillo son subgrupos de su grupo aditivo. Para el anillo no hay mas subgrupos, ya que multiplicar a por un elemento de equivale a sumar a o ;a varias veces consigo mismo. 3. El conjunto + de los numeros reales positivos es un subgrupo del grupo multiplicativo . 4. El conjunto de los numeros complejos de modulo 1 es un subgrupo del grupo multiplicativo  . 5. Pueden revisarse las tablas de los grupos nitos considerados en la seccion anterior para encontrar subgrupos de D4 , S3 o D3 . As, el conjunto de las rotaciones en D3 (esto es, f1 r r2g) es un subgrupo. Otro subgrupo de D3 es fe sg. 6. En el grupo lineal GLn ( ), el subconjunto de las matrices cuyo determinante es 1 es un subgrupo, como se comprueba facilmente a partir de la igualdad det(A B) = det(A) det(B) y usando la Proposicion 5.3.2. Este subgrupo se llama el grupo lineal especial y se denota por SLn ( ). Un argumento semejante prueba que el conjunto de las matrices en GLn ( ) cuyo determinante tiene valor absoluto 1 es tambien un subgrupo de GLn ( ), que denotaremos por ScLn ( ). 7. Dado un entero positivo k 3, consideremos el grupo simetrico Sk . Dentro de Sk podemos tomar todas aquellas permutaciones de k que dejan jos algunos elementos de k por ejemplo, sea T  Sk el conjunto de las permutaciones que dejan jos los numeros k ; 1 k. Es claro que la composicion de dos elementos de T esta en T, luego T es un subgrupo de Sk por el Corolario 5.3.3. Es facil establecer un isomorsmo entre los grupos T y Sk;2. 8. Sea G = S( 2) el grupo simetrico sobre el plano real. El subconjunto Isom( 2) de G formado por las isometras del plano es un subgrupo de G. Otro subgrupo de G (y de Isom( 2)) esta formado por las isometras que jan un cierto punto. Q 9. Sea fGi : i 2 I g una familia de grupos y sea G = i2I Gi el grupo producto. Entonces Z Z R R R N R i2I Gi = f(ai )i2I 2 Y i2I RC R R R N RR Gi : ai = 0 para casi todo i 2 I g es un subgrupo de G, llamado la suma directa de los subgrupos fGi : i 2 I g. Si I es nito, entonces la suma directa coincide con el producto directo. Cerramos esta seccion con algunos ejemplos que tendran interes teorico. Ejemplo 5.3.5 El centro de un grupo. Dado un grupo G, denimos su centro como el subconjunto Z(G) = fa 2 G : ag = ga para cada g 2 Gg: Es facil comprobar que Z(G) es subgrupo de G. Ademas G es abeliano si y solo si Z(G) = G. Si se conoce la tabla de un grupo, un elemento esta en el centro si y solo si, en su la y en su columna, los elementos de G aparecen en el mismo orden por tanto Z(S3 ) = feg, Z(D3 ) = f1g y Z(D4 ) = f1 r2g. 5.4. OPERACIONES CON SUBGRUPOS 125 Ejercicio 5.3.6 Demostrar que el centro de Dn es trivial si n es impar, y es fe Rn=2g si n es par. Ejercicio 5.3.7 Si G es un grupo, se dene el centralizador en G de un elemento x 2 G como el subconjunto 1. 2. 3. 4. CenG (x) = fg 2 G : gx = xgg: Demostrar que CenG (x) es un subgrupo de G. Demostrar que Z(G) = \x2G CenG (x). Demostrar que x 2 Z(G) , CenG (x) = G. Calcular CenG (x) para cada x 2 G, cuando G es S3 o D4 . Ejemplo 5.3.8 Automorsmos. Si G es un grupo, los isomorsmos de grupos G ! G se llaman automorsmos de G. En general, la composicion de dos isomorsmos y el inverso de un isomorsmo siguen siendo isomorsmos, y por tanto el conjunto Aut(G) de todos los automorsmos de G es un subgrupo del grupo de permutaciones S(G), llamado grupo de automorsmos de G. Analogamente, si en lugar de los automorsmos de un grupo consideramos los de un espacio vectorial o los de un anillo (con las deniciones obvias), obtendremos subgrupos de los correspondientes grupos de permutaciones. En el caso de un espacio vectorial V de dimension nita n sobre un cuerpo K, la conocida relacion entre las aplicaciones lineales V ! V y las matrices n  n sobre K (para una base ja de V ) nos da un isomorsmo de grupos entre el grupo de los automorsmos de V (como espacio vectorial) y el grupo lineal GLn (K) del Ejemplo 5.2.13. 5.4 Operaciones con subgrupos En adelante, para denotar que H es un subgrupo de un grupo G escribiremos H G. En esta seccion examinaremos diversas construcciones dentro de un grupo que dan lugar a subgrupos. El primer resultado es de esperar, en vista de los que obtuvimos para anillos, y aparte del interes que tiene en s mismo, resulta necesario para las construcciones que siguen. Ejercicio 5.4.1 Demostrar que la interseccion de cualquier familia de subgrupos de un grupo G es un subgrupo de G. De nicion 5.4.2 Sea G un grupo y sea S un subconjunto de G. Llamamos subgrupo de G generado por S , y lo denotamos por hS i, a la interseccion de todos los subgrupos de G que contienen a S es decir, hS i = \fH : H G y S  H g: Es claro que hS i es el menor subgrupo de G que contiene a S es decir, hS i es un subgrupo de G y todos los subgrupos de G que contienen a S tambien contienen a hS i. Ejemplos 5.4.3 Subgrupos generados. Sea G un grupo arbitrario entonces: 1. El subgrupo generado por es el subgrupo trivial, h i = f1g. 2. Si S es un subgrupo de G entonces hS i = S. 3. Si x 2 G, entonces hfxgi = fxn : n 2 g. Usualmente denotaremos este subgrupo por hxi. La descripcion dada no implica que hxi sea innito, puesto que los valores de xn se pueden repetir para distintos n. Por ejemplo, el subgrupo de  generado por i es f1 i ;1 ;ig. Si se usa notacion aditiva entonces hxi = fnx : n 2 g. Por tanto, los subgrupos de son precisamente los de la forma hni con n 2 (son todos innitos, excepto el trivial). Z C Z N Z CAPITULO 5. GRUPOS 126 Veamos ahora una manera mas explcita de identicar el subgrupo generado por un conjunto, que ademas generaliza el ultimo de los ejemplos anteriores. Dado S  G, pondremos S ;1 = fx 2 G : x;1 2 S g = fx;1 : x 2 S g y Sb = S  S ;1 : Empleando esta notacion se tiene: Proposicion 5.4.4 Sea G un grupo y sea S  G un subconjunto. Los elementos del subgrupo generado por S son todos los productos nitos de elementos de Sb. Esto es, hS i = fx1x2 xt : t 0 y cada xi 2 Sbg Z donde interpretamos un producto vaco (con 0 factores) como el neutro 1. Tambien se verica hS i = fxn1 1 xn2 2 xnr r : r 0, cada xi 2 S y cada ni 2 g: Demostracion. Las dos expresiones del enunciado son iguales porque podemos pasar de una del segundo tipo a una del primero sustituyendo xk por x x o por x;1 x;1 (jkj veces) segun el signo de k. Llamemos H al conjunto fx 2 G : x = x1 x2 xt , con t 0 y cada xi 2 Sbg y veamos que H es el menor subgrupo de G que contiene a S. Considerando productos de un solo elemento es claro que H contiene a S tambien es claro que H contiene al 1 y que es cerrado para productos ademas es cerrado para inversos por el Ejercicio 5.1.2, por lo que H es un subgrupo de G (Proposicion 5.3.2). Por ultimo, si un subgrupo K de G contiene a cada elemento de S entonces, de nuevo por la Proposicion 5.3.2, K contiene a todo H, de modo que, en efecto, H es el menor subgrupo de G que contiene a S. Observacion 5.4.5 Si G es abeliano y usamos notacion aditiva, el subgrupo generado por S es hS i = (X s 2S Z ) nss : ns 2 para todo s 2 S y ns = 0 para casi todo s 2 S : De nicion 5.4.6 Dado un subgrupo H del grupo G, pueden existir diversos subconjuntos S  G tales que hS i = H . Cada uno de estos se llama un sistema generador (o de generadores) del subgrupo H , y en muchos casos nos interesaran los sistemas generadores del subgrupo impropio G. Un grupo es cclico si posee un sistema generador unitario (es decir, formado por un solo elemento), y es nitamente generado si posee un sistema generador nito. Ejercicio 5.4.7 Demostrar que todo grupo cclico es abeliano. Todo subgrupo H de G tiene al menos un sistema generador: el propio H. Sin embargo, la idea que hay detras del concepto es poder expresar todos los elementos de un grupo a partir de unos pocos, as que, en general, nos interesan los sistemas generadores que tengan el menor numero posible de elementos. Z Ejemplos 5.4.8 Sistemas generadores. ZZ Z ZZ ZZ 1. El grupo abeliano es cclico generado por el 1 (usando notacion aditiva). El ;1 tambien es un generador, y no hay otros conjuntos generadores unitarios. El conjunto f2 3g tambien es un sistema generador de . 2. Los grupos abelianos n son cclicos generados por 1]n. De hecho, dado a 2 , el elemento a]n genera a n si y solo si a es coprimo con n (>por que?). 3. El grupo producto  (notacion aditiva) esta claramente generado por f(1 0) (0 1)g, pero no es cclico (>por que?). 4. El grupo de Klein 2  2 no es cclico, y esta generado por cualesquiera dos de sus elementos distintos del neutro. Z Z 5.4. OPERACIONES CON SUBGRUPOS 127 5. El grupo multiplicativo 8 no es cclico, pues tiene 4 elementos y cada uno de ellos genera un subgrupo que solo tiene 1 o 2 elementos. 6. Como S3 no es abeliano, no es cclico. Sin embargo, es facil ver que todo conjunto de dos elementos de S3 que no contenga al neutro y no sea f 2g es un sistema generador. 7. Como Dn no es abeliano (n 3), no es cclico. Por la descripcion dada en el Ejemplo 5.2.11, Dn esta generado por el conjunto fR S g. C Ejemplo 5.4.9 El grupo de los cuaterniones Q8. Se conoce con este nombre, y se denota por Q8, al subgrupo de GL2( ) generado por las matrices       i = 0i ;0i  j = ;01 10  k = 0i 0i Si denotamos por I la matriz identidad de tama~no 2  2, es elemental vericar las siguientes relaciones i2 = j2 = k2 = ;I ij = k = ;ji jk = i = ;kj ki = j = ;ik que en particular muestran que cualesquiera dos de las tres matrices i j k generan a la otra, de modo que podemos eliminar una cualquiera de ellas del sistema generador inicial sin alterar el subgrupo denido. Por otra parte, esas relaciones tambien muestran que Q8 = fI i j k ;I ;i ;j ;kg y que la tabla de operacion en Q8 es I i j k ;I ;i ;j ;k I I i j k ;I ;i ;j ;k i i ;I ;k j ;i I k ;j j k ;I ;i j k ;I ;i k ;j ;i I ;I i ;j k ;i ;I ;k ;j ;j ;k I i ;k j i ;I I ;i j ;k i I k j ;j ;k ;j ;k ;k j I ;i i I j k k ;j ;I i ;i ;I \ En general, la union de dos subgrupos H y K de un grupo G no es un subgrupo de G. El subgrupo que genera su union, hH  K i, se suele denotar por H _ K y es el menor subgrupo de G que contiene tanto a H como a K. Como es claro que H  K = H  K, la Proposicion 5.4.4 nos dice que H _ K = hH  K i = fx1 x2 xn : n 1 y cada xi 2 H o xi 2 K g: Recuerdese que, cuando considerabamos ideales en anillos, el ideal generado por una union de ideales era la suma de esos ideales (Proposicion 2.5.7). Esto sugiere la denicion del producto de dos subgrupos H y K de un grupo G como el subconjunto HK = fhk : h 2 H k 2 K g pero en general este subconjunto no es un subgrupo de G. Por ejemplo, si G = S3 entonces, con la notacion del Ejemplo 5.2.5, H = fe  g y K = fe  g son subgrupos, pero su producto HK = fe   2 g no lo es (>por que?). El siguiente ejercicio muestra que HK es un subgrupo en cuanto hay \un poco de conmutatividad". Ejercicio 5.4.10 Demostrar que, si H y K son subgrupos de un grupo G, entonces HK es un subgrupo de G si y solo si HK = KH . En este caso, HK es el menor subgrupo de G que contiene a H y a K es decir, HK = H _ K . CAPITULO 5. GRUPOS 128 Otro resultado util sobre estos productos de subgrupos es el siguiente: Lema 5.4.11 Sean A y B dos subgrupos nitos de un grupo G (no es necesario asumir que AB es un subgrupo de G). Entonces jAB j jA \ B j = jAj jB j: Demostracion. En el conjunto A  B denimos la relacion (a b)  (c d) , ab = cd, que es claramente de equivalencia. Si C denota el conjunto cociente de A  B por esa relacion, es claro que la aplicacion : C ! AB dada por (a b) = ab esta bien denida (no depende de representantes) y es biyectiva. Ademas, la clase de equivalencia de (a b) 2 A  B para esta relacion es f(ax;1 xb) : x 2 A \ B g. En efecto, es obvio que un elemento de la forma (ax;1 xb) esta en la clase de (a b) y si (c d)  (a b) entonces x = c;1 a = db;1 esta en A \ B y se tiene (c d) = (ax;1 xb). Como cada una de estas clases tiene cardinal jA \ B j, hemos demostrado que A  B se divide en jAB j clases de equivalencia, cada una de ellas con jA \ B j elementos, y por tanto jAj jB j = jA  B j = jAB j jA \ B j. 5.5 Clases laterales y Teorema de Lagrange En el Captulo 2 vimos que si I es un ideal de un anillo A, entonces existe una particion de A formada por los conjuntos de la forma a+I = fa+x : x 2 I g con a 2 A. En realidad, solo utilizamos la estructura aditiva de A para construir esta particion. Estas particiones se pueden construir de forma analoga a partir de un grupo G y un subgrupo suyo H. Pero la posible no conmutatividad de la operacion del grupo hace que la version de la relacion de equivalencia que utilizabamos para construir las clases a + I (a  b mod I , a ; b 2 I) tenga dos deniciones alternativas en el caso de grupos: a i b mod H , a;1b 2 H a d b mod H , ab;1 2 H: Ejercicio 5.5.1 Demostrar que, para cualquier grupo G y cualquier subgrupo H , las dos relaciones binarias recien denidas son de equivalencia en G. Las relaciones i y d inducen particiones en G. Veamos como son las clases de equivalencia: La clase de equivalencia de a 2 G por la relacion i mod H es fx 2 G : a i x mod H g = fx 2 G : a;1x 2 H g = fah 2 G : h 2 H g = aH y analogamente la clase de equivalencia de a por la relacion d es Ha = fha : h 2 H g: Los conjuntos aH se llaman clases laterales por la izquierda de G modulo H y los conjuntos Ha se llaman clases laterales por la derecha de G modulo H. El conjunto de las clases laterales por la izquierda se denota G=H y el conjunto de las clases laterales por la derecha por H nG. Es decir, G=H = fgH : g 2 Gg y H nG = fHg : g 2 Gg: Es claro que, para todo a b 2 G, se tiene aH = bH , a 2 bH , b 2 aH , a;1 b 2 H , b;1a 2 H , aH \ bH 6= y Ha = Hb , a 2 Hb , b 2 Ha , ba;1 2 H , ab;1 2 H , Ha \ Hb 6= : En particular H = 1H y aH = H , a 2 H , Ha = H: Ejemplos 5.5.2 Clases laterales. 1. Si I es un ideal de un anillo A, entonces I es un subgrupo del grupo (A +) y las clases laterales (por la izquierda y por la derecha) coinciden con las denidas en el Captulo 2. 5.5. CLASES LATERALES Y TEOREMA DE LAGRANGE 129 2. Consideremos en D4 el subgrupo de las rotaciones K = hri = f1 r r2 r3g. Como Ks = sK = fs rs r2s r3sg, vemos que en este caso G=K = K nG = fK Ksg es decir, solo hay dos clases por la derecha y coinciden con las dos clases por la izquierda. Podemos tambien calcular las clases laterales modulo el subgrupo H = f1 sg en este caso las clases por la derecha son H1 = Hs = f1 sg Hr = Hr3 s = fr r3sg Hr2 = Hr2 s = fr2  r2sg Hr3 = Hrs = fr3 rsg mientras que las clases por la izquierda son 1H = sH = f1 sg rH = rsH = fr rsg r2 H = r2sH = fr2 r2sg r3 H = r3sH = fr3 r3sg: En particular, hay elementos tales que aH 6= Ha y los conjuntos G=H y H nG son distintos. La siguiente proposicion estudia los cardinales de cada clase lateral de G modulo H, y tambien los de G=H y H nG. Proposicion 5.5.3 Si H es un subgrupo de G, entonces: 1. La aplicacion H ! Ha dada por h 7! ha es una biyeccion con inversa x 7! xa;1. En particular, si H es nito entonces todas las clases por la derecha modulo H tienen cardinal jH j. La aplicacion H ! aH dada por h 7! ah es una biyeccion con inversa x 7! a;1x. En particular, si H es nito entonces todas las clases por la izquierda modulo H tienen cardinal jH j. 2. La aplicacion H nG ! G=H dada por Ha 7! a;1H es una biyeccion con inversa bH 7! Hb;1. En particular, H nG es nito si y solo si lo es G=H , y en ese caso sus cardinales coinciden. Demostracion. El primer apartado es claro, y el segundo lo sera si vemos que las aplicaciones dadas estan bien denidas. Pero esto es as pues Ha = Hb implica que ab;1 = (a;1 );1 b;1 2 H, por lo que a;1 H = b;1H. De nicion 5.5.4 Se llama orden de un grupo nito G a su cardinal jGj. Si G es un grupo arbitrario con un subgrupo H , decimos que H tiene ndice nito en G si el cardinal comun de H nG y G=H es nito llamamos ndice de H en G a ese cardinal, y lo denotamos por G : H]. ZZ Es evidente que todo subgrupo de un grupo nito tiene ndice nito. Por su parte, un grupo innito tiene al menos un subgrupo de ndice innito (el trivial), y puede tener subgrupos impropios de ndice nito: por ejemplo, si n 1 entonces  : n ] = n. El siguiente teorema es fundamental en el estudio de un grupo nito G, pues establece una relacion entre el orden de G y el de sus subgrupos a saber, el orden de cualquier subgrupo divide al orden del grupo. En general, la relacion entre divisores de jGj y subgrupos de G es profunda y complicada, y este es el primer paso que daremos para desentra~narla. Teorema 5.5.5 (Teorema de Lagrange) Si G es un grupo nito y H es un subgrupo de G, entonces el orden de H y el ndice de H en G son ambos divisores del orden de G. De hecho, se tiene jGj = jH j G : H] (o sea, jG=H j = G : H] = jGj=jH j): Demostracion. Por los dos resultados anteriores, G se divide en jG=H j = G : H] clases por la derecha modulo H, cada una de ellas con jH j elementos, lo que nos da las igualdades postuladas. Del Teorema de Lagrange se deduce que un subgrupo de un grupo nito es tanto mayor cuanto menor es su ndice. Por lo tanto el ndice de H en G sirve para medir lo grande que es H \dentro de G": si G : H] = 1 entonces H es todo G, si G : H] = 2 entonces H es \la mitad de grande" que G, etcetera. Como hemos dicho, el Teorema de Lagrange es un instrumento poderoso para estudiar los grupos nitos en funcion de su cardinal. Un ejemplo es el siguiente, que es el primero de un tipo de resultados que perseguiremos a lo largo del curso, en los que se trata de deducir propiedades de un grupo nito en funcion unicamente de su orden. CAPITULO 5. GRUPOS 130 Corolario 5.5.6 Si G es un grupo nito de orden primo p, entonces G es cclico (generado por cualquier elemento distinto del neutro) y no tiene mas subgrupos que el trivial y el impropio. Demostracion. Como los unicos divisores positivos de jGj = p son 1 y p, cualquier posible subgrupo de G ha de tener orden 1 o p. Pero el unico subgrupo de orden 1 es el trivial, y el unico de orden p es todo G, luego estos son los unicos subgrupos de G. Por otra parte, G contiene un elemento a 6= 1 (pues jGj = p > 1), luego hai es un subgrupo no trivial del parrafo anterior deducimos que G = hai es decir, G es cclico (y esta generado por cualquier elemento distinto del neutro). Ejercicio 5.5.7 Si G es un grupo nito con subgrupos H y K tales que H  K , demostrar que se tiene H = K , jH j = jK j , G : H] = G : K]: Ejercicio 5.5.8 Por el teorema de Lagrange, los posibles subgrupos propios y no triviales de 6 o de S3 han de tener orden 2 o 3, y los de en cada caso. Z Z 8 o de D4 deben tener orden 2 o 4. Determinar todos los subgrupos 5.6 Subgrupos normales y grupos cociente En esta seccion abordamos la construccion del grupo cociente de un grupo G por un subgrupo N deniendo de forma natural un producto entre las clases laterales modulo N. Esta construccion no es valida para cualquier subgrupo, y en lo que sigue se describen los subgrupos para los que s lo es. Dados subconjuntos A y B de un grupo G, pondremos AB = fab : a 2 A b 2 B g. Si X = fxg pondremos xA en lugar de XA y Ax en lugar de AX, lo que es consistente con la notacion usada para las clases laterales. Por otra parte, la asociatividad de G implica que (AB)C = A(BC) para subconjuntos A, B y C arbitrarios, lo que nos permite escribir ABC sin ambig#uedad obviamente ABC = fabc : a 2 A b 2 B c 2 C g. Proposicion 5.6.1 Las condiciones siguientes son equivalentes para un subgrupo N de un grupo G: 1. N nG = G=N . 2. Para cada x 2 G se tiene Nx = xN (o equivalentemente x;1Nx = N ). 3. Para cada x 2 G se tiene Nx  xN (o equivalentemente x;1Nx  N ). 4. Para cada x 2 G se tiene xN  Nx (o equivalentemente xNx;1  N ). 5. Para cualesquiera a b 2 G se tiene aNbN = abN . 6. Para cualesquiera a b 2 G se tiene NaNb = Nab. Demostracion. Es obvio que 2 implica 1. El recproco es cierto pues, dado x 2 G, la hipotesis 1 implica que existe y 2 G tal que Nx = yN, por lo que x 2 yN y as xN = yN = Nx. A continuacion demostramos de forma cclica la equivalencia entre 2, 5 y 3. El lector puede ver de forma analoga que 2, 6 y 4 son equivalentes, lo que concluira la demostracion. Usaremos dos hechos de facil comprobacion: Por ser N subgrupo, se tiene NN = N, y si A  B, entonces xA  xB y Ax  Bx. 2 implica 5. Asumiendo 2 y tomando a b 2 G, se tiene aNbN = abNN = abN. 5 implica 3. Dados n 2 N y x 2 G se tiene nx = 1nx1 2 1NxN = 1xN = xN, por lo que Nx  xN. 3 implica 2. Fijemos x 2 G y veamos que xN  Nx. Como 3 vale para cualquier elemento de G, se tiene Nx;1  x;1 N, y multiplicando a la derecha por x en ambos lados de la igualdad obtenemos la inclusion deseada. Supongamos que se cumplen las condiciones de la Proposicion 5.6.1. Entonces el producto de dos elementos de G=N (o de N nG) es un elemento de G=N, y es elemental comprobar que esta operacion dota a G=N de una estructura de grupo. Observese que, para realizar un producto aN bN en G=N, no necesitamos describir el conjunto resultante, pues este queda determinado por cualquier representante suyo, por ejemplo ab. El elemento neutro de G=N es la clase N = 1N, y el inverso de aN es a;1 N. 5.6. SUBGRUPOS NORMALES Y GRUPOS COCIENTE 131 De nicion 5.6.2 Un subgrupo N de un grupo G es un subgrupo normal de G (tambien se dice que N es normal en G) si verica las condiciones equivalentes de la Proposicion 5.6.1. En ocasiones escribiremos N E G (respectivamente N C G) para indicar que N es un subgrupo normal (respectivamente normal y propio) de G. Si N es normal en G, el grupo G=N recien descrito se llama grupo cociente de G modulo N . Ejemplos 5.6.3 Subgrupos normales. 1. Es claro que, en un grupo abeliano, todo subgrupo es normal. De hecho, si I es un ideal de un anillo A, entonces el grupo cociente A=I es el grupo aditivo del anillo cociente. 2. Si G es un grupo y H es un subgrupo contenido en el centro Z(G), entonces H es normal en G. En particular, el centro es un subgrupo normal. 3. Si H es un subgrupo de G de ndice 2, entonces H es normal en G. En efecto, como las clases por la derecha modulo H constituyen una particion de G, solo hay dos, y una de ellas es H, la otra ha de ser el complementario fg 2 G : g 62 H g. El mismo argumento vale para las clases por la izquierda y en consecuencia G=N = N nG. R R 4. Sea G = GLn ( ) el grupo lineal general sobre . Usando el hecho de que, si a b 2 G, entonces R det(ba) = det(b) det(a) = det(a) det(b) = det(ab) R es facil ver que tanto el grupo lineal especial S = SLn( ) como el grupo S^ = ScLn( ) (ver los Ejemplos 5.3.4) son subgrupos normales de G. Vamos a describir el grupo cociente G=S. Sean m x 2 G y sea  = det(m). Si x 2 mS entonces x = ms con s 2 S, por lo que det(x) = det(m) det(s) = . Recprocamente, si det(x) =  entonces det(m;1 x) = ;1 = 1 y as m;1 x 2 S, o sea x 2 mS. En consecuencia, la clase mS consiste en todas las matrices de G cuyo determinante es , y la denotaremos por % . Como para cada  2  hay matrices con determinante  (<muestra una!), deducimos que G=S = f% :  2 g. Ademas, si  = en  entonces % % = % en G=S, como se comprueba facilmente. En consecuencia, la aplicacion  ! G=S dada por  7! % es un isomorsmo de grupos, y tenemos un ejemplo de grupo no abeliano con cociente abeliano. ^ Dejamos que el lector establezca un isomorsmo de grupos entre + y el cociente G=S. RR R R R 5. El siguiente es el diagrama de todos los subgrupos de D4 ordenados por inclusion: una lnea entre dos subgrupos signica que el de arriba contiene al de abajo. Los subgrupos de la segunda la tienen orden 4, y los de la tercera la tienen orden 2. En el diagrama estan subrayados los subgrupos que no son normales en D4 , que son precisamente los subgrupos generados por cada una de las simetras: D4 hr2si hr2 si hr i hsi hr2 i feg hr2  rsi hrsi hr3 si CAPITULO 5. GRUPOS 132 Los que aparecen son subgrupos y las relaciones de inclusion son claras, pero el lector debera comprobar esos subgrupos son distintos entre s y que no hay mas, as como la normalidad de los subgrupos no subrayados. Otro ejercicio interesante consiste en demostrar que los subgrupos hr2 si y hr2  rsi no son cclicos. Observese que cualquier subgrupo del diagrama es normal en cualquiera de los subgrupos que lo contengan y esten en el nivel inmediatamente superior. Por ejemplo, hsi E hr2 si y hr2  si E D4 como hsi no es normal en D4 , este ejemplo muestra que la relacion \ser normal en" no es transitiva. 6. El diagrama de los subgrupos de Q8 es el siguiente. Q8 hii hji hki h;I i fI g Observese que todos son normales (>por que?). Sea Z = Z(Q8 ) = fI ;I g. Los elementos del grupo cociente Q8=Z son Z, Z i = fi ;ig, Z j = fj ;jg y Zk = fk ;kg, y su tabla es la siguiente: Z Zi Z Z Zi Zi Zi Z Zj Zj Zk Zk Zk Zj Zj Zj Zk Z Zi Zk Zk Zj Zi Z Observese que Q8 es un grupo no abeliano pero es \casi abeliano" en dos sentidos: todos sus subgrupos son normales, y todos los subgrupos propios dan cocientes abelianos. Por otra parte, en vista de la tabla, es sencillo establecer un isomorsmo entre el grupo cociente Q8=Z y el grupo de Klein 2  2. ZZ Ejercicio 5.6.4 Demostrar que un grupo cociente de un grupo nitamente generado es nitamente generado. Acabamos la seccion con una version para grupos del Teorema de la Correspondencia (2.4.8). Teorema 5.6.5 (Teorema de la Correspondencia) Sea N un subgrupo normal de un grupo G. La asignacion H 7! H=N establece una biyeccion entre el conjunto A de los subgrupos de G que contienen a N y el conjunto B de los subgrupos de G=N . Ademas, esta biyeccion conserva las inclusiones, las intersecciones y la normalidad. Es decir, dados H K 2 A, se tiene: 1. H  K si y solo si (H=N)  (K=N). 2. (H \ K)=N = (H=N) \ (K=N). 3. H E G si y solo si (H=N) E (G=N). 5.6. SUBGRUPOS NORMALES Y GRUPOS COCIENTE 133 Demostracion. Si H esta en A entonces es claro que N es un subgrupo normal de H, por lo que podemos considerar el grupo cociente H=N, que es un subgrupo de G=N (<compruebese!). Llamemos * : A ! B a la aplicacion dada por *(H) = H=N, y veamos que es inyectiva y suprayectiva. Sean H y K elementos de A tales que *(H) = *(K). Entonces, si h 2 H, se tiene Nh 2 H=N = K=N y en consecuencia existe k 2 K tal que Nh = Nk, luego hk;1 2 N  K multiplicando por k se obtiene h 2 K. En consecuencia H  K, y analogamente se prueba que K  H, lo que nos dice que * es inyectiva. Por otra parte, si X 2 B entonces H = fx 2 G : Nx 2 X g es un elemento de A tal que *(H) = X (la comprobacion de los detalles se deja al lector), lo que demuestra que * es suprayectiva. La demostracion de la segunda parte del enunciado se deja como ejercicio para el lector. Z ZZ Ejemplos 5.6.6 Aplicaciones del Teorema de la Correspondencia. 1. Dado un entero positivo n 2 +, vamos a describir los subgrupos del grupo cociente n = =hni. Escribiremos a = a + hni. Sabemos que los subgrupos de son precisamente los de la forma hdi con d 0, y que hdi  hd0 i si y solo si d0 j d. Por tanto, los subgrupos de n son precisamente los de la forma hhndii = hdi, donde d es un divisor positivo de n, y ademas hdi  hd0 i si y solo si d0 j d. As, el diagrama de los subgrupos de n puede construirse de modo elemental a partir de Z Z Z los divisores de n como muestran los siguientes diagramas (en el de la izquierda se ha tomado n = 125, y en el de la derecha n = 72): h1i h1i h2i h5i h25i h125i h4i h8i h3i h6i h12i h24i h9i h18i h36i h72i En general, si r es el numero de divisores primos distintos de n, se necesita un diagrama en r dimensiones por ejemplo, para n = 180 necesitaramos un diagrama tridimensional. Observese que, si n = dt, entonces hdi = f0 d 2d : : : (t ; 1)dg tiene t elementos. Por tanto, si t es cualquier divisor positivo de n y tomamos d = n=t, entonces hdi es el unico subgrupo de n con t elementos. Es decir, n verica el \recproco del Teorema de Lagrange" con una condicion extra de unicidad. Z Z 2. Aplicando el Teorema de la Correspondencia al diagrama de los subgrupos de D4 (Ejemplos 5.6.3), obtenemos el siguiente diagrama de los subgrupos de D4 =hr2 i. D4 =hr2i hr2  si=hr2i hri=hr2 i hr2  rsi=hr2i hr2 i=hr2i = f1g CAPITULO 5. GRUPOS 134 5.7 Homomorsmos y Teoremas de Isomorfa Aunque ya hemos usado la nocion de homomorsmo e isomorsmo de grupos, es en esta seccion donde las estudiamos de modo sistematico. Comenzamos repitiendo las deniciones: De nicion 5.7.1 Un homomorsmo del grupo (G ) en el grupo (H ) es una aplicacion f : G ! H que conserva la operacion es decir, que verica f(a b) = f(a)  f(b) para cualesquiera a b 2 G. Si G = H decimos que f es un endomorsmo de G. Si f : G ! H es un homomorsmo biyectivo, diremos que es un isomorsmo y que los grupos G y H son isomorfos. Un isomorsmo de G en G se dira un automorsmo de G. Dado un homomorsmo de grupos f : G ! H , se denen su imagen y su nucleo como Im f = f(G) = ff(x) : x 2 Gg  H Ker f = f ;1 (1H ) = fx 2 G : f(x) = 1H g  G: Ejercicio 5.7.2 Sea f : G ! H un homomorsmo de grupos. Demostrar que se verican las siguientes propiedades para a a1 : : : an 2 G: 1. (f conserva el neutro) f(1G ) = 1H . 2. (f conserva inversos) f(a;1 ) = f(a);1 . 3. (f conserva productos nitos) f(a1 an) = f(a1 ) f(an ). 4. (f conserva potencias) Si n 2 entonces f(an ) = f(a)n . 5. Si f es un isomorsmo entonces la aplicacion inversa f ;1 : H ! G tambien lo es. 6. Si g : H ! K es otro homomorsmo de grupos entonces g  f : G ! K es un homomorsmo de Z 7. 8. 9. 10. grupos. Si H1 es un subgrupo de H entonces f ;1 (H1 ) = fx 2 G : f(x) 2 H1g es un subgrupo de G. Si ademas H1 es normal en H entonces f ;1 (H1 ) es normal en G en particular, Ker f es un subgrupo normal de G. f es inyectivo si y solo si Ker f = f1g. Si G1 es un subgrupo de G entonces f(G1 ) es un subgrupo de H en particular, Im f es un subgrupo de H . Si ademas G1 es normal en G y f es suprayectiva entonces f(G1 ) es normal en H . La hipotesis de suprayectividad en la propiedad anterior no es superua es decir, dar un ejemplo de un homomorsmo de grupos f : G ! H y un subgrupo normal G1 de G, tal que f(G1 ) no es normal en H . Ejemplos 5.7.3 Homomorsmos de grupos. 1. Si H es un subgrupo de G, la inclusion de H en G es un homomorsmo inyectivo. 2. Si N es un subgrupo normal de G, la aplicacion : G ! G=N dada por (x) = xN es un homomorsmo suprayectivo que recibe el nombre de proyeccion canonica de G sobre G=N. Su nucleo es Ker = N. 3. Dados dos grupos G y H, la aplicacion f : G ! H dada por f(a) = 1H para cada a 2 G es un homomorsmo llamado homomorsmo trivial de G en H. Su nucleo es todo G. 4. La aplicacion f : ! dada por f(n) = 2n es un homomorsmo inyectivo y no suprayectivo. 5. Si G es cualquier grupo y x 2 G es cualquier elemento, la aplicacion ! G dada por n 7! xn es un homomorsmo de grupos como en usamos notacion aditiva y en G multiplicativa, la armacion anterior es equivalente al hecho, que ya conocemos, de que xn+m = xnxm . ZZ Z Z 5.7. HOMOMORFISMOS Y TEOREMAS DE ISOMORFIA 135 R RRR 6. Otro ejemplo en el que se mezclan las notaciones aditiva y multiplicativa es el siguiente: Fijado un numero real positivo , la aplicacion ! + dada por r 7! r es un isomorsmo de grupos cuya inversa es la aplicacion + ! dada por s 7! log s. Claramente, si f : G ! H es un homomorsmo inyectivo de grupos entonces f : G ! Im f es un isomorsmo de grupos que nos permite ver a G como un subgrupo de H. El siguiente teorema nos dice que todo grupo nito puede verse como un subgrupo de un grupo simetrico. Teorema 5.7.4 (Cayley) Todo grupo G es isomorfo a un subgrupo del grupo de permutaciones S(G). En particular, todo grupo nito G de orden n es isomorfo a un subgrupo de Sn . Demostracion. Por el comentario anterior, se trata de establecer la existencia de un homomorsmo inyectivo : G ! S(G). Si, para cada g 2 G, denimos (g) como la permutacion de G dada por x 7! gx (es una biyeccion con inversa (g;1 )), es elemental ver que es el homomorsmo buscado. La segunda parte se sigue del hecho de que, si jGj = n, entonces S(G) es isomorfo a Sn (Ejercicio 5.2.3). El siguiente ejercicio puede considerarse como una version del Teorema de la Correspondencia. Ejercicio 5.7.5 Sea N un subgrupo normal de un grupo G y sea : G ! G=N la proyeccion canonica. Demostrar que: 1. Si H es un subgrupo de G, entonces HN es un subgrupo de G que contiene a N , y se tiene (H) = HN=N . Ademas, HN es normal en G si lo es H . 2. Si H es un subgrupo de G que contiene a N , entonces (H) = H=N es la imagen de H por la aplicacion * del Teorema de la Correspondencia (5.6.5). Los Teoremas de Isomorfa que vimos para anillos tienen una version para grupos. Las demostraciones son analogas, de modo que solo indicamos sus lneas generales. Teorema 5.7.6 (Primer Teorema de Isomorfa) Sea f : G ! H un homomorsmo de grupos. Entonces existe un unico isomorsmo de grupos f% : G=Ker f ! Im f que hace conmutativo el diagrama f - H G p ? G=Ker f f% 6i - Im f es decir, i  f%  p = f , donde i es la inclusion y p es la proyeccion canonica. En particular G  Ker f = Im f: % Demostracion. Si K = Ker f, la aplicacion f% : G=K ! Im f dada por f(xK) = f(x) satisface las propiedades del enunciado. Usando el Teorema de la Correspondencia se obtiene el siguiente corolario. Corolario 5.7.7 En la situacion del Primer Teorema de Isomorfa, f% induce una biyeccion entre el conjunto de los subgrupos de G que contienen a Ker f y el de los subgrupos de H contenidos en Im f . Teorema 5.7.8 (Segundo Teorema de Isomorfa) Sean N y H subgrupos normales de un grupo G con N  H . Entonces H=N es un subgrupo normal de G=N y se tiene G=N  G=H: H=N = En particular, si G=N es nito, se tiene G=N : H=N] = G : H]: CAPITULO 5. GRUPOS 136 Demostracion. La aplicacion f : G=N ! G=H dada por f(xN) = xH es un homomorsmo suprayectivo con nucleo H=N. Teorema 5.7.9 (Tercer Teorema de Isomorfa) Sean G un grupo, H un subgrupo de G y N un subgrupo normal de G. Entonces N \ H es un subgrupo normal de H y se tiene H  NH N \H = N : Demostracion. La aplicacion f : H ! G=N dada por f(x) = Nx es un homomorsmo con nucleo H \ N e imagen NH=N. C R Ejemplos 5.7.10 Aplicaciones de los Teoremas de Isomorfa. C R R R R R R R RR RR R R R Z ZZ Z Z Z C R C R R 1. Consideremos los grupos multiplicativos  y , y la aplicacion norma  :  !  dada por (a + bi) = a2 + b2. Entonces  es un homomorsmo que tiene por nucleo a la circunferencia de radio 1 en , y por imagen a +. Por tanto, el grupo cociente de  por la circunferencia de radio 1 es isomorfo a +. 2. La aplicacion det : GLn ( ) !  que lleva una matriz a su determinante es un homomorsmo suprayectivo de grupos con nucleo SLn ( ). Esto nos dice que el cociente de GLn ( ) por SLn( ) es isomorfo a , cosa que ya habamos visto por metodos mas elementales (pero con mas trabajo). Usando la aplicacion \valor absoluto del determinante" se demuestra con la misma facilidad que el cociente de GLn( ) por ScLn ( ) es isomorfo a +. Por ultimo, la aplicacion det : ScLn( ) !  tiene nucleo SLn( ) e imagen f1 ;1g, luego el cociente de ScLn( ) por SLn( ) es isomorfo a 2. Cuando n = 2, podemos  tomar  0 como  represen1 1 0 tantes de cada una de las clases laterales a las matrices a = 0 1 y b = 1 0 . ZZ 3. Sea n un entero positivo. Hemos visto (Ejemplos 5.6.6) que todo subgrupo de n = =hni es de la forma hdi = hdi=hni, para cierto divisor positivo d de n. El Segundo Teorema de Isomorfa nos permite identicar el cociente n=hdi, pues n = =hni  hdi hdi=hni = hdi = d: 5.8 O rdenes de elementos y grupos cclicos De nicion 5.8.1 Sea G un grupo. El orden de un elemento g 2 G, denotado por o(g), es el cardinal del subgrupo hgi generado por g. Si hgi es innito escribimos o(g) = 1. Recordemos que, en general, hgi = fgn : n 2 g (o fng : n 2 g si usamos notacion aditiva). Z Z Aunque los posibles exponentes sean innitos, estos conjuntos pueden ser nitos si los gn se repiten. De hecho es claro que, en un grupo nito, todo elemento tiene orden nito.  Ejemplos 5.8.2 Ordenes de elementos. 1. 2. 3. 4. Z Es claro que o(g) = 1 si y solo si g es el neutro de G. Cualquier elemento no nulo de tiene orden innito. En , el elemento ;1 tiene orden 2 y cualquier elemento distinto de 1 tiene orden innito. p En  , los elementos ;1 ! = ;1+2 3i e i tienen ordenes 2 3 y 4, respectivamente. Mas generalmente, el numero complejo e 2ki = cos 2k + isen 2k tiene orden k. Un elemento cuya norma no sea 1 tiene orden innito. RC  5.8. ORDENES DE ELEMENTOS Y GRUPOS CICLICOS R 5. En GL2 ( ), la matriz 137  0 ;1  1 1 tiene orden 4, y la matriz a = tiene orden innito 1 0 1 n 0 1 Z pues, como se demuestra facilmente por induccion, an = 0 1 . 6. Si n es un entero positivo, los siguientes tienen orden  n: el elemento 1]n en n el giro  1 elementos 2 : : : n ; 1 n de angulo 2 =n en Dn el elemento 2 3 : : : n 1 en Sn . ZZ En las dos proposiciones siguientes damos diversas interpretaciones alternativas del concepto de orden de un elemento, separando los casos nito e innito. Dado un elemento g de un grupo G, denotaremos por g : ! G al homomorsmo dado por g (n) = gn . Es obvio que Im (g ) = hgi y que Ker (g ) = fn 2 : gn = 1g. Proposicion 5.8.3 Las condiciones siguientes son equivalentes para un elemento g de un grupo G y un entero positivo n: 1. o(g) = n. Z Z 2. El subgrupo hgi es isomorfo a n. 3. Ker (g ) = n . 4. Para un entero arbitrario m se tiene gm = 1 si y solo si n j m. 5. n es el menor entero positivo m tal que gm = 1. Z Z ZZ 6. hgi = f1 g g2 : : :  gn;1g con los gi distintos dos a dos cuando 0 i < n. Z Demostracion. Por el Primer Teorema de Isomorfa se tiene hgi = Im (g ) = =Ker (g ), y ademas sabemos que Ker (g ) es de la forma n para un unico n 0 y que n = =n tiene exactamente n elementos si n > 0 y es innito si n = 0. Usando esto, la equivalencia entre 1, 2 y 3 es clara. Tambien es obvio a partir de las deniciones que 3 es equivalente a 4. Dejaremos que el lector compruebe que 4 implica 5 y que 5 implica 3. Como es obvio que 6 implica 1, solo falta demostrar que 3 implica 6. Supongamos pues que se verica 3. Entonces los gi son distintos dos a dos cuando 0 i < n, pues una relacion gi = gj con 0 i < j < n implicara que 0 < j ; i < n y que j ; i 2 Ker (g ) = n , lo cual es absurdo. Ademas, todo elemento gm 2 hgi esta en f1 g g2 : : :  gn;1g, pues dividiendo con resto encontramos q i 2 con m = nq + i y 0 i < n, y entonces tenemos gm = (gn )q gi = 1q gi = gi . Por tanto, se verica la condicion 6. Z Z Aplicando el Teorema de Lagrange se obtiene: Corolario 5.8.4 Si G es un grupo nito entonces el orden de cualquier elemento g de G es nito y divide al orden de G. En particular se tiene gjGj = 1. Observese que el Teorema de Euler (1.8.1) puede obtenerse como una consecuencia inmediata de este ultimo resultado. Proposicion 5.8.5 Las condiciones siguientes son equivalentes para un elemento g de un grupo G: 1. o(g) = 1. 2. g : ! G es inyectiva. 3. Los elementos gm (con m 2 ) son distintos dos a dos. 4. Ker (g ) = 0 es decir, gm = 6 1 para todo m 1. 5. El subgrupo hgi es isomorfo a . Z Z Z 138 CAPITULO 5. GRUPOS Z Z Z Z Demostracion. De la Proposicion 5.8.3 deducimos que 1 implica 2, pues si g no fuera inyectiva entonces se tendra Ker (g ) = n con n 1, lo que implicara o(g) = n < 1. La equivalencia entre 2, 3 y 4 es evidente, y tambien lo es que 4 implica 5. Por ultimo, como es innito, 5 implica 1. Nuestros primeros ejemplos de grupos cclicos fueron y los n con n 1. Una consecuencia inmediata de los resultados precedentes es que, esencialmente, no hay otros ejemplos: Z Z Z ZZ Z Teorema 5.8.6 (Clasi cacion de los Grupos Cclicos) Salvo isomorsmos, es el unico grupo cclico innito y n es el unico grupo cclico nito con n elementos (n 2 +). En otras palabras, todo grupo cclico innito es isomorfo a , y todo grupo cclico nito G es isomorfo a jGj. Esto, a su vez, nos permite renar el Corolario 5.5.6, con lo que avanzamos en nuestro objetivo de describir los grupos en terminos de su orden. Teorema 5.8.7 Si p es un entero positivo primo entonces, salvo isomorsmos, nito de orden p. Z Z p es el unico grupo Vamos a obtener otros dos corolarios del Teorema 5.8.6 usando las descripciones de los subgrupos y cocientes de y de los n que hemos obtenido en los Ejemplos 5.4.3, 5.6.6 y 5.7.10. El segundo corolario nos dice que los grupos cclicos nitos verican el recproco del Teorema de Lagrange con una condicion extra de unicidad. Corolario 5.8.8 Los subgrupos y los grupos cociente de un grupo cclico son cclicos. Corolario 5.8.9 Sea G = hgi un grupo cclico de orden nito n. Entonces, para cada divisor positivo m de n, existe un unico subgrupo de orden m en G a saber, hgk i con k = n=m. Ejercicio 5.8.10 Mostrar un grupo nito G que tenga al menos un subgrupo de orden m para cada divisor m de jGj y tenga mas de uno para cierto divisor. Una consecuencia mas del Teorema de Clasicacion de los Grupos Cclicos (5.8.6) es la siguiente: Proposicion 5.8.11 Sea g un elemento de un grupo G entonces: 1. Si o(g) es innito entonces los unicos elementos de hgi que generan todo hgi son g y g;1. 2. Si o(g) = n es nito y k es cualquier entero, entonces n : o(gk ) = mcd(n k) En consecuencia, los elementos de hgi que generan todo hgi son exactamente los gk en los que k es coprimo con n. Z Z Z Demostracion. Por el Teorema 5.8.6, si o(g) es innito podemos suponer que hgi = y que g = 1, y el apartado 1 es entonces evidente. Supongamos pues que o(g) es nito y vale n. Podemos suponer que hgi = n y que g = 1]. Solo hay entonces que demostrar que el orden de k] en n es n=d, donde d = mcd(n k). El subgrupo de n generado por k] es hk ni=hni = hdi=hni, que, como vimos en los Z Ejemplos 5.6.6, tiene n=d elementos, con lo que el resultado queda demostrado. 5.9 Conjugacion y Ecuacion de Clases El concepto de elementos conjugados y de clases de conjugacion es muy importante en Teora de Grupos, y lo emplearemos con frecuencia en el resto del curso. En esta seccion, ademas de las deniciones y propiedades elementales, establecemos la Ecuacion de Clases, que es una herramienta fundamental para contar elementos en un grupo nito, y la empleamos para demostrar algunos resultados en los que se establecen propiedades de un grupo en funcion solo de su orden.  Y ECUACION  DE CLASES 5.9. CONJUGACION 139 De nicion 5.9.1 Sea G un grupo y sean a x 2 G. El conjugado de a por x es el elemento ax = x;1 ax un elemento b 2 G es un conjugado de a si es de la forma b = ax para algun x 2 G. Si A es un subconjunto de G llamamos conjugado de A por x al subconjunto Ax = fax : a 2 Ag. La notacion ax para el producto x;1ax es util en muchas situaciones. Aunque aparentemente puede confundirse con la notacion introducida al principio de la Seccion 5.1, la naturaleza del exponente (elemento del grupo multiplicativo que estemos manejando o numero entero) nos sacara de dudas en cada caso concreto. En un grupo abeliano es claro que ax = a para cada x, luego en el contexto abeliano la conjugacion no tiene interes. Las propiedades elementales de la conjugacion son faciles de demostrar, y las proponemos como ejercicio. Ejercicio 5.9.2 Sea G un grupo y sean a b x 2 G. Demostrar que: 1. (ax );1 = (a;1)x . 2. (ab)x = ax bx . 3. o(ax ) = o(a). 4. Si H es un subgrupo de G entonces H x es un subgrupo de G del mismo orden que H (los subgrupos de este tipo se dice que son conjugados de H ). 5. Si A y B son subconjuntos de G entonces A  B , Ax  B x . 6. ax = a , ax = xa. 7. b = ax , xb = ax , a = b(x 1 ) . ; 8. axy = (ax )y . 9. La relacion \ser conjugados" es una relacion de equivalencia en G. Ejercicio 5.9.3 Dados un grupo G y un subgrupo N , demostrar que las siguientes condiciones son equivalentes: 1. N es normal en G. 2. N x = N para cada x 2 G (es decir, N coincide con todos sus conjugados). 3. N x  N para cada x 2 G (es decir, N contiene a todos sus conjugados). Ejercicio 5.9.4 Dado un grupo G, demostrar que: 1. Dado x 2 G, la aplicacion tx : G ! G dada por tx (a) = xax;1 (conjugacion por x;1 ) es un automorsmo de G, con inverso tx 1 , llamado automorsmo interno (o interior) de G inducido por x. Los elementos jados por tx son precisamente los que pertenecen al centralizador CenG (x). ; 2. El conjunto Inn(G) de los automorsmos internos de G es un subgrupo normal del grupo Aut(G) (Ejemplo 5.3.8) que se denomina grupo de los automorsmos internos de G. 3. La aplicacion t : G ! Aut(G) dada por x 7! tx es un homomorsmo de grupos con imagen Inn(G) y nucleo Z(G) (el centro de G). En consecuencia, Inn(G)  = G=Z(G). Las clases de equivalencia para la relacion \ser conjugados" en un grupo G se llaman clases de conjugacion de G, de modo que G es la union disjunta de sus clases de conjugacion. La clase de conjugacion de a se denota por aG es decir, aG = fax : x 2 Gg: Observese que todos los elementos de una clase de conjugacion tienen el mismo orden. CAPITULO 5. GRUPOS 140 Proposicion 5.9.5 (Ecuacion de Clases) Sea G un grupo arbitrario y sea a 2 G. Entonces: 1. La clase de conjugacion de a es unitaria (es decir, solo contiene al propio a) si y solo si a 2 Z(G). 2. jaGj = G : CenG (a)] (donde CenG (a) es el centralizador de a). En particular, jaG j jCenG (a)j = jGj, y por tanto jaGj divide a jGj. 3. Si G es nito y , es un conjunto de representantes de las clases de conjugacion no unitarias de G, entonces X X jGj = jZ(G)j + b2 jbGj = jZ(G)j + G : CenG (b)]: b2 Esta igualdad se conoce con el nombre de Ecuacion de Clases de G. Demostracion. 1. Es claro, pues ax = a si y solo si ax = xa. 2. Pongamos H = CenG (a). Como G : H] es el cardinal del conjunto H nG de las clases por la derecha modulo H, el resultado estara probado si vemos que la aplicacion H nG ! aG dada por Hx 7! ax es una biyeccion. Las equivalencias Hx = Hy , xy;1 2 H , axy;1 = xy;1 a , x;1 ax = y;1 ay , ax = ay muestran que la aplicacion esta bien denida y es inyectiva, mientras que la suprayectividad es obvia. 3. Como las clases de conjugacion constituyen una particion de G, el cardinal jGj es la suma de los cardinales de cada clase de conjugacion, y la ecuacion de clases lo unico que hace es recoger P la suma de los cardinales de las clases unitarias en jZ(G)j (por el apartado 1), y el resto en la suma G : CenG (b)] (por el apartado 2). Ejemplo 5.9.6 Clases de conjugacion en Dn . Sea R la rotacion de angulo 2 =n en Dn . Como hRi  CenDn (Ri ) para todo i, y hRi tiene ndice 2 en Dn , tenemos dos opciones: CenDn (Ri ) = Dn o CenDn (Ri ) = hRi: Por la Proposicion 5.9.5, la clase de conjugacion Ri Dn tiene uno o dos elementos. El primer caso se da si Ri es central, o sea, por el Ejercicio 5.3.6, si i = 0 o si n es par e i = n=2. En el segundo caso, Ri Dn = fRi R;ig. Para calcular las demas clases de conjugacion, observese que un elemento de Dn es una simetra respecto de una recta precisamente si deja ja una recta de 2. Si g 2 Dn y S es la simetra respecto de cierta recta L, entonces gSg;1 deja ja la recta gL, y por tanto es otra simetra. Si L pasa por un vertice, entonces gL tambien pasa por un vertice, porque g permuta los vertices. Utilizando esto, se ve facilmente que, si n es impar, las simetras respecto de rectas forman una clase de conjugacion. Sin embargo, si n es par, las simetras se dividen en dos clases de conjugacion: una esta formada por las simetras respecto de rectas que unen dos vertices, y otra por las simetras que unen puntos medios de lados opuestos. El lector puede tratar de obtener los mismos resultados empleando solo la operacion en Dn , en lugar de usar argumentos geometricos. La Ecuacion de Clases nos da informacion sobre jGj, y cuando este entero tiene una factorizacion comoda es posible sacarle bastante partido. Un tipo de enteros con factorizacion especialmente comoda son las potencias de numeros primos veamos que cosas se pueden decir sobre este tipo de grupos: Proposicion 5.9.7 Si jGj = pn para cierto entero primo p y cierto n 1, entonces Z(G) 6= f1g. Demostracion. La ecuacion de clases nos dice que pn = jZ(G)j + PG : CenG (a)], y como p divide a cada G : CenG (a)] y a pn deducimos que p divide a jZ(G)j, lo que nos da el resultado. R Cuando el orden del grupo G es p2 se puede armar algo mas contundente: G es abeliano. El resultado se basa en este lema, que tiene interes por s mismo.  Y ECUACION  DE CLASES 5.9. CONJUGACION 141 Lema 5.9.8 Si G=Z(G) es cclico entonces G es abeliano (luego G=Z(G) es trivial). En particular, G : Z(G)] no puede ser un entero primo para ningun grupo G. Z Demostracion. Sea Z = Z(G). Si G=Z es cclico y aZ es un generador de G=Z, entonces todo elemento de G es de la forma an z, con n 2 y z 2 Z, y es elemental ver que dos elementos de esta forma conmutan. Proposicion 5.9.9 Si p es un entero primo, entonces todo grupo nito G de orden p2 es abeliano. Demostracion. Por el Teorema de Lagrange, el orden de Z(G) solo puede ser 1, p o p2. Como jZ(G)j =6 1 por la Proposicion 5.9.7, y jZ(G)j 6= p por el Lema 5.9.8, ha de ser jZ(G)j = p2, de modo que G = Z(G). La Ecuacion de Clases nos permite encontrar una condicion que garantiza la normalidad de ciertos subgrupos de un grupo de orden pn. Proposicion 5.9.10 Sea G un grupo nito de orden pn , con p primo y n 1. Si H es un subgrupo de G de orden pn;1, entonces H es normal en G. Demostracion. Haremos induccion sobre n. Para n = 1, el resultado es obvio, de modo que supondremos n > 1. Por la Proposicion 5.9.7 y el Teorema de Lagrange se tiene jZ j = ps para cierto s con 1 s n. Es obvio que ZH = HZ, luego este es un subgrupo de G que contiene a H (Ejercicio 5.4.10), y en consecuencia solo hay dos opciones: o bien HZ = H o bien HZ = G. En el segundo caso H es normal en G. En efecto, dado g 2 G = HZ se tiene g = hz con h 2 H y z 2 Z, luego H g = (H h )z = H h = H. Por tanto, podemos asumir que HZ = H. Entonces Z es un subgrupo normal de H, y as n jG=Z j = jjGZ jj = pps = pn;s y n;1 jH=Z j = jjHZ jj = p ps = pn;s;1: Si aplicamos la hipotesis de induccion al grupo GZ observamos que HZ es normal en GZ , y ahora el resultado es consecuencia del Teorema de la Correspondencia (5.6.5). Grupos de orden peque~no Como parece apreciarse por los resultados obtenidos hasta ahora, en general podremos describir mejor los grupos de orden n cuanto mas sencilla sea la factorizacion de n en primos. En este parrafo vamos a recopilar lo que ya sabemos para algunos valores bajos de n, y a plantear algunos objetivos para el futuro. Z ZZ Z ZZ 1. Consideremos los siguientes grupos de orden 4: el grupo cclico 4, el producto 2  2 y el grupo cociente Q8 =N de los Ejemplos 5.6.3. No es difcil dar explcitamente un isomorsmo entre 2  2 y Q8=N. Por otra parte, en estos dos grupos todo elemento a verica o(a) j 2, cosa que no ocurre en 4. Esto nos dice que hay al menos dos grupos de orden 4 no isomorfos, y sabemos que cualquier otro debe ser abeliano (Proposicion 5.9.9). Veremos en el Captulo 7 que de hecho no hay mas. Z Z ZZ 2. Consideremos los siguientes grupos de orden 6: el grupo cclico 6, el producto 2  3, S3 y D3 . Ya vimos que los dos ultimos son isomorfos (Ejercicio 5.2.9), y los dos primeros tambien lo son (>por que?). Por otra parte, 6 es abeliano y S3 no, luego no pueden ser isomorfos. Esto nos dice que hay al menos dos grupos no isomorfos de orden 6 ( 6 y D3 ), y veremos en el Captulo 8 que no hay mas. Z 142 Z ZZZ CAPITULO 5. GRUPOS ZZ 3. Consideremos los siguientes grupos de orden 8: el grupo cclico 8, los productos 2  4 y 2  2  2, y los grupos no abelianos D4 y Q8. Entre ellos no hay dos isomorfos: Ninguno de los abelianos puede ser isomorfo a uno de los no abelianos D4 y Q8 no son isomorfos porque en el segundo solo hay un elemento de orden 2 y en el primero hay cuatro dejamos que el lector use argumentos de este tipo para ver que tampoco puede haber isomorsmos entre los abelianos. Esto nos dice que hay al menos cinco grupos no isomorfos de orden 8, y veremos en el Captulo 8 que no hay mas. 4. Consideremos los siguientes grupos de orden 9: el grupo cclico 9 y el producto 3  3 (no los hay no abelianos por la Proposicion 5.9.9). No son isomorfos porque en el segundo no hay elementos de orden 9 y en el primero s. Veremos en el Captulo 7 que, salvo isomorsmos, no hay mas grupos de orden 9. 5. Consideremos los siguientes grupos de orden 10: el grupo cclico 10, el producto 2  5, y el grupo no abeliano D5 . Los dos primeros son isomorfos, y desde luego no son isomorfos al no abeliano. En cuanto a grupos de orden 14 tenemos 14  6= D7 . Veremos en el Captulo 8 = 2 7  que, salvo isomorsmos, solo hay estos dos grupos de cada uno de los ordenes 10 y 14. 6. Veremos en el Captulo 8 que, salvo isomorsmos, solo hay un grupo de orden 15 (>cual crees que va a ser?), y hay exactamente 5 grupos de orden 12. Z ZZ Z Z ZZ ZZ 5.10 Problemas NN R R 1. Decir cuales de los siguientes conjuntos y operaciones son grupos. (a) El conjunto con la operacion maximo comun divisor. (b) El conjunto con la operacion mnimo comun multiplo. (c) El conjunto con la operacion a  b = a + b + ab. x+y . (d) El conjunto A = fx 2 : ;1 < x < 1g, con la operacion x  y = 1+ xy 2. Construir la tabla de multiplicacion de los siguientes grupos. (a) Los grupos de unidades de 7 y 16. (b) GL2( 2). (c) El subgrupo de GL2 ( ) generado por las matrices Z C C ZZ a= i 0 0 i  b= 0 i i 0 : (d) El subgrupo de GL2 ( ) generado por las matrices p Q ! 0  a = 0 !;1  0 1 b= 1 0  donde ! = 1+ 2 ;3 . (e) El subgrupo de GL2 ( ) generado por las matrices a=  0 1 ;1 0  Z b=  0 ;1  : 1 ;1 (f) El grupo cociente G=h;I i, donde G es el grupo del apartado anterior e I es la matriz identidad. (g) El grupo de los automorsmos del grupo 5. R 5.10. PROBLEMAS (h) El subgrupo del grupo de la permutaciones de A = n f0 1 2g generado por f y g, donde f(x) = 2 ; x y g(x) = x2 : (i) 16 con la operacion x  y = x + (;1)x + y. Decidir si alguno de estos grupos es isomorfo a algun otro grupo conocido. Construir el diagrama de los subgrupos de los grupos anteriores, indicando cuales de ellos son normales. Sea f : G ! H un isomorsmo de grupos. Demostrar (mientras no se considere una perdida de tiempo) que: (a) Los elementos a y a0 conmutan en G si y solo si f(a) y f(a0 ) conmutan en H. (b) G es abeliano si y solo si H es abeliano. (c) El orden de a en G coincide con el orden de f(a) en H. (d) Un subconjunto K de G es un subgrupo de G si y solo si f(K) es un subgrupo de H. En este caso, los ndices G : K] y H : f(K)] coinciden. (e) Un subconjunto N de G es un subgrupo normal de G si y solo si f(N) es un subgrupo normal de H. (f) Un subconjunto X de G genera el subgrupo K si y solo si el subconjunto f(X) de H genera el subgrupo f(K). (g) G es cclico si y solo si H es cclico. (h) Si Z es el centro de G entonces f(Z) es el centro de H. (i) Si C es el centralizador de a en G entonces f(C) es el centralizador de f(a) en H. (j) Un subconjunto C de G es una clase de conjugacion de G si y solo si f(C) es una clase de conjugacion de H. Probar que todo grupo G de orden menor o igual a cinco es abeliano. Sean H, K y L subgrupos de un grupo G con H  L. Probar que (HK) \ L = H(K \ L). Esta identidad de conoce como identidad de Dedekind. Sea G un grupo. Para cada una de las dos armaciones que siguen, dar una demostracion o poner un contraejemplo: (a) Para cada a 2 G, existe un a0 2 G tal que aa0 a = 1: (b) Para cada a 2 G, existe un a0 2 G tal que a0 aa0 = 1: (c) G posee a lo sumo un elemento a que verica a2 = a: Sea G un grupo. Probar que las siguientes armaciones son equivalentes: (a) G es abeliano. (b) (ab)2 = a2 b2 para cualesquiera a b 2 G. (c) (ab);1 = a;1 b;1 para cualesquiera a b 2 G. (d) (ab)n = an bn para todo n 2 y para cualesquiera a b 2 G. (e) *] (ab)n = anbn para tres enteros consecutivos jos y para cualesquiera a b 2 G. Probar, ademas, que la ultima condicion no es equivalente a las demas si se sustituye \tres" por \dos". Mostrar que la union de dos subgrupos de un grupo no es necesariamente un subgrupo. Aun mas, probar que un grupo nunca puede expresarse como union de dos subgrupos propios. Z 3. 4. 5. 6. 7. 8. N 9. 143 144 Z CAPITULO 5. GRUPOS 10. Para n = 1 : : : 10, determinar cuales de los grupos n son cclicos. 11. Supongamos que H y K son dos subgrupos de un grupo G, tales que existe una clase lateral por la derecha de G modulo H que es igual a otra de G modulo K. Probar que H = K. 12. Sea G un grupo arbitrario. Mostrar que si K y L son subgrupos de G de ndice nito y K  L entonces G : K] = G : L] L : K]. 13. Sea G un grupo con subgrupos H y K. Demostrar que, si la interseccion de dos clases laterales Hx y Ky no es vaca, entonces es una clase lateral modulo H \ K. Deducir que si H y K tienen ndice nito en G, entonces tambien lo tiene H \ K. 14. Calcular el orden de cada elemento de los grupos Dn .   15. Calcular el orden en S5 de la permutacion 13 24 35 42 51 . 16. Demostrar las siguientes variantes del Teorema Chino de los Restos: (a) Si G1 : : :  Gr son grupos cclicos nitos de ordenes n1  : : :  nr , entonces G = G1  : : :  Gr es cclico (de orden n1 nr ) precisamente si los ni son coprimos dos a dos. (b) Si a y b son dos elementos de un grupo que conmutan entre si y tienen ordenes nitos n y m, entonces ha bi es cclico de orden nm precisamente si mcd(n m) = 1. 17. >Es cclico el producto directo de dos grupos cclicos innitos? 18. Sea K un cuerpo. Demostrar que:   (a) El subconjunto G de GL2 (K) formado por las matrices invertibles de la forma a0 bd es un subgrupo.   (b) El subconjunto N de las matrices de la forma 10 b1 es subgrupo normal de G. 19. 20. 21. 22. 23. 24. (c) El cociente G=N es abeliano. Demostrar que la interseccion de una familia de subgrupos normales de un grupo tambien es un subgrupo normal. Sea H un subgrupo de un grupo G. Demostrar que \g2G g;1 Hg es el mayor subgrupo normal de G contenido en H y el subgrupo generado por g2G g;1 Hg es el menor subgrupo normal de G que contiene a H. Sea P una particion de un grupo G con la propiedad de que para cualquier par de elementos A B de la particion, el producto AB = fab : a 2 A b 2 B g es otro elemento de la particion. Sea N el elemento de la particion que contiene al neutro 1 2 G. Probar que N es un subgrupo normal de G y que P consiste en las clases laterales de G modulo N. Demostrar que la propiedad de \ser normal" no es transitiva. Es decir, dar un ejemplo de un grupo G con subgrupos H y K tales que H sea normal en K, K sea normal en G, y H no sea normal en G. Un grupo se dice que es simple si no tiene ningun subgrupo normal propio no trivial. Un subgrupo normal maximal del grupo G es un subgrupo normal propio de G que no esta contenido propiamente en ningun subgrupo normal propio de G. Demostrar que N es un subgrupo normal maximal de G si y solo si G=N es simple. Demostrar que si N es un subgrupo normal maximal de G y H es un subgrupo de G que no esta contenido en N, entonces N \ H es un subgrupo normal maximal de H. Encontrar todos los grupos cclicos G, salvo isomorsmos, que tengan exactamente dos generadores (es decir, tales que existan exactamente dos elementos x 2 G con G = hxi). 5.10. PROBLEMAS 145 25. Sean a b dos elementos en un grupo G y sea c = a b] = aba;1 b;1 su conmutador. Probar que, si c conmuta con a y con b, entonces se verica, para todo par r s de enteros positivos, ar  bs] = crs . 26. Probar que todo grupo de orden par posee un elemento de orden 2. 27. Sean H y K subgrupos de un grupo G, y sea g 2 G cualquiera. El conjunto HgK = fx 2 G : x = hgk para ciertos h 2 H k 2 K g se llama una doble clase lateral. (a) Probar que las dobles clases laterales son una particion de G. (b) >Es cierto que todas las dobles clases laterales tienen el mismo cardinal? 28. Sean N y M subgrupos normales de un grupo G tales que N \ M = f1g. Probar que nm = mn para todo n 2 N y m 2 M. 29. Sea N un subgrupo normal de ndice n de un grupo G. Demostrar que gn 2 N para todo g 2 G, y dar un ejemplo que muestre que esta propiedad falla si N no es normal en G. 30. Si N es un subgrupo normal en un grupo G y a 2 G tiene orden n, probar que el orden de Na en G=N es un divisor de n. 31. Si G y H son grupos, Hom(G H) denota el conjunto de los homomorsmos de G a H. (a) Demostrar que si H es abeliano, entonces Hom(G H) es un grupo con la operacion natural: (' )(g) = '(g) (g) (g 2 G): 32. 33. 34. 35. 36. 37. 38. Z ZZZ Z Z Z Z Z Z Z (b) Demostrar que si G es abeliano, entonces Hom(  G)  = G y Hom( n G)  = fg 2 G : gn = eg. (c) Calcular Hom( 3 8) y Hom( 3 21). (d) Probar que Aut( n)  = n. (e) Mostrar que, aun cuando G sea cclico, Aut(G) no tiene por que ser cclico. (f) Describir Aut( ). Probar que si n j m entonces existen un homomorsmo inyectivo n ! m y un homomorsmo suprayectivo m ! n. Demostrar que, si el grupo G no es abeliano, entonces existe un subgrupo abeliano de G que contiene estrictamente al centro Z(G). Demostrar que si H es un subgrupo de G y g 2 G, entonces H g = fhg : h 2 H g es un subgrupo de H tal que jH j = jH g j. Demostrar ademas que H es normal en G si y solo si lo es cualquier Hg. Un subgrupo H del grupo G es caracterstico si, para cualquier automorsmo f de G, se verica f(H)  H. Se pide: (a) Demostrar que todo subgrupo caracterstico de G es un subgrupo normal de G. (b) Dar un ejemplo de un grupo con un subgrupo normal que no sea caracterstico. (c) Si H es un subgrupo caracterstico de G y K es un subgrupo caracterstico de H, demostrar que K es un subgrupo caracterstico de G. (d) Demostrar que el centro de un grupo es un subgrupo caracterstico. Supongamos que H es un subgrupo de un grupo G, y que ningun otro subgrupo de G contiene un subgrupo del mismo cardinal que H. Demostrar que H es un subgrupo caracterstico (y por tanto normal) de G. Sea G un grupo nito con un subgrupo normal H tal que jH j y G : H] son coprimos. Si jH j = n, probar que H es el unico subgrupo de G de orden n. *] Sea G un grupo para el que existe un n > 1, tal que la aplicacion x 7! xn es un automorsmo. Probar que xn;1 esta en el centro de G para todo x 2 G. ZZ CAPITULO 5. GRUPOS 146 39. Sean N1 y N2 dos subgrupos normales de dos grupos G1 y G2. Demostrar que N1  N2 es un subgrupo normal de G1  G2 y que (G1  G2)=(N1  N2 )  = G1 =N1  G2=N2: 40. Sea f : G ! H un homomorsmo de grupos y sean G1 y H1 dos subgrupos normales de G y H, respectivamente, tales que f(G1 )  H1. Demostrar que existe un unico homomorsmo de grupos f% : G=G1 ! H=H1 que hace conmutativo el siguiente diagrama, donde las echas verticales designan las proyecciones canonicas: G # !f f H # G=G1 ! H=H1 41. 42. 43. 44. 45. 46. 47. 48. 49. 50. 51. % e Im (f) % en funcion de G1, H1, Ker (f) e Im (f). Identicar Ker (f) Sean H y N subgrupos de G. Supongamos que H tiene orden nito, que N tiene ndice nito en G y que jH j y G : N] son coprimos. Se pide: (a) Mostrar que si N es normal en G entonces H  N: (b) Mostrar que si H es normal en G entonces NH : N] = H : N \ H]. (c) Deducir que si H es normal en G, entonces H  N: Sea K un subgrupo normal nito de un grupo G, sea n un entero positivo coprimo con jK j, y sea x 2 G. Probar: (a) Si o(x) = n entonces el orden de xK en el grupo cociente G=K es tambien n. (b) Si el orden de xK en el cociente es n, entonces existe y 2 G de orden n tal que xK = yK. *] Sean a y p enteros con p primo impar. Demostrar que el polinomio X 2 ; a]p tiene una raz en p precisamente si a(p+1)=2  a mod p. Sea p un entero primo. Comprobar que p = fa=b + 2 Q : a b 2 b = pn para algun n 2 g Z es un subgrupo innito de = en el que el orden de cada elemento es una potencia de p. Demostrar que, si G el grupo diedrico D4 o el de cuaterniones Q8, entonces Z(G)  = 2 y G=Z(G)  =   , y sin embargo D 6 Q . = 2 2 4 8 Probar que, salvo isomorsmos, solo hay dos grupos no abelianos de orden 8. >Cuales son? Probar que todo grupo no abeliano de orden 6 es isomorfo a S3 . Sean N1 y N2 subgrupos normales de G1 y G2 respectivamente. Dar ejemplos que muestren que cada una de las siguientes implicaciones es falsa. (a) G1  = N2 ) G1 =N1  = G2 =N2. = G2 N1    (b) G1 = G2 G1=N1 = G2 =N2 ) N1  = N2 .   (c) N1 = N2  G1=N1 = G2=N2 ) G1  = G2. Probar las siguientes armaciones sobre el grupo abeliano de los numeros racionales. (a) Si un subgrupo H de es nitamente generado, entonces H es cclico. (b) no es cclico ni siquiera es nitamente generado. Demostrar que si H es un subgrupo abeliano de un grupo G tal que HZ(G) = G, entonces G es abeliano. Deducir que si G=Z(G) es cclico, entonces G es abeliano. Describir todos los subgrupos normales del grupo diedrico Dn . Z ZZ Q QZ Z Q 1 Z Z Q Z N 5.10. PROBLEMAS 147 52. *] Demostrar que el grupo de automorsmos de Dn tiene n (n) elementos, donde denota la funcion de Euler. 53. Calcular los centros de GLn ( ), GLn ( ), SLn ( ) y SLn ( ). 54. Demostrar que un grupo es nito precisamente si tiene un numero nito de subgrupos. 55. Si p es primo, probar que el centro de cualquier grupo no abeliano de orden p3 tiene orden p. 56. Sea G un grupo con un subgrupo N. Demostrar que N es normal en G si y solo si N es la union de ciertas clases de conjugacion de G. 57. *] Sea G un grupo abeliano nito en el que, para cada n 2 +, la ecuacion xn = e tiene a lo sumo n soluciones. Demostrar que G es cclico. Deducir que un subgrupo nito del grupo de unidades de un dominio es cclico. (Indicacion: Elegir un elemento de orden maximo y observar que para cada g 2 G de orden n, el subgrupo hgi contiene n soluciones de la ecuacion xn = e.) R C R C Z Bibliografa del captulo Allenby 1], Clark 9], Delgado-Fuertes-Xambo 11], Dorronsoro-Hernandez 13], Herstein 20], Jacobson 23], Rotman 30]. 148 CAPITULO 5. GRUPOS Captulo 6 Grupos de permutaciones Se estudian los grupos de permutaciones en un conjunto nito y sus subgrupos alternados. Introduccion El grupo simetrico Sn (grupo de permutaciones de un conjunto nito de n elementos) no es solo un ejemplo relevante de grupos no abelianos nitos, sino que tiene una gran importancia teorica e historica en el desarrollo de las Ciencias que se maniesta en muy diversas situaciones. Nosotros ya conocemos el Teorema de Cayley, y de hecho representaremos en la practica algunos grupos nitos como subgrupos de grupos simetricos. La aplicacion teorica fundamental de los grupos simetricos se vera en la asignatura Ecuaciones Algebraicas de Tercer Curso. El primer objetivo de este captulo es conseguir una representacion comoda de los elementos de Sn que nos permita operarlos con facilidad. Para ello describimos unos elementos sencillos de Sn , los ciclos, y demostramos que toda permutacion de Sn admite una expresion, unica salvo el orden, como producto de ciclos \disjuntos". Esta expresion, facil de obtener en la practica, permite calcular automaticamente el orden y la clase de conjugacion de un elemento de Sn , y proporciona diversos sistemas generadores comodos de Sn . En la segunda parte del captulo denimos el signo (o la paridad) de una permutacion y estudiamos el subgrupo alternado de Sn , formado por las permutaciones pares. Los grupos alternados An nos permiten obtener un contraejemplo para el recproco del Teorema de Lagrange (cuando n = 4) y nos proporcionan una familia de grupos simples (cuando n 5) que, en el curso de Ecuaciones Algebraicas, se usaran para demostrar un importante teorema sobre la irresolubilidad por radicales de ecuaciones polinomicas de grado 5. Objetivos del captulo Saber expresar una permutacion  de Sn como producto de ciclos disjuntos, y calcular a partir de esa expresion el orden y la clase de conjugacion de , as como el conjugado de  por cada elemento  de Sn . Conocer el concepto de signo de una permutacion, sus propiedades y las formas de calcularlo. Conocer el grupo alternado y sus propiedades basicas. Conocer distintos sistemas generadores de los grupos simetricos y alternados. Conocer el concepto de grupo simple y el Teorema de Abel sobre la simplicidad de los grupos alternados An para n 5. 149 CAPITULO 6. GRUPOS DE PERMUTACIONES 150 Desarrollo de los contenidos 6.1 Ciclos y trasposiciones N N N Recordemos que, para cada numero natural n, Sn denota el grupo simetrico sobre n es decir, el grupo de las aplicaciones biyectivas f : n ! n con la composicion de aplicaciones como operacion. Generalizando la notacion que introdujimos en el Ejercicio 5.2.5, describiremos a veces un elemento f 2 Sn dando la lista de sus imagenes en la forma  1 2 ::: n  f(1) f(2) : : : f(n) : Esta notacion puede ser bastante incomoda, y en esta primera seccion la simplicamos destacando ciertos elementos que ademas nos proveen con sistemas de generadores \comodos" de Sn . N De nicion 6.1.1 Diremos que una permutacion  2 Sn ja un entero i 2 n si (i) = i en caso contrario diremos que  cambia o mueve i, y denotaremos por M() al conjunto de los enteros cambiados por : N M() = fi 2 n : (i) 6= ig: Es claro que M() es vaco si y solo si  = 1, y que M() no puede tener exactamente un elemento. Diremos que dos permutaciones  y  de Sn son disjuntas si lo son los conjuntos M() y M(). Es decir, si todos los elementos que cambia una de ellas son jados por la otra. Cuando digamos que ciertas permutaciones 1  : : :  r son disjuntas entenderemos que lo son dos a dos. Ejercicio 6.1.2 Sean   2 Sn. Demostrar que M() = M(;1), y que si  y  son disjuntas entonces conmutan ( = ) y se tiene M() = M()  M(). De nicion 6.1.3 La permutacion  2 Sn es un ciclo de longitud s (o un s-ciclo) si M() tiene s elementos y estos pueden ordenarse de manera que se tenga M() = fi1  i2 : : :  is g y (i1 ) = i2  (i2 ) = i3  : : : (is;1 ) = is  (is ) = i1 : Observese que para cualquier i 2 M() se tiene M() = fi (i) 2 (i) : : :  s;1(i)g. Por ejemplo, los siguientes elementos de S4 son ciclos de longitudes 2, 3 y 4, respectivamente:        = 14 22 33 41 = 11 24 32 43 = 13 21 43 24 : Buscamos una representacion mas sencilla de las permutaciones, y esta depende de probar que toda permutacion es producto de ciclos, cosa que haremos pronto, y de encontrar una representacion adecuada para los ciclos. Esto ultimo es facil: como esta claro que un ciclo  queda determinado por la descripcion de M() en el orden adecuado, denotaremos el ciclo general de la Denicion 6.1.3 por  = (i1 i2 i3 : : : is ) o  = (i1  i2  i3 : : : is ): Observese que esta representacion no es unica. Por ejemplo, los ciclos de S4 que hemos dado como ejemplo se escribiran as (lo que ayuda a justicar el nombre de ciclo):  = (1 4) = (4 1) = (2 4 3) = (4 3 2) = (3 2 4) = (1 3 4 2) = (3 4 2 1) = (4 2 1 3) = (2 1 3 4): Ejercicio 6.1.4 Sea  = (i1 : : :is ) un ciclo de longitud s en Sn . Demostrar que: 1. Para cada t 2 f2 : : :  sg se tiene  = (it : : :is i1 : : :it;1 ). 2. Para cada t 2 f2 : : :  sg se tiene it = t;1(i1 ). 3. El orden de  coincide con su longitud s. 6.1. CICLOS Y TRASPOSICIONES 151 El resultado que nos da la representacion buscada de las permutaciones de Sn es el siguiente: Teorema 6.1.5 Toda permutacion  6= 1 de Sn se puede expresar de forma unica (salvo el orden) como producto de ciclos disjuntos. Z Demostracion. Denimos en M() 6= la siguiente relacion binaria: i  j , existe n 2 tal que n (i) = j: Dejamos que el lector compruebe que  es una relacion de equivalencia. Fijado i 2 M(), sea s el menor entero positivo tal que s (i) = i (>por que existe uno?). Entonces la clase de equivalencia que contiene a i es fi (i) 2(i) : : :  s;1(i)g (>por que son distintos?). En particular, s 2, y s es el cardinal de la clase de equivalencia que contiene a i. Sea ahora fi1 : : :  ik g un conjunto de representantes de las clases de equivalencia de  es decir, el conjunto tiene exactamente un elemento de cada clase de equivalencia. Para cada j = 1 : : : k, sea sj el cardinal de la clase de ij . Entonces  = 1 k es la factorizacion buscada, donde j = (ij  (ij ) 2(ij ) : : :  sj ;1 (ij )): El orden de los ciclos se puede alterar por la propia demostracion o por el Ejercicio 6.1.2, y tambien de la demostracion se deduce la unicidad salvo el orden, pues si  = 1 k es una factorizacion de  en producto de ciclos disjuntos, entonces las clases de equivalencia de  son los conjuntos M(1 ) : : :  M(k ). Ejemplo 6.1.6 Factorizacion de una permutacion como producto de ciclos disjuntos. Consideremos la permutacion de S11   9 10 11 :  = 16 25 31 44 52 67 73 88 11 9 10 Elegimos un elemento arbitrario cambiado por , por ejemplo el 1, y calculamos sus imagenes sucesivas por : (1) = 6 2 (1) = (6) = 7 3 (1) = (7) = 3 4 (1) = (3) = 1: Entonces (1 6 7 3) es uno de los factores de . Elegimos ahora un elemento de M() que no haya aparecido aun, por ejemplo el 2, y le volvemos a seguir la pista, lo que nos da un nuevo factor (2 5). Empezando ahora con el 9 obtenemos un tercer ciclo (9 11 10) que agota el proceso (el 4 y el 8 son jados por ) y nos dice que  = (1 6 7 3)(2 5)(9 11 10). Veamos como se puede calcular el orden de una permutacion en terminos de su factorizacion como producto de ciclos disjuntos: Proposicion 6.1.7 Sea  = 1 k la factorizacion de una permutacion  como producto de ciclos disjuntos, y sea si la longitud del ciclo i . Entonces N o() = mcm(s1  : : :  sk ): Demostracion. Sea m 2 . Como los i conmutan entre s, se tiene m = 1m km . Por otra parte, para cada i se tiene M(im )  M(i ) y por tanto los im son disjuntos. Esto implica, por la unicidad en el Teorema 6.1.5, que m = 1 precisamente si cada im = 1, y entonces el resultado es claro, pues si es el orden de i . A continuacion vamos a describir las clases de conjugacion de Sn . De nicion 6.1.8 El tipo de una permutacion  6= 1 de Sn es la lista s1  : : :  sk ] de las longitudes de los ciclos que aparecen en su factorizacion en ciclos disjuntos, ordenadas en forma decreciente. Por convenio, la permutacion identidad tiene tipo 1]. CAPITULO 6. GRUPOS DE PERMUTACIONES 152 Por ejemplo, el tipo de un s-ciclo es s], el de la permutacion (1 2)(3 4 5)(6 7) 2 S7 es 3 2 2], y el de la permutacion de S11 del Ejemplo 6.1.6 es 4 3 2]. Teorema 6.1.9 Dos elementos de Sn son conjugados precisamente si tienen el mismo tipo. En consecuencia, cada clase de conjugacion de Sn esta formada por todos los elementos de un mismo tipo. Demostracion. Fijemos una permutacion . Para un s-ciclo  = (i1 i2 : : : is ), se comprueba facilmente que   = (i1 i2 : : : is ) = (;1(i1 ) ;1 (i2 ) : : :;1(is )) (6.1.1) y en particular   es un s-ciclo. Tambien es facil ver que, si dos ciclos 1 y 2 son disjuntos, entonces lo son 1 y 2. Como, en general, (1 k ) = 1 k, es claro que dos elementos conjugados de Sn tienen el mismo tipo. Recprocamente, supongamos que  y 0 tienen el mismo tipo. Entonces las descomposiciones de  0 y  en producto de ciclos disjuntos son de la forma  = 1 2 k y 0 = 10 20 k0 donde i y i0 tienen la misma longitud. Por tanto existen biyecciones i : M(i0) ! M(i ) que conservan la estructura de los ciclos es decir, si i = (j1 j2 : : : js ) y i0 = (j10 j20 : : : js0 ), entonces i (jt0 ) = jt , para todo t. Ademas, como jM()j = jM(0)j, existe una biyeccion : n n M(0 ) ! n n M(). Sea ahora  2 Sn la biyeccion que se obtiene \pegando" las i y . Es decir, (x) = i (x) si x 2 M(i0 ) y (x) = (x) si x 62 M(0 ). De 6.1.1 se deduce que i0 = i, para todo i y, por tanto 0 =  . N N N Observacion 6.1.10 De la primera parte de la demostracion anterior se deduce que la factorizacion en ciclos disjuntos de  se obtiene sustituyendo, en la de , cada elemento i 2 n por ;1(i). Por ejemplo, si  = (1 4 3)(2 5 6) y  = (1 3)(2 4 7), entonces  = (3 4)(6 1 7). Ejemplo 6.1.11 Clases de conjugacion de Sn . Las 6 permutaciones de S3 se dividen en una permutacion de tipo 1] (la identidad), tres 2-ciclos o permutaciones de tipo 2] (a saber, (1 2), (1 3) y (2 3)), y dos 3-ciclos o permutaciones de tipo 3] (a saber, (1 2 3) y (1 3 2)). En S4 hay mas variedad, y en particular aparecen permutaciones que no son ciclos. Sus 24 permutaciones se dividen en los siguientes tipos: Tipo 1] 2] 3] 4] 2,2] Permutaciones 1 (1 2) (1 3) (1 4) (2 3) (2 4) (3 4) (1 2 3) (1 3 2) (1 2 4) (1 4 2) (1 3 4) (1 4 3) (2 3 4) (2 4 3) (1 2 3 4) (1 2 4 3) (1 3 2 4) (1 3 4 2) (1 4 2 3) (1 4 3 2) (1 2)(3 4) (1 3)(2 4) (1 4)(2 3) Por tanto, cada la de elementos a la derecha de la barra es una clase de conjugacion de S4 . Ademas de los ciclos, en S5 hay permutaciones de los tipos 2 2] y 3 2] y en S6 las hay de los tipos 2 2], 3 2], 2 2 2] y 3 3]. En estos casos, por el gran numero de elementos en los grupos, es pesado construir tablas como la que acabamos de dar para S4 , pero se puede al menos calcular cuantas permutaciones hay de cada tipo (vease el Problema 13). Como consecuencia del Teorema 6.1.5, sabemos que el grupo simetrico Sn esta generado por el conjunto de todos los ciclos. Vamos a terminar la seccion mostrando otros conjuntos generadores de Sn mas economicos. En particular, encontraremos uno con 2 elementos, que para n 3 es lo menos que podemos esperar puesto que Sn no es abeliano en tal caso. De nicion 6.1.12 Llamaremos trasposicion de Sn a cualquier ciclo de longitud 2. As, una trasposicion cambia exactamente dos elementos, permutandolos entre s. Por el Teorema 6.1.9, el conjunto de todas las trasposiciones es una clase de conjugacion en Sn . 6.2. GRUPOS ALTERNADOS 153 Proposicion 6.1.13 Para n > 2, los siguientes son conjuntos generadores de Sn : 1. 2. 3. 4. 5. El conjunto de todos los ciclos. El conjunto de todas las trasposiciones. El conjunto de n ; 1 trasposiciones: f(1 2) (1 3) (1 4) : : : (1 n ; 1) (1 n)g. El conjunto de n ; 1 trasposiciones: f(1 2) (2 3) (3 4) : : : (n ; 1 n)g. El conjunto de una trasposicion y un n-ciclo: f(1 2) (1 2 3 : : : n ; 1 n)g. Demostracion. 1. Es una consecuencia inmediata del Teorema 6.1.5. Para demostrar el resto de apartados bastara con comprobar que los elementos del conjunto dado en cada apartado se expresan como productos de los elementos del conjunto del apartado siguiente. 2. Cada ciclo  = (i1 i2 : : : is ) puede escribirse como producto de trasposiciones (no disjuntas):  = (i1 is )(i1 is;1 ) (i1 i3 )(i1 i2 ): 3. Es consecuencia de la igualdad (i j) = (1 i)(1 j)(1 i). 4. Dado j 2, sea  = (2 3)(3 4)(4 5) (j ; 1 j). Usando la Observacion 6.1.10 se obtiene (1 2) = (1 j). 5. Sean  = (1 2) y  = (1 2 : : : n ; 1 n). Como j ;1 lleva 1 7! j y 2 7! j +1, la Observacion 6.1.10 nos dice que j ;11;j = (j j + 1). Aunque toda permutacion de Sn se puede expresar como un producto de trasposiciones, estas expresiones no tienen las buenas propiedades que vimos en las descomposiciones en ciclos. Por una parte, no podemos esperar que una permutacion arbitraria sea producto de trasposiciones disjuntas (tendra orden 2). Por otra, tampoco se tiene conmutatividad (por ejemplo, (1 3)(1 2) 6= (1 2)(1 3)) ni unicidad, ni siquiera en el numero de factores por ejemplo (1 2 3) = (1 3)(1 2) = (2 3)(1 3) = (1 3)(2 4)(1 2)(1 4) = (2 3)(2 3)(1 3)(2 4)(1 2)(1 4): Notese que en todas estas factorizaciones de (1 2 3) hay un numero par de trasposiciones esto es consecuencia de un hecho general que analizaremos en la seccion siguiente (Proposicion 6.2.3). 6.2 Grupos alternados Z Z Fijemos un entero positivo n 2 y una permutacion  2 Sn . Por la Propiedad Universal de los Anillos de Polinomios, existe un homomorsmo de anillos % : X1 : : :  Xn] ! X1 : : :  Xn ] tal que % (Xi ) = X (i) para cada i (Ejemplos 4.7.4). Es decir, dado un polinomio Q, su imagen % (Q) se obtiene sustituyendo cada Xi por X (i) en la expresion de Q. En lo que sigue, P designara al polinomio de X1 : : :  Xn ] dado por P= Y i<j (Xj ; Xi ) = (X2 ; X1 )(X3 ; X1 ) Z (Xn ; X1 )(X3 ; X2 ) (Xn ; X2 ) (Xn ; Xn;1): La condicion i < j implica que cada diferencia entre dos indeterminadas distintas aparece, en cierto orden, exactamente una vez en esa factorizacion. Como % es un homomorsmo de anillos, se tiene % (P) = Y i<j % (Xj ; Xi ) = Y i<j (X (j ) ; X (i) ): Como  es una biyeccion, cada diferencia entre dos indeterminadas distintas sigue apareciendo, en cierto orden, exactamente una vez en esta factorizacion. Fijados i < j pueden ocurrir dos cosas: Que sea (i) < (j), en cuyo caso el factor X (j ) ; X (i) aparece en % (P) igual que en P. Que sea (i) > (j), en cuyo caso el factor X (j ) ; X (i) aparece en % (P ) en el orden contrario que en P en este caso diremos que  presenta una inversion para el par (i j). CAPITULO 6. GRUPOS DE PERMUTACIONES 154 Como cada inversion se traduce en un cambio de signo en % (P) con respecto a P , se tiene % (P) = P , donde el signo es + si y solo si el numero de pares (i j) (con i < j) para los que  presenta una inversion es par. Esto sugiere las deniciones que siguen: De nicion 6.2.1 La permutacion  2 Sn es par si %(P ) = P es decir, si  presenta un numero par de inversiones y es impar si % (P) = ;P es decir, si  presenta un numero impar de inversiones. El signo de  se dene como sg() = (;1)k , donde k es el numero de inversiones que presenta . Es decir, sg() = 1 si  es par y sg() = ;1 si  es impar. Por el comentario previo a esta denicion se tiene % (P ) = sg()P . Proposicion 6.2.2 La \aplicacion signo" sg : Sn !  = f1 ;1g es un homomorsmo de grupos. Demostracion. Sean   2 Sn . Es claro que %  % =   , y por tanto sg(  )P =   (P ) = % (% (P )) = % (sg()P ) = sg()%(P ) = sg()sg()P y por tanto sg(  ) = sg()sg(). Z Proposicion 6.2.3 En Sn se verica: 1. El signo de una permutacion  es el mismo que el de su inversa ;1 y que el de cualquiera de sus conjugadas  . 2. Toda trasposicion es impar. 3. Si  = 1 r , donde las i son trasposiciones, entonces sg() = (;1)r . 4. Una permutacion  es par (respectivamente impar) si y solo si es producto de un numero par (respectivamente impar) de trasposiciones. 5. Un ciclo de longitud s tiene signo (;1)s;1 es decir, un ciclo de longitud par es impar, y viceversa. 6. La paridad de una permutacion coincide con la del numero de componentes pares de su tipo. Demostracion. Por la Proposicion 6.2.2, sg()sg(;1 ) = 1 y sg( ;1) = sg();1sg()sg() = sg(), de donde se deduce 1. Ahora, como toda trasposicion es conjugada de (1 2) y esta es impar (presenta exactamente una inversion), el apartado 2 se sigue del 1, y entonces 3 y 4 son claros puesto que toda permutacion es producto de trasposiciones y sg es un homomorsmo. Finalmente, el apartado 3 aplicado a la igualdad (i1 i2 : : : is ) = (i1 is )(i1 is;1) (i1 i3 )(i1 i2 ) nos da el apartado 5 y 6 es una consecuencia inmediata de 5. Ejemplo 6.2.4 Calculando la paridad en funcion del tipo. Del Ejemplo 6.1.11 y del ultimo apartado de la Proposicion 6.2.3 se deduce que, ademas de la identidad, las permutaciones pares de S3 son las de tipo 3] las de S4 son las de los tipos 3] o 2 2] las de S5 son las de los tipos 3], 5] o 2 2] y las de S6 son las de los tipos 3], 5], 2 2] o 3 3]. De nicion 6.2.5 El grupo alternado en n elementos, denotado por An, es el nucleo del homomorsmo sg : Sn !  = f1 ;1g. Es decir, es el subgrupo de Sn formado por las permutaciones pares. Proposicion 6.2.6 An es un subgrupo normal de Sn, y para n 2 se tiene: Sn : An ] = 2 jAnj = n!2  y ASn = f1 ;1g = 2: n Demostracion. Al estar denido como el nucleo de un homomorsmo, An es normal en Sn . El resto es consecuencia del Primer Teorema de Isomorfa si vemos que, para n 2, el homomorsmo sg es suprayectivo, para lo que basta notar que sg(1) = 1 y sg(1 2) = ;1. Z Z Z Es elemental ver que A2 es el grupo trivial y que A3 es el subgrupo cclico de S3 generado por el 3-ciclo (1 2 3), y por tanto A3  = 3. En el caso general, tenemos dos maneras sencillas de describir conjuntos de generadores de An . 6.2. GRUPOS ALTERNADOS 155 Proposicion 6.2.7 Los siguientes son sistemas de generadores de An: 1. El conjunto de todos los productos de dos trasposiciones (disjuntas o no). 2. El conjunto de todos los 3-ciclos. Demostracion. El apartado 1 es una consecuencia inmediata del apartado 4 de la Proposicion 6.2.3. Por la misma proposicion, todos los 3-ciclos estan en An por tanto, usando 1, para ver 2 solo hay que probar que cada producto de dos trasposiciones distintas (disjuntas o no) se puede escribir como producto de 3-ciclos, lo que se sigue de las igualdades (i j)(i k) = (i k j) e (i j)(k l) = (j l k)(i k j) donde asumimos que i j k l son distintos dos a dos. Observese que, como el conjunto vaco genera el subgrupo trivial, la Proposicion 6.2.7 es valida incluso cuando n = 1 o n = 2. A continuacion describimos los subgrupos de A4 . Esto nos dara un ejemplo en el que no se verica el recproco del Teorema de Lagrange: A4 tiene orden 12, pero no tiene subgrupos de orden 6. Ejemplo 6.2.8 Subgrupos de A4. En virtud del Ejemplo 6.2.4, la siguiente es la lista completa de los elementos de A4 : 1  = (1 2)(3 4)  = (1 3)(2 4)  = (1 4)(2 3)  = (1 2 3) = (1 2 4) = (1 3 4)  = (2 3 4) 2 = (1 4 3) 2 = (1 3 2) 2 = (1 4 2)  2 = (2 4 3) Por el Teorema de Lagrange, los subgrupos propios y no triviales de A4 han de tener orden 2, 3, 4, o 6. Los de orden 2 han de estar generados por elementos de orden 2, y por tanto son: hi = f1 g h i = f1  g hi = f1 g: Como  =  62 hi, deducimos que hi no es normal en A4 , y del mismo modo se ve que no lo son h i ni hi. Los subgrupos de orden 3 han de estar generados por elementos de orden 3, y por tanto son: hi = h2 i = f1  2g h i = h 2 i = f1  2g h i = h 2 i = f1  2 g h i = h 2 i = f1   2g: Un subgrupo de orden 4 no puede contener a ninguno de los elementos de orden 3 como el resto de elementos forman un subgrupo N = f1   g este es el unico subgrupo de orden 4, que ademas es normal en Sn por el Teorema 6.1.9. Por ultimo, veamos que no hay subgrupos de orden 6. Un tal subgrupo H sera normal en A4 por tener ndice 2, por lo que tambien N \ H sera normal en A4. Ademas se tendra NH = A4 (>por que?) y en consecuencia jN \ H j = 2 (Lema 5.4.11), en contra del hecho de que ninguno de los subgrupos de orden 2 de A4 es normal. Terminamos el captulo con un resultado de notable importancia historica, pues es una de las claves que permitio demostrar la inexistencia de una formula general para calcular \por radicales" las races de polinomios de grado 5 o superior (ver la nota al pie en el Ejemplo 9.2.2). Se trata del Teorema de Abel, que arma que los grupos alternados An son simples cuando n 5. Por supuesto, necesitamos la denicion de grupo simple. De nicion 6.2.9 Un grupo no trivial G es simple si sus unicos subgrupos normales son f1g y G. Por ejemplo, todo grupo de orden primo es simple. El recproco se verica para grupos abelianos: Ejercicio 6.2.10 Demostrar que un grupo abeliano es simple precisamente si su orden es primo. CAPITULO 6. GRUPOS DE PERMUTACIONES 156 Los grupos simples no abelianos son escasos1 . De hecho, el grupo simple no abeliano de menor tama~no es A5 , que tiene 60 elementos (Teorema 8.4.11). Es decir, si G es un grupo simple nito entonces o bien jGj es primo o bien jGj 60. Observese que A3 es simple, pero A4 no lo es, como muestra el Ejemplo 6.2.8. Para demostrar el anunciado Teorema de Abel necesitamos un lema: Lema 6.2.11 Si un subgrupo normal H de An (n 5) contiene un 3-ciclo, entonces H = An. Demostracion. Sea  un 3-ciclo en H. Por la Proposicion 6.2.7, basta ver que cualquier otro 3-ciclo 0 esta en H. Sabemos por el Teorema 6.1.9 que existe  2 Sn tal que 0 =  , de modo que si  2 An entonces 0 2 H, por la normalidad de H en An en consecuencia, podemos suponer que  es una permutacion impar. Como  solo cambia 3 elementos y n 5, existe una trasposicion disjunta con , por lo que  = . Por tanto  = ( ) =  = 0 y como  esta en An por ser el producto de dos permutaciones impares, la normalidad de H en An implica que 0 2 H, como queramos ver. Teorema 6.2.12 (Abel) Si n 5, entonces An es un grupo simple. Demostracion. Supongamos que H 6= f1g es un subgrupo normal de An y veamos que H = An. Por el Lema 6.2.11, bastara probar que H contiene un 3-ciclo. Por el Axioma de Buena Ordenacion, podemos elegir un elemento 1 = 6  2 H que cambie el menor numero posible de elementos de n es decir, existen 1 = 6  2 H y r 2 + tales que  cambia exactamente r elementos, y cualquier otro 1 = 6  2 H cambia al menos r elementos. Ahora veremos N Z que debe tenerse r = 3, por lo que  sera un 3-ciclo en H y habremos terminado. Desde luego, no puede ser r = 1 porque ninguna permutacion cambia exactamente un elemento, ni tampoco r = 2 porque todas las permutaciones de H son pares. Supongamos pues, en busca de una contradiccion, que r > 3. Se tienen entonces dos posibilidades: 1. Que, en la factorizacion de  en ciclos disjuntos, aparezca alguno de longitud 3. 2. Que  sea un producto de (al menos dos) trasposiciones disjuntas. En el primer caso,  debe cambiar al menos 5 elementos (si solo cambiase 4, como al menos aparece un 3-ciclo,  sera un 4-ciclo, lo que contradice el hecho de que  2 An). Podemos suponer, sin perdida de generalidad (>por que?), que 1 2 3 4 5 2 M() y que alguno de los ciclos disjuntos que componen  es de la forma (1 2 3 : : : ) (con longitud al menos 3). Sea  = (3 4 5). Como  2 An y H es normal en An, deducimos que  2 H, y as = ;1  2 H. Si (i) = i entonces i > 5 y por tanto (i) = i, de donde se sigue que (i) = i por tanto M( )  M(), y la inclusion es estricta pues (1) = 2 mientras que (1) = 1. En consecuencia, 2 H cambia menos de r elementos, as que debe ser = 1, por la eleccion de r. Esto signica que  = , y por tanto  = . Pero esto es falso, pues (2) = 4 y (2) = 3, de manera que la primera de las dos posibilidades consideradas nos lleva a una contradiccion. Pasamos al segundo caso. Reordenando los elementos de n podemos asumir que  = (1 2)(3 4) (puede haber mas trasposiciones en el producto o no). Sea de nuevo  = (3 4 5). Como antes, tomamos = ;1  2 H. Si i 6= 5 y (i) = i entonces i 6= 3 4 5 y por tanto (i) = i, de donde se sigue que (i) = i por tanto M( )  M()  f5g. Pero el 1 y el 2 son jados por y cambiados por , de modo que cambia menos de r elementos y as = 1, o sea  = . Pero se tiene (3) = 3 6= 5 = (3). En cualquier caso, pues, llegamos a la contradiccion que buscabamos. N 1 La clasicaci on de todos los grupos simples nitos esta considerada como el mayor trabajo colectivo que se ha hecho en Matematicas. La \demostracion" de este resultado, conocido como el \Teorema Enorme", se esparce en numerosos artculos de investigacion y se estima que ocupara unas 15.000 paginas. La mayora de estos grupos se distribuyen en 4 familias innitas con alguna caracterstica comun dos de ellas son la familia de los grupos cclicos de orden primo y la de los grupos alternados An con n  5. Los grupos simples nitos que no pertenecen a ninguna de estas familias se llaman \esporadicos". Hay exactamente 26 grupos esporadicos el menor tiene orden 24  35  5  11 = 7:920, y el mayor, conocido como el \grupo monstruo", tiene orden algo mayor que 8  1053 (exactamente 246  320  59  76  112  133  17  19  23  29  31  41  47  59  71). 6.3. PROBLEMAS 157 6.3 Problemas 1. Se pide, para las siguientes permutaciones: = 1 2 3 4 4 3 2 1  = 1 2 3 4 5 6 6 5 4 3 1 2  = (4 5 6)(5 6 7)(1 2 3)(2 3 4)(3 4 5): (a) Expresarlas como producto de ciclos disjuntos. (b) Calcular sus ordenes. (c) Expresarlas como producto de las trasposiciones (1 2) (2 3) : : :  (n ; 1 n). (d) Expresarlas como producto de las trasposiciones (1 2) (1 3) : : :  (1 n ; 1) (1 n). (e) Expresarlas como producto de las permutaciones (1 2) y (1 2 n ; 1 n). (f) Calcular su paridad. (g) Expresar las que sean pares como producto de ciclos de longitud 3. 2. Calcular ;1 , donde (a)  = (1 3 5)(1 2),  = (1 5 7 9). (b)  = (5 7 9),  = (1 2 3). 3. Determinar la paridad de las siguientes permutaciones: (a) (1 2 3)(1 2). (b) (1 2 3 4 5)(1 2 3)(4 5).   (c) 71 52 36 44 51 63 72 . 4. Calcular 1000,  1 2 3 4 5 6 7 8 9 10 11  donde  = 3 5 2 6 1 7 4 0 9 11 8 . 5. Se pide: (a) Resolver la ecuacion x(1 2)(3 4)x;1 = (5 6)(1 3) en S6 . (b) Probar que la ecuacion x(1 2 3)x;1 = (1 3)(5 7 8) no tiene solucion en S8 . (c) Encontrar, si existen, las soluciones en S5 de la ecuacion x(1 2)x;1 = (3 4)(1 5). 6. Encuentra los 13 pares para los que la permutacion  = (2 7 8)(1 4) de S8 presenta inversiones.   7. Dada la permutacion  = 15 21 39 48 52 63 74 86 97 , calcular el orden de 2. 8. Probar que si  2 Sn es un d-ciclo y (x) 6= x entonces  = (x (x) d;1 (x)). 9. Sea  6= 1 una permutacion de Sn entonces: (a)  es un ciclo si y solo si, para cualesquiera j k 2 M(), existe un entero m tal que m (j) = k. (b) Si  es producto de dos o mas ciclos disjuntos, entonces  no es un ciclo. 10. Probar que para toda permutacion  2 Sn se cumple (i1 ir );1 = ((i1 ) (ir )). 11. Demostrar que una permutacion tiene orden primo p si y solo si se factoriza como un producto de ciclos disjuntos, cada uno de longitud p. ; 12. Demostrar que para todo 1 k < n, Sn tiene al menos nk subgrupos isomorfos a Sk  Sn;k y que todos son conjugados es decir, para dos de estos grupos H y K existe  2 G tal que H = K. CAPITULO 6. GRUPOS DE PERMUTACIONES 158 N 13. Dados dos numeros naturales n y k con n k 2, se pide: (a) Demostrar que, para cada subconjunto A de n de cardinal k, el numero de k-ciclos  de Sn con M() = A es (k ; 1)!. ; (b) Demostrar que el numero de k-ciclos en Sn es nk (k ; 1)!. (c) >Cuantos elementos de tipo 2 2] hay en S5 ? >Cuantos de tipo 2 3]? (d) >Cuantos elementos de tipo 2 2] hay en S6 ? >Cuantos de tipo 2 3]? >Y de tipo 3 3]? (e) *] Calcular en general el numero de elementos de Sn de tipo k1 : : :  kr ]. 14. Sea G un grupo nito de orden n, y sea g 2 G de orden m. Como en la demostracion del Teorema de Cayley (5.7.4), se dene g : G ! G por g (x) = gx. Viendo a g como un elemento de Sn , demostrar que: (a) g es un producto de n=m ciclos de longitud m. (b) La paridad de g coincide con la paridad del entero (m ; 1) mn . (c) Si (m ; 1) mn es impar, entonces G tiene un subgrupo normal de ndice 2. 15. Demostrar que el centralizador de la permutacion  = (1 2 : : :  n) en Sn es hi. 16. Demostrar que el grupo alternado An es un subgrupo caracterstico del grupo simetrico Sn . 17. Sea n 2 y sea f : Sn ! Sn+2 la aplicacion dada por f() =  , donde  actua igual que  sobre los elementos 1 2 : : :  n, y  ja (respectivamente, intercambia) n + 1 y n + 2 cuando  es par (respectivamente, impar). Demostrar que f es un homomorsmo inyectivo de grupos y que su imagen esta contenida en An+2 . Deducir que todo grupo nito es isomorfo a un subgrupo de un grupo alternado. 18. Probar que si P es un subgrupo de orden 4 del grupo alternado A5, entonces P es isomorfo al grupo de Klein 2  2. 19. Demostrar que Dn es isomorfo al subgrupo h i de Sn , donde  = (1 2 : : : n ; 1 n) y  es el producto de las trasposiciones (i n + 1 ; i), donde i vara desde 1 hasta la parte entera de n=2. >Para que valores de n se tiene h i  An ? 20. Probar que si todo  2 Sn es producto de ciclos disjuntos de orden 2 entonces n 2. 21. Probar que para todo H Sn  con n > 2, se tiene, o bien H An o bien H : H \ An ] = 2: 22. Dado f 2 Aut(S3 ), probar que f induce una permutacion del conjunto X = f(1 2) (1 3) (2 3)g  S3 . Deducir que la aplicacion t : S3 ! Aut(S3 ) que lleva  2 S3 al automorsmo interno t (ver el Ejercicio 5.9.4) es un isomorsmo de grupos. 23. Demostrar que An esta generado por los 3-ciclos de la forma (1 2 i) con i = 3 : : :  n. 24. Para n 5, demostrar que Sn tiene exactamente tres subgrupos normales. 25. Para n 2, demostrar que An es el unico subgrupo de ndice dos de Sn . 26. *] Sea K un cuerpo. Un polinomio P 2 KX1 : : :  Xn] se dice que es simetrico si, para todo  2 Sn , se tiene % (P) = P (con la notacion de la Seccion 6.2). Se llaman polinomios simetricos elementales a los siguientes polinomios simetricos: ZZ 1 2 3 .. . n = = = Pn Pi=1 Xi X X P1i<jn iX jX X 1i<j<kn i j k = X1 X2 Xn : 6.3. PROBLEMAS 159 (a) Demostrar que, si r1  : : :  rn son las races en K del polinomio P = X n + a1 X n;1 + an;1X + an, entonces se tiene + ai = (;1)i i(r1  : : :  rn) para cada i = 1 : : :  n. Esto nos da los coecientes de un polinomio en funcion de sus races. (b) Demostrar que el conjunto de los polinomios simetricos de KX1  : : :  Xn ] coincide con el subanillo K1 : : :  n] generado por K y los polinomios simetricos elementales. (Indicacion: Hacer una doble induccion primero en el numero de indeterminadas y segundo en el grado del polinomio simetrico P que se quiere demostrar que pertenece a K1 : : :  n].) (c) Expresar los siguientes polinomios simetricos como polinomios en los polinomios simetricos elementales: (X1 ; X2 )2 (X1 ; X3 )2(X2 ; X3 )2  X13 + X23 + X33 : (d) Demostrar que el homomorsmo de sustitucion S : KX1  : : :  Xn] ! KX1  : : :  Xn] dado por S(P ) = P(1 : : :  n) es inyectivo y deducir que KX1 : : :  Xn ] es isomorfo al anillo de los polinomios simetricos elementales. 27. *] Sea p un primo impar y sea H un subgrupo propio de Sp que contiene una trasposicion. Demostrar que existen i j 2 p tales que (i) 6= j para todo  2 H. (Indicacion: Considerar en p la relacion de equivalencia en la que i  j si i = j o si (i j) 2 H, y comparar el numero de elementos de las clases de equivalencia.) 28. *] Sea S1 = S( ) el grupo de permutaciones del conjunto numerable . El grupo alternado innito es el subgrupo A1 de S1 generado por todos los 3-ciclos (donde un 3-ciclo se dene del modo obvio). Demostrar que A1 es un grupo simple innito. N N Bibliografa del captulo N Allenby 1], Delgado-Fuertes-Xambo 11], Lang 24], Rotman 30]. N 160 CAPITULO 6. GRUPOS DE PERMUTACIONES Captulo 7 Grupos abelianos nitamente generados Se estudian los grupos abelianos nitamente generados, dando una descripcion precisa de su estructura y asignandoles unas listas de invariantes numericos que los determinan salvo isomorsmos. Introduccion En este captulo y en el siguiente, tratamos el problema de clasicar grupos nitos en funcion de su orden. El objetivo es obtener, para cada entero positivo n, la descripcion de todos los grupos de orden n. Por ejemplo, obtendremos resultados como este: \Salvo isomorsmos, hay cinco grupos de orden 8, que son 8, 4  2, 2  2  2, D4 y Q8". Esto signica que entre esos grupos de orden 8 no hay dos isomorfos, y que cualquier grupo de orden 8 es isomorfo a uno (y solo uno) de ellos. El problema es tremendamente complicado, y de hecho no esta resuelto en general. En situaciones como esta, lo usual es abordar primero el problema con alguna hipotesis extra que lo haga mas sencillo, y tratar entonces de extraer consecuencias para el caso general. En este captulo clasicaremos los grupos abelianos nitos (mas generalmente, los nitamente generados), y algunas de las ideas que usaremos nos serviran mas tarde para obtener ciertos resultados de clasicacion en el caso no abeliano (clasicaremos los grupos de orden n para n 15 y para ciertos valores de n con una factorizacion prima sencilla). Hay un modo elemental de fabricar grupos abelianos nitamente generados: hacer productos nitos de grupos cclicos. Vamos a demostrar que, salvo isomorsmos, estos son todos los grupos abelianos nitamente generados que existen, lo que nos da una descripcion muy precisa de esos grupos. Mas aun, de forma similar (aunque mas sosticada) al modo en el que la dimension de un espacio vectorial lo determina salvo isomorsmos, ciertos invariantes numericos (llamados as porque permanecen invariantes por isomorsmos) asociados a cada grupo abeliano nitamente generado caracterizan su clase de isomorfa, en el sentido de que dos de tales grupos son isomorfos si y solo si los invariantes de ambos coinciden. En todo el captulo usamos notacion aditiva. Comenzamos estudiando el concepto de suma directa de subgrupos, que permite establecer isomorsmos entre un grupo abeliano y el producto directo de algunos de sus subgrupos. Despues analizamos los grupos abelianos libres, que interpretamos como grupos que cumplen ciertas propiedades analogas a las de los espacios vectoriales, y vemos que se les puede asignar un \rango" que representa en este contexto el papel que tiene la dimension en el caso vectorial. Denimos entonces el subgrupo de torsion de un grupo, y demostramos que todo grupo abeliano nitamente generado A es suma directa de su subgrupo de torsion t(A), que es nito, y de un subgrupo libre isomorfo al cociente A=t(A). Esta \parte libre" es isomorfa a un producto de r copias de , donde r es el rango de A=t(A), por lo que el problema queda reducido al estudio de grupos abelianos nitos. Es facil ver que tales grupos son sumas directas de subgrupos \indescomponibles", por lo que basta ver que todo grupo nito e indescomponible B es cclico. Para ello, considerando las p-componentes de un grupo (para cada entero positivo primo p), vemos en un primer paso que el orden de B es una potencia de un primo jo p, y usando esto obtenemos nalmente el resultado deseado. ZZ ZZ Z Z Z 161 162 CAPITULO 7. GRUPOS ABELIANOS FINITAMENTE GENERADOS Habremos demostrado as que la parte de torsion de A es producto de grupos cclicos cuyo orden es una potencia de primo. Viendo entonces que esta expresion es esencialmente unica (salvo el orden y salvo isomorsmos), la lista formada por el rango de A=t(A) junto con los ordenes (potencias de primo) de esos grupos cclicos sera una lista de invariantes que determina la clase de isomorfa de A (es decir, la clase formada por los grupos isomorfos a A). Sera entonces facil ver que hay una forma alternativa de descomponer A como producto de cclicos, en la que el orden de cada uno de ellos divide al del siguiente. El captulo naliza con un metodo matricial para obtener estas listas a partir de una presentacion del grupo \por generadores y relaciones". Objetivos del captulo Conocer el concepto de suma directa de subgrupos y su relacion con el producto de grupos. Conocer los conceptos de conjunto linealmente independiente, base, grupo abeliano libre y rango, as como la Propiedad Universal de las Bases. Saber calcular bases y rangos de grupos abelianos libres. Saber determinar la parte de torsion y las p-componentes de un grupo abeliano, y saber descomponer estas como suma directa de subgrupos cclicos. Saber utilizar el Teorema Chino de los Restos para pasar de una descomposicion invariante a una descomposicion indescomponible, y viceversa. Saber usar la notacion matricial para obtener los factores invariantes de un grupo abeliano nitamente generado a partir de una presentacion por generadores y relaciones. Desarrollo de los contenidos 7.1 Sumas directas Un modo habitual de estudiar un objeto matematico consiste en descomponerlo en objetos mas sencillos, estudiar estos y recomponer entonces el objeto inicial. Lo que se entiende por objeto sencillo y la manera de descomponer y recomponer un objeto dependen de cada caso. En este captulo el objeto estudiado sera un grupo abeliano nitamente generado A, y los objetos sencillos seran los grupos cclicos, que ya conocemos bien. En este contexto, el proyecto sugerido al principio del parrafo funciona porque existe un metodo muy efectivo para descomponer A de modo que es muy facil conocer A a partir de sus componentes. Se trata de la suma directa de subgrupos, que analizamos en esta seccion. Proposicion 7.1.1 Sean fB1 : : :  Bn g subgrupos de un grupo abeliano A. Entonces las condiciones siguientes son equivalentes: 1. El 0 se expresa de manera unica como suma de elementos de los Bi . Es decir, si b1 + + bn = 0 con cada bi 2 Bi , entonces se tiene bi = 0 para cada i = 1 : : :  n. 2. Cada elemento de B1 + + Bn se expresa de manera unica como suma de elementos de los Bi . Es decir, si b1 + + bn = b01 + + b0n con cada bi 2 Bi y cada b0i 2 Bi , entonces se tiene bi = b0i para cada i = 1 : : :  n. P 3. Para cada j = 1 : : :  n se verica Bj \ ( i6=j Bi ) = 0. Demostracion. La equivalencia entre las dos primeras condiciones se deduce de un argumento tpico que el lector conocera del algebra lineal, y se deja como ejercicio. Veamos pues que las condiciones 1 y 3 son equivalentes. P Si se verica 1 y x 2 Bj \ ( i6=j Bi ), entonces x 2 Bj y existen b1  : : :  bj ;1 bj +1 : : :  bn tales que x = b1 + + bj ;1 + bj +1 + + bn . Haciendo bj = ;x se tiene b1 + + bn = 0, luego bj = 0 y as x = 0. Recprocamente, si se verica 3 y se tiene b1 + + bn = 0 con P cada bi 2 Bi entonces, para cada ndice j, se tiene bj = ;(b1 + : : : + bj ;1 + bj +1 + : : : + bn) 2 Bj \ ( i6=j Bi ) = 0, luego bj = 0. 7.1. SUMAS DIRECTAS 163 La Proposicion 7.1.1 se puede generalizar a una familia innita de subgrupos. Explcitamente: Ejercicio 7.1.2 Sea fBi : i 2 I g una familia de subgrupos de un grupo abeliano A. Entonces las condiciones siguientes son equivalentes: P 1. El 0 se expresa de manera unica como suma de elementos de los Bi . Es decir, si i2I bi = 0 con cada bi 2 Bi y bi = 0, para casi todo i 2 I , entonces se tiene bi = 0 para cada i 2 I . P 2. Cada elemento P de Pi2I Bi se expresa de manera unica como suma de elementos de los Bi. Es decir, si i2I bi = i2I b0i con cada bi b0i 2 Bi , bi = 0 para casi todo i 2 I y b0i = 0 para casi todo i 2 I , entonces se tiene bi = b0i para cada i 2 I . P 3. Para cada j 2 I se verica Bj \ ( i6=j Bi ) = 0. De nicion 7.1.3 Si se verican las condiciones equivalentes de la Proposicion 7.1.1 (o del Ejercicio 7.1.2 en el caso innito), se dice que la familia P de subgrupos fB1 : : : Bng es independiente, o que los subgrupos Bi son independientes. Su suma, ni=1 Bi = B1 + + Bn , se llama entonces la suma directa de la familia fB1  : : : Bn g, y se denota por ni=1 Bi = B1   Bn (o por i2I Bi en el caso innito). La expresion \Sea A = B1   Bn " quiere decir que los Bi son subgrupos independientes del grupo abeliano A y que su suma vale A. Un subgrupo B de A es un sumando directo de A si existe otro subgrupo C de A tal que A = B  C es decir, tal que A = B + C y B \ C = 0. En este caso se dice que C es un complemento directo de B . Z R R R R C C R C 3 Ejemplos 7.1.4 Subgrupos independientes y sumas directas. 1. En el grupo A = 6 los subgrupos B = h2i y C = h3i son independientes y se tiene A = B  C. 2. En el grupo multiplicativo  se tiene  = h;1i  +. 3. En el grupo multiplicativo  los subgrupos + y U = fu 2 : juj = 1g son independientes y se tiene  = +  U. Eso es precisamente la expresion en forma polar de un numero complejo. 3 z = ru u 1 -r 4. Si A y B son grupos abelianos, entonces el grupo producto A  B es la suma directa de los subgrupos A  0 y 0  B. 5. El complemento directo de un sumando directo no es, en general, unico. Por ejemplo, para cualquier a 2 se tiene  = h(1 0)i  h(a 1)i: la interseccion es claramente nula, y un elemento arbitrario (x y) de  se puede expresar como (x y) = y(a 1) + (x ; ya)(1 0). 6. En no hay dos subgrupos no triviales que sean independientes. En efecto, si A y B son subgrupos no nulos y elegimos elementos no nulos na 2 A y mb 2 B, entonces 0 = bn na ; am mb nos da una expresion no trivial del 0 como suma de elementos de A y B. En ocurre lo mismo, por un argumento similar. Z ZZZZ Q Z Ejercicio 7.1.5 Demostrar las siguientes armaciones sobre un grupo abeliano A. 1. Toda subfamilia de una familia independiente de subgrupos de A es independiente. Es decir, si fBi : i 2 I g es una familia independiente de subgrupos de A y J es un subconjunto del conjunto de ndices I , entonces la familia fBi : i 2 J g es independiente. 2. Una familia de subgrupos es independiente precisamente si toda subfamilia nita suya lo es. 164 CAPITULO 7. GRUPOS ABELIANOS FINITAMENTE GENERADOS 3. Si la familia fBi : i 2 I g de subgrupos de un grupo abeliano A es independiente y otro subgrupo B0 de A verica B0 \ (i2I Bi ) = 0, entonces la familia fB0 gfBi : i 2 I g tambien es independiente. En particular, si a una familia independiente le a~nadimos el subgrupo trivial 0 (una o mas veces), seguimos teniendo una familia independiente. 4. Si A = i2I Bi entonces cada Bj es un sumando directo de A con complemento i6=j Bi . 5. Si A = B  C y, a su vez, B = B1  Bn  C1   Cm .  Bn y C = C1   Cm , entonces A = B1   6. Si A = B  C entonces la aplicacion A ! C dada por b+c 7! c (donde b +c es la expresion unica de un elemento arbitrario de A con b 2 B y c 2 C ) es un homomorsmo suprayectivo de grupos con nucleo B . En particular, C  = A=B . 7. Si B es un sumando directo de A, cualquier complemento directo suyo es isomorfo a A=B . Por tanto, aunque un sumando directo puede tener distintos complementos directos, todos ellos son isomorfos entre s. Cuando solo consideramos familias nitas, existe una estrecha relacion entre los conceptos de suma directa y producto directo de grupos, que describimos a continuacion dejando los detalles a cargo del lector. Supongamos primero que A = B1   Bn . Entonces, viendo cada Bi como grupo y considerando su producto B1   Bn , la aplicacion B1   Bn ! A dada por (b1 : : :  bn) 7! b1 + + bn es un isomorsmo de grupos. Es decir, si A es la suma directa de los Bi , entonces A es isomorfo al producto directo de los Bi . Recprocamente, sean B1  : : :  Bn grupos abelianos y sea A el grupo producto, A = B1   Bn . Si denotamos por B^i al subgrupo de A formado por los elementos que llevan ceros en todas las coordenadas excepto tal vez en la i-esima (o sea B^i = 0   0  Bi  0   0), entonces es elemental ver que cada B^i es isomorfo a Bi y que A = B^1   B^n . Es decir, si A es el producto directo de los Bi , entonces A es la suma directa de los B^i , que son isomorfos a los Bi . En vista de esto, a partir de ahora identicaremos B1   Bn con B1   Bn . Ejercicio 7.1.6 Extender la discusion anterior al caso de una familia innita independiente de subgrupos, sustituyendo el producto directo por el grupo del ultimo apartado de los Ejemplos 5.3.4. 7.2 Grupos abelianos libres De nicion 7.2.1 Sea A un grupo abeliano. Un subconjunto nito fa1 : : :  ang de A se dice que es linealmente independiente si la unica solucion, formada por numeros enteros x1  : : : xn, de la ecuacion n X i=1 xiai = 0 es x1 = = xn = 0. Un subconjunto de A se dice que es linealmente independiente si todo subconjunto nito suyo es linealmente independiente. Una base de A es un sistema generador de A que es linealmente independiente. Diremos que A es un grupo abeliano libre si tiene una base. Ejercicio 7.2.2 Demostrar que un subconjunto X de un grupo abeliano A es una base precisamente si cada elemento de A se puede expresar de forma unica como combinacion lineal con coecientes enteros de los elementos de X . Es decir, P si para cada a 2 A existe una unica familia fax : x 2 X g de enteros, casi todos nulos, tal que a = x2X ax x. 7.2. GRUPOS ABELIANOS LIBRES 165 Ejemplos 7.2.3 Grupos abelianos libres. Z 1. Sea n un numero natural y sea A = n. Entonces el conjunto Z e1 = (1 0 : : :  0) e2 = (0 1 : : :  0) : : : en = (0 0 : : :  1) es una base de A, llamada base canonica. 2. En A = , el conjunto f2 3g es un sistema generador minimal (en el sentido de que, si quitamos algun elemento, deja de ser sistema generador) que no es linealmente independiente. Por otra parte, el conjunto f2g es un conjunto linealmente independiente maximal (en el sentido de que, en cuanto le a~nadamos un elemento, dejara de ser independiente) que no es generador. Esto no ocurre con los espacios vectoriales: en un espacio vectorial, todo sistema generador minimal es una base (en el sentido de algebra lineal) y todo conjunto linealmente independiente maximal es una base. Z Z Z 3. Sea I un conjunto y consideremos el grupo A = I = f(ai )i2I : ai 2 g con la suma componente a componente (a este grupo se le suele llamar el producto directo de jI j copias de ). Para cada i 2 I, sea ei = (ij )j 2I , donde ij = 0 si i 6= j y ii = 1 (este smbolo se conoce como la delta de Kronecker). Entonces E = fei : i 2 I g es un conjunto linealmente independiente de A. Sin embargo, E solo es una base si I es nito. De hecho, el subgrupo generado por E es Z Z (I) = f(a i )i2I 2 Z I : ai = 0 para casi todo i 2 I g: Luego (I ) es un grupo libre y E es una base suya llamada base canonica. Z Q 4. Sea P el conjunto de los enteros primos positivos. Entonces la aplicacion f : (P ) ! es un isomorsmo. Luego Q f((np )p2P ) = + Y p2P pnp + dada por es libre con base P . Ejercicio 7.2.4 Demostrar que una familia fai : i 2 I g de elementos de un grupo A es linealmente independiente si y solo si cada ai tiene orden innito y la familia fhai i : i 2 I g de los subgrupos cclicos generados por los ai es independiente. En realidad, el apartado 3 del Ejemplo 7.2.3 agota, salvo isomorsmos, todos los posibles ejemplos de grupos libres, como muestra el siguiente ejercicio. Ejercicio 7.2.5 SeaPX un subconjunto de un grupo abeliano A, y sea f : por f((ax )x2X ) = Demostrar: Z (X ) ! A la aplicacion dada x2X ax x (las sumas tienen sentido porque casi todos los sumandos son nulos). 1. f es un homomorsmo de grupos. 2. f es inyectiva precisamente si X es linealmente independiente. 3. f es suprayectiva precisamente si X es un conjunto generador de A. Z Z 4. X es una base precisamente si f es un isomorsmo. En este caso A  = (X ). 5. A es libre precisamente si A es isomorfo a (I) para cierto conjunto I . Los conceptos de linealmente independiente, generador y base nos recuerdan a los correspondientes de algebra lineal. En el siguiente ejercicio vemos algunas relaciones entre nuestro concepto y el de algebra lineal. CAPITULO 7. GRUPOS ABELIANOS FINITAMENTE GENERADOS 166 Q Ejercicio 7.2.6 Sea I un conjunto. Consideremos A = Z Q (I ) como un grupo abeliano libre y V = (I ) como un espacio vectorial sobre . Demostrar las siguientes propiedades para un subconjunto S de A: 1. S es linealmente independiente como subconjunto del grupo A precisamente si S es linealmente independiente como subconjunto del espacio vectorial V . 2. Si S es un sistema generador del grupo A, entonces S es un sistema generador del espacio vectorial V. 3. Dar un ejemplo de un subconjunto de A que sea generador de V pero que no sea generador de A. 4. Demostrar que todas las bases de A tienen el mismo cardinal, a saber jI j. Deducir que si J es otro conjunto, entonces (I )  = (J ) precisamente si jI j = jJ j. Z Z De nicion 7.2.7 El cardinal de una base (cualquier base) de un grupo abeliano libre A se llama rango de A y se denota r(A). De los Ejercicios 7.2.5 y 7.2.6 se deduce que el rango es un invariante que caracteriza los grupos abelianos libres salvo isomorsmos es decir: Proposicion 7.2.8 Si A y B son grupos abelianos libres, entonces A = B si y solo si r(A) = r(B). Los grupos abelianos libres que mas nos interesan son los nitamente generados. Obviamente, los grupos de la forma n son grupos abelianos libres nitamente generados. De hecho no hay mas, salvo isomorsmos, ya que si fa1 = (a1i )i2I  : : :  an = (ani)i2I g es un conjunto generador de A = (I ), entonces F = fi 2 I : aki 6= 0 para algun k = 1 : : :  ng es un subconjunto nito de I.PVamos a ver que I = F . Si i 2 I n F , entonces la coordenada i-esima de todo elemento de la forma nk=1 mk ak es 0. Como en (I ) hay elementos cuya coordenada i-esima no es 0 (<encuentra uno!), eso nos lleva a una contradiccion, de donde deducimos que I = F y, por tanto I es nito. Junto con la Proposicion 7.2.8, esto demuestra que: Corolario 7.2.9 Todo grupo abeliano libre nitamente generado A es isomorfo a n, donde n = r(A). Un grupo abeliano libre nitamente generado tambien se dice que es un grupo abeliano libre de tipo nito o de rango nito. Como consecuencia del Ejercicio 7.2.5 se deduce: Proposicion 7.2.10 Sea A un grupo abeliano. 1. A es isomorfo a un cociente de un grupo abeliano libre. 2. A es nitamente generado precisamente si es isomorfo a un cociente de un grupo abeliano libre Z Z Z Z Z de tipo nito. 3. A es cclico precisamente si es isomorfo a un cociente de . ZZ Demostracion. Sea A un grupo abeliano, y sea I un conjunto generador de A (>existe siempre?). Entonces la aplicacion f : (I ) ! A del Ejercicio 7.2.5 es un epimorsmo. Por el Primer Teorema de Isomorfa se tiene A  = (I )=Ker f, lo que demuestra la primera armacion y la condicion necesaria de las otras dos armaciones. Las condiciones sucientes son evidentes. ZZZ Del isomorsmo evidente n  m  = n+m se deduce: Proposicion 7.2.11 Si A y B son grupos abelianos libres de tipo nito entonces A  B es tambien libre, y se tiene r(A  B) = r(A) + r(B). El lector puede probar que el producto directo de un numero nito de grupos libres (no necesariamente de tipo nito) es libre. No es cierto que el producto directo innito de grupos libres sea libre. Por ejemplo, Nno es libre, pero la demostracion de este hecho excede los objetivos del curso. Las bases de los grupos abelianos libres verican una propiedad analoga a las bases de espacios vectoriales, en el sentido de que podemos describir homomorsmos que salgan de un grupo abeliano libre eligiendo arbitrariamente (en el grupo imagen) las imagenes de los elementos de la base. Explcitamente: Z 7.2. GRUPOS ABELIANOS LIBRES 167 Proposicion 7.2.12 (Propiedad Universal de las Bases) Sea A un grupo abeliano libre y sea I una base de A. Si B es un grupo abeliano y f : I ! B es una aplicacion, entonces existe un unico % = f(i) cuando i 2 I ). homomorsmo de grupos f% : A ! B que extiende f (es decir, tal que f(i) Observese que, si u : I ! A es la inclusion, entonces el homomorsmo f% completa el siguiente diagrama I u f ? qK 1 Z f A Demostracion. Por el Ejercicio 7.2.5 existe un isomorsmo g : A ! (I) tal que g(i) = ei P , donde fei : % i )i2I ) = i2I ai f(i). i 2 I g es la base canonica de (I ). Sea f% : (I ) ! B la aplicacion dada por f((a Por el Ejercicio 7.2.5, f% es un homomorsmo de grupos y la composicion g  f% : A ! B satisface las condiciones requeridas. La unicidad es consecuencia del hecho obvio de que dos homomorsmos que toman los mismos valores en un conjunto generador son iguales. Z Z Lema 7.2.13 Un subgrupo B de un grupo A es un sumando directo de A si y solo si existe un homomorsmo  : A ! B que es la identidad en B es decir, tal que (b) = b para cada b 2 B . Si tal  existe, entonces A = B  Ker . Demostracion. La condicion necesaria es consecuencia del Ejercicio 7.1.5. Sea  : A ! B un homomorsmo suprayectivo que es la identidad en B. Entonces B \ Ker  = 0 y, si a 2 A, entonces (a ; (a)) = (a) ; (a) = 0 y a = (a) + a ; (a) 2 B + Ker : Luego A = B  Ker . Corolario 7.2.14 Sea f : A ! L un homomorsmo suprayectivo de grupos abelianos. Si L es libre, entonces existe un subgrupo B de A isomorfo a L tal que A = B  Ker f . Demostracion. Sea I una base de L y, para cada i 2 I, sea ai 2 A tal que f(ai ) = i. Por la Proposicion 7.2.12, existe un unico homomorsmo de grupos g : L ! A tal que g(i) = ai , para todo i 2 I. Entonces f  g = 1L (>por que?) por tanto g es inyectiva y as B = Im g  = L. Entonces, la composicion p = g  f : A ! B es la identidad sobre B, pues un elemento b 2 B es de la forma b = g(x) con x 2 L y entonces p(b) = p(g(x)) = g(f(g(x))) = g(1L (x)) = g(x) = b: Ahora el resultado es una consecuencia inmediata del Lema 7.2.13. Teorema 7.2.15 Sea A un grupo abeliano libre de tipo nito y sea B un subgrupo de A. Entonces B es libre de tipo nito y r(B) r(A). Z Demostracion. Sea n = r(A). Por el Corolario 7.2.9, podemos suponer que A = n. Razonamos por induccion sobre n, con el caso n = 1 resuelto por el Corolario 5.8.8. Supongamos pues que n > 1 y que se verica el teorema para grupos abelianos libres de rango menor que n. Sea e1  : : :  en la base canonica de A. Sean A1 = he1  : : :  en;1i y B1 = B \ A1 . Obviamente, A1 es libre de rango n ; 1, y por la hipotesis de induccion B1 es libre de rango n ; 1. Sea f : A ! el homomorsmo dado por f(x1  : : :  xn) = xn, sea C = f(B) y sea g : B ! C la restriccion de f a B. Entonces C es un grupo abeliano libre de rango 1 y Ker g = B1 . Del Corolario 7.2.14 se deduce que B = B1  C1, donde C1 es un subgrupo de B isomorfo a C. Aplicando la Proposicion 7.2.11 se deduce que B es libre de rango menor o igual que n. Z CAPITULO 7. GRUPOS ABELIANOS FINITAMENTE GENERADOS 168 7.3 Grupos de torsion y libres de torsion De nicion 7.3.1 Sea A un grupo abeliano. Z El subgrupo de torsion de A es el conjunto t(A) formado por los elementos de orden nito de A: Z t(A) = fa 2 A : existe 0 6= n 2 tal que na = 0g: Se dice que A es un grupo de torsion si t(A) = A. Es decir, si para cada a 2 A existe 0 6= n 2 tal que na = 0. Se dice que A es un grupo libre de torsion si t(A) = 0. Es decir, si para cada 0 6= a 2 A, el unico n 2 tal que na = 0 es n = 0 (lo que equivale a que el conjunto fag sea linealmente independiente). Si un entero n verica na = 0 para todo a 2 A, escribiremos nA = 0. Si existe algun n 1 con nA = 0, llamamos periodo de A al menor entero positivo con esa propiedad. Si no existe tal entero positivo, decimos que A tiene periodo 0. Denotaremos con p(A) al periodo de A. Dejamos que el lector compruebe algunas propiedades elementales de los conceptos recien denidos, y en particular las relaciones entre ellos. Ejercicio 7.3.2 Si A es un grupo abeliano, demostrar que: 1. El conjunto t(A) es un subgrupo de A que es de torsion, y el grupo cociente A=t(A) es libre de Z 2. 3. 4. 5. 6. 7. 8. 9. 10. 11. 12. torsion. Si A es nito entonces p(A) 6= 0. Si p(A) 6= 0 entonces A es de torsion. Si A es libre entonces A es libre de torsion. Si A es libre de torsion y no trivial, entonces p(A) = 0. Si A es cclico y p(A) = n entonces A  = n (incluidos los casos n = 0 y n = 1). Si A es de torsion y B es un subgrupo de A entonces B y A=B tambien son de torsion. Si A es libre de torsion entonces cualquier subgrupo B es tambien libre de torsion >lo es A=B ? Si A = B  C entonces t(A) = t(B)  t(C). Si p(A) = n 6= 0 y m 2 entonces ma = 0 para cada a 2 A si y solo si n j m. Si A = B1   Bn entonces p(A) = mcm(p(B1 ) : : :  p(Bn )). Si A es de torsion y fai : i 2 I g es un sistema generador entonces p(A) = mcmfo(ai) : i 2 I g. Z Z ZZ QZ Z Z Ejemplos 7.3.3 Torsiones y periodos. 1. El grupo A = 2  2  (producto numerable de copias de 2) tiene periodo 2. Esto nos da 2. 3. 4. 5. 6. ZZ un contraejemplo para el recproco del apartado 2. El grupo = es de torsion, pues cada ab + es anulado por b 6= 0. Ademas este grupo tiene periodo 0, pues dado n 6= 0 en se tiene n( p1 + ) 6= 0, donde p es cualquier primo que no divide a n. Esto nos da un contraejemplo para los recprocos de los apartados 3 y 5. Existen grupos abelianos libres de torsion que no son libres por ejemplo (cualquier subconjunto linealmente independiente tiene un solo elemento, y por tanto no es un sistema generador). Esto nos da un contraejemplo para el recproco del apartado 4. Existen grupos abelianos que no son de torsion ni libres de torsion por ejemplo, ,  o  . Si A =  n, con n > 0, entonces t(A) = 0  n. Q Si A = p p, donde p recorre el conjunto de los enteros positivos primos, entonces t(A) = p p. Este subgrupo t(A) es otro ejemplo de grupo de torsion con periodo 0. Z ZZ Z Q QR C Z  Y LIBRES DE TORSION  7.3. GRUPOS DE TORSION C 169 Z 7. Sea z 2  . Si r = jz j entonces se tiene z = rei = r(cos  + isen ), donde  es el argumento de z. Entonces z n = rneni, con lo que z n = 1 precisamente si r = 1 y n = 2k para algun k 2 . Por tanto t(C  ) = fe2qi : q 2  0 q < 1g: O sea, t(C ) esta formado por los vertices de los polgonos regulares centrados en el origen con un vertice en el punto 1 (el lector puede representar gracamente, por ejemplo, todos los elementos de orden 10.) Observese que la aplicacion f : !  dada por f(q) = e2qi es un homomorsmo de grupos tal que t(C  ) = Im f y Ker f = , con lo que t(C )  = = . Los siguientes tres resultados nos dicen que, para grupos abelianos nitamente generados, todo lo relativo a la torsion se simplica: Ser de torsion equivale a ser nito, ser libre de torsion equivale a ser libre y el subgrupo de torsion es un sumando directo1 . Q ZQ C QZ Proposicion 7.3.4 Las condiciones siguientes son equivalentes para un grupo abeliano nitamente generado A: 1. A es nito. 2. p(A) 6= 0. 3. A es de torsion. Demostracion. Por el Ejercicio 7.3.2, basta ver que 3 implica 1. Supongamos pues que A es de torsion, con un sistema generador fa1 : : :  ak g, y sea ni = o(ai ) < 1. Obviamente, la familia de los elementos de A de la forma r1a1 + + rk ak con 0 ri < ni para cada i = 1 : : :  k, es nita (tal vez incluso se repitan elementos), y las condiciones implican que cada elemento de A es uno de esos, luego A es un conjunto nito. Teorema 7.3.5 Un grupo abeliano nitamente generado es libre de torsion precisamente si es libre. Demostracion. Todo grupo libre es libre de torsion, por el Ejercicio 7.3.2. Sea A un grupo abeliano nitamente generado y libre de torsion. Sea X = fa1 : : :  ang un conjunto de generadores de A. Entre todos los subconjuntos de X que sean linealmente independientes elegimos uno maximal Y podemos suponer, reordenando los ai si es necesario, que Y = fa1 : : :  ak g. Sea L = hY i, que claramente es libre. Sea i 2 fk + 1 k + 2 : : : ng. Por la maximalidad de Y , el conjunto fa1 : : :  ak  aig es linealmente dependiente, luego hay una relacion ti1a1 + + tik ak + ti ai = 0 donde los coecientes son enteros, no todos nulos. Como Y es linealmente independiente, se tiene ti 6= 0. Sea t = tk+1 tn. Entonces cada tai 2 L y la aplicacion a 7! ta es un homomorsmo de grupos f : A ! L. Como A es libre de torsion y t 6= 0, la aplicacion f es inyectiva y, por tanto A es isomorfo a un subgrupo de L. Del Teorema 7.2.15 se deduce que A es libre. Corolario 7.3.6 Sea A un grupo abeliano nitamente generado. Entonces A = t(A)  L para un subgrupo abeliano libre L de A. Ademas t(A) es nito y L es isomorfo a A=t(A), y por tanto L es unico salvo isomorsmos. Demostracion. A=t(A) es libre de torsion por el Ejercicio 7.3.2, y es nitamente generado por serlo A, luego es libre por el Teorema 7.3.5. Sea : A ! A=t(A) la proyeccion canonica. Del Corolario 7.2.14 se deduce que A = t(A)  L, donde L es un subgrupo de A isomorfo a A=t(A), luego L es un grupo abeliano libre. Ademas t(A) es nitamente generado (es isomorfo al cociente A=L por el Ejercicio 7.1.5) y de torsion, luego es nito (Proposicion 7.3.4). Existen grupos abelianos A tales que t(A) no es un sumando directo de A. Un ejemplo de esta situacion es el grupo Q 1Z p del apartado 6 de los Ejemplos 7.3.3, pero no es sencillo comprobar esta propiedad. p 170 CAPITULO 7. GRUPOS ABELIANOS FINITAMENTE GENERADOS Z Z Z RZ R Z C ZZ R ZZ Ejemplos 7.3.7 La torsion como sumando directo. 1. Si A =   6  36, entoncest(A) = 0  0  6  36, y podemos tomar L =   0  0. 2. Ya hemos visto que  = h;1i  +, y es claro que h;1i = t( ). 3. Sea G el subgrupo de  generado por f2 ig. Por los Ejemplos 7.3.3, si z = 2n im 2 t(G) (con n m 2 ), entonces 1 = jz j = 2n, lo que implica que n = 0. O sea t(G) = hii. Como h2i es libre de torsion, G = h2i  hii es la descomposicion del Corolario 7.3.6. 4. Sea R un anillo. En el anillo cociente RX]=(X 2), es claro que cada elemento tiene un unico representante de la forma r + sX con r s 2 R. Dados r + sX + (X 2 ) y r0 + s0 X + (X 2 ) en RX]=(X 2), se tiene r + sX + (X 2 )] + r0 + s0 X + (X 2 )] = (r + r0) + (s + s0 )X + (X 2 ) y r + sX + (X 2 )] r0 + s0 X + (X 2 )] = (rr0 ) + (rs0 + r0s)X + (X 2 ): Por tanto, podemos identicar RX]=(X 2 ) con el anillo cuyo grupo abeliano subyacente es R  R y en el que el producto viene dado por (r s) (r0  s0) = (rr0 rs0 + r0s): El elemento identidad de este anillo es (1 0). Si (r s) es invertible en este anillo, es claro que r 2 R . Recprocamente, si r 2 R entonces (r;1 ;sr;2 ) es el inverso de (r s) en este anillo. En consecuencia, el grupo multiplicativo de sus unidades, que denotaremos con R / R, es el conjunto R  R con el producto denido en el parrafo anterior. Vamos a determinar el orden de un elemento (r s) 2 R / R. Si n 2 +, es facil ver (bien por induccion o bien considerando la formula del binomio de Newton en RX]=(X 2 )) que (r s)n = (rn nrn;1s): Como r es invertible en R, se tiene (r s)n = (1 0) precisamente si rn = 1 y ns = 0. Por tanto, (r s) tiene orden nito en R / R precisamente si r tiene orden nito en (R  ) y s tiene orden nito en (R +). Es decir, t(R / R) = t(R )  t(R). Ademas, si (r s) 2 t(R / R) entonces o(r s) = mcm(om (r) oa (s)) donde om (r) es el orden de r en el grupo multiplicativo R y oa (s) es el orden de s en el grupo aditivo R. Pasemos a un caso concreto: Sea A =  / (recuerdese que  = f1 ;1g). Por el parrafo anterior se tiene t(A) = f(1 0) (;1 0)g. Por otra parte, L = h(1 1)i es un subgrupo libre de A, y el lector puede ahora comprobar que se tiene A = t(A)  L. Por el Teorema 7.2.15, todo subgrupo B de un grupo abeliano libre A es libre y r(B) r(A). Los resultados anteriores nos permiten determinar cuando se da la igualdad entre los rangos. Proposicion 7.3.8 Sea A un grupo abeliano libre nitamente generado y B un subgrupo de A. Entonces r(A) = r(B) precisamente si A=B es un grupo nito. Demostracion. Como A es nitamente generado, lo es tambien A=B, as que A=B es nito si y solo si es de torsion (Proposicion 7.3.4). Se trata pues de ver que A=B es de torsion si y solo si r(B) = r(A). Sean n = r(A) (por lo que podemos asumir que A = n) y k = r(B), y jemos una base b1  : : :  bk de B. Es facil ver que, para un elemento a 2 A, el elemento a + B 2 A=B tiene orden innito si y solo si los elementos b1 : : :  bk  a son linealmente independientes en A. Podemos ya demostrar la equivalencia: Si A=B no es de torsion, existe un elemento a + B en A=B de orden innito, luego b1  : : :  bk  a son linealmente independientes en A y as k < k + 1 n. Y si k < n y vemos a A = n dentro del espacio vectorial racional V = n, entonces existe  2 V tal que b1  : : :  bk  son linealmente independientes en V , y ademas existe un entero no nulo t tal que a = t 2 A. Como t 6= 0, los elementos b1 : : :  bk  a son linealmente independientes en V y, por el Ejercicio 7.2.6, tambien lo son en A, por lo que a + B tiene orden innito y as A=B no es de torsion. Z ZZ Q Z Z Z 7.4. GRUPOS INDESCOMPONIBLES Y P -GRUPOS 171 7.4 Grupos indescomponibles y p-grupos Como hemos comentado en la introduccion del captulo, nuestro objetivo es descomponer un grupo abeliano nitamente generado A como suma directa de subgrupos con algunas propiedades especiales. Con las herramientas desarrolladas en las secciones anteriores, en esta veremos primero que A es suma directa de subgrupos que no pueden descomponerse mas (indescomponibles), y a continuacion demostraremos que estos subgrupos indescomponibles son cclicos, de lo que deduciremos que A es suma directa de subgrupos cclicos. Comenzamos deniendo con precision los grupos indescomponibles. De nicion 7.4.1 Un grupo abeliano no nulo se dice que es indescomponible si no es suma directa de dos subgrupos propios. Es decir A es indescomponible si A = X  Y implica X = 0 o Y = 0 (y por tanto X = A o Y = A). ZQ Ejemplos 7.4.2 Grupos Indescomponibles. Z 1. y son indescomponibles, por un argumento usado en los Ejemplos 7.1.4. 2. Por el Corolario 7.2.9 y el Teorema 7.3.5, es, salvo isomorsmos, el unico grupo abeliano libre de torsion y nitamente generado que es indescomponible. 3. Si p es un numero primo entonces p es indescomponible, pues no tiene subgrupos propios no triviales. De hecho, pn es indescomponible para cada n 1. En efecto, por el Teorema de la Correspondencia, los subgrupos de pn forman una cadena (Ejemplos 5.6.6), y es claro que cualquier grupo cuyos subgrupos esten linealmente ordenados es indescomponible. 4. Sean n m 2 dos enteros coprimos por el Teorema Chino de los Restos se tiene nm  = n m, por lo que nm no es indescomponible (recuerdese la relacion entre entre suma directa y producto directo descrita al nal de la Seccion 7.1). En consecuencia, los grupos cclicos nitos indescomponibles son exactamente aquellos cuyo orden es pr para cierto primo p y cierto entero r 1. Z Z Z Z Z Z Z Como consecuencia inmediata del Corolario 7.3.6 se obtiene: Corolario 7.4.3 Un grupo abeliano nitamente generado indescomponible es de torsion (y por tanto nito) o libre de torsion. Proposicion 7.4.4 Todo grupo abeliano nitamente generado y no nulo A es una suma directa de subgrupos indescomponibles. Z Demostracion. Por el Corolario 7.3.6, y teniendo en cuenta que los grupos abelianos libres de rango nito son sumas directas de copias de (Corolario 7.2.9) y que los grupos abelianos nitamente generados de torsion son nitos (Proposicion 7.3.4), basta demostrar la armacion para grupos abelianos nitos. Sea A un grupo abeliano nito. Razonamos por induccion en jAj, con el caso jAj = 2 trivial. Si A es indescomponible no hay nada que demostrar. En caso contrario A = B  C y los cardinales de B y C son estrictamente menores que el de A. Por hipotesis de induccion, B y C son sumas directas de grupos indescomponibles, y \pegando" las descomposiciones de B y C como en el Ejercicio 7.1.5 obtenemos una descomposicion de A como suma directa de grupos indescomponibles. Z Q En los Ejemplos 7.4.2 han aparecido dos tipos de grupos nitamente generados e indescomponibles: y los cclicos de orden pn ( no es nitamente generado por el Problema 49 del Captulo 5). El resto de esta seccion lo dedicaremos a ver que, salvo isomorsmos, no hay otros. Para ello, sera importante considerar ciertos grupos que comparten una caracterstica con pn , y que denimos a continuacion: Z Lema 7.4.5 Dados un grupo abeliano nito A y un entero positivo primo p, las siguientes condiciones son equivalentes: 1. El orden de A es una potencia de p. 2. El orden de cada elemento de A es una potencia de p. CAPITULO 7. GRUPOS ABELIANOS FINITAMENTE GENERADOS 172 Demostracion. Si jAj = pn entonces cada a 2 A tiene orden pm con m n por el Teorema de Lagrange. Demostraremos el recproco por induccion en el orden jAj, con el caso jAj = 1 trivial. Si jAj > 1 entonces existe 0 6= b 2 A, y si ponemos B = hbi entonces tenemos jB j = o(b) = pm para cierto m 1. El grupo cociente A=B tiene cardinal menor que jAj, y es elemental ver que el orden de todos sus elementos es una potencia de p. Por la hipotesis de induccion se tiene jA=B j = pn para cierto n 0, y en consecuencia jAj = jB j jA=B j = pm+n , como queramos ver. De nicion 7.4.6 Un grupo abeliano nito que verique las condiciones equivalentes del Lema 7.4.5 se llama un p-grupo. ZZ Z Esta denicion la extenderemos a grupos no abelianos en la Denicion 8.3.3. Como en la denicion no se excluyen las potencias de exponente 0, el grupo trivial es un p-grupo para cualquier primo p. Un ejemplo mas sosticado es 25  625  625, que es un 5-grupo. De nicion 7.4.7 Dados un grupo abeliano A y un entero primo p, el subgrupo de p-torsion de A es tp (A) = fa 2 A : existe n 2 tal que pna = 0g = fa 2 A : o(a) es una potencia de pg: Dejamos que el lector compruebe que ambos conjuntos son iguales y que forman un subgrupo de A. De hecho, si A es nito, tp (A) es claramente el mayor p-subgrupo de A (es decir, el mayor subgrupo de A que es un p-grupo). Proposicion 7.4.8 Sea A un grupo abeliano nito y sean p1  : : :  pk los divisores primos de jAj. En- N tonces con cada tpi (A) 6= 0. A = tp1 (A)   tpk (A) Z Demostracion. Sea a 2 A y sea o(a) = n = p1 1 pk k (>por que no pueden aparecer otros primos en la factorizacion de n?). Para cada i = 1 : : :  k sea qi = n=pi i . Es claro que ningun primo divide a la vez a todos los qi , por lo que mcd(q1 : : : qk) = 1 y por tanto existen m1  : : :  mk 2 tales que m1 q1 + + mk qk = 1. Como pi i qia = 0, se tiene qia 2 tpi (A), luego a = m1 q1a + + mk qk a 2 tp1 (A) + + tpk (A): En consecuencia, A = tp1 (A) + + tpk (A). Para ver que la suma es directa, supongamos que a1 + + ak = 0 con cada ai 2 tpi (A). Por tanto, para cada i = 1 : : : k, existe i tal que pi i ai = 0. Sea m = p1 1 pkk . Para cada ndice i ponemos ti = m=pi i , de modo que ti aj = 0 cuando i 6= j, y as ti ai = ;ti X j 6=i aj = 0: Entonces o(ai ) divide a ti y a pi i , y como estos son coprimos, se tiene o(ai ) = 1 y por tanto ai = 0. Esto prueba que la familia es independiente. Por ultimo, de la igualdad A = tp1 (A)   tpk (A) se deduce que jAj = jtp1 (A)j jtpk (A)j. Como el orden de cada tpi (A) es una potencia de pi (Lema 7.4.5) y cada pi divide a jAj, deducimos que ese orden es mayor que 1 y por tanto tpi (A) 6= 0. El siguiente corolario es inmediato: Corolario 7.4.9 Un grupo nito e indescomponible es un p-grupo para cierto primo p. Ejemplos 7.4.10 Descomposicion en suma directa de p-grupos Z Z ZZ Z 1. Sea n = p1 1 pk k una factorizacion prima irredundante del entero n. Por el Teorema Chino de los Restos, n  = p1 1   pk k y claramente los factores de esta descomposicion van a corresponder con los factores tp ( n) de la descomposicion de la Proposicion 7.4.8. Mas concretamente, si qi = n=pi i para cada i = 1 : : :  k, entonces qi = qi + n genera un grupo de orden pi i , y por tanto tpi ( n) = hqii. Z ZZZ Z 7.4. GRUPOS INDESCOMPONIBLES Y P -GRUPOS 173 2. Sea A = 12 / 12 (veanse los Ejemplos 7.3.7 para la denicion de este grupo). Como j 12j = 4 y j 12j = 12, se tiene jAj = 48 = 24 3. Usando la descripcion del orden de cada elemento que se dio en los Ejemplos 7.3.7, se tiene t3(A) = f(1 b) : b = 0 4 8g y t2(A) = f(a b) : b = 0 3 6 9g (donde identicamos cada entero con su clase modulo 12). Sea B un subgrupo del grupo abeliano A, y sea a 2 A. Si na = 0 (con n 2 ), entonces, en A=B, se tiene n(a+B) = 0. Eso implica que el orden de a+ B divide al orden de a. En general estos ordenes no coinciden por ejemplo, no lo hacen si a es un elemento no nulo de B. Se dice entonces que a \baja de orden" en el cociente A=B. El siguiente lema muestra que, en algunas clases laterales, podemos elegir un representante que no baja el orden. Lema 7.4.11 Sea A un p-grupo nito. Entonces: 1. Existe a 2 A tal que o(a) = p(A). 2. Si B = hai (donde a es el del apartado anterior) entonces todo elemento del cociente A=B tiene un representante que no baja de orden. Es decir, para todo 2 A=B existe x 2 A tal que x + B = y o(x) = o( ). Demostracion. El primer apartado se tiene porque el periodo de un grupo abeliano es el mnimo comun multiplo de los ordenes de sus elementos (Ejercicio 7.3.2). Para el segundo, comenzaremos eligiendo un representante cualquiera de , y veremos que podemos sustituirlo por otro con la propiedad requerida. Sea pues y 2 A tal que y + B = . Supongamos que o(a) = p(A) = pm , o(y) = ps y o( ) = pk . Por el parrafo anterior al lema, se tiene k s m. Si k = s, tomamos x = y y hemos terminado. Supongamos pues que k < s. Como pk (y + B) = pk = 0, se tiene que pk y 2 B = hai es decir, pk y = qa, para algun q 2 . Dividiendo q por la mayor potencia posible de p, podemos poner q = rpt con mcd(p r) = 1. Entonces pm+k;t y = pm;t pk y = pm;t qa = rpm a = 0 y, por tanto, s m + k ; t. Por otro lado, pm+k;t;1y = pm;t;1qa = rpm;1 a 6= 0 de donde se deduce que s = m + k ; t. Sea ahora x = y ; rpm;s a entonces x + B = y + B = , y por tanto o( ) = pk divide a o(x). Pero ademas, pk x = pk y ; rpm+k;s a = pk y ; rpta = 0, de donde se deduce que o(x) = pk = o( ), como queramos ver. N Z Ahora podemos caracterizar los grupos abelianos nitamente generados que son indescomponibles. Proposicion 7.4.12 Las siguientes condiciones son equivalentes para un grupo abeliano nitamente generado A: 1. A es indescomponible. 2. A es isomorfo a o a pn con p primo y n 2 ZZ Z +. Z Demostracion. Ya hemos observado (Ejemplos 7.4.2) que los grupos del apartado 2 son indescompo- nibles. Supongamos pues que A es indescomponible y veamos que es isomorfo a uno de ellos. Por el Corolario 7.4.3, A es libre de torsion o de torsion. En el primer caso A es isomorfo a por el Teorema 7.3.5 y el Corolario 7.2.9. Supongamos pues que A es de torsion, por lo que debe ser un p-grupo nito (Proposicion 7.3.4 y Corolario 7.4.9) y en consecuencia jAj = pn para cierto n 1. Solo falta demostrar que A es cclico, cosa que vamos a hacer por induccion sobre n. El caso n = 1 lo resuelve el Teorema 5.8.7. En el caso general, por el Lema 7.4.11, A contiene un elemento a cuyo orden coincide con el periodo de A. Sean B = hai y C = A=B. Por la Proposicion 7.4.4 se tiene C = C1   Ck para ciertos C1 : : :  Ck indescomponibles. Por hipotesis de induccion, cada Ci es cclico. Es decir, existen x1  : : :  xk 2 A tales que Ci = hxi + B i para cada i, y por el Lema 7.4.11 174 CAPITULO 7. GRUPOS ABELIANOS FINITAMENTE GENERADOS Z podemos suponer que o(xi ) = o(xi +B) para cada i. Claramente A = B+ hx1 i + hx2 i + + hxk i. Vamos a ver que esta suma es directa. Sean b 2 B y m1  : : :  mk 2 tales que b + m1 x1 + + mk xk = 0. Entonces 0 = m1 (x1 + B) + + mk (xk + B) y, por tanto, cada mi (xi + B) = 0. De aqu se deduce que mi es multiplo de o(xi + B) = o(xi ) y por tanto mi xi = 0 y b = 0. Como A es indescomponible y B 6= 0, deducimos que A = B = hai es cclico. Combinando las Proposiciones 7.4.4 y 7.4.12 se obtiene: Corolario 7.4.13 Todo grupo abeliano nitamente generado es suma directa de subgrupos cclicos (y los que sean nitos se pueden tomar de manera que su orden sea potencia de primo). 7.5 Descomposiciones primarias e invariantes El Corolario 7.4.13 va a ser fundamental para clasicar los grupos abelianos nitamente generados salvo isomorsmos. La idea es que cada clase de isomorfa de grupos abelianos nitamente generados estara dada por una lista de numeros que van a representar los cardinales de los factores que aparecen en una descomposicion de cualquiera de los elementos de la clase como suma directa de grupos cclicos. Vamos a elegir dos tipos de listas de numeros: En la primera los numeros que admitimos son potencias de primos y 0 en la segunda los numeros van a ser numeros naturales arbitrarios pero con la exigencia de que cada uno de ellos divida a los anteriores. ZZ De nicion 7.5.1 Sea A un grupo abeliano nitamente generado. Una descomposicion primaria o indescomponible de A es una expresion de A como suma directa de subgrupos indescomponibles. Como cada uno de estos sumandos es isomorfo a o a pn , con p primo y n 1, siempre podemos reordenarlos de modo que se tenga A = (nj=1 Aj )  (mj =11 A1j )  = A1  A2   An A11  A12   A1m1  Ak1  Ak2   (mj =1k Akj )  Akmk con p(Ai ) = 0 (es decir, Ai es cclico innito) y p(Aij ) = pi ij para ciertos enteros primos positivos p1 < p2 < < pk y ciertos enteros positivos ij con i1 i2 imi 1 para cada i = 1 : : : k. Con esta terminologa, la Proposicion 7.4.4 se reenuncia como: Teorema 7.5.2 Todo grupo abeliano nitamente generado tiene una descomposicion primaria. Para obtener una descomposicion primaria de un grupo abeliano nitamente generado seguimos los pasos indicados en la seccion anterior es decir, dado un grupo abeliano nitamente generado A: 1. Se calcula T = t(A) y un subgrupo libre de torsion L de A tal que A = T  L (Corolario 7.3.6). Z 2. Se busca una base b1  : : :  bn de L y por tanto se tiene L = hb1i   hbn i  = n. Esta es la \primera la" en la ordenacion de los sumandos que se propone en la Denicion 7.5.1. 3. Se calcula tp (T) para cada divisor primo de jT j entonces T = tp1 (T ) tpk (T ) (Proposicion 7.4.8). 4. Para cada divisor primo p de jT j se calcula a 2 tp (T ) tal que o(a) coincida con el periodo de tp (T ) (Proposicion 7.4.11) y pasamos a estudiar tp (T)=hai, que tiene orden menor que el de tp (T). Por recurrencia vamos pasando a grupos de orden cada vez mas peque~no hasta obtener un grupo cclico. Volvemos para atras siguiendo la demostracion de la Proposicion 7.4.12 y as obtendremos una descomposicion primaria de tp (T), que ocupara una la en la ordenacion de los sumandos segun la Denicion 7.5.1. 7.5. DESCOMPOSICIONES PRIMARIAS E INVARIANTES 175 Ejemplos 7.5.3 Descomposiciones primarias. 1. Sea A = h2 ii el grupo del Ejemplo 7.3.7. Ya vimos que t(A) = hii y A = hii  h2i. Como hii es ZZ ZZ cclico de orden 4, hemos obtenido una descomposicion primaria de A. 2. Sea A =  / . Vimos que t(A) = h(;1 0)i y A = h(;1 0)i  h(1 1)i. Como h(;1 0)i es cclico de orden 2, la anterior es una descomposicion primaria de A. 3. Sea A = 12 / 12. Como este grupo es nito, no hay que dar los dos primeros pasos, y el tercero lo habamos dado en el Ejemplo 7.4.10. Claramente t3(A) = f(1 b) : b = 0 4 8g es cclico de orden 3, generado por (1 4). Sin embargo t2 (A) = f(a b) : b = 0 3 6 9g no es cclico ya que su cardinal es 16 y su periodo 4. Un elemento de orden 4 es (1 3). Pongamos B = h(1 3)i = f(1 0) (1 3) (1 6)(1 9)g y C = t2(A)=B, que tiene orden 4. Observese que x2 2 B para todo x 2 t2(A). Por tanto C = 2  2, de donde se deduce que si C1 y C2 son dos subgrupos C de orden 2 y distintos, entonces C = C1  C2 es una descomposicion primaria de C. Uno de estos subgrupos puede ser h(;1 3)B i (notacion multiplicativa), pero como (;1 3) no tiene orden 2 en A, es necesario cambiarlo, como hicimos en la demostracion del Lema 7.4.11, por otro elemento de la misma clase modulo B que no baje el orden, por ejemplo (;1 3)(1 3) = (;1 0) esta en (;1 3)B y tiene orden 2. El otro subgrupo puede ser h(5 0)B i, de donde se obtiene que C = h(;1 0)B i  h(5 0)B i y, por tanto, t2 (A) = h(1 3)i  h(;1 0)i  h(5 0)i: Uniendo toda la informacion obtenemos A = h(1 3)i  h(;1 0)i  h(5 0)i  h(1 4)i  = 4  2  2  3: ZZ ZZZZ De nicion 7.5.4 Sea A un grupo abeliano nitamente generado. Una descomposicion invariante de A es una expresion del tipo A = ni=1 Ai  donde cada Ai es un grupo cclico no trivial y se verica p(Ai ) j p(Ai;1 ) para cada i = 2 : : : n. Ejercicio 7.5.5 Demostrar que, si A = ni=1Ci es una descomposicion invariante A y el subndice k es tal que p(Ck ) = 0 = 6 p(Ck+1 ), entonces el subgrupo de torsion de A es t(A) = ni=k+1 Ci . Utilizando el Teorema 7.5.2 podemos obtener tambien: Teorema 7.5.6 Todo grupo abeliano nitamente generado tiene una descomposicion invariante. Demostracion. Sea A un grupo abeliano nitamente generado. A~nadiendo sumandos triviales a una descomposicion primaria suya, tenemos A = A1  A2   An  A11  A12   A1m  Ak1  Ak2   Akm  donde cada sumando es cclico y se tiene p(Ai ) = 0 y p(Aij ) = pi ij , para ciertos primos positivos distintos p1 p2 : : : pk y ciertos enteros ij tales que, para cada i, 11 12 : : : 1m 0: (7.5.1) Los ij que valen cero se corresponden con los sumandos triviales que hemos a~nadido para que, en cada la de la descomposicion de A, a partir de la segunda, haya el mismo numero de sumandos. Para obtener la descomposicion primaria basta con \agrupar los sumandos por columnas", a partir de la segunda la. Explcitamente, para cada j = 1 : : :  m, sea Bj = A1j  A2j   Akj : CAPITULO 7. GRUPOS ABELIANOS FINITAMENTE GENERADOS 176 Entonces A = A1  A2   An  B1  B2   Bm y, por el Teorema Chino de los Restos, cada Bj es cclico de orden dj = p1j p2j pmj . Como consecuencia de las desigualdades (7.5.1) se tiene que dj j dj ;1 para todo j = 2 : : :  n. La demostracion del Teorema 7.5.6 nos dice como se obtiene una descomposicion invariante a partir de una descomposicion primaria. Ejemplos 7.5.7 Descomposiciones invariantes a partir de descomposiciones primarias. 1. Supongamos dada una descomposicion primaria de A, digamos A = (A1  A2)  (A21  A22  A23  A24)  (A31  A32)  (A71  A72  A73 ) donde Aij = haij i y los ordenes de los respectivos sumandos son (por este orden) 0,0,16, 4, 2, 2, 27, 3, 7, 7, 7. Entonces: B1 = A21  A31  A71 = ha21 + a31 + a71i es cclico de orden 16 27 7 = 3:024 B2 = A22  A32  A72 = ha22 + a32 + a72i es cclico de orden 4 3 7 = 84 B3 = A23  A73 = ha23 + a73i es cclico de orden 2 7 = 14 y B4 = A24 es cclico de orden 2. Entonces ZZZ Z Z Z ZZZZ Z ZZ A = A1  A2  B1  B2  B3  B4  =   3:024  es una descomposicion invariante de A. 2. Sea A = 12 / 12. En los Ejemplos 7.5.3 vimos que ZZ A = h(1 3)i  h(;1 0)i  h(5 0)i  h(1 4)i  = es una descomposicion primaria de A. Por tanto A = h(1 7)i  h(;1 0)i  h(5 0)i  = es una descomposicion invariante de A. 84  14  2 4 2 2 3 12  2  2 Tambien es facil sacar consecuencias de la demostracion del Teorema 7.5.6 para obtener descomposiciones primarias a partir de descomposiciones invariantes. Z Z Z ZZ ZZZ Ejemplo 7.5.8 Descomposiciones primarias a partir de descomposiciones invariantes. Sea A = ha1 i  ha2 i  ha3 i  ha4 i  = 2:025  135  9 (donde el isomorsmo es componente a componente). Observando que 2:025 = 34 52 y 135 = 33 5, denimos  81 B51 = h81a1i = 25 B31 = h25a1i = B32 = h5a2i  = 27 B52 = h27a2i  = 5 B33 = ha4 i  = 9: Entonces A = B31  B32  B33  B51  B52 es una descomposicion primaria de A. ZZZ Por supuesto, es posible descomponer un grupo abeliano nito como suma directa de subgrupos cclicos sin ajustarse a ninguno de los \formatos" de las descomposiciones primarias o invariantes. Por ejemplo, si A = 6  3  2 entonces la descomposicion A = h(1 0 0)i  h(0 1 0)i  h(0 0 1)i 7.5. DESCOMPOSICIONES PRIMARIAS E INVARIANTES 177 ZZZZ ZZ no es de ninguno de esos dos tipos, aunque no es difcil obtener una descomposicion primaria A = h(2 0 0)i  h(3 0 0)i  h(0 1 0)i  h(0 0 1)i  = y una invariante 3 2 3 2 A = h(1 0 0)i  h(0 1 1)i  = 6  6: Lo importante de estas descomposiciones es que presentan buenas condiciones de unicidad. En efecto, como vamos a ver, cualesquiera dos descomposiciones primarias de un grupo A (abeliano y nitamente generado) son \esencialmente iguales", lo que nos permite asignarle a un tal grupo una lista de numeros enteros (los periodos de los sumandos que aparecen en una de esas descomposiciones). Otro tanto podra decirse de las descomposiciones invariantes. Ademas, estas listas de numeros determinan salvo isomorsmos al grupo A, en el mismo sentido en el que la dimension determina salvo isomorsmos a un espacio vectorial de dimension nita. Aunque el caso que nos ocupa es mas sosticado, en ambos somos capaces de asociar a un objeto (grupo o espacio vectorial) una lista de numeros (uno solo, la dimension, en el caso vectorial) de tal modo que dos objetos son isomorfos si y solo si tienen la misma lista. De nicion 7.5.9 Sean A y B dos grupos abelianos nitamente generados. Dos descomposiciones primarias de A y B son semejantes si los sumandos que intervienen son isomorfos dos a dos. Si ordenamos las descomposiciones como se ha indicado en la Denicion 7.5.1, digamos y A = (nj=1 Aj )  (mj =11 A1j )   (mj =1k Akj ) B = (nj =1 Bj )  (mj =11 B1j )   (mj =1k Bk j ) es claro que estas son semejantes si y solo si n = n0, k = k0 , cada mi = m0i y p(Aij ) = p(Bij ) para cada 0 0 0 0 0 posible par de ndices. Dos descomposiciones invariantes A = ni=1 Ai y B = ni=1 Bi son semejantes si los sumandos que intervienen son isomorfos dos a dos, lo que claramente equivale a que tengan el mismo numero de sumandos (n = n0 ) y las mismas listas de periodos (p(Ai ) = p(Bi ) para todo i = 1 : : : n). 0 Es facil ver que, si A y B tienen descomposiciones primarias (o invariantes) semejantes, entonces A y B son isomorfos. El siguiente teorema nos dice, esencialmente, que se verica el recproco: Teorema 7.5.10 Sea A un grupo abeliano nitamente generado. Entonces: 1. Todas las descomposiciones primarias de A son semejantes. 2. Todas las descomposiciones invariantes de A son semejantes. Demostracion. En vista de que se puede pasar de una descomposicion primaria a una invariante y viceversa, bastara con demostrar una de las dos armaciones. Demostraremos la primera. Sea A = (nj=1 Aj )  (mj =11 A1j )   (mj =1k Akj ) una descomposicion primaria de A con p(Ai ) = 0 y p(Aij ) = pi ij para ciertos enteros primos positivos p1 < p2 < < pk y ciertos enteros positivos ij con i1 i2 imi 1 para cada i = 1 : : : k. Observese que nj=1 Aj  = A=t(A), por lo que n es el rango del grupo libre A=t(A) y por tanto esta determinado por A (no depende de la descomposicion particular elegida). Por otro lado, es claro que, para cada i = 1 : : : k, se tiene mj =1i Aij = tpi (A) por lo que estos subgrupos tambien estan determinados por A. En consecuencia, podemos limitarnos a demostrar la unicidad asumiendo que A es un p-grupo nito. En esta situacion, dos descomposiciones primarias de A seran de la forma A = A1   An = B1   Bm  CAPITULO 7. GRUPOS ABELIANOS FINITAMENTE GENERADOS 178 donde cada sumando es cclico y, si ponemos p(Ai ) = pi y p(Bi ) = pi , se tiene 1 2 n y 1 2 m . Vamos a ver, por induccion en i, que i = i para cada i. Observese que p1 = p(A) = p1 , lo que resuelve el caso i = 1. Supongamos pues que j = j para cada j = 1 : : :  i ; 1, y veamos que i = i . Podemos suponer sin perdida de generalidad que i i . Observemos lo siguiente: Sea C un grupo cclico de orden pr y sea s 2 . Se tiene ps C = 0 si y solo si s r. Por otra parte, si s r, entonces ps C es cclico de orden pr;s por la Proposicion 5.8.11. En consecuencia, si ponemos q = pi , se tiene N qA  = qA1   qAi;1 = (qB1   qBi;1 )  (qBi   qBm ): Como qA1   qAi;1 y qB1   qBi;1 tienen el mismo cardinal, deducimos que qBi   qBm = 0. En particular 0 = qBi = pi Bi , de modo que i i , y por tanto i = i , como queramos ver. De nicion 7.5.11 Sea A un grupo abeliano nitamente generado. Sea A = ni=1 Ai (7.5.2) una descomposicion primaria ordenada como en la Denicion 7.5.1. Entonces la lista (p(A1 ) : : :  p(An )) (que no depende de la descomposicion primaria elegida, por el Teorema 7.5.10) se conoce como la lista de los divisores elementales de A. Analogamente, si (7.5.2) es una descomposicion invariante, entonces la lista (p(A1 ) : : :  p(An)) (que tampoco depende de la descomposicion invariante elegida) se conoce como la lista de los factores invariantes de A. En ambas listas, cada sumando cclico innito aporta un 0 al principio de la lista. A menudo se simplica la notacion escribiendo (m p(Am+1 ) : : :  p(An)), donde m es el numero de ceros en la lista original. Ejemplos 7.5.12 Listas de divisores elementales y factores invariantes. 1. Si A es el grupo del primer apartado de los Ejemplos 7.5.7, la lista de sus divisores elementales es (2 16 4 2 2 27 3 7 77), y la de sus factores invariantes es (2 3:024 8414). ZZ 2. Para el grupo  / , las listas de divisores elementales y de factores invariantes coinciden, y son (0 2). >Para que tipo de grupos coinciden ambas listas? ZZ 3. Los divisores elementales de 12 / 12 son (4 2 2 3), y sus factores invariantes son (12 2 2). Todo lo visto en esta seccion se resume en el siguiente Teorema: Teorema 7.5.13 (Teorema de Estructura de Grupos Abelianos Finitamente Generados) 1. Todo grupo abeliano nitamente generado tiene una descomposicion primaria y una descomposicion invariante. 2. Las siguientes condiciones son equivalentes para dos grupos abelianos: (a) (b) (c) (d) (e) Son isomorfos. Tienen descomposiciones primarias semejantes. Tienen descomposiciones invariantes semejantes. Tienen la misma lista de divisores elementales. Tienen la misma lista de factores invariantes. 7.6. PRESENTACIONES POR GENERADORES Y RELACIONES Ejercicio 7.5.14 179 Z Z 1. Demostrar que, si n 2 + es libre de cuadrados, entonces todo grupo abeliano nito de orden n es cclico (y por tanto isomorfo a n). 2. Si p es un primo positivo, demostrar que, salvo isomorsmos, los unicos grupos abelianos de orden p2 son p2 y p  p. Ademas se puede borrar \abelianos". 3. Describir, salvo isomorsmos, todos los grupos abelianos de ordenes 8, 12, 16, 20 y 24. Ejercicio 7.5.15 Demostrar el recproco del Teorema de Lagrange para grupos abelianos nitos. Es decir, demostrar que un grupo abeliano de orden n tiene un subgrupo de orden m para cada divisor m de n. Ejemplos 7.5.16 Algunas decomposiciones invariantes y primarias. ZZZ ZZ Z ZZ Z ZZ ZZ ZZ ZZ Z Z Z ZZZ ZZ ZZZZ ZZZZZ Z Z Z Q Z Q ZZ Z Z Z Z Z N Z Z Z Z Z ZZ Z 1. Los grupos multiplicativos 5, 10 y 12 tienen 4 elementos (pues (5) = (10) = (12) = 4). Los dos primeros son cclicos (busca un generador), y por tanto isomorfos entre s. El tercero no es cclico (tiene periodo 2), y por el ejercicio anterior debe ser isomorfo a 2  2 (da un isomorsmo explcito). 2. Los grupos multiplicativos 15, 16, 20, 24 y 30 tienen 8 elementos, por lo que han de ser isomorfos a uno de estos tres: 8, 4  2 o 2  2  2. Considerando los ordenes de los elementos se deduce que ninguno es cclico, que 24 es isomorfo a 2  2  2 y que los otros cuatro son isomorfos a 4  2. 3. Vamos a calcular todos los grupos abelianos de orden 420 salvo isomorsmos y sus descomposiciones invariantes y primarias. Como 420 = 22 3 5 7, las posibles listas de divisores elementales son (4 3 5 7) o (2 2 3 5 7). Por tanto, salvo isomorsmos, los grupos abelianos de orden 420 son A = 4  3  5  7 y B = 2  2  3  5  7: E stas son sus descomposiciones primarias. Sus descomposiciones invariantes seran A = 420 y B = 210  2: Por el Teorema de Estructura 7.5.13, todo grupo abeliano nitamente generado es suma directa de cclicos. Esto no es cierto para grupos abelianos en general, considerese ni siquiera para grupos abelianos de torsion, como muestra el siguiente ejemplo. Ejemplo 7.5.17 Un grupo abeliano de torsion que no es suma directa de grupos cclicos. Sea p un numero primo. El conjunto Xp de los numeros racionales de la forma pmn , donde m 2 y n es un entero no negativo, es un subgrupo de . Ademas es un subgrupo de Xp . Se dene p = Xp = . Para cada entero no negativo n, sea An el subgrupo de p generado por an = p1n + . Como an tiene orden pn , entonces An es isomorfo a pn. Ademas 0 = A0  A1  A2  y n2NAn = p . Vamos a ver que todo subgrupo propio H de p es igual a An para algun n 2 . El conjunto de los numeros naturales n tales que an 2 H esta acotado (>por que?). Sea n el maximo de dicho conjunto. Entonces An  H. Si pmt + 2 H, con mcd(m p) = 1, entonces existen x y 2 tales que xm +ypt = 1. Luego at = p1t + = x pmt + y + = x( pmt + ) 2 H, y por tanto t n, de donde se concluye que m n;t pt + = mp an 2 An . Deducimos que H = An , como queramos. La conclusion nal es que p es indescomponible y, como no es cclico, tampoco es suma directa de grupos cclicos. 1 1 1 1 1 7.6 Presentaciones por generadores y relaciones Sea L un grupo abeliano libre con base fa1 : : : ang y sea S un subgrupo de L. Sabemos que S ha de estar generado por un conjunto nito fr1 : : : rm g de elementos de L es decir, cada ri sera una combinacion lineal con coecientes enteros de los aj , digamos ri = ki1a1 + + kinan (kij 2 ): Z CAPITULO 7. GRUPOS ABELIANOS FINITAMENTE GENERADOS 180 Consideremos ahora el grupo cociente L=S. Abusando de la notacion, escribiremos aj = aj + S. Entonces fa1  : : : ang es un conjunto generador de L=S, y para cada i = 1 : : : m se tiene ki1a1 + + kinan = 0 (en L=S): Estas igualdades se llaman \relaciones" entre los generadores a1  : : : an del grupo L=S. Es decir, los ai son generadores \libres" (linealmente independientes, sin relaciones no triviales) cuando los vemos en L, pero satisfacen ciertas relaciones en el cociente L=S. Por la Proposicion 7.2.10, todo grupo abeliano nitamente generado A es isomorfo a uno de la forma recien descrita, y de hecho es usual encontrar grupos abelianos dados de esa manera. Dados L y S en la situacion anterior y tales que A  = L=S, escribiremos A = ha1 : : :  an j r1 : : :  rmi y diremos que esta es una presentacion de A por generadores y relaciones. Un resultado basico para el manejo de las presentaciones por generadores y relaciones es el siguiente: Ejercicio 7.6.1 Sea A un grupo abeliano que es suma directa de subgrupos: A = A1   An . Para cada i = 1 : : : n, sea Bi un subgrupo de Ai . Entonces la familia B1  : : : Bn es independiente y, si B = B1   Bn , se verica A  A1 An B = B1   Bn donde, si algun Bi coincide con Ai , el correspondiente factor es trivial y se puede eliminar del producto. (Indicacion: Usar el Primer Teorema de Isomorfa.) Como consecuencia, las presentaciones en las que cada relacion es un multiplo entero de un generador nos permiten ver al grupo en cuestion como suma directa de cclicos de modo inmediato. Explcitamente, el hecho de que A tenga una presentacion del tipo Z A = ha1  : : : ar j d1a1  : : : dsas i Z Z Z Z Z Z Z ZZZ Z ZZ (con s r y cada di 2 +) equivale a decir que A = d1   ds     con r ; s factores iguales a , y podemos eliminar los factores con di = 1. Por ejemplo, las siguientes son varias expresiones por generadores y relaciones de grupos abelianos nitamente generados: n = ha1  : : :  ani n = ha j nai  2  3 = ha b c j 2b 3ci: Un grupo puede tener diversas presentaciones. Por ejemplo, = hai = ha b j bi o, utilizando el Teorema Chino de los Restos, 2  3 = ha b j 2a 3bi = hc j 6ci. En esta seccion veremos como conseguir, a partir de una presentacion por generadores y relaciones de un grupo abeliano nitamente generado A, otras presentaciones mas manejables, con el objetivo ultimo de obtener una presentacion del tipo ha1  : : : ar j d1a1  : : : dsas i con d1 j d2 j j ds, lo que nos dara la descomposicion invariante de A. La segunda herramienta basica para la manipulacion de las presentaciones es: Z Ejercicio 7.6.2 Sean a1 a2 : : : an elementos de un grupo abeliano A. Si, o bien a01 = a1 + ta2, donde t 2 , o bien a01 = ;a1 , entonces: 1. ha1 a2 : : : ani = ha01 a2 : : : ani. 2. El conjunto fa1 a2 : : : ang es linealmente independiente si y solo si lo es fa01 a2 : : : ang. En otras palabras, si en un conjunto sumamos a un elemento un multiplo entero de otro, o si cambiamos de signo un elemento, no cambian ni el subgrupo generado ni la dependencia o independencia lineal. Aplicando reiteradamente el Ejercicio 7.6.2, vemos que la armacion sigue valiendo si sumamos a un elemento una combinacion lineal (con coecientes enteros) del resto de elementos. Veamos con un ejemplo como pueden usarse estos resultados para simplicar las presentaciones: 7.6. PRESENTACIONES POR GENERADORES Y RELACIONES 181 Ejemplo 7.6.3 Simplicacion de una presentacion por generadores y relaciones. Sea A = ha b c j 2a + b + 6c 2a + 2b + 2ci. Esto signica que A  = L=S, donde fa b cg es base de L y S = h2a + b + 6c 2a+ 2b + 2ci. Si hacemos b0 = 2a + b + 6c, entonces fa b0 cg sigue siendo base de L y se tiene 2a + 2b + 2c = ;2a + 2b0 ; 10c, luego S = hb0  ;2a + 2b0 ; 10ci restando al segundo generador el doble del primero, y cambiando luego el signo del resultado, obtenemos S = hb0  2a+10ci. Por tanto, A = hb0  a c j b0  2a + 10ci. Podemos simplicar mas, haciendo a0 = a + 5c. Entonces fb0 a0 cg sigue siendo base de L y ademas S = hb0  2a0i, de modo que A = hb0 a0 c j b0 2a0i = ha0  c0 j 2a0 i. Por tanto A = 2 . En lo que sigue vemos como las ideas usadas en el Ejemplo 7.6.3 son sucientes para simplicar cualquier presentacion. Lo primero que haremos sera adoptar una notacion matricial para las presentaciones que las hace mas manejables. Supongamos que partimos de una expresion de un grupo por generadores y relaciones A = ha1 : : :  an j r1 : : :  rmi, donde las relaciones vienen dadas por las combinaciones lineales ri = ki1a1 + + kinan (kij 2 ): (7.6.3) Representamos este conjunto de relaciones por una matriz K = (kij ). Recprocamente, a cada matriz K de numeros enteros con m las y n columnas, le asociaremos el grupo cuya presentacion por generadores y relaciones es ha1  : : : an j r1  : : : rm i, donde cada ri viene dado por la igualdad (7.6.3). En particular, una matriz m  n de la forma 0d 1 0 0 0 1 B@ ... . . . ... ... .. C (7.6.4) .A 0 dm 0 0 se corresponde con el grupo abeliano A  = d1   ds    , con n ; m factores iguales a . Comenzaremos notando que ciertas transformaciones en una matriz no alteran el grupo que dene, y despues veremos como combinar esas transformaciones para alcanzar una matriz del tipo (7.6.4). Lema 7.6.4 Si K es una matriz de numeros enteros y K 0 es una matriz obtenida a partir de K mediante una de las operaciones que siguen, entonces los grupos asociados a K y K 0 son isomorfos. ZZ Z Z ZZ Z Z F0: F1: F2: F3: Eliminar una la formada por ceros. Reordenar las las. Cambiar el signo a todos los elementos de una la. Sumar a una la un multiplo entero de otra. (Si a la la i-esima le sumamos la j -esima multiplicada por t, escribiremos Fi + tFj ). C1: Reordenar las columnas. C2: Cambiar el signo a todos los elementos de una columna. C3: Sumar a una columna un multiplo entero de otra. (Si a la columna i-esima le sumamos la j -esima multiplicada por t, escribiremos Ci + tCj ). Demostracion. Con la notacion L=S que venimos usando, las operaciones en las las se traducen en manipulaciones de los generadores de S (o sea, de las relaciones en L=S) que no afectan al subgrupo: quitar un generador nulo, reordenar los generadores, sustituir uno por su opuesto, o sumarle a uno un multiplo de otro. Vemos ahora que la operacion C3 no afecta a L ni a S, y dejamos que el lector analice por que son tambien admisibles las operaciones de los tipos C1 y C2. Supongamos, para simplicar, que la operacion es C1 + tC2. Si fa1 : : : ang es la base de L y ponemos a02 = a2 ; ta1 , entonces fa1  a02 : : : ang tambien es base de L. Si la matriz de partida es (kij ) entonces el generador ri es ri = ki1a1 + ki2a2 + ki3a3 + + kinan = ki1a1 + ki2(a02 + ta1 ) + ki3a3 + + kinan = (ki1 + tki2)a1 + ki2a02 + ki3a3 + + kinan  CAPITULO 7. GRUPOS ABELIANOS FINITAMENTE GENERADOS 182 por lo que la matriz obtenida al aplicar C3 representa a los mismos generadores de S, aunque expresados en una base distinta. En conclusion, la operacion C3 no supone ningun cambio en L ni en S. A continuacion describimos un metodo para pasar, mediante operaciones de los tipos anteriores, de una matriz cualquiera con coecientes enteros a una matriz del tipo (7.6.4) en la que d1 j d2 j j dm . Cada vez que hablemos de \la matriz K" nos estaremos reriendo a la ultima matriz obtenida a partir de la inicial mediante las operaciones que se hayan descrito. Comencemos notando el siguiente hecho: Sea a una entrada no nula de K con el menor valor absoluto (podemos suponer que a > 0, cambiando si hace falta el signo de su la), y supongamos que a no divide a todas las entradas de K. Entonces podemos transformar K hasta hacer aparecer una entrada r con 0 < r < a. Para ello, comenzamos haciendo operaciones F1 y C1 para poner a en la entrada (1 1) este paso lo damos solo por comodidad en la notacion. Supongamos que a no divide a cierta entrada de la primera la, digamos k1j (con j 6= 1). Dividiendo con resto, encontramos q r 2 con 0 < r < a y k1j = aq + r. Entonces la operacion Cj ; qC1 nos da una matriz con r en la entrada (1 j), como queramos. Si a no divide a una entrada de la primera columna procedemos de modo analogo, operando esta vez por las. Podemos pues suponer que a divide a todas las entradas de la primera la y a todas las de la primera columna, pero no divide a cierto kij con i j 6= 1. Por hipotesis, existen enteros b y c tales que k1j = ba y ki1 = ca. Haciendo primero la operacion Fi ; cF1 (para poner un 0 en la entrada (i 1)) y despues la operacion F1 ; Fi, obtenemos una matriz con a en la entrada (1 1) y ba + bca ; kij en la entrada (1 j). Como a no divide a esta entrada de la primera la, procedemos como al principio de este parrafo para obtener una entrada r con 0 < r < a. Como el valor absoluto no puede bajar indenidamente, repitiendo el proceso anterior llegara un momento en el que K tendra una entrada a > 0 que dividira al resto de entradas de K, y podemos llevar a hasta el lugar (1 1). Ponemos entonces ceros en el resto de los lugares (i 1) de la primera columna: Tomamos q 2 tal que ki1 = qa y hacemos la operacion Fi ; qF1. Hecho esto, podemos cambiar todas las entradas de la primera la, excepto la (1 1), por ceros (>por que?). Hemos llegado pues a una matriz de la forma 0a 0 1 0 B 0 b22 b2n C B C B . . @ .. .. . . . ... CA 0 bm2 bmn en la que a es positivo y divide a cada bij . A partir de ahora no haremos operaciones con la primera la ni con la primera columna, por lo que ni a ni los ceros de esos lugares van a variar. De hecho, podemos eliminar esa la y esa columna de la matriz y \apuntar" el valor de a (incluso olvidarlo, si a = 1). Ademas, las operaciones que podemos hacer no van a cambiar el hecho de que todas las entradas que se obtengan sean multiplos de a (>por que?). Pues bien, procediendo con la submatriz (bij ) como se acaba de describir, podremos llegar a una matriz del tipo 0a 0 0 1 0 BB0 b 0 0 C C BB0 0 c33 c3n C B@ .. .. .. . . . .. CCA . . . . 0 0 cm3 cmn en la que a divide a b y b divide a los cij . Continuando de este modo, y eliminando las las de ceros que puedan aparecer, conseguiremos la matriz que buscamos. Por supuesto, el proceso se puede simplicar por procedimientos heursticos. Z Z Ejemplos 7.6.5 Transformaciones en la matriz de generadores y relaciones. 1. Sea A el grupo abeliano con generadores a b c d e f y relaciones: 4a + 13b + 3e + f 5a ; 7b + 6c + ; f 3a + 3b + 3d + 3a + 6b + 3e a + 7b + f = = = = = 0 0 0 0 0 7.6. PRESENTACIONES POR GENERADORES Y RELACIONES La matriz asociada a dicho grupo es 0 BB BB @ 183 1 4 13 0 0 3 1 5 ;7 6 0 0 ;1 C C 3 3 0 3 0 0C C 3 6 0 0 3 0A 1 7 0 0 0 1 En este ejemplo veremos que no es necesario seguir estrictamente los pasos descritos anteriormente. Por ejemplo, el papel que antes ha representado la entrada de arriba a la izquierda lo asumira ahora la entrada de abajo a la derecha. Observamos que la primera la es combinacion lineal de las dos ultimas. Luego, restando a la primera la suma de las dos ultimas (lo que representaremos por F1 ; F4 ; F5) obtenemos: 00 0 0 B 5 ;7 6 B B 3 3 0 B @3 6 0 1 0 0 3 0 7 0 0 0 0 0 ;1 0 0 3 0 0 1 1 CC CC : A Eliminando la primera la, y haciendo sucesivamente F1 +F4 (con la nueva numeracion), C1 ; C6 y C2 ; 7C6, obtenemos 06 0 6 0 0 01 06 0 6 0 0 01 B BB 3 3 0 3 0 0 CC : 3 3 0 3 0 0C B C  @3 6 0 0 3 0A @3 6 0 0 3 0A 1 7 0 0 0 1 0 0 0 0 0 1 Podemos eliminar la ultima la y ultima columna, y haciendo entonces (con la nueva numeracion) C1 ; C3 ; C4 ; C5 y C2 ; C4 ; 2C5, obtenemos por n la matriz 00 0 6 0 01 @ 0 0 0 3 0 A 0 0 0 0 3 de la que se deduce que A  =   6  3  3 es la descomposicion invariante de A. A partir de esta podemos obtener la descomposicion indescomponible   2  3  3  3. 2. Sea A un grupo abeliano con matriz de generadores y relaciones dada por 0 0 12 24 0 1 @ 4 10 12 6 A 4 8 0 4 Es claro que, por mas que operemos, las entradas van a ser siempre pares. Por otra parte, no es difcil conseguir que una sea 2, por ejemplo, haciendo F2 ; F3 (escriba el lector las matrices que se van obteniendo). De los dos \2" que aparecen, el mas comodo es de la ultima columna. Podemos poner ceros en el resto de esa columna haciendo F3 ; 2F2, siguiendo el metodo descrito, pero es mas facil hacer C4 ; C1. Ahora podemos poner ceros en la segunda la, haciendo C2 ; C4 y C3 ; 6C4. Pasando entonces la primera la al ultimo lugar, y pasando despues la ultima columna al primer lugar, habremos puesto el 2 en la entrada (1 1). Haciendo entonces, sucesivamente, C3 ; 2C2 y C4 ; 2C3 , se obtiene por n 02 0 0 01 @ 0 4 0 0 A 0 0 12 0 por lo que A  = 2  4  12  . ZZZ Z Z ZZZ Z ZZZ Z Z Z CAPITULO 7. GRUPOS ABELIANOS FINITAMENTE GENERADOS 184 7.7 Problemas 1. Sea f : A ! B un isomorsmo entre dos grupos abelianos. Demostrar (hasta aburrirse) que: (a) La familia fAi gi2I de subgrupos de A es independiente si y solo si la familia ff(Ai )gi2I de subgrupos de B es independiente. (b) Un subgrupo C de A es un sumando directo de A si y solo si el subgrupo f(C) es un sumando directo de B. (c) A es indescomponible si y solo si B es indescomponible. (d) La familia fai gi2I de elementos de A es linealmente independiente si y solo si la familia ff(ai )gi2I de elementos de B es linealmente independiente. (e) A es libre si y solo si B es libre. (f) Si T es el subgrupo de torsion de A entonces f(T ) es el subgrupo de torsion de B. (g) A es de torsion si y solo si B es de torsion. (h) A es libre de torsion si y solo si B es libre de torsion. 2. Sea A un grupo abeliano libre de rango n. Decidir sobre la verdad o falsedad de las siguientes armaciones: (a) Todo subconjunto linealmente independiente de A tiene a lo sumo n elementos. (b) Todo subconjunto generador de A tiene al menos n elementos. (c) Todo subconjunto linealmente independiente de A con n elementos es una base de A. (d) Todo subconjunto generador de A con n elementos es una base de A. 3. Sean L M N grupos abelianos nitamente generados con L  N  = M  N. Demostrar que L = M. 4. Determinar el subgrupo de torsion de los siguientes grupos aditivos: , n,  n, = , = . p 5. Determinar el subgrupo de torsion del grupo de las unidades de los anillos , , , i] y  2]. 6. Sea P un polinomio monico de X] de grado n. Demostrar que el grupo aditivo de X]=(P) es libre de rango n. >Puede fallar el resultado si P no es monico? 7. Probar que si A es un grupo abeliano libre y n 2 , entonces A tiene un subgrupo de ndice n. 8. Encontrar un subgrupo B de A = 16 tal que A=B  = 2  2. 9. Sea A el subconjunto de  formado por las parejas de numeros enteros (a b) tales que a  b mod 10. Demostrar que A es un subgrupo de  y determinar una base de A. 10. Para un grupo abeliano arbitrario A, demostrar que la familia ftp (A)g, donde p recorre el conjunto de todos los enteros primos positivos, es independiente, y que su suma directa es t(A). 11. Demostrar que tp ( = ) es el subgrupo p del Ejemplo 7.5.17, y que = = p p , donde p recorre el conjunto de todos los enteros primos positivos. 12. Demostrar que el grupo aditivo (A +) de un anillo A es de torsion precisamente si la caracterstica de A es diferente de 0. Si A es un dominio, demostrar que las condiciones son equivalentes a que (A +) no sea libre de torsion. El anillo X]=(2X) muestra que, en la segunda parte, la hipotesis de que A sea un dominio no es superua, >por que? 13. Encontrar bases para los siguientes subgrupos de grupos abelianos libres: (a) h3a 4b 6a + 2bi, siendo a b generadores de un grupo abeliano libre de rango 2. (b) hx + 2y + 4z 3x + 6y + 12z ;12x ; 24y ; 48z ;2x + y + 7z i, siendo x y z generadores de un grupo libre de rango 3. Z NZZ ZZ Z ZZ QZ Z Z 1 Z Z ZZR ZC ZQ ZZR Z Z QZ Z 1 7.7. PROBLEMAS 185 14. Demostrar que el grupo aditivo de los numeros racionales es indescomponible. 15. Calcular las descomposiciones primaria e invariante de los siguientes grupos abelianos: (a) 20  40  108 (b) 21. (c) ha b j 3a + 6b = 9a + 24b = 0i. (d) ha b c j 2a + b = 3a + c = 0i. (e) ha b c j ;4a + 2b + 6c = ;6a + 2b + 6c = 7a + 4b + 15c = 0i. (f) ha b c j a + 2b + 4c = 3a + 6b + 12c = ;2a + b + 7c = 0i. 16. Clasicar el grupo abeliano presentado por los generadores y relaciones dados: (a) Generadores a b c y relaciones 7a + 8b + 9c = 0 4a + 5b + 6c = 0 a + 2b + 3c = 0 ZZ Z Z (b) Generadores a b c d e y relaciones a ; 7b ; 21c + 14d = 0 5a ; 7b ; 2c + 10d ; 15e = 0 3a ; 3b ; 2c + 6d ; 9e = 0 a ; b + 2d ; 3e = 0 17. Encontrar todos los grupos abelianos, salvo isomorsmos, de ordenes 30, 60, 72, 90, 180, 360, 720 y 1830, calculando para cada uno de ellos las descomposiciones primaria e invariante. 18. Determinar salvo isomorsmos todos los grupos abelianos de orden 30 y dar la lista de sus divisores elementales y factores invariantes. 19. Demostrar que la lista de factores invariantes de n  m es (nm) o (mcm(n m) mcd(n m)). 20. Demostrar que si A es un p-grupo abeliano que es la suma directa de n grupos cclicos no nulos, entonces la ecuacion px = 0 tiene exactamente pn soluciones. 21. Sea G un p-grupo abeliano nito en el que la ecuacion px = 0 tiene a lo sumo p soluciones. Demostrar que G es cclico. Demostrar que tambien es cclico un grupo abeliano nito (no necesariamente un p-grupo) en el que la ecuacion px = 0 tenga a lo sumo p soluciones para todo primo p. 22. Resolver el Problema 57 del Captulo 5 usando los resultados de este captulo. 23. Del Ejercicio 7.5.15 se deduce que, si G es un grupo abeliano nito y p es un divisor primo de jGj, entonces G contiene un elemento de orden p. Demostrar que este resultado sigue siendo valido si G no es abeliano. E ste es el Teorema de Cauchy que demostraremos en el captulo siguiente con otros metodos. (Indicacion: Usar el Ejercicio 7.5.15, la Ecuacion de Clases e induccion en jGj.) 24. Sea p primo. Demostrar que si G es un grupo abeliano nito en el que todo elemento no nulo tiene orden p, entonces G  = np para algun n. 25. Demostrar que todo grupo abeliano nito no cclico contiene un subgrupo isomorfo a p  p para algun primo p. 26. Sea B un subgrupo de un grupo abeliano nito A. Demostrar que A contiene un subgrupo isomorfo a A=B. >Es cierto el resultado si A es innito? 27. Sea A un grupo abeliano nito y sea a un elemento de A orden maximo. Demostrar que hai es un sumando directo de A. ZZ Z ZZ 186 CAPITULO 7. GRUPOS ABELIANOS FINITAMENTE GENERADOS 28. Se pide: (a) Encontrar un numero natural n tal que existan exactamente 5 grupos abelianos de orden n salvo isomorsmos. (b) *] Encontrar todos los numeros naturales n tales que, salvo isomorsmos, existen exactamente 6 grupos abelianos de orden n. (c) *] Demostrar que para ningun numero natural n hay exactamente 13 grupos abelianos de orden n salvo isomorsmos. 29. *] Un subgrupo propio H de un grupo G que no esta contenido en ningun otro subgrupo propio de G se dice que es maximal (en G). Para un grupo abeliano nito A, demostrar: (a) Los subgrupos maximales son precisamente los de ndice primo. (b) A tiene exactamente 2 subgrupos maximales si y solo si A es cclico y jAj tiene exactamente dos divisores primos. P 30. Sea G el grupo abeliano denido por los generadores g1 : : :  gn y las relaciones nj=1 aij gi = 0 (i = 1 : : :  n), con aij 2 . Demostrar que G es nito precisamente si el determinante de la matriz (aij ) es diferente de 0 y que, en tal caso, el orden de G coincide con el valor absoluto de dicho determinante. Z Bibliografa del captulo Artin 4], Delgado-Fuertes-Xambo 11], Dorronsoro-Hernandez 13], Herstein 20], Jacobson 23], Rotman 30]. Captulo 8 Estructura de los grupos nitos Se estudian los grupos nitos usando las acciones de grupos en conjuntos como principal herramienta. Se demuestran los Teoremas de Sylow y, como consecuencia de estos resultados y otras tecnicas auxiliares, se clasican todos los grupos con 15 o menos elementos. Introduccion La nocion de accion de un grupo en un conjunto, cuya denicion abre este captulo, es una de las mas importantes en Teora de Grupos. De hecho, historicamente, las acciones fueron usadas antes incluso de la formalizacion del concepto de grupo. No es exagerado armar que, en gran medida, los grupos son importantes en Matematicas porque actuan, de formas muy variadas y casi siempre muy naturales, sobre otros conjuntos. Los conceptos de orbita y estabilizador de un elemento surgen tambien de modo natural y se relacionan de modo que, en el caso de grupos nitos, son muy utiles para hacer argumentos de recuento. En la parte central del captulo usamos tecnicas basadas en acciones para demostrar los Teoremas de Cauchy y Sylow, que constituyen recprocos parciales del Teorema de Lagrange. E ste arma que el orden de cualquier subgrupo de un grupo nito G debe dividir a jGj, pero sabemos que pueden existir divisores d de jGj tales que G no posea ningun subgrupo de orden d. Podemos preguntarnos si podra imponerse una condicion extra sobre el divisor d de jGj que garantice la existencia de subgrupos de orden d en G. Un primer paso en esta direccion lo proporciona el Teorema de Cauchy, que responde armativamente a la pregunta cuando d = p es primo. Esto permite deducir diversas propiedades de los p-grupos (grupos cuyo orden es una potencia del primo p), en particular el hecho de que verican el \recproco del Teorema de Lagrange". El Primer Teorema de Sylow va mas alla, y responde armativamente a la pregunta cuando d = pn es cualquier potencia de primo que divide a jGj. Los otros dos Teoremas de Sylow tratan sobre los subgrupos de orden pn en los que el exponente n es lo mayor posible, llamados p-subgrupos de Sylow de G. El segundo arma que todos ellos son conjugados entre s en particular, si G posee un unico p-subgrupo de Sylow (para cierto primo p), este es normal. El tercero nos da informacion, muy util en la practica, sobre el numero de p-subgrupos de Sylow de G, y de el se deduce un criterio de no simplicidad que nos permite demostrar que ningun grupo simple no abeliano tiene menos de 60 elementos (recuerdese que el grupo alternado A5 es simple y tiene 60 elementos). El nal del captulo lo dedicamos a obtener resultados sobre la estructura de los grupos de orden bajo, para lo que necesitaremos usar informacion obtenida a lo largo de todos los captulos sobre grupos y dos ultimos recursos tecnicos: Uno de ellos es la descripcion de ciertas situaciones en las que un grupo es isomorfo a un producto directo de dos de sus subgrupos, y otro es el concepto de producto semidirecto de grupos. Con estos recursos, clasicamos salvo isomorsmos todos los grupos de orden menor o igual que 15. Estos resultados muestran que, en general, la clasicacion de los grupos nitos es muy laboriosa y requiere de toda la teora de la que se pueda disponer. Clasicar grupos es un tema todava abierto a la investigacion y esta muy lejos de resolverse. 187 CAPITULO 8. ESTRUCTURA DE LOS GRUPOS FINITOS 188 Objetivos del captulo Conocer el concepto de accion de un grupo sobre un conjunto, as como los de orbita y estabilizador y sus propiedades basicas. Conocer las acciones por traslaciones, por conjugacion y por conjugacion en subgrupos, y sus consecuencias teoricas. Conocer el Teorema de Cauchy y sus aplicaciones a p-grupos. Conocer los Teoremas de Sylow, y saber emplearlos para analizar la estructura de ciertos grupos en funcion de su orden. Conocer algunos criterios de no simplicidad para grupos. Conocer el concepto de producto semidirecto de grupos. Conocer algunas tecnicas de clasicacion de grupos de orden bajo. Desarrollo de los contenidos 8.1 Accion de un grupo sobre un conjunto De nicion 8.1.1 Sean G un grupo y , un conjunto. Una accion por la izquierda de G en , (o sobre ,) es una aplicacion G  , ! , tal que, si denotamos la imagen del par (g ) por g, se verica: 1. 1 =  para cada  2 ,. 2. h(g) = (hg) para cualesquiera h g 2 G y  2 ,. La segunda condicion es una \asociatividad formal" que permite escribir hg sin ambig#uedad. Observando en que conjuntos estan los elementos que se operan, no debe haber confusion sobre si se esta usando el producto de G o la accion. Ocasionalmente, en lugar de g se emplean otras notaciones como g  para designar a la imagen de (g ) por la accion que se considere. Las condiciones de la Denicion 8.1.1 son entonces: 1 =  y g (h ) = gh : Dada una accion G  , ! , de un grupo G sobre un conjunto ,, cada g 2 G determina una aplicacion g : , ! , dada por g () = g. Combinando las dos propiedades de la denicion se observa que la aplicacion g 1 es la inversa de g , por lo que esta es una permutacion de ,. Ademas, si g y h son elementos de G, es claro que h  g = hg . Con esto hemos probado: Proposicion 8.1.2 Dada una accion G  , ! , de un grupo G sobre un conjunto ,, la aplicacion : G ! S(,) dada por (g) = g es un homomorsmo de grupos. Llamaremos nucleo de la accion al nucleo de este homomorsmo , y diremos que la accion es el si es inyectivo. La proposicion anterior admite una recproca de demostracion elemental: Proposicion 8.1.3 Dados un conjunto ,, un grupo G y un homomorsmo de grupos : G ! S(,), la aplicacion G  , ! , dada por (g ) 7! (g)() es una accion por la izquierda de G sobre ,. De las dos proposiciones anteriores resulta que las acciones de un grupo G sobre un conjunto , y los homomorsmos G ! S(,) se determinan mutua y biyectivamente, por lo que pueden ser considerados como conceptos equivalentes. Todo lo expuesto hasta aqu puede traducirse sin dicultad para las acciones \por la derecha". Una accion por la derecha de un grupo G sobre un conjunto , es una aplicacion ,  G ! ,, en la que la imagen del par ( g) se escribe g, que verica 1 =  y (g)h = gh ;  8.2. ORBITAS Y ESTABILIZADORES 189 para cualesquiera  2 , y g h 2 G. A menudo, la imagen del par ( g) se denota por g , y entonces las condiciones anteriores se traducen en 1 =  y (g )h = gh : Como antes, se demuestra que dar una accion por la derecha de G sobre , es equivalente a dar un antihomomorsmo de G en S(,) es decir, una aplicacion : G ! S(,) que verica (gh) = (h) (g) cuando g h 2 G. Deniendo el nucleo de un antihomomorsmo del modo obvio, podemos denir el nucleo de una accion por la derecha y el concepto de accion por la derecha el. En realidad, la diferencia entre acciones por la izquierda y por la derecha es meramente formal. En efecto, si  : G ! G es la aplicacion dada por g 7! g;1 , entonces la asignacion 7!   dene una biyeccion entre el conjunto de los homomorsmos de G en S(,) y el conjunto de los antihomomorsmos de G en S(,) (la biyeccion inversa tambien se obtiene componiendo con ). Por lo dicho anteriormente, esta biyeccion se traduce en una biyeccion entre las acciones por la izquierda de G sobre , y las correspondientes acciones por la derecha. Explcitamente, esta biyeccion lleva una accion por la izquierda dada por (g ) 7! g a la accion por la derecha dada por ( g) 7! g;1 . Cada resultado o ejemplo, por su naturaleza (o en muchos casos para respetar costumbres notacionales), sugiere el uso de acciones por uno u otro lado. Por tanto, es conveniente usar ambos conceptos, y eso haremos en lo que sigue. NN N Ejemplos 8.1.4 Acciones. N 1. Para un numero natural n, la aplicacion Sn  n ! n dada por i = (i) es una accion por la izquierda del grupo simetrico Sn en el conjunto n, y el homomorsmo asociado Sn ! S( n) es la identidad. Mas generalmente, si , es un conjunto arbitrario y G es un subgrupo del grupo simetrico S(,), entonces la aplicacion ( x) 7! (x) es una accion por la izquierda el de G sobre ,. 2. Sea G un grupo y sea , = G. La aplicacion (g x) 7! gx es una accion de G sobre s mismo. Se denomina accion de G sobre s mismo por traslaciones por la izquierda, y es claramente el. >Que relacion guarda este ejemplo con el Teorema de Cayley 5.7.4? Analogamente, la aplicacion dada por (x g) 7! xg es una accion el por la derecha de G sobre s mismo por traslaciones por la derecha. 3. Sean G un grupo, H un subgrupo de G y , el conjunto G=H de las clases laterales por la izquierda modulo H. La aplicacion (g xH) 7! gxH es una accion por la izquierda de G sobre ,, que tambien se llama accion por traslaciones por la izquierda de G en G=H. Cuando H = 1 tenemos la accion del apartado anterior. El lector podra describir una accion por traslaciones por la derecha de G sobre el conjunto H nG de las clases laterales por la derecha de G modulo H. 4. Dado un grupo G, la aplicacion (a x) 7! ax = x;1 ax es una accion por la derecha de G sobre s mismo. Mas generalmente, si H y N son subgrupos de G y N es normal en G, la conjugacion induce una accion por la derecha de H sobre N que se denomina accion de H sobre N por conjugacion. >Cuando es el esta accion? La accion por la izquierda de H sobre N asociada a la anterior viene dada por (x a) 7! x a = xax;1 (conjugacion por x;1). En este caso, la imagen del homomorsmo H ! S(N) esta contenida en Aut(N). 8.2 O rbitas y estabilizadores De nicion 8.2.1 Sea G un grupo que actua por la izquierda sobre un conjunto ,, y sean  2 ,. Se dice que  y son equivalentes para la accion cuando existe g 2 G tal que g = . Se propone como ejercicio la comprobacion de que la anterior es una relacion de equivalencia en ,. Las correspondientes clases de equivalencia reciben el nombre de orbitas. As, la orbita de  es el conjunto G = fg : g 2 Gg: CAPITULO 8. ESTRUCTURA DE LOS GRUPOS FINITOS 190 El estabilizador de  en G es el conjunto EstabG () = E() = fg 2 G : g = g (escribiremos E() cuando no haya riesgo de confusion en cuanto a la accion considerada). Es claro que la orbita de  es unitaria es decir, G = fg, precisamente si EstabG () = G, pues ambas condiciones equivalen a que se tenga g =  para cada g 2 G. Cuando esto ocurre decimos que  es un punto jo para la accion. Cuando todos los elementos estan en la misma orbita se dice que la accion es transitiva, o que G actua transitivamente en ,. Si usamos la notacion g , o si trabajamos con acciones por la derecha, haremos los cambios pertinentes en la notacion. Por ejemplo, las orbitas se denotaran por G , G o G . Ejercicio 8.2.2 Dada una accion de G sobre ,, demostrar que el estabilizador de cada elemento de , es un subgrupo de G, y que la interseccion de todos los estabilizadores es el nucleo de la accion.  Ejemplos 8.2.3 Orbitas y estabilizadores. N N N 1. Sea G el subgrupo cclico de S6 generado por la permutacion  = (1 2 3)(4 5), y consideremos la accion G  6 ! 6 denida en los Ejemplos 8.1.4. Entonces las orbitas son f1 2 3g, f4 5g y f6g, el unico punto jo de 6 es el 6, y los estabilizadores son E(1) = E(2) = E(3) = f1 3g E(4) = E(5) = f1 2 4 g E(6) = G: N N 2. La accion natural del subgrupo G = f1 (1 2)(3 4) (1 3)(2 4) (1 4)(2 3)g de S4 sobre 4 es transitiva, y cada elemento de 4 tiene estabilizador trivial. 3. La accion de un grupo G sobre s mismo por traslaciones por la izquierda (o por la derecha) es transitiva, pues cada ecuacion aX = b tiene solucion unica en G. Tambien lo es la accion de G en G=H por traslaciones por la izquierda. Como las orbitas son clases de equivalencia, forman una particion de ,. Cuando este es nito se tiene j,j = X 2 jGj (8.2.1) 0 donde ,0 es un conjunto de representantes de las orbitas (es decir, en ,0 hay exactamente un elemento  de cada orbita). Esta igualdad se conoce con el nombre de Ecuacion de Orbitas . Ejercicio 8.2.4 Se considera la accion (por la derecha) de un grupo G sobre s mismo por conjugacion: 1. Comprobar que las orbitas son las clases de conjugacion, que el estabilizador de un elemento g es su centralizador CenG (g), que los puntos jos de la accion son los elementos de Z(G), que la accion es el precisamente si Z(G) = f1g, y que la accion es transitiva precisamente si G = f1g. 2. Interpretar la Ecuacion de Clases (Proposicion 5.9.5) como un caso particular de la Ecuacion de  Orbitas (8.2.1). Proposicion 8.2.5 Sea G un grupo que actua por la izquierda sobre un conjunto , y sea  2 ,. Entonces: 1. G : E()] = jGj. 2. Si la accion es transitiva entonces E() es el grupo trivial para cualquier  2 ,. 3. Si G es nito se tiene jGj = jGj jE()j por tanto, el cardinal de cualquier orbita divide a jGj. 4. E(g) = g E() g;1 (para cada g 2 G).  8.2. ORBITAS Y ESTABILIZADORES 191 Demostracion. Para ver 1, notemos que dados g h 2 G se tiene g = h , h;1 g =  , h;1 g 2 E() , gE() = hE(): En consecuencia, la aplicacion E()nG ! G dada por gE() 7! g esta bien denida y es inyectiva. Como es obviamente suprayectiva, tenemos una biyeccion que nos da la igualdad entre cardinales buscada. Los apartados 2 y 3 se siguen del 1 y del Teorema de Lagrange. El 4 se tiene por las equivalencias h 2 E(g) , hg = g , g;1 hg =  , g;1hg 2 E() , h 2 gE()g;1 : En el siguiente ejercicio, dado un subgrupo H de un grupo nito G, describimos el mayor subgrupo de H que es normal en G y damos una condicion bajo la cual el propio H es normal en G (esta condicion generaliza el hecho de que todo subgrupo de ndice 2 es normal). Ejercicio 8.2.6 Sea G un grupo con un subgrupo H , y consideremos la accion por traslaciones por la derecha de G en H nG descrita en los Ejemplos 8.1.4. Demostrar que: 1. La accion es transitiva. 2. EstabG (H) = H , por lo que EstabG(Hg) = H g para cada g 2 G. 3. El nucleo de la accion, que denotaremos por c(H) (se suele llamar el corazon de H ), es la interseccion de todos los conjugados de H en G es decir, c(H) = \ g2G Hg : 4. c(H) es el mayor subgrupo normal de G contenido en H . 5. Si H tiene ndice nito n en G, existe un homomorsmo inyectivo de grupos c(GH ) ! Sn . 6. Si G es nito y G : H] = p, donde p es el menor divisor primo de jGj (esto signica que H tiene el mayor orden posible entre los subgrupos propios de G), entonces H es normal en G. (Indicacion: usar el apartado anterior para probar que H : c(H)] divide a (p ; 1)!). 7. Si jGj = m, G : H] = n > 1 y m 6 j n! entonces c(H) 6= f1g y por tanto G no es simple. A continuacion damos un nuevo ejemplo de accion del que se extraen consecuencias teoricas interesantes: Ejemplo 8.2.7 Accion por conjugacion en subgrupos normalizadores. Sea G un grupo y sea , el conjunto de todos los subgrupos de G. La asignacion (H g) 7! H g = g;1 Hg dene una accion por la derecha de G en , llamada accion por conjugacion en subgrupos. La orbita del subgrupo H esta formada por los subgrupos conjugados de H, y es claro que los subgrupos normales de G son precisamente los puntos jos para esta accion es decir, los que tienen estabilizador G. Este hecho se generaliza en el ejercicio que sigue. Ejercicio 8.2.8 Sea H un subgrupo de un grupo G, y consideremos el conjunto NG (H) = fg 2 G : H = H g g que recibe el nombre de normalizador de H en G. Demostrar que: 1. NG (H) es el estabilizador de H para la accion de Ejemplo 8.2.7. 2. NG (H) es el mayor subgrupo de G en el que H es normal. Es decir, H es un subgrupo normal de NG (H) y, si K es un subgrupo de G tal que H es un subgrupo normal de K , entonces K  NG (H). 3. Si NG (H) tiene ndice nito en G, entonces G : NG (H)] coincide con el numero de subgrupos conjugados de H en G. 192 CAPITULO 8. ESTRUCTURA DE LOS GRUPOS FINITOS 8.3 Teorema de Cauchy y p-grupos Sea p un entero primo positivo. Los grupos cuyo orden es una potencia de p han tenido un papel destacado en la descripcion de los grupos abelianos nitos (Seccion 7.4), y tambien aparecieron en la Seccion 5.9. En el caso abeliano, vimos que un grupo G tiene orden potencia de p si y solo si lo mismo le ocurre a todos sus elementos, lo que dio lugar a la denicion de los p-grupos. Por otra parte, hemos visto que no se verica el \recproco del Teorema de Lagrange": Si G es un grupo nito y m es un divisor de jGj, en general no es cierto que G tenga un subgrupo de orden m (por ejemplo, no lo es si G = A4 y m = 6). Sin embargo, este resultado es cierto cuando m es una potencia de primo, como veremos en la seccion siguiente. En esta seccion damos un primer paso en la demostracion de ese resultado. Es el Teorema de Cauchy, que se ocupa del caso en que m es primo y nos permite extender la denicion de los p-grupos al caso no abeliano. En el Problema 23 del Captulo 7 propusimos una demostracion alternativa de este teorema. Teorema 8.3.1 (Cauchy) Sea G un grupo nito. Si p es un divisor primo de jGj, entonces G posee un elemento de orden p. Demostracion. Sea , el conjunto de las p-tuplas de elementos de G cuyo producto es 1: , = f(x1 : : :  xp) : xi 2 G para cada i y x1 xp = 1g: Es claro que una p-tupla (x1 : : :  xp) de elementos de G esta en , si y solo si xp = (x1 x2 xp;1 );1. Por tanto, para dar un elemento de , podemos elegir sus p ; 1 primeras componentes arbitrariamente en G, y la ultima queda entonces determinada por estas. En consecuencia se tiene j,j = jGjp;1, y en particular p divide a j,j. Sea C = hci un grupo cclico de orden p, y consideremos su accion por la izquierda en , dada por (c (x1 : : :  xp)) 7! (xp  x1 x2 : : : xp;1): Los puntos jos para esta accion son los elementos de , de la forma (x x : : : x), que se corresponden con los elementos de G tales que xp = 1. Por tanto, el resultado quedara demostrado si vemos que , contiene algun punto jo distinto del (1 1 : : : 1). Sea pues k 1 el numero de orbitas unitarias (o de puntos jos) para la accion dada, y sea q el numero de orbitas no unitarias. Por la Proposicion 8.2.5, cada orbita no unitaria tiene cardinal p, y as j,j = k +pq. Como p divide a j,j deducimos que p divide a k, y como k 6= 0 deducimos que hay puntos jos diferentes del (1 1 : : : 1), como queramos ver. Como consecuencia de los Teoremas de Lagrange y Cauchy se tiene: Ejercicio 8.3.2 Dados un grupo nito G y un entero positivo primo p, demostrar la equivalencia de las siguientes condiciones: 1. El orden de G es una potencia de p. 2. El orden de cada elemento de G es una potencia de p. De nicion 8.3.3 Un grupo nito que verique las condiciones equivalentes del Ejercicio 8.3.2 se llama un p-grupo. Otra consecuencia notable del Teorema de Cauchy es la siguiente: Proposicion 8.3.4 Los p-grupos verican el recproco del Teorema de Lagrange. Es decir, un p-grupo G posee un subgrupo de orden d para cada divisor d de jGj. Demostracion. Sea jGj = pn con n 1, y procedamos por induccion en n, con el caso n = 1 trivial. En el caso general, el divisor d ha de ser de la forma pm con m n. Por la Proposicion 5.9.7, el centro Z = Z(G) no es trivial, y por tanto contiene un elemento g de orden p. El subgrupo N = hgi es normal en G (>por que?), y el cociente G=N tiene orden pn;1. Por tanto pm;1 divide a jG=N j, y la hipotesis de induccion nos dice que G=N posee un subgrupo X de orden pm;1 . Por el Teorema de la Correspondencia, existe un subgrupo H de G que contiene a N y tal que X = H=N, y entonces se tiene jH j = jX j jN j = pm;1 p = pm , luego H es el subgrupo que buscabamos. 8.4. LOS TEOREMAS DE SYLOW 193 8.4 Los Teoremas de Sylow Acabamos de ver que los p-grupos verican el recproco del Teorema de Lagrange. En este sentido, el Primer Teorema de Sylow constituira un \recproco parcial" del Teorema de Lagrange: armara que, dado un grupo nito jGj, para cada divisor d de jGj que sea potencia de un primo, existe un subgrupo de G de orden d. Observese que estos subgrupos son precisamente los subgrupos de G que son p-grupos para algun primo p (les llamaremos p-subgrupos de G). El resto de los Teoremas de Sylow profundizaran mas en el estudio de los subgrupos de este tipo. Particularmente importantes seran los p-subgrupos de G que tengan el mayor orden posible, y esto nos llevara a menudo a la siguiente situacion: Si p es un divisor primo de jGj, podemos poner jGj = pnr con n 1 y p 6 j r es decir, pn es la mayor potencia de p que divide a jGj, y entonces pn es el mayor orden que puede tener un p-subgrupo de G. De nicion 8.4.1 Sea G un grupo nito y sea p un entero primo positivo. Un subgrupo de G que sea un p-grupo se llama un p-subgrupo de G. Por el Teorema de Lagrange, G solo puede poseer p-subgrupos no triviales si p divide a jGj. Si jGj = pn r con p 6 j r, un subgrupo de G de orden pn (es decir, un p-subgrupo del mayor orden posible) se llama un p-subgrupo de Sylow de G. Para grupos abelianos nitos, los subgrupos de Sylow han aparecido con otro nombre: Ejercicio 8.4.2 Sea G un grupo abeliano nito y sea p un divisor primo de jGj. Demostrar que tp(G) es el unico p-subgrupo de Sylow de G. El Primer Teorema de Sylow Teorema 8.4.3 (Primer Teorema de Sylow) Sea G un grupo nito y sea p un divisor primo de jGj. Entonces G contiene algun p-subgrupo de Sylow. Demostracion. Sea m = jGj y pongamos m = pn r con p 6 j r. Se trata de demostrar que G tiene un subgrupo de orden pn, y lo hacemos por induccion sobre m. Como p divide a m, el menor caso posible es m = p y el resultado es entonces trivial. As, supondremos que m > p y que el enunciado del teorema es valido para cualquier grupo de orden menor que m. Distinguiremos dos casos: Si existe g 2 G n Z(G) tal que p no divide a G : CenG (g)] entonces, por el Teorema de Lagrange y el Teorema Fundamental de la Aritmetica, pn divide a jCenG (g)j. Como este es un subgrupo propio de G, la hipotesis de induccion implica que CenG (g) posee un subgrupo de orden pn , y por tanto G tambien. En otro caso, se tiene que p divide a G : CenG (g)] para cada g 2 G n Z(G). Aplicando la Ecuacion de Clases (Proposicion 5.9.5) se deduce que p ha de dividir a Z(G). Por el Teorema de Cauchy (8.3.1), Z(G) contiene un subgrupo N de orden p, que sera normal en G. Como jG=N j = m=p < m y pn;1 es la mayor potencia de p que divide a jG=N j, la hipotesis de induccion nos dice que existe un subgrupo X de G=N tal que jX j = pn;1. Por el Teorema de la Correspondencia (5.6.5) se tiene X = H=N para algun subgrupo H de G que contiene a N. Entonces jH j = jN j jX j = pn, y por tanto H es el subgrupo que buscabamos. Aplicando ahora la Proposicion 8.3.4 a un subgrupo de Sylow de G se tiene: Corolario 8.4.4 Un grupo nito posee p-subgrupos de todos los ordenes posibles. Es decir, si G es un grupo nito y k es un entero tal que pk divide a jGj, entonces G contiene algun subgrupo de orden pk . Estos resultados nos dan informacion sobre la existencia de ciertos subgrupos de los grupos nitos. Por ejemplo, sea G un grupo del que sabemos que jGj = 28:600 = 23 52 11 13. Entonces G tiene subgrupos de cualquiera de los ordenes 2, 4, 8, 5, 25, 11 o 13. >Que pasa con los subgrupos de otros ordenes? Los resultados anteriores no arman nada sobre su existencia, y de hecho no es posible hacer ninguna armacion general en ese sentido. Por ejemplo, si jGj = 12, lo anterior nos dice que G tiene subgrupos de ordenes 2, 3 y 4 (y por supuesto 1 y 12) en cuanto a los subgrupos de orden 6, existen para el grupo cclico G = 12 o para el grupo diedrico D6 , pero no existen para el grupo alternado G = A4 (Ejemplo 6.2.8). Z CAPITULO 8. ESTRUCTURA DE LOS GRUPOS FINITOS 194 El Segundo Teorema de Sylow A lo largo de este parrafo suponemos que jGj = pnr con p 6 j r. Por el Primer Teorema (8.4.3) podemos jar un p-subgrupo de Sylow P de G es decir, un subgrupo tal que jP j = pn. Denotaremos por SP el conjunto de todos los subgrupos de G que son conjugados de P es decir, todos los subgrupos de la forma P x = x;1 Px, para algun x 2 G. Como \conjugar por x" es un automorsmo de G, todos los subgrupos de SP tienen el mismo cardinal que P y, en consecuencia, son p-subgrupos de Sylow de G. Una parte del Segundo Teorema de Sylow arma que, recprocamente, todo p-subgrupo de Sylow de G esta en SP (es decir, es conjugado de P). Para demostrar esto necesitamos un par de lemas previos. Lema 8.4.5 Con la notacion anterior, jSP j es un divisor de r (es decir, jSP j es un divisor de jGj que no es multiplo de p). Demostracion. Consideraremos la accion por la derecha por conjugacion de G sobre el conjunto , de todos los subgrupos de G, y usaremos la notacion y los resultados del Ejemplo 8.2.7 y del Ejercicio 8.2.8. Entonces SP es la orbita de P , y N = NG (P) es un subgrupo de G que contiene a P y verica jSP j = G : N], de donde jGj = jN j jSP j. De aqu se deduce inmediatamente que jSP j divide a jGj. Ademas, como pn = jP j divide a jN j (pues P  N), p no puede dividir a jSP j pues de lo contrario pn+1 dividira a jGj, lo cual es imposible por la eleccion de n. El lema que sigue contiene esencialmente la demostracion del Segundo Teorema de Sylow, pero lo enunciamos separadamente porque sus apartados seran usados tambien en el Tercer Teorema. Lema 8.4.6 Sea H un p-subgrupo de G. Entonces: 1. La aplicacion SP  H ! SP dada por (Q x) 7! Qx es una accion por la derecha de H en SP . 2. Esta accion tiene puntos jos. 3. Un elemento Q de SP es un punto jo para la accion si y solo si H  Q. Demostracion. El apartado 1 es elemental y se deja como ejercicio.  2. Apliquemos la Ecuacion de Orbitas (8.2.1) a esta accion: tenemos jSP j = X jQH j = X H : EstabH (Q)] donde Q recorre un conjunto de representantes de las orbitas. Por el Lema 8.4.5, p no divide a jSP j y en consecuencia p no divide a H : EstabH (Q)] para cierto Q 2 SP . Como H es un p-grupo, ese ndice debe valer 1 y as EstabH (Q) = H es decir, Q es un punto jo. 3. Si x 2 Q entonces Qx = Q, por lo que los elementos de SP que contienen a H son puntos jos para la accion. Recprocamente, supongamos que Q es un punto jo de SP para esa accion. La hipotesis nos dice que xQ = Qx para cada x 2 H, por lo que HQ = QH y as HQ es un subgrupo de G que j jQj contiene a Q y a H (Ejercicio 5.4.10). Ademas, como H y Q son p-grupos y se tiene jHQj = jjH H \ Qj (Lema 5.4.11), deducimos que HQ es un p-subgrupo de G. Como Q es maximal entre los p-subgrupos de G, entonces HQ = Q y por tanto H  Q. Podemos ya demostrar el Segundo Teorema de Sylow. Teorema 8.4.7 (Segundo Teorema de Sylow) Sean G un grupo nito, p un divisor primo de jGj, P un p-subgrupo de Sylow de G y H un p-subgrupo de G. Entonces H esta contenido en algun subgrupo conjugado de P . En particular, todos los p-subgrupos de Sylow de G son conjugados entre s. Demostracion. La accion del Lema 8.4.6 tiene al menos un punto jo Q, y este es un conjugado de P que contiene a H. Si, ademas, H es un p-subgrupo de Sylow de G, entonces la inclusion H  Q es una igualdad (>por que?), y en consecuencia H es un conjugado de P. El siguiente corolario se usa a menudo para contar elementos de un grupo. 8.4. LOS TEOREMAS DE SYLOW 195 Corolario 8.4.8 Sea G un grupo nito y sea p un divisor primo de jGj. La union de todos los psubgrupos de Sylow de G coincide con el conjunto de todos los elementos de G cuyo orden es una potencia de p. Demostracion. Es claro que el orden de cualquier elemento de esa union es una potencia de p (incluyendo al neutro, cuyo orden es p0). Recprocamente, si o(x) = pk entonces x esta en el p-subgrupo hxi, y por tanto esta en algun p-subgrupo de Sylow por el Segundo Teorema. El Tercer Teorema de Sylow Una consecuencia inmediata del Segundo Teorema de Sylow es el hecho de que SP es el conjunto de todos los p-subgrupos de Sylow de G, y tambien el conjunto de los conjugados de cualquier p-subgrupo de Sylow de G (no solo de un p-subgrupo de Sylow prejado). En particular, si denotamos por np al numero de p-subgrupos de Sylow de G, se tiene np = jSP j: El Tercer Teorema de Sylow establece ciertas condiciones que debe cumplir el numero np recien denido. Teorema 8.4.9 (Tercer Teorema de Sylow) Sea G un grupo nito, sea p un divisor primo de jGj y sea jGj = pn r con p 6 j r. Entonces el numero np de p-subgrupos de Sylow de G verica: 1. np divide a r. 2. np  1 mod p. 3. np = 1 si y solo si algun p-subgrupo de Sylow de G es normal en G. Demostracion. 1. Es consecuencia del Lema 8.4.5 y de la igualdad np = jSP j. 2. Sea H un p-subgrupo de Sylow de G la accion del Lema 8.4.6 tiene al menos un punto jo, y de hecho no tiene mas porque ahora, para un elemento Q de SP , contener a H equivale a coincidir con H (por la igualdad de cardinales). Por tanto, en la Ecuacion de O rbitas (8.2.1) jSP j = X jQH j = X H : EstabH (Q)] exactamente uno de los ndices H : EstabH (Q)] vale 1, y cada uno de los otros es multiplo de p por ser un divisor de jH j distinto de 1. En consecuencia, np  1 mod p. 3. Puesto que SP es el conjunto de todos los conjugados de cualquier p-subgrupo de Sylow de G, el resultado es una consecuencia directa del hecho de que un subgrupo es normal si y solo si coincide con todos sus conjugados (Ejercicio 5.9.3). Criterios de no simplicidad En este parrafo usaremos los Teoremas de Sylow para obtener un criterio que nos permite armar que ciertos grupos no son simples. Este resultado, junto con otros del mismo tipo que se han visto antes, nos permitira demostrar que no hay grupos simples no abelianos con menos de 60 elementos. Proposicion 8.4.10 Sea G un grupo nito con jGj = pq o jGj = pq2, donde p q son dos primos distintos. Entonces se tiene np = 1 o nq = 1, y en consecuencia G no es un grupo simple. Demostracion. Usaremos en varias ocasiones el Tercer Teorema de Sylow (8.4.9). Si jGj = pq podemos asumir que p < q, luego p 6 1 mod q y en consecuencia nq 6= p, por lo que nq = 1. Suponemos pues que jGj = pq2 , y distinguimos tres casos: Si p < q entonces p 6 1 mod q, y por tanto nq = 1. Si p > q2 (> q) entonces q q2 6 1 mod p y en consecuencia np 6= q q2, por lo que np = 1. Por ultimo, si q < p < q2 entonces q 6 1 mod p y en consecuencia np 6= q. Por tanto o bien np = 1, y hemos terminado, o bien np = q2 , y basta ver que esta ultima opcion implica que nq = 1. Supongamos 196 CAPITULO 8. ESTRUCTURA DE LOS GRUPOS FINITOS pues que np = q2. Entonces el numero de elementos de orden p que hay en el grupo G es q2 (p ; 1), pues cada subgrupo de orden p aporta p ; 1 elementos de orden p que no se repiten, ya que dos subgrupos distintos de orden p tienen interseccion trivial. Por tanto, el numero de elementos de G cuyo orden no es p es pq2 ; q2(p ; 1) = q2 : Sea ahora K un q-subgrupo de Sylow de G como jK j = q2 y K no contiene elementos de orden p, K debe consistir en los q2 elementos de orden distinto de p, lo que muestra que K es el unico q-subgrupo de Sylow de G y as nq = 1. Teorema 8.4.11 Z 1. El menor entero positivo n para el que existe un grupo simple no abeliano de orden n es n = 60. 2. Si G es un grupo simple y jGj < 60, entonces G  = p para cierto numero primo p. Demostracion. Por el Ejercicio 6.2.10, el apartado 2 sera consecuencia del 1, por lo que nos limitamos a demostrar este. Ademas, como A5 es un grupo simple con 60 elementos, basta ver un grupo no abeliano con menos de 60 elementos no es simple. Si p y q son primos distintos, ya sabemos que los grupos de orden p son abelianos y que los de ordenes pn (con n > 1), pq y pq2 no son simples (Proposiciones 5.9.7 y 8.4.10). Esto resuelve todos los casos excepto los de ordenes 24, 30, 36, 40, 42, 48 y 56. Por el Tercer Teorema de Sylow 8.4.9, si si jGj = 40 entonces n5 = 1, y si jGj = 42 entonces n7 = 1, por lo que ningun grupo de esos ordenes puede ser simple. Por el Primer Teorema de Sylow 8.4.3, todo grupo de orden 24 tiene un subgrupo de ndice 3 todo grupo de orden 36 tiene un subgrupo de ndice 4 y todo grupo de orden 48 tiene un subgrupo de ndice 3. Por el ultimo apartado del Ejercicio 8.2.6, ningun grupo de esos ordenes puede ser simple. Supongamos ahora que jGj = 30 = 2 3 5. Del Tercer Teorema de Sylow se deduce que n3 puede valer 1 o 10, y que n5 puede valer 1 o 6. Si n3 = 10 entonces, como cada par de 3-subgrupos de Sylow tiene interseccion trivial, la union de todos ellos tiene 10 2 = 20 elementos distintos del neutro, todos de orden 3. Analogamente, si n5 = 6 entonces hay 6 4 = 24 elementos de orden 5 en G. Por tanto, no puede tenerse a la vez n3 = 10 y n5 = 6, por lo que uno de ellos vale 1 y por tanto G no es simple. Solo nos queda por estudiar el caso jGj = 56 = 237, que estara resuelto si vemos que o bien n7 = 1 o bien n2 = 1. Supongamos que n7 6= 1 y veamos que n2 = 1. Del Tercer Teorema de Sylow se deduce que n7 = 8. Como en el parrafo anterior, G contiene 8 6 = 48 elementos de orden 7. Solo quedan pues el neutro y 7 elementos de orden 2 para formar los 2-subgrupos de Sylow (de orden 23 = 8), por lo que ha de ser n2 = 1. 8.5 Productos directo y semidirecto de subgrupos En el Captulo 7 hemos visto que, si un grupo abeliano A es la suma directa de dos subgrupos B y C, entonces A es isomorfo al producto directo B  C. En esta seccion describiremos situaciones en las que un grupo (no necesariamente abeliano) es isomorfo al producto directo de dos subgrupos, y estudiaremos una construccion que generaliza el producto directo, y que nos permitira encontrar nuevos ejemplos de grupos. Sean H y K dos grupos y sea G = H  K su producto directo. Si a cada h 2 H le asociamos el elemento ^h = (h 1) de G, a cada k 2 K le asociamos k^ = (1 k) 2 G, y hacemos H^ = f^h : h 2 H g y K^ = fk^ : k 2 K g, se comprueba facilmente que: H^ y K^ son subgrupos normales de G. ^ entonces ^hk^ = k^h^ . Si h^ 2 H^ y k^ 2 K, H^ \ K^ = f1g. ^ y si son nitos entonces jGj = jH^ j jK^ j. G = H^ K, 8.5. PRODUCTOS DIRECTO Y SEMIDIRECTO DE SUBGRUPOS 197 Los resultados que siguen muestran que algunas de estas condiciones son sucientes para que un grupo sea isomorfo al producto directo de dos de sus subgrupos. Lema 8.5.1 Sea G un grupo nito con subgrupos H y K . Si se verican las condiciones siguientes: 1. H y K conmutan elemento a elemento es decir, si h 2 H y k 2 K entonces hk = kh. 2. H \ K = f1g. 3. jGj = jH j jK j. Entonces existe un isomorsmo G  = H  K. Demostracion. La condicion 1 implica que la aplicacion : H  K ! G dada por (h k) = hk es un homomorsmo de grupos. La condicion 2 implica que es inyectivo, pues si 1 = (h k) = hk (con h 2 H y k 2 K) entonces k = h;1 2 H \ K, luego (h k) = (1 1). Por tanto, Im es un subgrupo de G isomorfo H  K y en consecuencia su cardinal es jH  K j = jH j jK j la condicion 3 nos dice entonces que es suprayectivo, lo que termina la demostracion. Z Z Ejemplo 8.5.2 Si n es impar, D2n es el producto directo de Dn y 2. Consideremos el grupo diedrico D2n = hr s : r2n = 1 s2 = 1 srs = r;1 i y el subgrupo H = hr2 si. Como o(r2 ) = n y sr2 s = (r2 );1, se tiene H  = Dn . Por el Ejercicio 5.3.6, se tiene Z(D2n ) = hrn i  = 2. n Como n es impar, r 62 H. Ahora es evidente que H y Z(D2n ) verican las hipotesis del Lema 8.5.1 y por tanto D2n  = Dn  2. Z Lema 8.5.3 Sea G un grupo nito con subgrupos H y K . Si se verican las condiciones siguientes: 1. H y K son normales en G. 2. H \ K = f1g. 3. jGj = jH j jK j. Entonces existe un isomorsmo G  = H  K. Demostracion. Por el Lema 8.5.1, basta ver que H y K conmutan elemento a elemento. Sean pues h 2 H y k 2 K. Por la condicion 1 se tiene hk 2 H y (k;1)h 2 K, y as h;1k;1 hk = h;1 hk 2 H y h;1k;1hk = (k;1)h k 2 K: La condicion 2 implica entonces que h;1k;1 hk = 1, por lo que hk = kh, como queramos ver. La generalizacion del concepto de producto directo prometida al principio de la seccion es la siguiente: De nicion 8.5.4 Sean H y N dos grupos y sea : H ! Aut(N) un homomorsmo de grupos. Denotamos por g la imagen de g 2 H por y denimos en el producto cartesiano N  H la operacion (x g)(y h) = (x g (y) gh): o o Esta operacion dota a N  H de una estructura de grupo llamado producto semidirecto de N por H con accion , que denotaremos por N H , o por N  H si puede haber confusion con respecto a . o Ejercicio 8.5.5 Dados H , N y : H ! Aut(N) como antes, se pide: 1. Comprobar que el producto semidirecto N cada elemento. H es un grupo. Identicar el neutro y el inverso de o 2. Si es el homomorsmo trivial (es decir, g (x) = x para todo g 2 H y x 2 N ) entonces N H es el producto directo N  H . 198 CAPITULO 8. ESTRUCTURA DE LOS GRUPOS FINITOS o o 3. Demostrar que N H tiene un subgrupo H^ isomorfo a H y un subgrupo normal N^ isomorfo a N tales que H^ \ N^ es el subgrupo trivial y H^ N^ = N H . 4. >Cuando es N H abeliano? o Como ha ocurrido con el producto directo, ciertas condiciones en dos subgrupos de un grupo G nos permiten ver a este como un producto semidirecto de aquellos: Lema 8.5.6 Sea G un grupo nito con subgrupos N y H . Si se verican las condiciones siguientes: 1. N es normal en G. 2. N \ H = f1g. 3. jGj = jN j jH j. Entonces existe un isomorsmo G  = N  H , donde : H ! Aut(N) lleva g 2 H al automorsmo g : N ! N dado por g (x) = gxg;1 (conjugacion por g;1 ). Demostracion. Es claro que es un homomorsmo de grupos de hecho, es el homomorsmo asociado a la accion por la izquierda de H sobre N descrita en los Ejemplos 8.1.4. La aplicacion : N  H ! G dada por (x g) = xg es un homomorsmo de grupos pues ((x g)(y h)) = (x g (y) gh) = xgyg;1 gh = xgyh = (x g) (y h) y es inyectivo pues (x g) = 1 implica que x = g;1 2 N \ H = f1g y por tanto (x g) = (1 1). Ahora la condicion 3 implica que es suprayectivo, y por tanto es el isomorsmo que buscamos. Para dar ejemplos de productos semidirectos, es interesante estudiar los grupos de automorsmos Aut(N) cuando N es un grupo sencillo. o o Ejercicio 8.5.7 Demostrar que: 1. Si N es un grupo abeliano entonces (y) = y;1 (para cada y 2 N ) dene un automorsmo de N , y se tiene 2 = 1 en Aut(N). 2. Sea N = hxi un grupo cclico nito de orden n. Entonces cada homomorsmo  : N ! N queda determinado por el valor de (x), y  es un automorsmo si y solo si (x) = xi con mcd(n i) = 1. De hecho, hay un isomorsmo de grupos Aut(N)  = n. 3. Si N es cclico de orden 3 o 4, entonces Aut(N) = f1 g, donde  es el automorsmo del apartado 1. 4. Si N = f1 a b cg es el grupo de Klein, entonces cada automorsmo de N induce una permutacion de fa b cg, y de hecho esto dene un isomorsmo Aut(N)  = S3 . Los elementos de orden 2 de Aut(N) son los que jan exactamente uno de los elementos a, b o c. Ejemplos 8.5.8 Productos semidirectos. Z o En todos los ejemplos,  es el automorsmo del Ejercicio 8.5.7. 1. Sean H = hgi y N = hxi dos grupos cclicos de ordenes 2 y 3, respectivamente, y sea : H ! Aut(N) el homomorsmo dado por g 7! . Entonces es facil ver que N H  = S3 . 2. Mas generalmente, si N es un grupo abeliano y H = hgi es cclico de orden 2, entonces el homomorsmo : H ! Aut(N) dado por g =  induce un producto semidirecto. Si N es cclico de orden n, es facil dar un isomorsmo Dn  =N H = Zn  2. Si N es cclico innito, el grupo obtenido se llama diedrico innito. 3. Sean H = hgi y N = hxi grupos cclicos de ordenes 4 y 3, respectivamente. El producto semidirecto asociado al homomorsmo : H ! Aut(N), dado por (g) = , es un grupo de orden 12 que denotaremos por 3 4. Ejercicio 8.5.9 Sea H = hgi un grupo cclico de orden 3 y sea N = hx1 x2i = 2  2. Demostrar que existe un homomorsmo de grupos : H ! Aut(N) tal que g (x1) = x2 y g (x2 ) = x1x2 , y describir un isomorsmo explcito entre N H y A4 . Z oZ o oZ o ZZ 8.6. GRUPOS DE ORDEN BAJO 199 8.6 Grupos de orden bajo En esta seccion obtenemos informacion acerca de la estructura de ciertos grupos nitos cuyo orden tiene una factorizacion en primos sencilla, y a partir de ellos describimos todos los grupos de orden menor o igual que 15. Z Proposicion 8.6.1 Sea G un grupo nito con jGj = pr qs , donde p y q son primos distintos y r s 2 +. Si G tiene un unico p-subgrupo de Sylow H y un unico q-subgrupo de Sylow K , entonces G  = H  K. Demostracion. Bastara ver que H y K verican las tres condiciones del Lema 8.5.3. La primera se verica por el Tercer Teorema de Sylow (8.4.9) la segunda porque jH \ K j debe ser un divisor comun de jH j = pr y de jK j = qs y la tercera es evidente. Proposicion 8.6.2 Sea G un grupo nito con jGj = pq, donde p < q son primos y q 6 1 mod p. Entonces G es un grupo cclico. Z Demostracion. Sea np el numero de p-subgrupos de Sylow de G. El Tercer Teorema de Sylow (8.4.9) y la hipotesis q 6 1 mod p implican que np = 1. Es decir, G tiene un unico p-subgrupo de Sylow H,  p. Como la relacion p < q implica que p 6 1 mod q, se ve del que tiene orden p y por tanto H =  q. Por la Proposicion 8.6.1 y el Teorema mismo modo que G tiene un unico q-subgrupo de Sylow K =  H  K = p  q = pq, como queramos ver. Chino de los Restos se tiene G = ZZZ Z Proposicion 8.6.3 Si p es un primo impar, entonces un grupo de orden 2p o bien es cclico o bien es isomorfo al grupo diedrico Dp . Demostracion. Si G es abeliano el resultado es claro, considerando su descomposicion invariante. Supongamos pues que G no es abeliano. Con la notacion usual se tiene np = 1, de modo que G posee un subgrupo normal N de orden p. Como p es primo, existe x 2 G tal que N = hxi. Por otra parte, G contiene algun subgrupo H = hgi de orden 2. Como G = hx gi (>por que?) y no es abeliano, se tiene xg = 6 gx. Es claro que el orden de xg no es 1, y no es 2p pues G no es cclico. Tampoco es p pues, al ser hxi el unico p-subgrupo de Sylow, eso nos dara xg 2 hxi y por tanto g 2 hxi, que es absurdo. En conclusion, debe ser (xg)2 = 1, o sea gxg;1 = x;1. Ahora el Lema 8.5.6 y los Ejemplos 8.5.8 nos dan el resultado. Podemos ya iniciar la descripcion de los grupos de orden menor o igual que 15. De hecho ya tenemos descritos casi todos esos grupos desde hace tiempo, y lo que nos permiten los ultimos resultados es asegurarnos de que no hay mas. Grupos de orden 2, 3, 5, 7, 11 o 13. Z Si n vale 2, 3, 5, 7, 11 o 13 el unico grupo (salvo isomorsmos) de orden n es n (Proposicion 5.5.6). Grupos de orden 4 o 9. Z ZZ Z Z Z ZZZ Z Z Si jGj = p2 con p primo, la Proposicion 5.9.9 nos dice que G es abeliano y entonces, por el Teorema de Estructura 7.5.13, G es isomorfo a p2 o a p  p. As, hay solamente dos grupos de orden 4 (el grupo cclico 4 y el grupo de Klein 2  2) y dos grupos de orden 9 ( 9 y 3  3). Grupos de orden 6, 10 o 14. Z Por la Proposicion 8.6.3 hay exactamente dos grupos (salvo isomorsmos) de cada uno de esos ordenes: el cclico y el diedrico. Es decir, los unicos grupos de orden 6 son 6 y D3 los unicos grupos de orden 10 son 10 y D5 y los unicos grupos de orden 14 son 14 y D7 . CAPITULO 8. ESTRUCTURA DE LOS GRUPOS FINITOS 200 Grupos de orden 8. ZZ Z Z Z Z Conocemos cinco grupos distintos y no isomorfos de orden 8 (ver el nal de la Seccion 5.8): son los grupos abelianos 8, 4  2 y 2  2  2 y los grupos no abelianos D4 y Q8. Por el Teorema de Estructura 7.5.13 no hay mas grupos abelianos de orden 8. Por tanto, para ver que esos son todos los grupos de orden 8, hemos de probar que un grupo G no abeliano y de orden 8 debe ser isomorfo a D4 o a Q8 . Como G no contiene elementos de orden 8 (>por que?) y no todos sus elementos pueden tener orden 2 (>por que?), G debe contener un elemento x de orden 4. Como hxi tiene ndice 2, es normal en G y G=hxi  = 2. Si jamos ahora y 62 hxi, se tiene G = hx yi, y en consecuencia xy 6= yx. Como ademas xy 2 hxi (por la normalidad) y o(xy ) = o(x) = 4, ha de ser xy = x o xy = x3 , pero la primera posibilidad equivale a xy = yx y por tanto se ha de dar la segunda: xy = x3 = x;1. Por otra parte, como G=hxi  = 2, se tiene y2 2 hxi. Si fuera y2 = x o y2 = x3 deduciramos que 2 o(y) = 8, por lo que ha de ser y = 1 o y2 = x2. La primera posibilidad nos lleva a la situacion Z Z G = hx yi x4 = 1 G = hx yi x4 = 1 y;1 xy = x;1 y2 = 1 y es entonces claro que G  = D4 . La segunda posibilidad nos lleva a la situacion y;1 xy = x;1  y 2 = x2  y entonces es facil establecer un isomorsmo G  = Q8 que lleve x 7! i e y 7! j. Grupos de orden 15. Por la Proposicion 8.6.2, el unico grupo de orden 15 es Grupos de orden 12. Z Z ZZoZ Z 15. Z Z oZ Z oZ Z Conocemos cinco grupos de orden 12. Por los resultados del Captulo 7, hay exactamente dos que son abelianos, 12 y 6  2. Otros tres no son abelianos se trata del grupo alternado A4 , del grupo diedrico D6 y del grupo 3 4 de los Ejemplos 8.5.8 (producto semidirecto de 3 = hxi por 4 = hgi con homomorsmo : hgi ! Aut(hxi) dado por g (x) = x;1 ). E stos no son isomorfos entre s porque A4 no tiene subgrupos de orden 6, D6 tiene elementos de orden 6 y 3 4 tiene elementos de orden 4. Vamos a demostrar que no hay mas grupos no abelianos de orden 12. Para ello, basta con demostrar que un grupo G no abeliano y de orden 12 debe ser isomorfo a D6 , A4 o 3 4. Sea G un tal grupo, y sean H un 2-subgrupo de Sylow (de orden 4) y K un 3-subgrupo de Sylow (de orden 3). Como H y K son abelianos y G no lo es, de las Proposiciones 8.4.10 y 8.6.1 se deduce que exactamente uno de estos dos subgrupos es normal en G. Como jK j = 3, existe k 2 G de orden 3 tal que K = hki. Para H tenemos dos opciones: o bien es cclico, H = hhi con o(h) = 4, o bien es isomorfo al grupo de Klein, H = hh1 h2i con o(h1 ) = o(h2 ) = 2 y h1h2 = h2 h1. Distinguiremos casos segun cual de los subgrupos sea normal y segun sea H cclico o no. Supongamos primero que H es normal en G. Por el Lema 8.5.6 se tiene G  = H  K para el homomorsmo : K ! Aut(H) tal que k es conjugar por k;1 . Como no es trivial (de lo contrario G = H  K sera abeliano) y K es simple, debe ser inyectivo y por tanto k tiene orden 3 en Aut(H). Por los dos ultimos apartados del Ejercicio 8.5.7, deducimos que H ha de ser el grupo de Klein y que podemos elegir los generadores h1 y h2 de H de modo que se tenga (h1) = h2 y (h2) = h1h2 . Entonces el Ejercicio 8.5.9 nos dice que G  = A4 . Supongamos ahora que K es normal en G. Por el Lema 8.5.6 se tiene G  = K  H para el homomorsmo no trivial : H ! Aut(K) tal que g es conjugar por g;1 , para cada g 2 H. Por el Ejercicio 8.5.7 se tiene Aut(K) = f1 g con (k) = k;1 . Si H = hhi es cclico entonces debe ser h = , y por tanto G es el grupo 3 4 de los Ejemplos 8.5.8. Supongamos por ultimo que H es el grupo de Klein. Entonces el nucleo de tiene dos elementos y, eligiendo adecuadamente los generadores de H, podemos poner H = hh1 h2i con h1 (k) = k;1 y h2 (k) = k. Entonces D = hk h1i es un subgrupo de G isomorfo a D3 . Comprobando que D y E = hh2 i  = 2 verican las hipotesis del Lema 8.5.1, deducimos que G  =DE = D3  2, y por tanto G  = D6 por el Ejemplo 8.5.2. o Z oZ o Z Z 8.7. PROBLEMAS 201 Resumen: Grupos de orden menor o igual que 15. La siguiente tabla resume la clasicacion de los grupos de orden menor o igual que 15 (excepto los de orden primo). En la primera columna aparecen los posibles ordenes, en la segunda el numero N de grupos no isomorfos de cada orden, con el numero de abelianos entre parentesis, en la tercera se da la lista de los abelianos, y en la cuarta la de los no abelianos. jGj N Abelianos No abelianos 4 2 (2) 4 2  2 6 2 (1) 6 D3 8 5 (3) 8 4  2 2  2  2 D4  Q8 9 2 (2) 9 3  3 10 2 (1) 10 D5 12 5 (2) 12 6  2 D6  A4  3 4 14 2 (1) 14 D7 15 1 (1) 15 Clasicar grupos de ordenes mayores puede ser muy laborioso (por ejemplo, existen 14 grupos distintos de orden 16), aunque ya hemos visto que es sencillo cuando el orden tiene una factorizacion adecuada (Teorema 5.8.7 y Proposiciones 5.9.9 y 8.6.3). Con las tecnicas vistas en este captulo resulta facil clasicar los grupos de ordenes 18, 20, 21 o 30. En el libro Group Tables, de A. Thomas y G. Wood (Shiva Math. Series), se describen todos los grupos de orden menor o igual que 32, y para cada uno de ellos se da la tabla y se describen todos sus subgrupos y sus clases de conjugacion. ZZ ZZ ZZ ZZ ZZ ZZ ZZ Z Z Z ZZ Z oZ 8.7 Problemas 1. Sea , = fA B C Dg el conjunto de los vertices de un cuadrado, y sea S(,) su grupo de permutaciones. Se pide: (a) Describir un homomorsmo inyectivo D4 ! S(,). (b) Describir las orbitas y los estabilizadores de la correspondiente accion por la izquierda de D4 en ,. (c) Determinar todas las orbitas y los estabilizadores de la restriccion de esta accion al subgrupo de las rotaciones. 2. Considerese S3 actuando en el conjunto , de sus subgrupos por conjugacion. Determinar la orbita y el estabilizador de < (12) >. p C 3. Sea ! = ;1+2 ;3 una raz cubica de 1. Demostrar que D3 = ha b j a3 = b2 = e bab = a;1 i actua en la circunferencia unidad C que identicaremos con C = fz 2 : jz j = 1g de forma que b z = z% a z = !z: 4. 5. 6. 7. Calcular las orbitas de esta accion. Generalizar esta accion a una accion del grupo diedrico Dn en C. Vericar que, para grupos no abelianos, la accion por conjugacion gxg;1 es una accion por la izquierda pero no por la derecha, y viceversa para g;1 xg: Sean x e y elementos conjugados en un grupo nito G: Probar que el numero de elementos g 2 G tales que xg = y es igual al orden del subgrupo CenG (x). Probar que si en un grupo G existe un elemento que tiene exactamente dos conjugados entonces G posee un subgrupo normal propio y no trivial. *] Sea G un grupo nito y sea p el menor divisor primo de jGj. Demostrar que si H es subgrupo normal de G de orden p entonces H  Z(G). (Indicacion: Considerar G actuando por conjugacion sobre H.) 202 CAPITULO 8. ESTRUCTURA DE LOS GRUPOS FINITOS 8. *] Sea G un grupo nito que actua sobre un conjunto nito ,. Para cada g 2 G, designamos por ,g al conjunto f 2 , : g = g de los punto jados por g. Demostrar que el numero de orbitas de G en , es 1 X j, j jGj g2G g 9. 10. 11. 12. 13. 14. es decir, el promedio del numero de puntos jados por los elementos de G. (Indicacion: Contar el numero de elementos del conjunto f(g ) 2 G  , : g = g de dos maneras distintas.) *] Sea G un grupo nito que actua en un conjunto nito X. Sea k el numero de orbitas. Para cada g 2 G sea v(g) el cardinal del conjunto fx 2 X : g x 6= xg. Demostrar: P (a) g2G v(g) = jGj(jX j ; k). (b) Si k = 1, entonces jGj = jX j si y solo si para todo g 2 G n f1g y todo x 2 X, g x 6= x. Sean G un grupo nito, H un subgrupo propio y no trivial de G, X el conjunto de los subgrupos conjugados de H y S(X) el grupo simetrico sobre el conjunto X. (a) Demostrar que la aplicacion ' : G ! S(X) dada por '(h)(K) = hKh;1 es un homomorsmo de grupos. Deducir que si G es simple, entonces jGj divide a jS(X)j. (b) Utilizar el apartado anterior para probar que ningun grupo de orden 300 es simple. Si un grupo G posee un unico p-subgrupo de Sylow P , demostrar que P es un subgrupo caracterstico de G. Sea P un p-subgrupo de Sylow de un grupo nito G y sea N un subgrupo normal de G. Probar que PN=N es un p-subgrupo de Sylow de G=N. Demostrar que si H es un subgrupo de ndice nito de G entonces N = \g2G g;1 Hg tiene ndice nito en G. Concluir que si G tiene un subgrupo propio de ndice nito, entonces tambien tiene un subgrupo propio normal de ndice nito. Sean G un grupo que actua sobre dos conjuntos X e Y . Demostrar que g (x y) = (g x g y) es una accion de G en X  Y y que se tiene G(xy) = Gx \ Gy para cualesquiera x y 2 G. Utilizar esto para demostrar que, si H y K son dos subgrupos de G de ndice nito, entonces H \ K tambien tiene ndice nito en G y G : H \ K] G : H]G : K]. Demostrar que si ademas G : H] = G : K] = r, entonces G : H \ K] r(r ; 1). 15. Sea G un grupo que actua sobre un conjunto X. Demostrar: (a) G0 = \x2X Gx es un subgrupo normal de G. (b) La accion de G en X es el precisamente si G0 = feg. (c) G=G0 actua elmente en X de forma natural. 16. Dados H, N y un homomorsmo : H ! Aut(N), demostrar que las condiciones siguientes son equivalentes para un elemento (x g) de N  H: (a) (x g) esta en el centro de N  H. (b) (x g) conmuta con los elementos de la forma (1 h) y con los de la forma (y 1). (c) g 2 Z(H), g es la conjugacion por x, y h (x) = x para cada h 2 H. a 0 17. Demostrar que el conjunto G de las matrices de la forma b c tales que a b c 2 5 y ac 6= 0 es un grupo multiplicativo. Determinar su orden, encontrar los elementos de orden 5 y describir los 5-subgrupos de Sylow de G. o o Z 8.7. PROBLEMAS 203 18. Sea G un grupo que posee un sistema generador formado por tres elementos a b c que satisfacen las siguientes condiciones: o(a) = o(b) = 2 o(c) = 3 19. 20. 21. 22. 23. 24. 25. 26. 27. 28. 29. 30. 31. 32. 33. 34. 35. 36. ab = ba ac = b bc = ab: Demostrar que jGj = 12, y determinar todos los subgrupos normales de G. >A que grupo de los que hemos descrito en el texto es isomorfo G? Demostrar que si H y K son dos subgrupos de un grupo que verican K  NG (H) y H \ K = f1g, entonces HK  = H K para cierto homomorsmo K ! Aut(H). Probar que ningun grupo de orden 2pr (donde p es primo y r 1) es simple. Sean G un grupo nito, p un divisor primo de jGj y N la interseccion de todos los p-subgrupos de Sylow de G. Demostrar que N es el mayor p-subgrupo de G que es normal en G (es decir, N es normal y cualquier otro p-subgrupo que sea normal esta contenido en N). Probar que no hay grupos de orden 490 que sean simples. *] Se pide: (a) Demostrar que si a y b son elementos de un grupo con o(a) = n y o(b) = m, y si existe un entero i tal que b;1ab = ai , entonces im  1 mod n. (b) Recprocamente, demostrar que si n, m e i son enteros positivos tales que im  1 mod n, entonces existe un grupo de orden nm generado por dos elementos a y b tales que o(a) = n, o(b) = m y b;1ab = ai . (c) Mostrar que las permutaciones  = (1 2 3 4 5 6 7) y  = (1 4 2)(3 5 6) de S7 verican  = 2 , y describir un subgrupo de S7 no abeliano de orden 21. (d) Demostrar que todos los grupos de orden impar menor que 21 son abelianos. *] Sean n y m enteros positivos tales que m y (n) son coprimos. Demostrar que si a y b son dos elementos de ordenes n y m de un grupo y hai es un subgrupo normal, entonces ab = ba. Probar que todo grupo de orden 255, 455 o 1645 es cclico. Probar que ningun grupo tiene un centro de ndice 77. Demostrar que dos subgrupos conjugados de un grupo G tienen el mismondice en G. (Indicacion: Considerar G actuando en el conjunto de los subgrupos por traslaciones.) *] Demostrar que un grupo de orden 108 tiene un subgrupo normal de orden 9 o 27. Sea H un subgrupo normal de un grupo nito G y p un divisor primo de jGj tal que G : H] no es multiplo de p. Demostrar que H contiene todos los p-subgrupos de Sylow de G. Demostrar que si G es un grupo nito de orden pq, con p < q primos y de forma que q 6 1 mod p, entonces G es cclico. Dar un ejemplo de un grupo no cclico de orden pq con p < q primos. Demostrar que un grupo de orden pnq, con p y q primos distintos y pn;1 < q, no es simple. Demostrar que un grupo simple no tiene orden p2q2 con p y q dos primos distintos. (Indicacion: Observese que qjp + 1 con p < q implica q = p + 1 = 3.) Sea G un grupo tal que jG=Z(G)j = pq, con p < q primos. Demostrar que q  1 mod p. Clasicar los grupos de orden 18, 20 y 30. Calcular el numero de p-subgrupos de Sylow de Sp , con p primo y del resultado obtenido deducir el Teorema de Wilson. *] Sea H un subgrupo propio del grupo alternado An , con n 5. Demostrar que An : H] n. Demostrar que, para cada divisor m 5 de 60, A5 tiene un subgrupo de ndice m. o 204 CAPITULO 8. ESTRUCTURA DE LOS GRUPOS FINITOS 37. Sea f(x1  : : :  xn) una funcion en n variables y, para cada  2 Sn , sea (f) la funcion en n variables (f)(x1  : : :  xn) = f(x (1)  : : :  x (n)): Demostrar que si n 5 y el numero k de funciones (f) distintas es > 2, entonces k n. 38. *] Probar que si G es nito y P es un subgrupo de Sylow de G, entonces todo subgrupo H de G que contenga al normalizador NG (P) es su propio normalizador es decir, verica H = NG (H). (Indicacion: Usar el Segundo Teorema de Sylow aplicado a H.) 39. *] Demostrar que no hay grupos simples de orden n no primo y n 6= 60, para n < 168. >Hasta donde puedes llegar para n > 168? 40. *] Sea G un grupo simple de orden 60. Demostrar: (a) Si G actua en un conjunto X con menos de 5 elementos entonces lo hace trivialmente es decir, g x = x, para todo g 2 G y x 2 X. Concluir que G no tiene subgrupos propios de ndice menor que 5. (b) Si H1 6= H2 son 2-subgrupos de Sylow de G, entonces K = hH1  H2i tiene ndice 1 o 5. (c) Si el numero de 2-subgrupos de Sylow de G es 15, entonces al menos dos de ellos H1 y H2 se intersecan no trivialmente y en tal caso K = hH1  H2i ha de tener ndice 5. (Indicacion: Para la primera armacion, contar elementos de orden potencia de 2 y de orden potencia de 5. Para la segunda, observar que K tiene centro no trivial.) (d) G actua de forma no trivial en un conjunto con 5 elementos. (e) G  = A5 . 41. *] Sea G un grupo. Un G-conjunto (por la izquierda) es un conjunto X junto con una accion por la izquierda de G en X. Un homomorsmo de G-conjuntos es una aplicacion : X1 ! X2 entre dos G-conjuntos X1 y X2 tal que (g x) = g (x) para cada x 2 X1 . Demostrar: (a) La composicion de dos homomorsmos de G-conjuntos es un homomorsmo de G-conjuntos. (b) Si : X1 ! X2 es un homomorsmo de G-conjuntos biyectivo, entonces ;1 : X2 ! X1 es un homomorsmo de G-conjuntos. Se dice entonces que es un isomorsmo de G-conjuntos. (c) Sea X un G-conjunto. Un G-subconjunto de X es un subconjunto Y de X tal que Gy  Y para cada g 2 G y cada y 2 Y . En este caso, la restriccion a Y de la accion de G dota a Y de una estructura de G-conjunto, y la inclusion Y ,! X es un homomorsmo de G-conjuntos. (d) Las orbitas de un G-conjunto X son G-subconjuntos de X. (e) Si fXi : i 2 I g es una familia de G-conjuntos disjuntos, entonces la union X = i2I Xi tiene una unica estructura de G-conjunto en`la que cada Xi es un G-subconjunto. Este G-conjunto se llama coproducto y se denota por i2I Xi . (f) Sean X un G-conjunto, x 2 X y H = EstabG (x). Consideremos la accion por la izquierda por traslaciones de G sobre G=H. Entonces la aplicacion : G=H ! X dada por (gH) = gx esta bien denida y es un homomorsmo de G-conjuntos. (g) Todo G-conjunto (por la izquierda) es isomorfo a un coproducto de G-conjuntos de la forma G=H, donde cada H es un subgrupo de G y G actua en G=H por traslaciones. ` ` (h) Sean H1 : : : Hn K1  : : : Km subgrupos de G, y sean X = ni=1 G=Hi e Y = mi=1 G=Ki los G-conjuntos coproducto, donde en cada cociente se considera la accion por traslaciones. Demostrar que X e Y son isomorfos si y solo si n = m y existe una permutacion  2 Sn tal que Hi y K (i) son conjugados para cada i = 1 : : : n. Bibliografa del captulo Delgado-Fuertes-Xambo 11], Dorronsoro-Hernandez 13], Herstein 20], Jacobson 23], Rotman 30]. Captulo 9 Series normales Se estudian las series normales de un grupo, y se demuestran las propiedades basicas de los grupos resolubles y de los grupos de longitud nita. Introduccion Sea G un grupo con un subgrupo normal N, propio y no trivial. Como N y G=N son grupos \mas peque~nos" que G, puede ser que tengamos cierta informacion sobre ellos, y entonces es natural tratar de usarla para deducir propiedades de G. Es decir, podemos pensar que G \se descompone" de algun modo en las \piezas" N y G=N, y la tarea es \recomponer" G a partir de esas piezas. Hay que advertir que, en general, N y G=N no determinan a G, pero s pueden conocerse algunas propiedades de G a partir de las de estas piezas. A su vez, podemos tratar de estudiar cada pieza descomponiendola en otras: Si H es un subgrupo normal de N podemos estudiar N a partir de H y N=H, y si K=N es un subgrupo normal de G=N (es decir, si se tiene N E K E G) podemos estudiar G=N a partir de su subgrupo K=N y del correspondiente cociente (G=N)=(K=N)  = G=K. Globalmente, estamos tratando de conocer G a partir del conocimiento de los grupos H, N=H, K=N y G=K, donde H, N y K son subgrupos de G tales que H E N E K E G. Esto sugiere la denicion de una serie normal de un grupo G como una cadena de subgrupos G = G0 D G1 D G2 D D Gn;1 D Gn = f1g: Dada una tal serie, y generalizando la idea anterior, podemos interpretar que G esta compuesto a partir de las piezas (que llamaremos factores de la serie) Gi;1=Gi con i = 1 : : : n. Cuando estos factores son sencillos en algun sentido (por ejemplo, abelianos, cclicos, simples...), es posible obtener informacion sobre G a partir de la de los factores, y en este captulo estudiaremos algunas situaciones de este tipo. En lugar de comenzar con la denicion general de serie normal, analizamos primero un caso concreto. A cada grupo G se le asocia un subgrupo normal G0 tal que el cociente G=G0 es \el mayor grupo cociente abeliano" de G. Repitiendo este proceso, se obtiene una cadena de subgrupos G D G0 D G00 D (la \serie derivada" de G) que, si en algun paso alcanza al subgrupo trivial, proporciona una serie normal de G con factores abelianos (decimos entonces que G es resoluble). Esto motiva nuestra denicion general de serie normal, y demostramos entonces que un grupo G es resoluble si y solo si posee una serie normal con factores abelianos. En el caso de grupos nitos, esto equivale a que exista una serie normal de G con factores cclicos y, como se vera en la asignatura de Ecuaciones Algebraicas, este hecho hara especialmente relevantes a los grupos resolubles en el estudio de las ecuaciones (de hecho, el nombre se debe a la conexion de estos grupos con la resolubilidad por radicales de las ecuaciones). Otra clase importante de series normales son las que tienen sus factores simples es decir, aquellas en las que no pueden a~nadirse nuevos terminos. Estas series se llaman series de composicion, y un grupo que posea una se dice que tiene longitud nita. En el contexto de las series normales, los grupos simples han de considerarse como los \bloques basicos", puesto que no se pueden descomponer en el sentido que estamos considerando en este captulo. Por tanto, los grupos de longitud nita son los que admiten descomposiciones en terminos de estos bloques basicos. En la segunda mitad del captulo 205 CAPITULO 9. SERIES NORMALES 206 estudiamos estos grupos hasta demostrar el Teorema de Jordan-H#older, que arma que todas las series de composicion de un grupo de longitud nita G tienen los mismos factores. El hecho de que todo grupo nito tenga longitud nita, combinado con la existencia de una descripcion (complicada) de todos los grupos simples nitos, puede interpretarse como un teorema de estructura para los grupos nitos. Pero este teorema es mucho menos contundente que el que vimos para grupos abelianos, por dos motivos: Por una parte, los \bloques basicos de estructura" en el caso abeliano (grupos cclicos) son mucho mas sencillos que los grupos simples (a pesar de su nombre). Por otra parte, en cuanto a la manera de \componer" esos bloques basicos, las sumas directas son mas manejables que las series normales, y ademas determinan salvo isomorsmos al grupo en cuestion. Objetivos del captulo Conocer las propiedades del subgrupo derivado de un grupo. Conocer el concepto de serie normal y las distintas caracterizaciones y propiedades de los grupos resolubles y de longitud nita. Saber calcular series derivadas y series de composicion en ejemplos concretos. Conocer las propiedades de la longitud de composicion, y saber usarlas para su calculo en ejemplos. Desarrollo de los contenidos 9.1 El subgrupo derivado Dado un grupo no abeliano G, es interesante medir de algun modo lo cerca que esta G de ser abeliano. El centro del grupo es util para esto: cuanto mayor es Z(G) mas cerca esta G de ser abeliano. En esta seccion vamos a desarrollar una herramienta alternativa para evaluar la \falta de abelianidad" de G. Observemos dos cosas: Por una parte, un elemento de la forma aba;1b;1 vale 1 precisamente cuando a y b conmutan, por lo que tales elementos (y el subgrupo que generan) seran interesantes para nuestro proposito. Por otra parte, es obvio que G tiene grupos cociente abelianos (al menos G=G, que es trivial), y sera interesante considerar los mayores de ellos es decir, los que se obtienen tomando el cociente por un subgrupo lo menor posible. En lo que sigue vemos como estas dos ideas conuyen en el concepto de subgrupo derivado de G. De nicion 9.1.1 Sea G un grupo. El conmutador de los elementos a b 2 G es el elemento a b] = aba;1b;1 : El subgrupo de G generado por los conmutadores se llama subgrupo derivado de G y se denota por G0. Las siguientes propiedades se verican facilmente: Ejercicio 9.1.2 Dados un grupo G y elementos a b 2 G, demostrar que: 1. G es abeliano si y solo si G0 = f1g. 2. a b];1 = b a]. 3. G0 consiste en los productos nitos de conmutadores. 4. Si f : G ! H es un homomorsmo de grupos entonces f(a b]) = f(a) f(b)]. 5. En particular, si N es normal en G, entonces a b]N = aN bN] en G=N . 6. a b]x = ax  bx] para cada x 2 G. 9.2. LA SERIE DERIVADA GRUPOS RESOLUBLES 207 Ejemplo 9.1.3 El subgrupo derivado de Sn. Vamos a demostrar que Sn0 = An. En efecto, como la \aplicacion signo" Sn ! f1 ;1g es un homomorsmo, es claro que todo conmutador es par, y en consecuencia Sn0  An . Recprocamente, la igualdad (ijk) = (ij)(ik)(ij)(ik) = (ij) (ik)] nos dice que cada ciclo de longitud 3 de Sn esta en Sn0 , y como tales ciclos generan An (Proposicion 6.2.7) deducimos que An  Sn0 , lo que nos da la igualdad buscada. La propiedad basica del subgrupo derivado es la siguiente. Teorema 9.1.4 Dado un grupo G, su subgrupo derivado G0 es el menor subgrupo normal de G que da un cociente abeliano es decir, se verican: 1. G0 es un subgrupo normal de G. 2. El cociente G=G0 es abeliano. 3. Si N es un subgrupo normal de G tal que el cociente G=N es abeliano, entonces G0  N . Demostracion. 1. Vemos que, si g 2 G0 y x 2 G, entonces gx 2 G0. En efecto, por el Ejercicio 9.1.2, se tiene g = a1 b1] an bn] para ciertos elementos de G, y por tanto gx = a1 b1]x an bn]x = a1x bx1 ] axn bxn] 2 G0 : 2. Es una consecuencia inmediata del Ejercicio 9.1.2. 3. Si N es como en el enunciado y a b 2 G, entonces a b]N = aN bN] = N, luego a b] 2 N. Es decir, N contiene a cada conmutador de G y en consecuencia contiene a G0 . El Teorema 9.1.4 nos dice que, en cierto sentido, G0 convierte a G en un grupo abeliano perdiendo la menor informacion posible (si entendemos que al hacer el cociente por G0 se pierde la informacion sobre G0 , pues sus elementos representan al neutro en el cociente). Podemos decir que, cuanto mas peque~no es G0, mas cerca esta G de ser abeliano. En la proxima seccion consideraremos una manera mas precisa de medir lo lejos que esta G de ser abeliano. Concluimos esta con un ejemplo. Ejemplo 9.1.5 El subgrupo derivado de Q8 . Sabemos que el grupo de los cuaterniones Q8 no es abeliano y que todos sus subgrupos son normales (Ejemplos 5.6.3). En particular lo es el subgrupo Z = fI ;I g, y el cociente Q8=Z es abeliano por tener orden 4. Como el unico subgrupo menor que Z es el trivial, que no da un grupo abeliano al pasar al cociente, el Teorema 9.1.4 nos dice que G0 = Z = fI ;I g. Observese que, en este caso, el subgrupo derivado coincide con el centro >es esto cierto en general? 9.2 La serie derivada grupos resolubles Consideremos el ultimo ejemplo: Q8 no es abeliano, pero s lo es su derivado Q08 . De este modo, Q8 se \descompone" mediante un subgrupo normal abeliano Q08 y el cociente correspondiente Q8=Q08, que tambien es abeliano (Teorema 9.1.4). En este sentido podramos decir que, aunque Q8 no es abeliano (su derivado no es trivial), esta proximo a serlo: el derivado de su derivado es trivial. Analogamente, podramos considerar grupos para los que el derivado del derivado de su derivado es trivial, etc. Todos estos grupos se pueden considerar como pertenecientes a una clase amplia de grupos \parecidos a los abelianos" el grado de parecido vendra dado por el numero de veces que hace falta calcular el derivado para llegar a obtener el subgrupo trivial. Sistematizamos estas observaciones a continuacion. Recordemos que N E G (o G D N) signica que N es un subgrupo normal de G, mientras que N C G (o G B N) signica que N es un subgrupo normal y propio de G. 208 CAPITULO 9. SERIES NORMALES Z De nicion 9.2.1 Sea G un grupo. Se dene por recurrencia el t-esimo derivado del grupo G, denotado G(t) (donde t 2 +) del modo siguiente: G(1) = G0 , el derivado de G. G(t+1) = G(t) 0, el derivado de G(t). La cadena de subgrupos G D G0 D G(2) D se conoce como la serie derivada de G, y se dice que G es resoluble si su serie derivada alcanza al grupo trivial es decir, si existe t 1 tal que G(t) = f1g. Es evidente que todo grupo abeliano es resoluble, y el comentario que abrio la seccion muestra que el grupo Q8 , no abeliano, es resoluble con serie derivada Q8 B f1 ;1g B f1g. El siguiente ejemplo muestra nuevos grupos resolubles, y tambien otros que no lo son. Usaremos el hecho obvio de que, en cuanto un termino se repite en la serie derivada, esta se estabiliza en ese termino es decir, si G(t) = G(t+1) entonces G(t) = G(t+k) para cualquier k 1. Ejemplos 9.2.2 Resolubilidad de los grupos simetricos1. 1. Hemos visto que Sn0 = An . Cuando n = 3 tenemos S30 = A3 , que es abeliano. Por tanto S3 es resoluble con serie derivada S3 B A3 B f1g. 2. Por el Ejemplo 6.2.8, el unico subgrupo normal, propio y no trivial de A4 es V = f1 (1 2)(3 4) (1 3)(2 4) (1 4)(2 3)g: Como A4 no es abeliano y A4 =V s lo es (tiene orden 3), del Teorema 9.1.4 se deduce que V = A04 = S4(2) . Y como V es abeliano, deducimos que S4(3) es trivial. En consecuencia, S4 es resoluble con serie derivada S4 B A4 B V B f1g. 3. Si n 5 entonces An es simple (Teorema de Abel, 6.2.12) y no abeliano, luego A0n = An por el Teorema 9.1.4. Es decir, la serie derivada de Sn se estabiliza en An y nunca alcanza al grupo trivial. En consecuencia, Sn no es resoluble cuando n 5. Vamos a estudiar las propiedades basicas de los grupos resolubles, y comenzamos con un lema. Lema 9.2.3 Sea f : G ! H un homomorsmo de grupos. Entonces: 1. f(G0 )  H 0 mas generalmente, f(G(t) )  H (t) para cada t 1. 2. Si f es suprayectiva entonces f(G0 ) = H 0 mas generalmente, f(G(t) ) = H (t) para cada t 1. Demostracion. 1. Como f(a b]) = f(a) f(b)] para cualesquiera a b 2 G, se tiene f(G0 )  H 0. El caso general lo vemos por induccion en t, con el caso t = 1 resuelto por lo anterior. Si el resultado vale para cierto entero positivo t entonces f se restringe a un homomorsmo f : G(t) ! H (t), y aplicando a este el caso inicial deducimos que G(t+1)  H (t+1) , lo que completa la demostracion. 2. Supongamos ahora que f es suprayectiva, y sea u v] un conmutador en H. Tomando a b 2 G con f(a) = u y f(b) = v se tiene f(a b]) = u v], lo que prueba que u v] 2 f(G0 ) es decir, H 0  f(G0 ), y el apartado anterior nos da la igualdad. El caso general se demuestra por induccion como antes. p Es bien conocida la formula (;b  b2 ; 4ac)=2a para obtener las soluciones de la ecuacion general de segundo grado aX + bX + c = 0. Cuando es posible expresar las soluciones de una ecuacion polinomica usando solo sus coecientes y las operaciones de suma, resta, producto, division y extraccion de races, se dice que la ecuacion es resoluble por radicales. Un profundo resultado, que se demostrara en la asignatura de Tercer Curso Ecuaciones Algebraicas, arma que la ecuacion general de grado n (es decir, la ecuacion con coecientes arbitrarios an X n +   + a1 X + a0 = 0) es resoluble por radicales si y solo si el grupo simetrico Sn es resoluble. Por tanto, los Ejemplos 9.2.2 implican que esto solo es posible para las ecuaciones lineales, cuadraticas, cubicas y cuarticas. Tambien es posible asignar un grupo Gf a cada polinomio particular f (en lugar de considerarpolinomioscon coecientes indeterminados) de tal manera que la ecuacion f (X ) = 0 es resoluble por radicales si y solo si el grupo Gf es resoluble (de ah el nombre de estos grupos). El grupo Gf es un subgrupo del grupo de permutaciones de las races de f en algun cuerpo. De hecho, el origen de la Teora de Grupos esta, en buena medida, en el uso de estos grupos de permutaciones para el estudio de la resolubilidad por radicales de las ecuaciones polinomicas. 1 2 9.2. LA SERIE DERIVADA GRUPOS RESOLUBLES 209 Proposicion 9.2.4 Sea G un grupo con un subgrupo H y un subgrupo normal N . Se verican: 1. Si G es resoluble entonces H es resoluble (los subgrupos de resolubles son resolubles). 2. Si G es resoluble entonces G=N es resoluble (los cocientes de resolubles son resolubles). 3. Si N y G=N son resolubles entonces G es resoluble. Demostracion. 1. Aplicando el Lema 9.2.3 a la inclusion H ,! G, tenemos H (t)  G(t) para cada t 1. Si G es resoluble entonces existe un t tal que G(t) = f1g y por tanto H (t) = f1g, por lo que H es resoluble. 2. El Lema 9.2.3 aplicado a la proyeccion canonica p : G ! G=N nos dice que p(G(t)) = (G=N)(t) para cada t 1. Si G es resoluble entonces existe un t tal que G(t) es trivial, y por tanto (G=N)(t) = p(G(t)) = p(f1g) tambien es trivial, por lo que G=N es resoluble. 3. Por hipotesis existe t 1 tal que (G=N)(t) = f1g. Aplicando como antes el Lema 9.2.3 deducimos que p(G(t)) es trivial, lo que signica que G(t)  Ker p = N. Por el apartado 1, G(t) es resoluble es decir, existe s 1 tal que (G(t))(s) = f1g. Como es claro que (G(t))(s) = G(t+s), deducimos que G es resoluble. Esto nos permite encontrar otros ejemplos de grupos resolubles: Proposicion 9.2.5 Todo p-grupo nito G es resoluble. Demostracion. Por hipotesis se tiene jGj = pn para cierto n 1 demostraremos el resultado por induccion sobre n, con el caso n = 1 evidente. En el caso general G tiene un subgrupo N de orden pn;1 (Corolario 8.3.4) que es normal en G (Proposicion 5.9.10 o Ejercicio 8.2.6). Como G=N es resoluble (de hecho, abeliano) por tener orden primo y N lo es por la hipotesis de induccion, la Proposicion 9.2.4 nos dice que G es resoluble. La serie derivada de un grupo resoluble sugiere la siguiente denicion mas general: De nicion 9.2.6 Sea G un grupo arbitrario. Una serie normal de G es una cadena de subgrupos de G de la forma D Gn;1 D Gn = f1g (es decir, es una cadena descendente nita que empieza en G y termina en f1g y en la que cada subgrupo G = G0 D G1 D G2 D es normal en el anterior). Los grupos Gi se llaman terminos de la serie, el numero n es la longitud de la serie, y cada grupo cociente Gi;1=Gi (para i = 0 1 : : :  n) se llama un factor de la serie. Observese que la longitud n mide el numero de factores, y no el de terminos (que es n + 1). Podemos considerar una serie normal de G como una \descomposicion" del grupo G: En efecto, G podra obtenerse \componiendo" de algun modo los grupos G1 y G=G1. A su vez, G1 es \compuesto" de G2 y G1=G2, de modo que G esta compuesto a partir de los tres grupos G2, G1 =G2 y G=G1. En general, dada una serie normal como la denicion, G esta compuesto por los factores de la serie G = G0  G1  G2  : : :  Gn;2  Gn;1 = Gn;1 = G : n;1 G1 G1 G2 G3 Gn;1 Gn f1g En particular, si G es resoluble, su serie derivada nos dice que G se puede considerar compuesto a partir de grupos abelianos. El siguiente teorema nos dice que, recprocamente, todo grupo compuesto a partir de grupos abelianos es resoluble. Teorema 9.2.7 Un grupo G es resoluble si y solo si existe una serie normal de G con todos sus factores abelianos. Demostracion. Si G es resoluble entonces su serie derivada es una serie normal con factores abelianos, lo que nos da el \solo si". Recprocamente, supongamos que G tiene una serie normal como la de la Denicion 9.2.6 con todos los factores abelianos, y veamos que G es resoluble por induccion en la longitud n de la serie. Si n = 1 entonces G es el unico factor de la serie, por lo que es abeliano y en consecuencia resoluble. En el caso general, la hipotesis de induccion aplicada a G1 nos dice que este es resoluble (tiene una serie de longitud n ; 1 con factores abelianos), y como G=G1 tambien es resoluble (de hecho es abeliano, por ser un factor de la serie inicial), la Proposicion 9.2.4 nos dice que G es resoluble, como queramos ver. 210 CAPITULO 9. SERIES NORMALES 9.3 Series de composicion grupos de longitud nita Supongamos dada una serie normal cualquiera de un grupo G: G = G0 D G1 D G2 D D Gn;1 D Gn = f1g (9.3.1) Desde luego, siempre es posible introducir nuevos terminos en esa serie: entre Gi y Gi+1 podemos intercalar un subgrupo Gi D H D Gi+1 tomando, por ejemplo, H = Gi o H = Gi+1. Aparte de este modo trivial de alargar la serie, puede haber otros. Por ejemplo, en la serie D4 B hr2 i B f1g podemos introducir otro termino de modo no trivial: D4 B hri B hr2 i B f1g: As, muchas series normales pueden ser \renadas" como en el caso que acabamos de ver. Introducimos la denicion que precisa esta idea. De nicion 9.3.1 Un renamiento de la serie normal (9.3.1) de un grupo G es otra serie normal de G que incluye a todos los terminos de (9.3.1). Si el renamiento aporta nuevos terminos, decimos que es un renamiento propio. Es decir, un renamiento de (9.3.1) se obtiene intercalando en cada \salto" Gi D Gi+1 de la serie original un cierto numero nito r 0 de subgrupos Ki en la forma Gi D K1 D K2 D D Kr D Gi+1 : En la denicion de serie normal no se excluye la posibilidad de que dos terminos consecutivos sean iguales, pero es claro que esas repeticiones no aportan nada al conocimiento de G. Por tanto, son especialmente interesantes las series en las que todos los contenidos son estrictos. Por ejemplo, si G es un grupo simple entonces la unica serie as que podemos dar en G es G B f1g en otras palabras, esta serie no admite renamientos propios. Mas generalmente, usando el Teorema de la Correspondencia (5.6.5), es facil ver que un factor Gi=Gi;1 de una serie normal es un grupo simple si y solo si no se pueden intercalar terminos entre Gi y Gi;1 de forma no trivial. Este es el contenido del siguiente ejercicio: Ejercicio 9.3.2 Sea G un grupo con subgrupos N y H tales que N E H . Demostrar que el grupo cociente H=N es simple si y solo si N 6= H (es decir, N C H ) y los unicos subgrupos K de G que verican N E K E H son K = N y K = H . Por tanto, si una serie (9.3.1) con contenidos estrictos no admite renamientos propios, es porque cada uno de los factores Gi =Gi+1 es un grupo simple. Esta es una buena situacion, pues nos da una \descomposicion" de G a partir de grupos simples. Destacamos este tipo de series normales mediante una denicion. De nicion 9.3.3 Una serie de composicion de G es una serie normal con contenidos estrictos G = G0 B G1 B G2 B B Gn;1 B Gn = f1g en la que todos los factores Gi =Gi+1 (para i = 0 1 :::n ; 1) son grupos simples, lo que equivale a que la serie no admita renamientos propios. Si G posee una serie de composicion decimos que G es de longitud nita, y llamamos longitud de composicion de G, denotada por `(G), al mnimo de las longitudes de sus series de composicion. Ejemplos 9.3.4 Series de composicion. 1. Si G es el grupo trivial, entonces G = G0 = f1g es una serie normal con un solo termino, y por tanto sin inclusiones ni factores. Por tanto, el grupo trivial tiene una serie de composicion y su longitud de composicion es 0. 2. Es claro que todo grupo simple admite una unica serie de composicion mas aun, los grupos simples son exactamente aquellos cuya longitud de composicion es 1.  GRUPOS DE LONGITUD FINITA 9.3. SERIES DE COMPOSICION Z 211 ZZ 3. El grupo aditivo de los enteros no es de longitud nita: Si tuviese una serie de composicion, digamos = G0 B G1 B B Gt;1 B Gt = f0g, entonces Gt;1 sera un subgrupo simple de , as que basta ver que no existe tal cosa. En efecto, por el Teorema 1.2.11, los subgrupos de son el trivial (que no es simple) y los de la forma n con n 1, que no son simples pues tienen por ejemplo a 2n como subgrupo (normal) propio y no trivial. 4. Sean p1 p2 : : :  pn enteros positivos (no necesariamente primos ni distintos) y sea n = p1p2 pn. Consideraremos el grupo G = n y abusaremos de la notacion identicando cada entero con su clase modulo n . Tenemos entonces una serie normal Z ZZ Z n = h1i B hp1 i B hp1 p2 i B 5. 6. 7. 8. 9. Z ZZ Z B hp1 p2 pn;1i B hp1p2 pn i = f0g Z (la normalidad es obvia, pues el grupo es abeliano) cuyos cocientes son p1  p2 : : : pn por el Corolario 5.8.9. Por tanto, si n = p1 p2 pn es una factorizacion de n en producto de primos positivos, la anterior es una serie de composicion de n. Observese que, como en la factorizacion de n se puede alterar el orden de los pi , podemos obtener otras series de composicion en las que los factores simples que aparecen son los mismos pero en orden distinto. Del apartado 4 deducimos que 4 tiene una serie de composicion de longitud 2 cuyos dos factores son isomorfos a 2. Como 4 no es simple, esto implica que `( 4) = 2. Exactamente lo mismo le pasa al grupo 2  2 (<compruebalo!), lo que nos dice que dos grupos no isomorfos pueden tener series de composicion con la misma longitud y los mismos factores. En otras palabras, las series de composicion de un grupo G no determinan a G, ni siquiera salvo isomorsmos. Si p es un entero positivo primo y r es la rotacion de angulo 2 =p en el grupo diedrico Dp , entonces Dp B hri B f1g es serie de composicion de Dp . En efecto, hri es normal en Dp por tener ndice 2, y los factores Dp =hri y hri son simples por tener orden primo. Como Dp no es simple, se tiene `(Dp ) = 2. >Puedes encontrar un grupo abeliano que tenga una serie con los mismos factores? Combinando este argumento con el del apartado 4, es facil ver que, si el entero n es producto de k primos, entonces Dn tiene una serie de composicion de longitud k + 1. La serie derivada de S4 es S4 B A4 B V B f1g, donde V = f1 (1 2)(3 4) (1 3)(2 4) (1 4)(2 3)g. E sta no es una serie de composicion porque V no es simple, pero puede renarse a la serie de composicion S4 B A4 B V B h(1 2)(3 4)i B f1g: Cambiando (1 2)(3 4) por cualquiera de los otros dos elementos no triviales de V se obtienen otras formas de renar la serie normal dada a una serie de composicion, pero es claro que las tres series de composicion que se obtienen son esencialmente iguales. Para n 5, la serie Sn B An B f1g es de composicion, por lo que `(Sn ) = 2. El grupo de los cuaterniones Q8 tiene subgrupos de orden 4 (por ejemplo, N = hii = fI i ;I ;ig) y de orden 2 (h;I i), y es claro que Q8 B N B Z B fI g es una serie de composicion. Mas generalmente, si G es un grupo de orden pn , con p primo, entonces G tiene una serie de composicion de longitud n con factores de orden p (Proposicion 8.3.4 y Ejercicio 8.2.6). Z ZZ Z Z Z Para proseguir con el estudio de las series de composicion necesitamos algunas propiedades basicas. Proposicion 9.3.5 Sea G un grupo con un subgrupo normal N y sea G = G0 D G1 D una serie normal de G. Entonces N = G0 \ N D G1 \ N D D Gn = f1g D Gn \ N = f1g es una serie normal de N . Si la serie de G es de composicion entonces, eliminando en la de N los terminos repetidos, se obtiene una serie de composicion de N . 212 CAPITULO 9. SERIES NORMALES Demostracion. Sean H y K dos subgrupos de G tales que H C K. Como K \ N es otro subgrupo de K, el Tercer Teorema de Isomorfa (5.7.9) nos dice que la interseccion H \ (K \ N) (que, obviamente, coincide con H \ N) es un subgrupo normal de K \ N. Es decir, H \ N E K \ N lo que claramente implica la primera parte del enunciado. El Tercer Teorema de Isomorfa nos da ademas un isomorsmo H(K \ N)  K \N  = H \N H y el mismo teorema aplicado a G, N y K nos dice que K \ N E K, por lo que H(K \ N) E K (Ejercicio 5.7.5) y as H(K \ N) E K H H (Teorema 5.6.5). Si el cociente K=H es simple, esta ultima condicion implica que H(K \ N)=H es trivial o simple, y en consecuencia lo mismo le pasa a (K \ N)=(H \ N). La demostracion de la segunda parte del enunciado a partir de este hecho es ahora evidente. Ejemplo 9.3.6 Intersecando series normales con subgrupos. Sea D8 = hr s j r8 = s2 = 1 srs = r;1 i el grupo diedrico. Entonces hr2  si = D4 . Una serie de composicion de D8 es D8 B hri B hr2i B hr4 i B f1g. Intersecando con D4 obtenemos la siguiente serie de composicion de D4 , que tiene un termino menos que la anterior porque, al intersecar con D4, los terminos segundo y tercero de la serie de D8 se transforman en el mismo: D4 B hri \ D4 = hr2 i \ D4 B hr4 i B f1g: Proposicion 9.3.7 Sea G un grupo con un subgrupo normal N y sea G = G0 D G1 D D Gn = f1g una serie normal G. Entonces G = G0N D G1 N D D GnN = f1g N N N N es una serie normal de G=N . Si la serie de G es de composicion entonces, eliminando en la de G=N los terminos repetidos, se obtiene una serie de composicion de G=N . Demostracion. Sean, como antes, H E K dos subgrupos de G. Como N es normal en G, los conjuntos HN y KN son subgrupos de G, y obviamente el primero esta contenido en el segundo. Armamos que, de hecho, HN E KN: En efecto, dados k 2 K y n 2 N, se tiene knHN = kHNn (pues n 2 N  HN) = HkNn (pues H E K) = HNkn (pues N E G): El Teorema de la Correspondencia nos dice ademas que HN=N E KN=N, de donde se sigue facilmente la primera armacion. El Tercer Teorema de Isomorfa aplicado a HN E KN y a K KN nos dice que K \ HN E K y K  KHN KN K \ HN = HN = HN : Ademas, el cociente K=(K \ HN) es a su vez un cociente de K=H pues, por el Segundo Teorema de Isomorfa, K  K=H K \ HN = (K \ HN)=H : Por tanto, si K=H es simple entonces el cociente K=(K \ HN) es o bien trivial o bien simple, y ahora la segunda parte del enunciado es clara.  GRUPOS DE LONGITUD FINITA 9.3. SERIES DE COMPOSICION 213 Ejemplo 9.3.8 Transportando series normales a cocientes. Sea G = hgi un grupo cclico de orden 12, y consideremos la serie de composicion G = hgi B hg3 i B hg6 i B hg12i = f1g. Aplicando la Proposicion 9.3.7 con el subgrupo normal hg4 i obtenemos la siguiente serie de composicion de G=hg4i  = 4: Z G = hg g4 i=hg4 i = hg3  g4 i=hg4 i B hg6  g4 i=hg4 i B hg12  g4i=hg4 i = f1g (notese que hg g4 i = hg3  g4i = hgi, que hg6  g4i = hg2 i y que hg12 g4 i = hg4 i). Proposicion 9.3.9 Sea G un grupo con un subgrupo normal N , y sean G G0 G1 Gn N = N B N B B N = f1g series normales de G=N y N . Entonces G = G0 B G1 B y N = Gn B Gn+1 B B Gn+m = f1g B Gn+m = f1g B Gn;1 B Gn B Gn+1 B es una serie normal de G. Si las series de G=N y de N son de composicion, entonces lo es la de G. Demostracion. Observamos primero que, por el Teorema de la Correspondencia, una serie normal de G=N debe ser de la forma dada, donde cada Gi (con i = 0 1 : : :  n) es un subgrupo de G que contiene a N. El mismo teorema aplicado a la relacion Gi+1 =N E Gi =N (con i = 0 1 : : :  n ; 1) nos dice que Gi+1 E Gi, lo cual prueba la primera armacion. Ademas, por el Segundo Teorema de Isomorfa, Gi=N  Gi Gi+1=N = Gi+1 para i = 0 1 : : :  n ; 1, de modo que los factores de la serie de G que hemos construido son los de las dos series iniciales, y ahora la segunda parte del enunciado es clara. Como consecuencia de los tres resultados anteriores se obtiene: Teorema 9.3.10 Sea G un grupo con un subgrupo normal N . Entonces G es de longitud nita si y solo si lo son a la vez N y G=N . Corolario 9.3.11 Todo grupo nito G es de longitud nita2 . Demostracion. Demostramos el resultado por induccion en el orden del grupo. Si jGj = 1 el resultado es obvio. Supongamos pues que jGj > 1. Si G es simple, entonces G B f1g es una serie de composicion. En otro caso G posee un subgrupo N normal, propio y no trivial por hipotesis de induccion, N y G=N tienen longitud nita, y entonces G tambien la tiene por el Teorema 9.3.10. Una serie normal que no sea de composicion se puede renar. Si es posible renarla hasta un punto en el que no se pueda renar mas, habremos obtenido una serie de composicion. Esto no es siempre posible. Por ejemplo, los siguientes renamientos sucesivos Z B f0g Z B 2Z B f0g ZZ B 2Z B 4Z B f0g Z B 2Z B 4Z B 8Z B f0g : : : nunca daran una serie de composicion de . Otra consecuencia importante del Teorema 9.3.10 es que este proceso s tiene n cuando trabajamos con un grupo de longitud nita: 2 Como consecuencia de este resultado, clasicar los grupos nitos se divide en las dos siguientes tareas: Clasicar los grupos simples nitos y determinar las posibles maneras de \componer" grupos simples nitos en series de composicion. Sobre la primera tarea, vease la nota al pie en la pagina 156. Como hemos visto en los Ejemplos 9.3.4, existe mas de una forma de componer dos grupos, y es todava un problema abierto conocer cuales son las posibles formas de componer los grupos simples nitos. CAPITULO 9. SERIES NORMALES 214 Corolario 9.3.12 Sea G un grupo de longitud nita. Entonces cualquier serie normal de G puede renarse a una serie de composicion. Demostracion. Sea G = G0 B G1 B G2 B B Gm = f1g una serie normal de G. Por el Teorema 9.3.10, cada Gi=Gi+1 tiene una serie de composicion Gi0 Gi1 Giki Gi Gi+1 = Gi+1 B Gi+1 B B Gi+1 B f1g (observese que hemos escrito los subndices de forma que Giki 6= Gi+1 viendo como sigue la demostracion, el lector puede analizar la ventaja de esta notacion). \Pegando" estas series como en la Proposicion 9.3.9, obtenemos la serie G = G00 B G01 B G10 B G11 B Gm0 B Gm1 B B G0k0 B B G1k1 B B Gmkm B f1g que es un serie de composicion de G. Corolario 9.3.13 Un grupo resoluble G es de longitud nita si y solo si es nito. En este caso G tiene una serie de composicion con todos sus factores de orden primo. Demostracion. Una implicacion es consecuencia directa del Corolario 9.3.11. Recprocamente, supongamos que G es resoluble y de longitud nita. Por el Corolario 9.3.12, su serie derivada se rena a una serie de composicion G = G0 B G1 B B Gn = f1g cuyos factores han de ser abelianos (>por que?). Por el Ejercicio 6.2.10, cada uno de esos factores Gi;1=Gi (para i = 1 : : : n) debe tener orden primo pi , y en particular debe ser nito. Aplicando reiteradamente el Teorema de Lagrange, deducimos que G es nito de orden p1 pn . Ejemplos 9.3.14 Series de composicion de grupos abelianos y resolubles. 1. Sea G un grupo abeliano nito con descomposicion invariante G = hxihyihz i, donde o(x) = 12, o(y) = 6, o(z) = 3. Entonces f0g C hx6i C hx2i C hxi C hxi  hy3 i C hxi  hyi C hxi  hyi  hz i = G es una serie de composicion de G. >Puedes dar otras series de composicion de G usando la misma idea? 2. En los Ejemplos 9.3.4 vimos como la serie derivada de S4 se renaba a una de composicion. 3. Sea D8 = hr s j r8 = s2 = 1 srs = r;1 i el grupo diedrico. El subgrupo hr2i es normal, pues (r2 )s = r;2 2 hri, y el cociente D8 =hr2i es abeliano por tener orden 4. Como el cociente D8 =hr4i no es abeliano (>por que?, >a que grupo de los de orden 8 es isomorfo?), deducimos que D8 B hr2 i B f1g es la serie derivada de D8 . Un renamiento a una serie de composicion es D8 B hri B hr2i B hr4 i B f1g: Los ejemplos del inicio de esta seccion sugieren que puede haber una cierta condicion de unicidad sobre las series de composicion de un grupo de longitud nita G, especialmente en cuanto al tipo de isomorfa de los grupos simples que aparecen como factores, y desde luego no en cuanto al orden en el que estos aparecen. El resultado fundamental en este sentido es el Teorema de Jordan-H#older, que asegura que esa sospecha es cierta. Comencemos deniendo con precision el concepto de \igualdad salvo el orden" para dos series de composicion de un grupo de longitud nita.  GRUPOS DE LONGITUD FINITA 9.3. SERIES DE COMPOSICION 215 De nicion 9.3.15 Si un grupo G tiene series de composicion, dos de ellas se dicen equivalentes si tienen la misma longitud y, salvo el orden y salvo isomorsmos, los mismos factores. Explcitamente, dos series de composicion G = G0 B G 1 B B Gn = f1g y G = H0 B H1 B B Hm = f1g son equivalentes si n = m y existe una permutacion  2 Sn tal que, para cada i = 0 1 : : :  n ; 1, existe un isomorsmo Gi  H (i) Gi+1 = H (i)+1 : Ejercicio 9.3.16 Demostrar que la relacion recien denida es una relacion de equivalencia en el conjunto de todas las series de composicion de un grupo de longitud nita. Teorema 9.3.17 (Jordan-Holder) Si un grupo G tiene longitud nita, entonces todas sus series de composicion son equivalentes. Demostracion. Sea n = `(G). Demostraremos el teorema por induccion en n, con el caso n = 1 resuelto porque entonces G es simple y G B f1g es su unica serie de composicion. En el caso general, es claro que la hipotesis de induccion puede enunciarse del modo siguiente: \Si un grupo tiene una serie de composicion de longitud menor que n, entonces todas las series de composicion de ese grupo son equivalentes". Fijemos una serie de composicion de G G = G0 B G1 B G2 B B Gn = f1g (9.3.2) de longitud mnima n. Por la transitividad de la relacion, el teorema quedara demostrado si vemos que cualquier otra serie de composicion de G, digamos G = H0 B H1 B H2 B B Hm = f1g (9.3.3) es equivalente a la serie (9.3.2). Observemos que G1 B G2 B B Gn = f1g (9.3.4) H1 B H2 B B Hm = f1g (9.3.5) son series de composicion de G1 y de H1, y que la primera de ellas nos permite aplicar la hipotesis de induccion a G1 . Consideremos primero el caso en el que G1 = H1. Entonces (9.3.4) y (9.3.5) son dos series de composicion de G1, y por la hipotesis de induccion ambas son equivalentes, de modo que n ; 1 = m ; 1 y as n = m. Ademas, los factores de ambas son los mismos (salvo quizas el orden), y a~nadiendo el factor G=G1 = G=H1 vemos que lo son los de las series (9.3.2) y (9.3.3), que por tanto son equivalentes, como queramos ver. Supongamos a partir de ahora que G1 6= H1 , y observemos que esto implica que ninguno de estos grupos contiene al otro: por ejemplo, si fuese G1  H1 , entonces H1 =G1 sera un subgrupo normal, propio y no trivial del grupo simple G=G1, lo cual es imposible. Como G1 y H1 son normales en G, tambien lo es su producto G1H1, que contiene estrictamente a G1 pues lo contrario implicara que H1  G1 . En consecuencia G1 H1=G1 es un subgrupo normal no trivial del grupo simple G=G1, de donde se sigue que G1H1 = G. Sea ahora K2 = G1 \ H1. E ste es un subgrupo normal tanto de G1 como de H1, y podemos identicar los correspondientes cocientes: H1 = H1  G1H1 = G : G1 H1 = G G1 = G1  y = K2 G1 \ H1 H1 H1 K2 G1 \ H1 = G1 G1 Por la Proposicion 9.3.5, K2 tiene una serie de composicion, digamos K2 B K3 B B Kl = f1g: (9.3.6) 216 CAPITULO 9. SERIES NORMALES Como G1=K2 y H1 =K2 son simples, las siguientes son series de composicion de G1 y de H1: G1 B K2 B K3 B B Kl = f1g: (9.3.7) H1 B K2 B K3 B B Kl = f1g: (9.3.8) Ahora, la hipotesis de induccion aplicada a G1 nos dice que las series (9.3.4) y (9.3.7) son equivalentes, y por tanto n = l. En consecuencia, la serie (9.3.8) tiene longitud n ; 1 y podemos aplicar la hipotesis de induccion a H1, por lo que las series (9.3.8) y (9.3.5) son equivalentes en particular n ; 1 = m ; 1 y as n = m. Por ultimo, las series iniciales (9.3.2) y (9.3.3) son equivalentes pues los siguientes conjuntos van siendo iguales cada uno al siguiente (las justicaciones de todas las igualdades o bien son claras o bien estan en los parrafos anteriores identifcalas): Factores de (9.3.2). G=G1 y los factores de (9.3.4). G=G1 y los factores de (9.3.7). G=G1, G1=K2 y los factores de (9.3.6). G=H1, H1=K2 y los factores de (9.3.6). G=H1 y los factores de (9.3.8). G=H1 y los factores de (9.3.5). Factores de (9.3.3). Si el grupo G tiene longitud nita, los grupos simples que aparecen como factores de una (cualquiera) de sus series de composicion, con las repeticiones pertinentes, se llaman factores de composicion de G. Los dos corolarios que siguen son consecuencias inmediatas del Teorema de Jordan-H#older (en el segundo hay que usar ademas la Proposicion 9.3.9): Corolario 9.3.18 Si un grupo G tiene longitud nita, entonces todas sus series de composicion tienen la misma longitud `(G). Ejercicio 9.3.19 Sea G un grupo nito y resoluble de orden p1 pk, con los pi primos tal vez repetidos. Demostrar que `(G) = k. Corolario 9.3.20 Si un grupo G tiene longitud nita y N es un subgrupo normal, propio y no trivial de G, entonces `(G) = `(N) + `(G=N) `(N) < `(G) y `(G=N) < `(G): Aplicando el Corolario 9.3.12 se obtiene. Corolario 9.3.21 Si G es un grupo de longitud nita, cualquier serie normal de G con factores no triviales (es decir, con inclusiones propias) tiene longitud `(G). 9.4 Problemas 1. Sea f : G ! H un isomorsmo de grupos. Demostrar que: (a) Si G0 es el subgrupo derivado de G entonces f(G0 ) es el subgrupo derivado de H. (b) G es resoluble si solo si lo es H. (c) G es de longitud nita si solo si lo es H. En este caso, `(G) = `(H). 9.4. PROBLEMAS 217 2. Decidir sobre la verdad o falsedad de las siguientes igualdades para elementos a b c de un grupo: (a) a b c]] = a b] c]. (b) a bc] = a c]a b]c. (c) a b]c = a b]ab]c]. 3. Demostrar que el subgrupo derivado de un grupo G es un subgrupo caracterstico de G. Deducir que todos los terminos de la serie derivada de G son caractersticos, y por tanto normales, en G. 4. Probar que si G es resoluble y no trivial entonces G0 6= G. 5. Probar que si G es simple y resoluble entonces G es cclico de orden primo. 6. Dado un grupo cclico G = hgi de orden 72, consideremos la serie de composicion Z G = hgi B hg2 i B hg6 i B hg12i B hg36 i B hg72 i = f1g: 7. 8. 9. 10. 11. 12. 13. 14. 15. 16. 17. 18. Calcular la serie de composicion de G=hg18i  = 18 que se obtiene al aplicar la Proposicion 9.3.7 con el subgrupo normal hg18i. Formar todas las series de composicion del grupo cclico de 20 elementos. Dar una serie de composicion del grupo de 21 elementos construido en el Problema 23 del Captulo 8. Probar que, si H y K son dos subgrupos normales de G, entonces H K] = hh k] : h 2 H k 2 K i es un subgrupo normal de G que esta contenido en H \ K. Demostrar que todo grupo de orden menor que 60 es resoluble. Sea n 5 un entero. Encontrar   2 An tales que  ] = (1 2 3), y usar esto para demostrar que An no es resoluble sin utilizar el Teorema de Abel. Probar que si p q r son primos distintos tales que pq < r entonces todo grupo nito de orden pqr es resoluble. Probar que todo grupo de orden 56, 63 o 440 es resoluble. Si G es un grupo resoluble nito cuyo orden se factoriza como Z jGj = p1 1 p2 2 pr r  demostrar que la longitud de composicion de G es `(G) = 1 + 2 + + r y que sus factores de composicion son los pi, cada uno repetido i veces. Sea G un grupo de orden 2925. (a) Probar que G tiene al menos dos subgrupos normales. (b) Mostrar que G es resoluble. (c) Calcular su longitud de composicion. Demostrar: (a) Todo grupo de orden 45 es abeliano. (b) Todo grupo de orden 765 es resoluble. (c) Sea G un grupo de orden 765. Construir una serie de composicion para G. Sea G un grupo nito con un subgrupo normal N de ndice 351 tal que N tiene una serie de composicion de longitud 6. Encontrar la longitud de la composicion de G. Demostrar que G  H es resoluble precisamente si G y H lo son. CAPITULO 9. SERIES NORMALES N 218 19. *] Demostrar que para cada n 2 existe un grupo resoluble G tal que G(n) 6= f1g. Utilizar esto para mostrar que el producto directo innito de grupos resolubles puede no ser resoluble. (Indicacion: En la primera parte, usar el isomorsmo S3  = Aut(S3 )). 20. Dar un ejemplo de un grupo resoluble que no sea de longitud nita, y otro de un grupo de longitud nita que no sea resoluble. 21. Dar una serie de composicion para cada uno de los grupos de orden 15. 22. Calcular la serie derivada de cada uno de los grupos de orden 15. 23. Demostrar que si H es un subgrupo normal de un grupo G de longitud nita, entonces G tiene una serie de composicion en la que aparece H. 24. Probar que si un grupo G es resoluble y G=Z(G) es simple entonces G es abeliano. 25. Dado un grupo G de orden 33 13, probar que es resoluble y dar una serie de composicion. 26. Si G es un grupo de longitud nita y H es un subgrupo de G, >es cierto que `(H) `(G)? 27. Demostrar que, si G es un grupo con series de composicion y N es un subgrupo normal de G tal que `(N) = `(G), entonces se tiene N = G. 28. Probar que el grupo G del Problema 17 del Captulo 8 es resoluble y calcular una serie de composicion. 29. Encontrar una serie de composicion del grupo 3 4 de orden 12. 30. Escribir la serie derivada de los grupos diedricos Dn y del grupo de cuaterniones. 31. Probar que si G es un grupo no abeliano de orden p3 (con p primo), entonces G0 = Z(G) y G=G0  = p  p. 32. >Es cierta la siguiente armacion? Si todos los subgrupos propios de un grupo G tienen longitud nita, entonces G tiene longitud nita. 33. Demostrar que un grupo nito abeliano tiene una unica serie de composicion si y solo si es un p-grupo cclico, para algun primo p. >Es cierto esto si el grupo no es abeliano? 34. Sean H y K dos subgrupos normales de un grupo G. Demostrar que si G=H y G=K son resolubles, entonces G=H \ K es resoluble. 35. Demostrar que todo grupo de orden p2q con p y q primos es resoluble. 36. *] Demostrar que si G es un grupo no resoluble de orden n < 300, entonces n = 60 120 168 180 o 240. 37. *] Sea G un grupo. Para cada n 2 denimos el n-esimo centro Zn (G) de G por recurrencia de la siguiente forma: Z0 (G) = f1g. Supongamos que hemos denido Zn (G) que resulta ser un subgrupo normal de G. Entonces Zn+1 (G) es el unico subgrupo (normal) de G que verica Zn+1 (G)=Zn(G) = Z(G=Zn(G)): En particular, Z1 (G) es el centro de G. La cadena de subgrupos f1g = Z0 (G) E Z1 (G) E Z2 (G) E se conoce como la serie central (ascendente) de G. Se dice que G es nilpotente si Zn (G) = G para algun n es decir, si la serie central alcanza al grupo G en algun paso. Demostrar: (a) Todo grupo abeliano es nilpotente. (b) Zn (G) = fx 2 G : x y] 2 Zn;1(G) para todo y 2 Gg. (c) Todo p-grupo nito (p primo) es nilpotente. Z oZ ZZ N 9.4. PROBLEMAS 219 (d) Un grupo G es nilpotente precisamente si tiene una serie normal f1g = G0 C G1 C (e) (f) (g) (h) (i) (j) C Gn;1 C Gn = G tal que Gi=Gi;1  Z(G=Gi;i) para todo i = 1 2 : : :  n. Todo grupo nilpotente es resoluble. Dar un ejemplo de un grupo resoluble que no sea nilpotente. Si G es nilpotente y H es un subgrupo de G, entonces H es nilpotente. Si G es nilpotente y N es un subgrupo normal de G, entonces G=N es nilpotente. Dar un ejemplo de un grupo G con un subgrupo normal N tales que N y G=N sean nilpotentes y G no lo sea. Demostrar que si G y H son dos grupos nilpotentes, entonces G  H es un grupo nilpotente. Bibliografa del captulo Allenby 1], Cohn 10], Delgado-Fuertes-Xambo 11], Rotman 30]. 220 CAPITULO 9. SERIES NORMALES Captulo 10 Formas canonicas de matrices Se estudia un endomorsmo f de un espacio vectorial V de dimension nita sobre un cuerpo K. Usando el anillo de polinomios KX] como principal herramienta, se obtienen diversas representaciones matriciales comodas de f y se asigna a f una lista de invariantes (polinomios de KX]) que lo determinan salvo semejanza. Introduccion Muchos problemas, puramente matematicos o planteados por otras ciencias, llevan a la consideracion de un endomorsmo f de un espacio vectorial V de dimension nita n sobre un cuerpo K. Por A lgebra Lineal sabemos que, para cada base de V , hay una matriz n  n sobre K que representa a f. En general, hay bases que \se adaptan mejor" a f que otras, lo que se traduce en que las correspondientes matrices son mas sencillas, lo que suele dar informacion sobre el endomorsmo. Por tanto, es importante disponer de metodos para encontrar esas bases adecuadas y esas matrices sencillas, y este es el principal objetivo del captulo. En el camino, asociaremos a f unos invariantes que determinaran su clase de semejanza, por lo que tendremos una clasicacion de los endomorsmos de V analoga a la clasicacion de los grupos abelianos nitamente generados. En la primera mitad del captulo nos enfrentamos principalmente a los aspectos teoricos del problema. El camino, salvando las complicaciones tecnicas, es muy similar al que seguimos al clasicar grupos abelianos nitamente generados de hecho, bastara con pensar en grupos abelianos nitos, pues en la analoga que desarrollaremos la \parte libre" sera trivial. Tras recordar algunos resultados de A lgebra Lineal, en particular los conceptos de endomorsmos y matrices semejantes, estudiamos los subespacios de V en los que la restriccion de f induce un endomorsmo, llamados subespacios invariantes. Es sencillo ver que V es suma directa de subespacios invariantes indescomponibles, de modo que se plantea el problema de dar una descripcion precisa de estos. Para ello, denimos un producto de polinomios de KX] por vectores de V , que se comporta en gran medida como el producto de enteros por elementos de un grupo abeliano. Las nociones de orden de un elemento, periodo de un grupo y subgrupo cclico tienen sus analogos en este contexto: polinomio anulador de un vector (Anuf (v)), polinomio mnimo del endomorsmo (min(f)) y subespacio cclico (Kf]v), respectivamente. Esta analoga nos permite adaptar los argumentos del Captulo 7 para demostrar que los subespacios invariantes indescomponibles son precisamente los del tipo Kf]v, donde Anuf (v) es una potencia de un polinomio irreducible de KX]. Ademas, en una descomposicion de V como suma directa de subespacios invariantes indescomponibles, V = Kf]v1   Kf]vk , la lista de los anuladores Anuf (vi ) determina unvocamente la clase de semejanza de f. Una descomposicion V = Kf]v1   Kf]vk como la anterior nos permite formar matrices de f a partir de matrices de sus restricciones a los Kf]vi , y cada una de estas adopta formas sencillas para bases adecuadas de Kf]vi . Por tanto, el problema de encontrar matrices sencillas (y las correspondientes bases) para f se reduce a saber encontrar esas descomposiciones de V y a elegir bases adecuadas en cada Kf]v. A esto dedicamos casi todo el resto del captulo. 221  CAPITULO 10. FORMAS CANONICAS DE MATRICES 222 Hay varios modos tpicos de construir bases adecuadas para Kf]vi (siempre a partir de vi y de Anuf (Vi )), y segun cual usemos se obtiene una de las llamadas \formas canonicas" de f. Comenzamos describiendo la forma canonica primaria, y la usamos para demostrar el Teorema de Cayley-Hamilton, que nos da un modo practico para calcular min(f). Despues, describimos las formas canonicas de Jordan y damos algoritmos para su calculo. Hay una forma canonica \generalizada" que posee cualquier endomorsmo y otra \clasica", mas sencilla, pero que solo poseen los endomorsmos para los que min(f) se descompone en factores lineales. En particular, todo endomorsmo de un espacio vectorial complejo tiene una forma canonica de Jordan, y la estrecha relacion entre y permite asignar a cada endomorsmo de un espacio vectorial real un nuevo tipo de forma canonica. El captulo termina mostrando como las formas canonicas de Jordan pueden usarse para calcular potencias de matrices (lo que permite describir el termino general de ciertas sucesiones recurrentes) y para encontrar \descomposiciones aditivas" de matrices que son de gran utilidad en el estudio de las ecuaciones diferenciales. CR Objetivos del captulo Conocer la nocion de subespacio invariante, y entender por que las descomposiciones en suma directa de subespacios invariantes son utiles para encontrar matrices sencillas de un endomorsmo. Conocer los conceptos de polinomio mnimo y caracterstico (de un endomorsmo) y anulador (de un vector), las relaciones entre ellos y metodos para calcularlos. Comprender por que la clase de semejanza de un endomorsmo queda determinada por sus formas canonicas o por sus divisores elementales. Saber calcular las distintas formas canonicas de una matriz: primaria, de Jordan generalizada, de Jordan (si existe) y real de Jordan (para endomorsmos de espacios vectoriales reales). Saber calcular, a partir de las formas canonicas de Jordan, las potencias de una matriz y la descomposicion aditiva de un endomorsmo de un espacio vectorial real o complejo. Desarrollo de los contenidos 10.1 Representaciones matriciales de endomorsmos En todo este captulo, K sera un cuerpo y V sera un espacio vectorial sobre K, no nulo y de dimension nita n. Denotaremos por E = EndK (V ) el conjunto de los endomorsmos de V (aplicaciones Klineales V ! V ), y por Mn (K) al conjunto de las matrices cuadradas de tama~no n con entradas en K. Usaremos resultados tpicos de A lgebra Lineal que seran citados o enunciados sin demostracion. Para cada base ordenada B = fv1  : : :  vn g de V hay una biyeccion E ! Mn(K) f 7! fB que asocia cada endomorsmo f 2 E, con la matriz fB asociada a f en la base B es decir, la j-esima columna de fB esta formada por las coordenadas en la base B del vector f(vj ). Diremos que f esta representado por la matriz fB o que fB es la matriz del endomorsmo f en la base B. El isomorsmo inverso lleva una matriz A = (aij ) al unico endomorsmo f 2 E que, sobre cada elemento vj de la base B, actua de la forma siguiente: n X aij vi : f : vj 7! i=1 Observese que el orden en el que se escriban los elementos de la base inuye en la representacion de una matriz en esa base. Por tanto, consideraremos las bases como conjuntos ordenados, de forma que dos bases formadas por los mismos elementos ordenados de diferente forma se consideran diferentes. Fijado un endomorsmo f 2 E, sus representaciones en distintas bases seran, por lo general, distintas. Algunas de estas matrices pueden ser mas sencillas que otras (por ejemplo, porque tengan muchos 10.1. REPRESENTACIONES MATRICIALES DE ENDOMORFISMOS 223 ceros), y nuestro objetivo en este captulo es encontrar una base B de forma que la matriz fB sea lo mas sencilla posible, y ofrezca por tanto una buena informacion visual de f. Veamos algunos ejemplos: Ejemplos 10.1.1 Representaciones sencillas de endomorsmos. 1. Sea f la homotecia de razon  2 K (es decir, f(v) = v para cada v 2 V ). Entonces, para cualquier base B, la matriz fB es la matriz escalar correspondiente a  (con  en cada entrada de la diagonal y ceros en el resto). En este caso, no hay ningun problema que resolver, pues todas las representaciones matriciales de f son iguales. 2. Si V es el plano real 2, f es la rotacion de angulo =2 (antihorario) y B es la base canonica, entonces  0 ;1  fB = 1 0 : De hecho, se obtiene la misma matriz para cualquier base formada por dos vectores ortogonales de la misma longitud y ordenados adecuadamente. Podemos decir que estas bases \se ajustan bien" a f, y en general otras bases dan lugar a representaciones mas complicadas. 3. Sea V = K 2 y sea f(x y) = (4y ; 7x 7y ; 12x). Si B es la base canonica y B 0 es la base fv1 = (1 2) v2 = (2 3)g, entonces  7 4 1 0  y f = fB = ;;12 B 7 0 ;1 : R 0 Ahora B 0 es \mejor base" para f que la base canonica B, y fB nos permite describir a f como la reexion sobre la recta hv1 i en la direccion de la recta hv2 i. 0 Ademas de encontrar bases que nos den representaciones adecuadas de f, deseamos que las matrices sencillas que buscamos sean unicas en cierto sentido, de forma que podamos clasicar los endomorsmos de V a partir de esas matrices (que llamaremos formas canonicas). Pero todava no hemos dicho que entendemos por una matriz sencilla. Nuestro modelo inicial de matriz sencilla es una matriz diagonal es decir, una matriz con ceros en las entradas que no estan en la diagonal. Observese, por ejemplo, que es muy facil calcular las potencias de una matriz diagonal, o el producto de una matriz diagonal por una matriz arbitraria. Un vector v 2 V es un vector propio de f si existe un escalar  2 K (llamado valor propio de f asociado al vector propio v) tal que f(v) = v. Es claro que fB es diagonal si y solo si cada vector de B es un vector propio de f, de donde se deduce, por ejemplo, que la rotacion de los Ejemplos 10.1.1 no puede ser representada por una matriz diagonal (no es diagonalizable). Es decir, no todos los endomorsmos pueden representarse por una matriz diagonal, as que tendremos que ampliar (y por lo tanto complicar) nuestro concepto de matriz sencilla. Existen varias posibilidades para elegir estos modelos de matrices sencillas, y en consecuencia existen distintos tipos de formas canonicas, como veremos. En el desarrollo de la teora sera fundamental el anillo de polinomios KX] y sus propiedades de divisibilidad, que en algunos aspectos dependen del cuerpo K. Por ejemplo, el caso en el que K es algebraicamente cerrado es especialmente bueno. Dedicaremos una seccion especial al caso K = , que es el de mayor interes en numerosas aplicaciones de la teora. R El resto de esta primera seccion lo dedicaremos a establecer con precision la relacion entre E y Mn (K). Como ya hemos comentado, muchos de estos resultados son bien conocidos en A lgebra Lineal y nos limitaremos a citarlos. En E = EndK (V ) podemos considerar tres operaciones (las dos primeras son internas y en la tercera intervienen elementos de K, a los que llamaremos escalares): Dados f g 2 E y  2 K, se denen los elementos f + g, fg y f de E mediante su accion en elementos v 2 V por las formulas: (f + g)(v) = f(v) + g(v) (suma) (fg)(v) = f(g(v)) (producto o composicion) (f)(v) = f(v) (producto por escalares) La suma y el producto (composicion) dotan a E de una estructura de anillo (no conmutativo salvo que V tenga dimension 1), mientras que la suma y el producto por escalares lo dotan de una estructura  CAPITULO 10. FORMAS CANONICAS DE MATRICES 224 de K-espacio vectorial. Dado un escalar  2 K, representaremos por la misma letra a la homotecia de razon , o sea al elemento de E dado por (v) = v para cada v 2 V . Esta notacion no induce a error, pues el producto f (con f 2 E) no depende de si vemos a  como escalar o como endomorsmo (homotecia). Ademas, es claro que las homotecias conmutan con los endomorsmos (es decir, f = f) y que la aplicacion K ! E dada por  7!  es un homomorsmo inyectivo de anillos, por lo que podemos ver a K como un subanillo de E. Con la notacion usual de potencias, si f 2 E entonces f n (con n 1) denotara la composicion de f consigo mismo n veces. Es decir, si v 2 V entonces f 2 (v) = f(f(v)), y en general f n (v) = f(f n;1 (v)). Algo similar ocurre en Mn (K) con las operaciones usuales de suma y producto de matrices, y de producto de un escalar por una matriz. De nuevo Mn (K) es un anillo (no conmutativo si n > 1) y un K-espacio vectorial. Ademas, si a cada  2 K se le asigna la matriz escalar  = In (con  en cada entrada de la diagonal y ceros en el resto), se tiene un homomorsmo inyectivo de anillos K ! Mn(K) tal que A = A para cada A 2 Mn (K), independientemente de que lo veamos como producto de matrices o como producto de matriz por escalar. Para cada base B de V , la aplicacion E ! Mn (K) descrita anteriormente es un isomorsmo de anillos (de hecho, la forma de denir el producto de matrices esta pensada para que estas aplicaciones conserven el producto) y un isomorsmo de espacios vectoriales (lo que implica que dimK (E) = n2 ). En vista de los objetivos que nos hemos marcado, es esencial describir como se relacionan, para dos bases distintas B y B 0 de V , las matrices fB y fB . Para ello, denotaremos por CB B a la matriz del cambio de base de B a B 0 es decir, en la j-esima columna de CB B aparecen las coordenadas en B 0 del j-esimo vector de B. Esta matriz es invertible y su inversa es la matriz del cambio de base de B a B 0 es decir, CB;1B = CBB . La relacion que buscamos es la siguiente (para cualquier f 2 E): 0 0 0 0 0 ;1 fB CBB : fB = CB B fB CBB = CBB 0 0 0 0 0 En particular, si A y A0 son dos matrices que representan al mismo endomorsmo, entonces existe una matriz invertible Q tal que A0 = Q;1 AQ. Dos endomorsmos se dice que son semejantes si son representables por la misma matriz. Es decir f f 0 2 E son semejantes si y solo si existen bases B y B 0 de V tales que fB = fB0 . Dos matrices se dice que son semejantes si representan el mismo endomorsmo es decir, A A0 2 Mn (K) son semejantes si existen f 2 E y bases B y B 0 tales que fB = A y fB = A0. Claramente dos matrices A y A0 son semejantes precisamente si son conjugadas es decir, si existe una matriz invertible U tal que A0 = U ;1 AU. Es facil ver que las relaciones de semejanza son relaciones de equivalencia en E y Mn (K). Las correspondientes clases de equivalencia se llaman clases de semejanza y la clase de semejanza que contiene a x se denotara por x], tanto si x es un endomorsmo como si es una matriz. Fijada una base B de V , la aplicacion f] 7! fB ] 0 0 esta bien denida y dene una correspondencia biunvoca entre los conjuntos de clases de semejanza de endomorsmos y matrices. Utilizando esto podemos reescribir nuestros objetivos de las siguientes formas equivalentes: Dado un endomorsmo f, queremos encontrar una base B tal que la matriz fB sea sencilla. Dada una matriz A, queremos encontrar una matriz semejante a ella que sea sencilla. Por lo tanto, nuestro problema consiste en encontrar una base adecuada o una matriz invertible adecuada, lo que tecnicamente es la misma cosa. En las secciones que siguen desarrollaremos la teora para un endomorsmo f. A menudo se quiere trabajar directamente con una matriz A 2 Mn (K), y en este caso no hay mas que considerar A como un endomorsmo del modo que sigue: Se toma V = K n (vectores columna n  1) y se considera el endomorsmo A que lleva un vector v al producto Av (que vuelve a ser un vector columna n  1). De este modo se tiene A = (A )B , donde B es la base canonica de V = K n. 10.2. SUBESPACIOS INVARIANTES 225 10.2 Subespacios invariantes En general, cuando se estudia un objeto con cierto tipo de estructura (grupo, anillo, espacio vectorial...), es interesante considerar los subconjuntos para los que se conserva esa estructura. Ahora estamos estudiando un endomorsmo f de un espacio vectorial V , de modo que nos interesan los subespacios vectoriales W de V para los que f pueda seguir considerandose un endomorsmo. Para eso, necesitamos que los vectores de W sigan en W despues de aplicarles f, y esa es la idea inicial en las siguientes deniciones. De nicion 10.2.1 Un subespacio vectorial W de V se dice que es invariante por el endomorsmo f 2 E (o que W es f -invariante) si f(W )  W . En este caso, la restriccion de f a W es un endomorsmo de W es decir, f jW 2 EndK (W). Cuando trabajemos directamente con matrices, usaremos la siguiente denicion analoga: El subespacio W de K n es invariante por la matriz A 2 Mn (K) si es invariante por el endomorsmo A . Una descomposicion por subespacios invariantes de f es una descomposicion de V en suma directa de subespacios invariantes V = V1   Vn: En este caso, si fi es la restriccion de f a Vi (de modo que fi 2 EndK (Vi )), escribimos f = f1   fn y decimos que f es la suma directa de f1  : : :  fn. Ejercicio 10.2.2 Sean V un espacio vectorial, W un subespacio, f un endomorsmo de V y fw1 : : : wmg un sistema generador de W . Demostrar que W es f -invariante si y solo si f(wi ) 2 W para cada i = 1 : : : m. Ejercicio 10.2.3 Demostrar que los siguientes subespacios de V son invariantes por f : 1. hv f(v) f 2 (v) : : :i, donde v es cualquier vector de V . En particular, si v es un vector propio de f , entonces hvi es una recta invariante. 2. Ker g, donde g es cualquier endomorsmo de V que conmuta con f , es decir, tal que fg = gf . A menudo nos encontraremos con sumas directas del tipo f = f1   fn . Entonces, salvo que se diga lo contrario, Vi representara el dominio de fi es decir, fi es la restriccion de f al subespacio Vi de V , que entendemos que es invariante por f. En tal caso, si Bi es una base de Vi para cada i, entonces B = B1   Bn es una base de V . Consideramos el orden en B de forma que los elementos de Bi van delante de los de Bi+1 y el orden dentro de cada Bi se mantiene. La representacion matricial de f en la base B adquiere entonces la siguiente forma: 0 f1 B B f2 B B fB = B ... @ 1 2 fn Bn 1 CC CA donde se entiende que las cajas vacas estan formadas por ceros. Cuanto menores sean las dimensiones de los Vi , mas cerca estara la matriz fB de ser diagonal. El caso extremo ocurre cuando todos los Vi tienen dimension 1, en cuyo caso fB es diagonal (y por tanto f es diagonalizable), pero ya hemos observado que no siempre es posible conseguir esto. En general, nuestro objetivo es obtener una descomposicion de f lo \mas na" posible, lo que nos lleva a considerar los endomorsmo que \no se pueden renar": De nicion 10.2.4 Diremos que f es indescomponible si V no admite una descomposicion por subespacios invariantes de f formada por dos subespacios vectoriales no nulos. Una descomposicion indescomponible o primaria de f es una descomposicion f = f1  la que cada fi es indescomponible.  fn en  CAPITULO 10. FORMAS CANONICAS DE MATRICES 226 Ejercicio 10.2.5 Demostrar que la rotacion de angulo =2 en el plano real es un endomorsmo indescomponible. Observese que esta aplicacion viene dada por f(x y) = (;y x). Comprobar que, si consideramos dicha aplicacion en C 2, entonces f es descomponible. Ejemplos 10.2.6 Descomposiciones indescomponibles. 1. Sea V = K 3 y sea f el endomorsmo dado por f(x y z) = (;x ; 2y ; 2z x + y z): Entonces u = (0 ;1 1) verica f(u) = u. En consecuencia, V1 = hui es un subespacio f-invariante de V con base B1 = fug, la restriccion f1 = f jV1 es la identidad, y su matriz 1  1 en B1 es (1). Por su parte, los vectores v = (1 0 0) y w = (;1 1 0) verican f(v) = w y f(w) = ;v. As, W = hv wi es un subespacio f-invariante con base B20 = fv wg, y es facil ver que g = f jW es indescomponible si K = y no lo es si K = (g esta en la situacion del Ejercicio 10.2.5). Como ademas V = V1  W, tenemos la descomposicion indescomponible f = f1  g para el endomorsmo f si K = . La matriz asociada a f en la base B 0 = B1  B 0 = fu v wg es R R C 01 1 fB = @ 0 ;1 A : C 0 1 0 Supongamos ahora que K = . Si ponemos v1 = u, v2 = (1;i ;1 0) y v3 = (1+i ;1 0), entonces f(v2 ) = iv2 y f(v3 ) = ;iv3 , y por tanto, V2 = hv2 i y V3 = hv3 i son subespacios f-invariantes. Si fi es la restriccion de f a Vi (i = 1 2 3), entonces f = f1  f2  f3 es una descomposicion indescomponible de f. La matriz asociada a f en la base B = fv1  v2 v3g es 01 fB = @ i ;i 1 A: 2. Sea f : K 2 ! K 2 el homomorsmo dado por f(x y) = (x + y y). Vamos a ver que f es indescomponible. Si f fuera descomponible, existira una base B = fv1  v2g de K 2 formada por vectores propios es decir, de forma que f(vi ) = i vi para ciertos 1 2 2 K. Eso implicara que, para i = 1 2, el sistema homogeneo de ecuaciones lineales (1 ; i )x + y = 0 + (1 ; i )y = 0 tiene solucion no trivial y por tanto  1 ;  1  0 i 1 ;  i   = 0 es decir, 1 = 2 = 1. De aqu se deduce facilmente que la segunda coordenada de v1 y v2 es cero en contra de que la independencia lineal de v1 y v2 . 3. Sea f el endomorsmo de K 3 dado por f(x y z) = (x x + y x + z). Si ponemos v1 = (0 0 1), v2 = (0 1 1) y v3 = (1 1 1), entonces f(v1 ) = v1 , f(v2 ) = v2 y f(v3 ) = v2 + v3. Por tanto U = hv1 i y W = hv2  v3i son subespacios f-invariantes. Observese que la matriz asociada a f en la base B = fv1  v2 v3g es 1 01 fB = @ 1 1 A : 0 1 >Es V = U  W una descomposicion indescomponible? 10.3. ENDOMORFISMOS INDESCOMPONIBLES 227 La existencia de descomposiciones indescomponibles para cualquier endomorsmo esta garantizada por la siguiente proposicion. Proposicion 10.2.7 Todo endomorsmo f de un espacio vectorial de dimension nita n es suma directa de endomorsmos indescomponibles es decir, tiene una descomposicion indescomponible. Demostracion. Razonaremos por induccion sobre n. Si n = 1 entonces V no tiene subespacios vectoriales propios no nulos, y por tanto f es indescomponible. Supongamos que n > 1 y la hipotesis de induccion. Si f es indescomponible no hay nada que demostrar. En caso contrario f = f1  f2 para ciertos fi denidos en subespacios vectoriales propios V1 y V2 . Por hipotesis de induccion f1 = g1   gk y f2 = h1   hl con los gi y los hi indescomponibles, y entonces f = g1   gk  h1   hl es una descomposicion indescomponible de f. 10.3 Endomorsmos indescomponibles En este momento nos encontramos en una situacion muy similar a la que tenamos despues de la Proposicion 7.4.4. Observemos el paralelismo entre ellas: La Proposicion 7.4.4 (respectivamente, la Proposicion 10.2.7) muestra que todo grupo abeliano nitamente generado (respectivamente, todo endomorsmo de un espacio vectorial de dimension nita) admite una descomposicion indescomponible. Lo que hicimos a continuacion en el Captulo 7 fue caracterizar los grupos abelianos nitamente generados indescomponibles. Eso es lo que vamos a hacer ahora para los endomorsmos. Recordaremos sucintamente los pasos dados en el Captulo 7 para un grupo abeliano A: 1. Descomponiendo A = t(A)  L con L libre de torsion (Corolario 7.3.6), observabamos que si A es indescomponible, entonces es de torsion o libre de torsion (Corolario 7.4.3). El caso libre de torsion se resuelve utilizando el Corolario 7.2.9 y pasamos a suponer que A es de torsion y por tanto nito. 2. Descomponiendo A como suma directa de p-grupos (Proposicion 7.4.8) observamos que si A es indescomponible, entonces es un p-grupo (Corolario 7.4.9) y pasamos a suponer que A es un p-grupo. 3. Despues de demostrar un lema tecnico sobre los elementos de orden maximo (Lema 7.4.11), demostramos que los grupos nitos indescomponibles son los cclicos de orden pn con p primo o, equivalentemente, los grupos de periodo pn con un elemento de orden pn. Los pasos que vamos a dar para caracterizar los endomorsmos indescomponibles son los mismos, aunque apareceran algunos problemas tecnicos. Para poder aplicar los metodos del Captulo 7 a endomorsmos necesitamos versiones de los siguientes conceptos: orden, periodo y torsion. Estos tres conceptos vienen asociados a la multiplicacion ng de enteros (n 2 ) por elementos del grupo (g 2 A). La siguiente tabla es un peque~no diccionario de traduccion de conceptos que iremos desarrollando poco a poco. Grupo abeliano A Endomorsmo f : V ! V Enteros Polinomios KX] ( n ) ng = g+ : : : +g P v = P (f)(v) Periodo p(A) Polinomio mnimo min(f) Orden o(g) Polinomio anulador Anuf (v) Grupo cclico hgi Subespacio cclico Kf]v Z Paso 1 Z Z A partir de ahora f denota un endomorsmo del espacio vectorial de dimension nita V sobre el cuerpo K. El papel que representaba el anillo de los numeros enteros va a ser representado ahora por el anillo de polinomios KX]. Para ello denimos, para cada P = p0 + p1X + + pnX n 2 KX] y cada v 2 V , el producto: P v = p0v + p1 f(v) + + pnf n (v):  CAPITULO 10. FORMAS CANONICAS DE MATRICES 228 Podemos ver esto de otra forma: Como f conmuta con las homotecias, la Observacion 4.2.2 nos permite aplicar la Propiedad Universal del Anillo de Polinomios al homomorsmo K ! E (que lleva  2 K a la homotecia ) y X al elemento f 2 E. Obtenemos as un homomorsmo de anillos Sf : KX] ! E dado por KX] 3 P = p0 + p1 X + + pn X n 7! Sf (P) = P(f) = p0 + p1f + + pnf n 2 E: Entonces P v = P (f)(v) = p0v + p1f(v) + + pnf n (v): Utilizando que Sf es un homomorsmo de anillos se pueden hacer los siguientes ejercicios: Ejercicio 10.3.1 Comprobar que, para cualesquiera P Q 2 KX] y v w 2 V , se verica: 1. P(f)Q(f) = Q(f)P(f). 2. 1 v = v. 3. P (v + w) = P v + P w. 4. (P + Q) v = P v + Q v. 5. (PQ) v = P (Q v) y por tanto, P (Q v) = Q (P v). Ejercicio 10.3.2 Demostrar que: 1. Si v 2 V entonces el conjunto fP v : P 2 KX]g coincide con el subespacio hv f(v) f 2 (v) : : :i, que es invariante por f por el Ejercicio 10.2.3. Denotaremos este subespacio por Kf]v y lo llamaremos subespacio cclico generado por v. 2. Si P 2 KX] entonces Ker P (f) es un subespacio invariante por f (usar el Ejercicio 10.2.3). Recordemos que KX] es un dominio de ideales principales y que los polinomios monicos (es decir, con coeciente principal 1) forman un conjunto de representantes salvo asociados de KX] nf0g (Ejercicio 3.1.7). Por tanto, cada ideal no nulo de KX] esta determinado por el unico polinomio monico que lo genera. Como Sf es un homomorsmo de anillos, Ker Sf es un ideal de KX]. Observemos ademas que Sf es homomorsmo de espacios vectoriales, que la dimension de KX] es innita (el conjunto f1 X X 2 : : : g es linealmente independiente) y que la dimension de E es nita (y vale n2 ). Por tanto Sf no puede ser inyectivo, y en consecuencia Ker Sf es un ideal no nulo de KX]. De nicion 10.3.3 El polinomio mnimo de f , denotado por min(f), es el generador monico de Ker Sf = fQ 2 KX] : Q(f) = 0g: Es decir, un polinomio Q 2 KX] verica Q(f) = 0 si y solo si Q es multiplo de min(f). Si A es una matriz cuadrada con coecientes en K entonces llamamos polinomio mnimo de A, min(A), al polinomio mnimo del endomorsmo A es decir, min(A) = min(A ). Si B es una base de V , A = fB y P = p0 + p1 X + + pnX n 2 KX], entonces la matriz asociada a P(f) en la base B es P (A) = p0 + p1 A + + pnAn : Por tanto P (f) = 0 precisamente si P (A) = 0, de donde se deduce: Lema 10.3.4 Si A es la matriz asociada a f en una base, entonces min(f) = min(A). Por tanto, dos endomorsmos semejantes tienen el mismo polinomio mnimo y dos matrices semejantes tienen el mismo polinomio mnimo. 10.3. ENDOMORFISMOS INDESCOMPONIBLES 229 Ejemplo 10.3.5 Calculo del polinomio mnimo. Sea f el endomorsmo de K 3 del apartado 3 de los Ejemplos 10.2.6. Como la matriz asociada a f en cierta base es 01 1 A = fB = @ 1 1 A 0 1 2 y se ve facilmente que (A ; I) = 0, deducimos que min(f) = (X ; 1)2 . Si interpretamos la torsion de V como el conjunto de los elementos v de V tales que P v = 0 para algun 0 6= P 2 KX], acabamos de ver que todo elemento de V es de torsion. Por tanto el primero de los pasos que dabamos para grupos abelianos no es necesario para endomorsmos. Ademas el polinomio mnimo representa el papel del periodo. Paso 2 Primero necesitamos introducir el concepto correspondiente al de orden. Es facil comprobar que para cada v 2 V el conjunto fQ 2 KX] : Q v = 0g es un ideal de KX], y es no nulo pues claramente contiene a min(f). De nicion 10.3.6 El polinomio anulador de v 2 V por f , denotado por Anuf (v), es el generador monico de fQ 2 KX] : Q v = 0g: Es decir, un polinomio Q 2 KX] verica Q v = 0 si y solo si Q es multiplo de Anuf (v). Cuando el polinomio f este claro por el contexto escribiremos Anu(v) en lugar de Anuf (v). Los ejercicios y ejemplos que siguen son utiles a la hora de calcular en la practica los polinomios recien denidos. El primer ejercicio dice como se puede obtener Anuf (v), y como se puede usar Anuf (v) para estudiar el subespacio invariante cclico Kf]v. El segundo dice que para obtener min(f) basta con calcular Anuf (v) para los vectores de una base. Ejercicio 10.3.7 Dados un polinomio monico P = p0 + p1X + + pm;1 X m;1 + X m (de grado m) y un vector v 2 V , demostrar que las condiciones siguientes son equivalentes: 1. P = Anuf (v). 2. P v = 0 y Q v 6= 0 para todo polinomio no nulo Q con gr(Q) < m. 3. P v = 0 y Kf]v = hv f(v) f 2 (v) : : :i tiene dimension m. Asumiendo ahora que se cumplen esas condiciones equivalentes, demostrar que: 1. B = fv f(v) f 2 (v) : : :  f m;1 (v)g es una base de Kf]v. 2. P es el polinomio mnimo de la restriccion de f al subespacio cclico Kf]v generado por v. 3. La matriz asociada a la restriccion de f a Kf]v en la base del apartado 1 es 0 0 BB 1 fB = C(P ) = B B@ : 0: : : : : 0 ;p0 : : : 0 ;p1 : : : 0 ;p2 ::: ::: ::: : : : 1 ;pm;1 Esta matriz C(P) se llama matriz compa~nera del polinomio P . 0 0 1 ::: 0 0 1 CC CC : A Ejercicio 10.3.8 Si B = fv1 : : :  vng es una base de V (mas generalmente, si es un sistema generador), demostrar que min(f) = mcm(Anuf (v1 ) : : :  Anuf (vn )): 230  CAPITULO 10. FORMAS CANONICAS DE MATRICES Ejemplos 10.3.9 Calculo del polinomio mnimo. 1. Sea V = K 3 y sea f el endomorsmo del apartado 1 de los Ejemplos 10.2.6. Recordemos que hay una base B = fu v wg de V tal que f(u) = u f(v) = w f(w) = ;v: Entonces, es claro que Anuf (u) = X ; 1. Por otra parte, se tiene f 2 (v) = ;v, de modo que el polinomio P = X 2 + 1 verica P v = 0. Ademas, si Q = aX + b verica 0 = Q v = ;aw +bv, entonces Q = 0 (por la independencia lineal de v y w), y en consecuencia P = Anuf (v). Analogamente P = Anuf (w), y del Ejercicio 10.3.8 deducimos que min(f) = mcm(Anuf (u) Anuf (v) Anuf (w)) = mcm(X ; 1 X 2 + 1) = (X ; 1)(X 2 + 1) (excepto si la caracterstica de K es 2, en cuyo caso el mnimo comun multiplo vale X 2 + 1). 2. Sea V = K 3 y sea f el endomorsmo cuya matriz en la base canonica B = fe1  e2  e3g es 0 1 0 ;1 1 @ 0 1 0 A: 1 0 1 Como en el apartado anterior, se comprueba que Anuf (e2 ) = X ; 1 y que Anuf (e1 ) = Anuf (e3 ) = X 2 ; 2X + 2, por lo que min(f) = (X ; 1)(X 2 ; 2X + 2). Para cada P 2 KX] irreducible sea tP (V ) = fv 2 V : P i v = 0 para algun i 0g = fv 2 V : Anuf (v) es una potencia de P g: Claramente, tP (V ) es un subespacio vectorial f-invariante de V . De hecho, esto se puede deducir del Ejercicio 10.3.2 y del siguiente: Ejercicio 10.3.10 Demostrar que si P 2 KX] es irreducible, entonces tP (V ) = Ker P i(f), donde P i es la mayor potencia de P que divide a min(f). La siguiente proposicion es la version de la Proposicion 7.4.8 en el contexto de endomorsmos (comparense las demostraciones). Proposicion 10.3.11 Si P1 P2 : : :  Pk son los polinomios monicos irreducibles que dividen a min(f), entonces V = tP1 (V )   tPk (V ) es una descomposicion por subespacios invariantes por f en la que cada sumando es no nulo. Ademas, existen polinomios F1  : : :  Fk 2 KX] tales que cada proyeccion i : V ! tPi (V ) viene dada por i = Fi(f). Demostracion. Sea min(f) = P11 Pkk con cada i  1, y veamos en primer lugar que se tiene V = tP1 (V ) + + tPk (V ). Para ello ponemos Qi = P=Pi i , de modo que los Qi son coprimos y por tanto existen R1 : : :  Rn 2 KX] tales que 1 = Q1R1 + + Qk Rk . As, dado v 2 V se tiene v = Q1 R1 v + + Qk Rk v (10.3.1) y cada sumando Qi Ri v esta en tPi (V ) pues Pii Qi Ri v = Pii QiRi v = Ri (P v) = 0: P Probemos ahora que la suma es directa es decir, que si ki=1 vi = 0 con cada vi 2 tPi (V ), entonces cada vi = 0. Como Qi vi = 0 (>por que?), Anuf (vi ) divide a Qi . Ademas, Anuf (vi ) es una potencia de Pi y mcd(Pi Qi) = 1. Por tanto Anuf (vi ) = 1 o equivalentemente vi = 0, como queramos ver. Observese que la ecuacion (10.3.1) muestra que i = Qi Ri(f). Veamos por ultimo que los sumandos tPi (V ) no son nulos. En efecto, como Qi no es multiplo de P = min(f) se tiene Qi (f) 6= 0, por lo que existe v 2 V tal que w = Qi v 6= 0. Ahora es claro que w es un elemento no nulo de tPi (V ). 10.3. ENDOMORFISMOS INDESCOMPONIBLES 231 Ejemplo 10.3.12 Calculo de tp (V ). R R R Sea f el endomorsmo de 3 del apartado 1 de los Ejemplos 10.2.6. Ya vimos en el Ejemplo 10.3.9 que min(f) = (X ; 1)(X 2 +1), Anu(v1) = X ; 1 y Anu(v) = Anu(w) = X 2 +1. Por tanto hv1 i  tX ;1 ( 3) y hv wi  tX 2 +1 ( 3) y de hecho de la Proposicion 10.3.11 se deduce que se dan ambas igualdades. Usando la Proposicion 10.3.11 obtenemos la siguiente condicion necesaria para que un endomorsmo sea indescomponible, que es la version para endomorsmos del Corolario 7.4.9. Corolario 10.3.13 Si f es indescomponible entonces min(f) es potencia de un polinomio irreducible. El recproco del Corolario 10.3.13 no se verica. En efecto, ya vimos en el apartado 3 de los Ejemplos 10.2.6 un endomorsmo descomponible cuyo polinomio mnimo es una potencia de un irreducible (Ejemplo 10.3.5). Paso 3 Vamos a terminar la caracterizacion de los endomorsmos indescomponibles. Recordemos que en el Captulo 7 demostramos que los grupos abelianos nitos que son indescomponibles son los cclicos con orden una potencia de un primo. Ya hemos demostrado que si un endomorsmo f de un espacio vectorial V es indescomponible, entonces su polinomio anulador es una potencia de un polinomio irreducible. Tambien hemos dado una denicion de subespacio cclico (Ejercicio 10.3.2): Un subespacio de V se dice que es cclico (con respecto a f) si es de la forma Kf]v para algun v 2 V . El resultado que vamos a demostrar no puede parecerse mas al caso de grupos abelianos nitos: Proposicion 10.3.14 Un endomorsmo f de un espacio vectorial de dimension nita V es indescomponible precisamente si min(f) es una potencia de un irreducible y V es cclico (con respecto a f ). Antes de demostrar la Proposicion 10.3.14 necesitamos el siguiente resultado. Ejercicio 10.3.15 Sea f un endomorsmo de V tal que min(f) = P es irreducible en KX]. Consideremos el cuerpo F = KX]=(P). Denotamos por Q la clase Q + (P ) de Q 2 KX] en F . Demostrar: 1. La operacion Qv = Q v: dota a V de una estructura de espacio vectorial sobre F . 2. Un K -subespacio vectorial de V es un F -subespacio precisamente si es f -invariante. 3. Para cada subespacio f -invariante W de V existe otro subespacio f -invariante W 0 tal que V = W  W 0. 4. f es indescomponible precisamente si dimF (V ) = 1. Demostracion de la Proposicion 10.3.14. Supongamos primero que V es cclico, pongamos V = Kf]v, y que min(f) = P n con P 2 KX] irreducible. Si f no es indescomponible, entonces existe una descomposicion V = V1  V2 en subespacios f-invariantes no triviales. Sea v = v1 + v2, con vi 2 Vi . Como Vi es invariante, Kf]vi  Vi y por tanto la dimension de Kf]vi es estrictamente menor que la dimension de V . Por el Ejercicio 10.3.7 el grado de Anuf (vi ) coincide con la dimension de Kf]vi y el grado de Anuf (v) = P n coincide con la dimension de V = Kf]v. Por tanto, Anuf (vi ) = P mi , con mi < n, de donde se deduce que P m v = 0, con m = maxfm1  m2 g < n, de donde se deriva una contradiccion. En consecuencia, f es indescomponible. Supongamos ahora que f es indescomponible. Por la Proposicion 10.3.11, el polinomio mnimo min(f) es una potencia de un irreducible, pongamos min(f) = P n. Razonamos por induccion sobre n. Si n = 1 y v 2 V no es cero, entonces del Ejercicio 10.3.15 se deduce que V = Kf]v. Supongamos ahora que n > 1 y que se verica la hipotesis de induccion y pongamos W = P V = fP v : v 2 V g. Claramente W es un subespacio f-invariante no nulo. Por la Proposicion 10.2.7, la restriccion g de f a W tiene una descomposicion indescomponible y, como min(g) = P n;1, por hipotesis de induccion los subespacios que aparecen en esta descomposicion indescomponible son cclicos. Es decir, existen  CAPITULO 10. FORMAS CANONICAS DE MATRICES 232 w1 : : :  wk 2 W tal que W = Kf]w1   Kf]wk (= Kg]w1   Kg]wk ). Para cada i = 1 : : :  k sea vi 2 V tal que wi = P vi y sea V 0 = Kf]v1 + + Kf]vk : Vamos a ver que la suma anterior es directa. Si Q1 v1 + + Qk vk = 0, entonces Q1 w1 + + Qk wk = P (Q1 v1 + + Qk vk ) = 0 y, por tanto, Qiwi = 0. Eso implica que Anuf (wi) j Qi y como Anuf (wi) es una potencia de P , se deduce que P divide a Qi. Si ponemos Qi = PRi, entonces 0= n X i=1 Qi vi = n X i=1 Ri wi y por tanto Ri wi = 0, de donde deducimos que Qi vi = RiP vi = R wi = 0. Ademas P V 0 = P V y, por tanto, V = V 0 + Ker P(f). La restriccion h de f a Ker P(f) satisface las condiciones del Ejercicio 10.3.15. Como V 0 \ Ker P (f) es un subespacio h-invariante de Ker P(f), existe un subespacio h-invariante (o sea, f-invariante) K de Ker P (f) tal que Ker P(f) = (V 0 \ Ker P(f))  K. Entonces V = V 0 + Ker P(f) = V 0 + (V 0 \ Ker P (f)) + K = V 0 + K y V 0 \ K = V 0 \ Ker P(f) \ K = 0 de donde obtenemos la siguiente descomposicion en suma directa de subespacios f-invariantes: V = V 0  K = Kf]v1   Kf]vk  K: Como f es indescomponible K = 0 y k = 1 y el resultado esta probado. Algunas consecuencias Algunos de los resultados de los pasos anteriores tienen consecuencias interesantes aparte de la caracterizacion de los endomorsmos indescomponibles. El resto de esta seccion lo dedicamos a ver algunas de ellas. Corolario 10.3.16 Un endomorsmo f de un espacio vectorial es diagonalizable precisamente si su polinomio mnimo es de la forma (X ; 1 ) (X ; k ) con los i distintos dos a dos. Demostracion. Supongamos que f es diagonalizable y sea v1 : : :  vn una base de vectores propios con valores propios 1  : : :  n. Entonces Anuf (vi ) = X ; i , para todo i y por tanto min(f) = mcm(Anuf (v1 ) : : :  Anuf (vn)) = (X ; 1) (X ; k ) donde 1  k son los diferentes valores propios. Supongamos que min(f) = (X ; 1) (X ; k ) con los i distintos. Por la Proposicion 10.3.11, V = tX ;1 (V )   tX ;k (V ) y la matriz asociada a la restriccion de f a tX ;i (V ) es i. Por tanto f es diagonalizable. Proposicion 10.3.17 Sean f g 2 E tales que fg = gf . Entonces existe una descomposicion V = V1   Vn y polinomios irreducibles P1  : : :  Pn Q1 : : :  Qn tales que, para cada i = 1 2 : : :  n se verican: 1. Vi es f -invariante y g-invariante. 2. Para todo v 2 Vi , Anuf (v) es una potencia de Pi y Anug (v) es una potencia de Qi .  PRIMARIA 10.4. DESCOMPOSICION 233 Demostracion. Por la Proposicion 10.3.11 y el Ejercicio 10.3.10 V = Ker R1 1 (f)   Ker Rl l (f) donde min(f) = R1 1 Rl k es la descomposicion irredundante de min(f) en producto de polinomios monicos irreducibles. Como g conmuta con f, tambien conmuta con Ri i (f) y por tanto Ker Ri i (f) es g-invariante (Ejercicio 10.2.3). Sea gi la restriccion de g a Ker Ri i (f). Aplicando de nuevo la Proposicion 10.3.11 y los Ejercicios 10.3.10 y 10.2.3 a gi obtenemos que Ker Ri i (f) = Vi1   Viki donde Vij es g-invariante y f-invariante y Anug (vij ) es una potencia de un polinomio irreducible para cualesquiera ndices i j y para cada v 2 Vij . \Pegando" estas descomposiciones se obtiene el resultado deseado. Corolario 10.3.18 Si f y g son dos endomorsmos diagonalizables de V tales que fg = gf , entonces existe una base B de V tal que fB y gB son diagonales (se dice que f y g son simultaneamente diagonalizables). Demostracion. Sean V = V1   Vn y P1 : : :  Pn Q1 : : :  Qn como en la demostracion de la Proposicion 10.3.17. Por el Corolario 10.3.16, para cada i = 1 : : :  n existen i y i tal que Anuf (v) = X ; i y Anug (v) = X ; i para todo v 2 Vi . O sea f(v) = iv y f(v) = i v. Pegando bases de los Vi obtenemos una base B de V tal que fB y gB son diagonales. 10.4 Descomposicion primaria Como consecuencia de las Proposiciones 10.2.7 y 10.3.14 se deduce. Teorema 10.4.1 Sea f un endomorsmo del espacio vectorial de dimension nita V sobre K . Entonces f = f1   fn , donde cada fi es un endomorsmo de un subespacio invariante cclico de V y min(fi ) es una potencia de un polinomio irreducible de KX]. De nicion 10.4.2 Una descomposicion f = f1   fn como la del Teorema 10.4.1 se llama descomposicion primaria de f . La lista de los polinomios min(fi ), con sus repeticiones si las hay, se llama la lista de divisores elementales de la descomposicion primaria. Teorema 10.4.3 Las listas de divisores elementales de todas las descomposiciones primarias de un endomorsmo son iguales salvo el orden. En consecuencia, podemos hablar de la lista de divisores elementales del endomorsmo. Demostracion. Supongamos que f = g11   g1k1  = h11   h1l1  son dos descomposiciones primarias de f con min(gij ) = Pimij y  gp1   gq1   gpkp  gqlq min(hij ) = Qni ij  siendo los Pi polinomios monicos irreducibles distintos entre s y los Qi polinomios monicos irreducibles distintos entre s. Por la Proposicion 10.3.14 existen v11  : : :  v1k1  : : :  vp1 : : :  vpkp  w11 : : :  w1l1  : : :  wq1 : : :  wqlq 2 V tales que V q l i Kf]v = pi=1 kj =1 ij = i=1 ji=1 Kf]wij  CAPITULO 10. FORMAS CANONICAS DE MATRICES 234 con Anuf (vij ) = Pimij y Anuf (wij ) = Qni ij . Entonces min(f) = mcm(Pimij : in j) = P1m1 Ppmp , donde n ij mi = max(mij : j). Analogamente, min(f) = mcm(Qi : i j) = Qn1 1 Qq q con ni = max(nij : j). Esto muestra que p = q y que, reordenando los Qi si es necesario, Pi = Qi y mi = ni para todo i. El lector puede comprobar como ejercicio que para cada i = 1 : : :  p i Kf]vij = li Kf]wij : tPi (V ) = kj =1 j =1 i gij = li hij , de manera que, para ver que mij = nij para cada par de Por tanto f jtPi (V ) = kj =1 j =1 ndices, podemos suponer que p = 1 es decir, que min(f) = P m para algun polinomio irreducible P. Es decir, podemos poner V = kj=1 Kf]vj = lj =1 Kf]wj donde Anuf (vj ) = P mj y Anuf (vj ) = P nj . Reordenando los vj y los wj , podemos suponer que los mj y los nj estan en orden decreciente. La demostracion estara terminada si vemos que mj = nj para cada j, y a continuacion vemos esto por induccion en j. Observese que P m1 = min(f) = P n1 , lo que resuelve el caso j = 1. Sea j > 1 y suponemos que mt = nt para todo t < j. Podemos suponer, sin perdida de generalidad que mj nj . Entonces P mj V = Kf]P mj v1   Kf]P mj vj ;1 = Kf]P mj w1   Kf]P mj wj ;1  Kf]P mj wj   Kf]P mj wk : Como, para cada t < j dimK Kf](P mj vt ) = gr(Anuf (P mj vt )) = gr(P mt ;mj ) = gr(P nt;mj ) = dimK Kf](P mj wt) deducimos que P mj wj = 0 y, por tanto mj = nj . La demostracion de la Proposicion 10.4.3 muestra que, para calcular la lista de los divisores elementales de un endomorsmo f, podemos suponer que su polinomio mnimo es potencia de un polinomio irreducible P. En este caso, todos los divisores elementales son potencias de P y se trata solo de decir cuantos de ellos son el propio P , cuantos son P 2, etcetera. El siguiente resultado nos da una formula que resuelve este problema. Proposicion 10.4.4 Sea f un endomorsmo de V tal que min(f) = P n con P irreducible de grado m en KX]. Para cada t = 1 : : :  n sea dt = dimK Ker P t(f) ; dimK Ker P t;1(f): Entonces el numero de divisores elementales de f de la forma P t es nt = dt ;mdt+1 : Demostracion. Pongamos k Kf]v V = nk=1 ni=1 ki k con Anuf (vki) = P , para todo i k. Observese que dimK Kf]vki = mk, para todo k e i. Por otro lado, para todo t = 1 : : :  n, nk t k Kf](P t v ) = n P t V = nk=1 ni=1 ki k=t+1 i=1 Kf](P vki) y Anuf (P t vki) = P k;t, para todo k > t. Luego, dimK Kf](P t vki ) = (k ; t)m y, por tanto, dimK P t(f)(V ) = dimK P t V = nk n X X k=t+1 i=1 dimK Kf](P t vki) = de donde dimK Ker P t(f) = dimK V ; dimK P t(f)(V ) = dimK V ; n X k=t+1 n X k=t+1 nk (k ; t)m nk (k ; t)m  PRIMARIA 10.4. DESCOMPOSICION y como consecuencia dt = n X k=t 235 nk (k ; t + 1)m ; n X k=t+1 nk (k ; t)m = m n X k=t nk  de donde se deduce facilmente que mnt = dt ; dt+1 . Sea f = f1   fk una descomposicion primaria de f asociada a la descomposicion V = V1   Vk . Como cada Vi es cclico, existen vectores vi tales que Vi = Kf]vi , y pondremos Qi = Anuf (vi ) = min(fi ), de modo que Q1 : : :  Qk son los divisores elementales de f. Como hemos visto en el Ejercicio 10.3.7, el conjunto Bi = fvi  f(vi ) f 2 (vi ) : : :  f mi ;1 (vi )g es una base de Vi (donde mi es el grado de Qi) y se tiene fi Bi = C(Qi). Por tanto, del Teorema 10.4.1 se deduce: Corolario 10.4.5 Para todo endomorsmo f de un espacio vectorial de dimension nita V existe una base B tal que 0 C(Q1) B C(Q2) fB = B B @ ... C(Qk ) 1 CC CA (10.4.2) donde (Q1  Q2 : : :  Qk ) es la lista de los divisores elementales de f y C(Qi) es la matriz compa~nera de Qi . La matriz de (10.4.2) se llama forma canonica primaria de f . A la forma canonica primaria se le puede \dar la vuelta". Supongamos que B es una base de V tal que fB es de la forma (10.4.2). Si descomponemos la base B de acuerdo con los bloques de la matriz (10.4.2) B = B1  B2   Bk , entonces cada Vi = hBi i es un subespacio f-invariante de V . Ademas, por la forma de la matriz compa~nera C(Qi), si vi es el primer elemento de Bi y mi es el grado de Qi , entonces Bi = fvi f(vi ) f 2 (vi ) : : :  f mi ;1(vi )g. Luego Vi = Kf]vi y fi = f jVi es indescomponible y por tanto f = f1  f2   fk es una descomposicion primaria de f. En conclusion, (Q1  Q2 : : :  Qk ) es la lista de divisores elementales de f y tenemos el siguiente corolario: Corolario 10.4.6 Dos endomorsmos son semejantes precisamente si sus listas de divisores elemen- tales coinciden salvo el orden, precisamente si sus formas canonicas primarias coinciden salvo el orden en el que escribimos las matrices compa~neras. Ejercicio 10.4.7 Demostrar que un endomorsmo es diagonalizable precisamente si su forma canonica primaria es diagonal. Ejemplos 10.4.8 Forma canonica primaria. R R R R Vamos a obtener la forma canonica primaria de los endomorsmos de los Ejemplos 10.2.6 y 10.3.9. 1. Sea f el endomorsmo de 3 dado por f(x y z) = (;x ; 2y ; 2z x + y z). Ya vimos en el Ejemplo 10.2.6 que 3 = hui  hv wi = f]u  f]v es una descomposicion indescomponible de f, donde u = (0 ;1 1), v = (1 0 0) y w = (;1 1 0). Tambien vimos en el Ejemplo 10.3.9 que Anuf (u) = X ; 1 y Anuf (v) = X 2 + 1. Entonces, la forma canonica primaria de f es  C(X ; 1) 1  01 @ 0 ;1 A C(X 2 + 1) = 1 0 Y la base en la que se obtiene la forma canonica primaria es fu v f(v) = wg. De hecho, ya habamos obtenido esta representacion matricial en el Ejemplo 10.2.6. 2. Sea f el mismo endomorsmo del ejemplo anterior pero considerado en 3 . En tal caso, (f)v no es indescomponible, como veamos en el Ejemplo 10.2.6. En dicho ejemplo veamos que f es diagonalizable, y claramente la forma diagonal es la forma canonica primaria. C C  CAPITULO 10. FORMAS CANONICAS DE MATRICES 236 3. Sea ahora f el endomorsmo de K 3 dado por f(x y z) = (x x + y x + z). En el Ejemplo 10.2.6 vimos que K 3 = hv1 i  hv2  v3i es una descomposicion indescomponible, donde v1 = (1 0 0), v2 = (0 1 1) y v3 = (1 1 1). Por tanto, hv2  v3i = Kf]v para algun vector v. Como f(v2 ) = v2, Kf]v2 = hv2 i y por tanto el vector v2 no puede hacer de v. Sin embargo, f(v3 ) = v2 + v3 es linealmente independiente con v3 y por tanto podemos coger v = v3. Como f 2 (v) = 2v2 + v3 = ;v3 + 2f(v), la matriz asociada a f en la base B = fv1 v f(v)g es 1 01 @ 0 1 A: 1 ;2 E sta es la forma canonica primaria de f y sus divisores elementales son X ; 1 = Anu(v1 ) y X 2 ; 2X + 1 = (X ; 1)2 = Anu(v). 4. Sea ahora f el endomorsmo de K 3 del segundo apartado de los Ejemplos 10.3.9. Ya vimos que el polinomio mnimo de f es (X ; 1)(X 2 ; 2X + 2). Supongamos que K = . Entonces, los polinomios X ; 1 y X 2 ; 2X + 2 son irreducibles y por tanto son los factores invariantes de f y la forma canonica primaria es 01 1 @ 0 ;2 A 1 2 2 Ya vimos que X ; 1 = Anuf (e2 ) y X ; 2X + 2 = Anuf (e1 ) y por tanto una base en la que se obtiene la forma racional es fe2 e1  f(e1 ) = e1 + e2 g. La unica forma que hemos visto hasta ahora para calcular el polinomio mnimo de un endomorsmo ha consistido en aplicar el Ejercicio 10.3.8. Sin embargo, hay un metodo mas efectivo, al que dedicamos el resto de la seccion. De nicion 10.4.9 Se llama polinomio caracterstico de una matriz A 2 Mn(K) al polinomio A = det(XI ; A) donde I 2 Mn (A) representa la matriz identidad. Dos matrices semejantes tienen el mismo polinomio caracterstico. En efecto, si A y B son matrices semejantes, entonces existe una matriz invertible U tal que B = U ;1AU. Luego, B = det(XI ; B) = det(XU ;1 U ; U ;1 AU) = det(U ;1(XI ; A)U) = det(U ;1) det(XI ; A) det(U) = det(XI ; A) = A : Por tanto todas las representaciones de un endomorsmo tienen el mismo polinomio caracterstico. Se llama polinomio caracterstico f de un endomorsmo f al polinomio caracterstico de cualquiera de sus representaciones matriciales. Ejercicio 10.4.10 Demostrar que el polinomio caracterstico de la matriz compa~nera C(P ) de un polinomio monico P es precisamente P . Utilizando el Corolario 10.4.5 y el Ejercicio 10.4.10 se demuestra facilmente el siguiente: Teorema 10.4.11 (Cayley-Hamilton) Si f es un endomorsmo entonces f (f) = 0, o equivalentemente min(f) j f . Ademas min(f) y f tienen los mismos divisores irreducibles. R R Ejemplo 10.4.12 Calculo de la forma canonica primaria. Consideremos el endomorsmo f = A de 5, donde 0 ;1 ;1 ;1 ;2 0 1 BB 1 0 0 2 0 CC A=B B@ 00 10 01 ;21 ;01 CCA : 0 0 0 0 1  10.5. FORMA CANONICA DE JORDAN 237 El polinomio caracterstico de A es (X ; 1)(X 2 + 1)2 . En vista del Teorema de Cayley-Hamilton (10.4.11), el polinomio mnimo de A es (X ; 1)(X 2 + 1) o (X ; 1)(X 2 + 1)2 . Comprobando que (A ; 1)(A2 + 1) 6= 0 deducimos que es el segundo y por tanto la forma canonica primaria de f ha de ser 1 01  BB 0 0 0 ;1 CC  C(X ; 1) B 1 0 0 0 CC : J= C((X 2 + 1)2 ) = B @ 0 1 0 ;2 A 0 0 1 0 Para obtener la descomposicion 5 = tX ;1 ( 5)  tX 2 +1 ( 5) observamos que las columnas de la matrices (A2 + I)2 y A ; I han de pertenecer a tX ;1 ( 5) y tX 2 +1 ( 5) respectivamente: Como 00 0 0 0 41 BB 0 0 0 0 ;4 CC (A2 + I)2 = B B@ 00 00 00 00 ;44 CCA 0 0 0 0 4 una base de tX ;1 ( 5) esta formada por el vector e0 = (1 ;1 1 ;1 1). Por otro lado, 0 ;2 ;1 ;1 ;2 0 1 BB 1 ;1 0 2 0 CC A;I = B B@ 00 10 ;11 ;20 ;01 CCA 0 0 0 0 0 de donde facilmente se deduce que una base de tX 2 +1 ( 5) esta formada por los cuatro primeros vectores de la base canonica C = fe1 : : :  e5g. Entonces, e1 = (1 0 0 0 0) f(e1 ) = (;1 1 0 0 0) f 2 (e1 ) = (0 ;1 1 0 0) f 3 (e1 ) = (0 0 ;1 1 0) forman una base B de W = tX 2 +1 ( 5) y por tanto la restriccion de f a W es indescomponible y W = f]e1. La forma canonica primaria de f se obtiene en la base B = fe0  e1 f(e1 ) f 2 (e1 ) f 3 (e1 )g. (Compruebese calculando explcitamente AU y UJ, donde U = CCB es la matriz de cambio de base.) Ejercicio 10.4.13 Resolver el ejercicio anterior considerando f = A como un endomorsmo de 5 . R RR R R R R R R 10.5 Forma Canonica de Jordan C En la seccion anterior hemos asociado a cada endomorsmo una matriz canonica llamada forma canonica primaria. En esta seccion vamos a ver una nueva forma canonica, llamada forma canonica de Jordan, que tiene mas aplicaciones (vease la Seccion 10.8). El proceso para conseguir la forma canonica primaria ha consistido en descomponer el endomorsmo f en una suma directa de endomorsmos indescomponibles f = f1   fk y despues, para cada fi , elegir una base del tipo vi  f(vi ) : : :  f n;1(vi ), donde fi es un endomorsmo de Kf]vi y n es la dimension de Kf]vi (y el grado del polinomio anulador de vi ). Podemos conseguir algo mejor si elegimos la base de otra forma aprovechando que el polinomio anulador es una potencia de un irreducible. De nicion 10.5.1 Sea f un endomorsmo del espacio vectorial V sobre K . Sea v 2 V tal que Anuf (v) = P n con P 2 KX] irreducible y monico de grado m. Entonces pondremos B(v) = Bf (v) = fX i P j v : 0 i < m 0 j < ng = fv f(v) f 2 (v) : : :  f m;1 (v) P(f)(v) (fP(f))(v) (f 2 P(f))(v) : : :  f m;1 P (f))(v) :::  P n;1(f)(v) (fP n;1 (f))(v) (f 2 P n;1(f))(v) : : :  (f m;1 P(f)n;1 )(v)g  CAPITULO 10. FORMAS CANONICAS DE MATRICES 238 Lema 10.5.2 Sea f un endomorsmo indescomponible de V . Sea min(f) = P n, con P irreducible y monico de grado m, y sea v 2 V tal que V = Kf]v. Entonces B = Bf (v) es una base de V y la matriz asociada a f en B = Bf (v) es 0 C(P ) BB N C(P ) B N C(P) fB = Jn(P) = B BB ... ... B@ N C(P) N donde hay n  n bloques, todos de tama~no m  m, y 00 0 B0 0 N =B B @ ... ... 0 0 1 0 . . . .. . 0 C(P) 1 CC CC CC  CA 1 CC CA 2 Mm (K): Recprocamente, si f es representable por una matriz del tipo Jn(P ), entonces f es indescomponible y P n es el polinomio mnimo de f . Demostracion. Por el Ejercicio 10.3.7, la dimension de V coincide con el grado nm del polinomio mnimo de f. Por tanto, para comprobar que B es una base, basta con demostrar que es un conjunto linealmente independiente. Pero esto es facil, ya que si mX ;1 nX ;1 i=0 j =0 ij X i P j v = 0 P P ;1  X i P j , se tiene Q(f)(v) = 0. Por tanto, Q es multiplo de P n, y entonces, haciendo Q = mi=0;1 nj =0 ij como gr(Q) < gr(P n) deducimos que Q = 0. Como es claro que los polinomios fXi P j : 0 i < m 0 j < ng son linealmente independientes, deducimos que ij = 0. El resto de la demostracion lo dejamos como ejercicio. De nicion 10.5.3 La matriz Jn (P) del Lema 10.5.2 se llama matriz de elemental de Jordan genera- lizada asociada al polinomio irreducible y monico P con multiplicidad n. Se llama matriz de Jordan generalizada a una matriz del tipo 0 J (P ) 1 k1 1 BB CC Jk2 (P2 ) (10.5.3) B@ CA .. . Jkm (Pm ) en la que cada Pi es un polinomio irreducible y monico de KX] (los Pi pueden repetirse). Corolario 10.5.4 Cada endomorsmo f 2 EndK (V ) es representable por una matriz de Jordan gene- ralizada, de forma unica salvo permutaciones en las matrices elementales de Jordan generalizadas que la componen. Ademas, en tal caso, los divisores elementales de f son P1k1  : : :  Pmkm . En consecuencia, toda matriz A 2 Mn(K) es semejante a una matriz de Jordan generalizada, unica salvo permutaciones en las matrices elementales de Jordan generalizadas que la componen. Demostracion. La existencia de una representacion por una matriz de Jordan generalizada es consecuencia del Teorema 10.4.1 y del Lema 10.5.2. Por otro lado, del Lema 10.5.2 se deduce que si f es representable por la matriz (10.5.3), entonces los divisores elementales de f son P1k1  : : :  Pmkm . De la unicidad de los divisores elementales (Teorema 10.4.3) se deduce la unicidad de la representabilidad por matrices de Jordan generalizadas.  10.5. FORMA CANONICA DE JORDAN 239 De nicion 10.5.5 Se llama forma canonica de Jordan generalizada de un endomorsmo (respectiva- mente, de una matriz) a cualquier matriz de Jordan generalizada que la represente (respectivamente, que sea semejante a ella). No distinguiremos en el orden en el que escribimos las matrices elementales que forman la forma canonica de Jordan generalizada y, por tanto, hablaremos de la forma canonica de Jordan generalizada. Dos endomorsmos son semejantes precisamente si sus formas canonicas de Jordan generalizadas son iguales. Ejemplo 10.5.6 Calculo de la forma de Jordan generalizada. Consideremos la matriz A del Ejemplo 10.4.12. Como Anuf (e1 ) = (X 2 +1)2 , podemos obtener la forma generalizada de Jordan de f con la base B 0 = fe0 g  Bf (e1 ) formada por e0 = (1 ;1 1 ;1 1) e1 = (1 0 0 0 0) f(e1 ) = (;1 1 0 0 0) (f 2 + 1)(e1 ) = (1 ;1 1 0 0) f(f 2 + 1)(e1 ) = (;1 1 ;1 1 0): Lo cual se puede ver comprobando la igualdad AU = UJ donde U es la matriz (invertible) cuyas columnas estan formadas por las coordenadas de la base B 0 y 1 01 BB 0 ;1 CC  J (X ; 1)  1 B C = J =B 1 0 J2(X 2 + 1) @ 0 1 0 ;1 CA 0 0 1 0 es la forma canonica de Jordan generalizada de A. Cuando los divisores elementales de un endomorsmo son potencias de polinomios lineales (de grado 1), su forma de Jordan generalizada toma un aspecto particularmente agradable. De nicion 10.5.7 Sean a 2 K y n 2 +. Se llama matriz elemental de Jordan de valor propio a y tama~no n a la matriz Jn (a) 2 Mn (K) dada por 0a 1 BB 1 a CC B CC 1 a Jn (a) = B B C Z ... ... @ 1 a A es decir, Jn(a) es la matriz elemental de Jordan generalizada asociada al polinomio irreducible X ; a, con multiplicidad n en smbolos, Jn(a) = Jn(X ; a). Una matriz de Jordan es una matriz del tipo 0 Jn (a1) B Jn (a2) B B @ 1 2 ... Jnk (ak ) 1 CC CA : (10.5.4) Como consecuencia del Corolario 10.5.4 se obtiene: Corolario 10.5.8 Las siguientes condiciones son equivalentes para un endomorsmo f de un espacio vectorial sobre el cuerpo K : 1. f tiene una representacion matricial que es una forma de Jordan con valores propios en K . 2. El polinomio mnimo min(f) es completamente descomponible. 3. El polinomio caracterstico f es completamente descomponible.  CAPITULO 10. FORMAS CANONICAS DE MATRICES 240 Ademas, si min(f) = (X ; a1)m1 (X ; ak )mk , con a1 : : :  ak 2 K distintos, entonces los valores propios de una matriz de Jordan J que represente a f son a1  : : :  ak y mi es el maximo de los tama~nos de las matrices elementales de Jordan, de valor propio ai , que forman J . Finalmente, todas las representaciones de f por matrices de Jordan son iguales, salvo el orden en que se escriben las matrices elementales de Jordan que las forman. Corolario 10.5.9 Las condiciones siguientes son equivalentes para una matriz A 2 Mn (K). 1. A es semejante a una matriz de Jordan con valores propios en K . 2. El polinomio mnimo min(A) es completamente descomponible en K . 3. El polinomio caracterstico A es completamente descomponible en K . Ademas, si min(f) = (X ; a1)m1 (X ; ak )mk , con a1 : : :  ak 2 K distintos, entonces los valores propios de una matriz de Jordan J semejante a A son a1  : : :  ak y mi es el maximo de los tama~nos de las matrices elementales de Jordan, de valor propio ai , que forman J . Finalmente, todas las matrices de Jordan semejantes a A son iguales, salvo el orden en que se escriben las matrices elementales de Jordan que las forman. Del Corolario 10.5.8 se deduce que no todos los endomorsmos son representables por una matriz de Jordan con valores propios en K (salvo si K es algebraicamente cerrado). Sin embargo, vamos a ver como podemos representar cada endomorsmo por una forma de Jordan con valores propios en un cuerpo que contenga a K como subcuerpo. Sea f un endomorsmo de un espacio vectorial V de dimension n sobre el cuerpo K. Para simplicar suponemos que V = K n y que f = A para una matriz A 2 Mn(K). Por el Corolario 4.4.4, existe un cuerpo K 0 que contiene a K como subcuerpo y tal que min(f) es completamente descomponible en K 0 . Si V1 = K 0 n , entonces f se puede extender de forma unica a un endomorsmo de EndK (V1 ). Este endomorsmo extendido, que tambien denotaremos por f, sera el endomorsmo que tiene por matriz asociada en la base canonica la matriz A. Por el Corolario 10.5.8, f es representable sobre K 0 por una matriz de Jordan. En resumen, como consecuencia del Corolario 10.5.8, tenemos. Teorema 10.5.10 Dado f 2 EndK (V ) existen: un cuerpo K 0 que contiene a K como subcuerpo un espacio vectorial V 0 sobre K 0 , que contiene a V como subespacio vectorial sobre K y de forma que una base de VK es una base de VK0 y un endomorsmo f 0 2 EndK (V 0) que se restringe a f sobre V , tales que f 0 es representable por una matriz de Jordan. Corolario 10.5.11 Para toda matriz A 2 Mn(K) existe un cuerpo K 0 que contiene a K como subcuerpo y tal que A es semejante a una unica matriz de Jordan sobre K 0 . La matriz de Jordan que representa a un endomorsmo f (o que es semejante a una matriz A) en algun cuerpo K 0 que contiene a K se llama forma canonica de Jordan1 de f (o de A). 0 0 0 10.6 Calculo efectivo En esta seccion vamos a ver como obtener efectivamente la forma canonica de Jordan de un endomorsmo o de una matriz. Para ello seguimos los pasos del siguiente algoritmo: Algoritmo de calculo de la forma de Jordan de un endomor smo f o una matriz A: Supondremos que f se descompone en factores lineales, bien por que eso sucede sobre K o porque consideramos un cuerpo K 0 que contiene a K como subcuerpo y en el que f descompone. 1. Calculamos el polinomio caracterstico: f (o A ). 2. Descomponemos f en la forma f = (X ; a1 )n1 (X ; ak )nk , donde a1  : : :  ak son distintos. Aunque pueden existir distintos cuerpos K que contengan a K y en los que min(f ) se descomponga en factores lineales, es posible demostrar que \el menor K con esa propiedad" es unico salvo isomorsmos. En el caso de mayor interes, K = R, ese cuerpo es R o C , dependiendo de si min(f ) se descompone o no en factores lineales en RX ]. 1 0 0  10.6. CALCULO EFECTIVO 241 3. Calculamos el polinomio mnimo. E ste sera el polinomio P = (X ; a1 )m1 : : :(X ; ak )mk con los mi mnimos entre los que veriquen P(f) = 0. Por el Teorema de Cayley-Hamilon (10.4.11), 1 mi ni , para todo i. 4. Para cada i = 1 : : :  k calculamos elementos vi1  : : :  viti de forma que Ker (f ; ai)mi = tji=1 Kf]vij Para hacer esto procedemos de la siguiente forma: Para simplicar pongamos a = ai , m = mi y los elementos vi1 : : :  viti que vamos a calcular los denotamos v1  : : :  vt Pongamos g = f ; a. Observese que Ker (g)  Ker (g2 )   Ker (gm;1 )  Ker (gm ) = Ker (gm+1 ) = y que si x 2 Ker (gi ), entonces g(x) 2 Ker (gi;1 ). Ademas, si x1 : : :  xk 2 Ker (gi ) y sus proyecciones en Ker (gi )=Ker (gi;1 ) son linealmente independientes, entonces las proyecciones de g(x1 ) : : :  g(xk ) en Ker (gi;1)=Ker (gi;2 ) son linealmente independientes (>por que?). Elegiremos v1  : : :  vj1  vj1+1  : : :  vj2  : : :  vjm 1 +1  : : :  vjm de forma que en cada la de la siguiente tabla, las clases de los elementos que aparecen a la izquierda en el espacio vectorial cociente de la derecha forman una base de este espacio: v1 : : : vj1 Ker (gm )=Ker (gm;1 ) g(v1 ) : : : g(vj1 ) vj1 +1 : : : vj2 Ker (gm;1 )=Ker (gm;2 ) g2 (v1 ) : : : g2 (vj1 ) g(vj1 +1 ) : : : g(vj2 ) vj2 +1 : : : vj3 Ker (gm;2 )=Ker (gm;3 ) ::: ::: ::: ::: ::: ::: ::: ::: ::: ::: ; 5. Cada uno de los vij proporcionara una matriz elemental de Jordan del tipo Jnij (ai ) donde nij viene determinado por la propiedad vij 2 Ker (f ; ai)nij n Ker (f ; ai )nij ;1 . 6. La base en la que la representacion matricial de f es su forma canonica de Jordan esta formada uniendo los conjuntos de la forma Bf (vij ) = fvij  (f ; ai )(vij ) (f ; ai)2 (vij ) : : :  (f ; ai)nij ;1(vij )g: Observese que cada uno de estos conjuntos corresponde a los elementos de una columna de la tabla del paso 4. Por tanto, los tama~nos de las cajas de la forma de Jordan coinciden con las alturas de las columnas de dicha tabla (sin contar las cajas vacas). 7. Si estamos buscando la forma de Jordan de una matriz A, entonces U ;1AU es la forma de Jordan, donde U se obtiene poniendo en columna las coordenadas de los elementos de la base descrita en el paso 6. Ejemplo 10.6.1 Calculo de la forma canonica de Jordan. 0 2 0 0 01 B ;1 1 0 0 CC : A=B @ 0 ;1 0 ;1 A Sea Entonces  X ; 2 0  A =  10 X 1; 1  ;1 ;1 1  1 1 2 0 0  0 0  = (X ; 1)(X ; 2)(X 2 ; 2X + 1) = (X ; 1)3(X ; 2): X 1  ;1 X ; 2  As que los valores propios de A son 1 y 2, de forma que la suma de los tama~nos de las matrices elementales de Jordan que forman la forma de canonica Jordan de A con valor propio 1 es 3 y solo hay 242  CAPITULO 10. FORMAS CANONICAS DE MATRICES una matriz elemental de Jordan con valor propio 2 y tama~no 1. Por tanto, la forma de Jordan A es una de las tres siguientes: 101 0 1 01 0 0 1 01 C B CC  BB 1 1 0 CC : BB 1 1 1 C B  A @ @ 1 A @0 1 1 A 1 2 2 2 Por el Teorema de Cayley-Hamilton (10.4.11), el polinomio mnimo de A es (X ; 1)n (X ; 2) con n = 1 2 o 3. Como 0 1 0 0 0 10 0 0 0 0 1 0 0 0 0 0 1 B ;1 0 0 0 CC BB ;1 ;1 0 0 CC = BB 0 0 0 0 CC 6= 0 (A ; I)(A ; 2I) = B @ 0 ;1 ;1 ;1 A @ 0 ;1 ;2 ;1 A @ 0 1 1 1 A 1 1 1 1 1 1 1 0 0 ;1 ;1 ;1 y 0 1 0 0 0 10 0 0 0 0 1 B ;1 0 0 0 CC BB ;1 ;1 0 0 CC = 0 (A ; I)2 (A ; 2I) = B @ 0 0 0 0 A @ 0 ;1 ;2 ;1 A 1 0 0 0 1 1 1 0 2 el polinomio mnimo de A es (X ; 1) (X ; 2). Eso implica que la forma de Jordan de de A es 1 01 0 B1 1 CC : J=B @ 1 A 2 Vamos a encontrar una matriz invertible U tal que U ;1 AU = J. Para ello primero buscamos un elemento en Ker (A ; I)2 n Ker (A ; I). El vector v1 = (0 1 0 0) satisface esta propiedad. Entonces v2 = (A ; I)v1 = (0 0 ;1 1) 2 Ker (A ; I). Ahora tenemos que completar v2 con un vector v3 de forma que v2 y v3 formen una base de Ker (A ; I) = f(0 x1 x2 x3) : x1 + x2 + x3 = 0g. Podemos elegir v3 = (0 1 ;1 0). Finalmente necesitamos un vector v4 2 Ker (A ; 2I). Calculando este nucleo observamos que v4 = (1 ;1 0 1) es una base de Ker (A ; 2I). Por tanto 00 0 0 11 B 1 0 1 ;1 CC U =B @ 0 ;1 ;1 0 A 0 1 0 1 es la matriz buscada. Compruebese la igualdad AU = UJ. Calculo de la forma de Jordan generalizada de un endomor smo f o una matriz A: El proceso de calculo de la forma canonica de Jordan generalizada de un endomorsmo o una matriz, en el caso en el que no exista la forma canonica de Jordan, es bastante similar al del caso en el que s exista. Enumeraremos aqu las diferencias con los siete pasos del algoritmo de calculo de la forma canonica de Jordan. 2. El momento en el que descubriremos que un endomorsmo no tiene forma canonica de Jordan sobre K sera al calcular, en el paso 2, la descomposicion del polinomio caracterstico f = P1n1 Pknk como producto de polinomios irreducibles de KX], si algun Pi tiene grado mayor que 1. 3. Entonces, en el paso 3, calcularemos el polinomio mnimo de forma similar, excepto que este debe tener la forma P1m1 Pkmk , con 1 mi ni para todo i. 4. En el paso 4, debemos calcular vi1  : : :  viti de forma que Ker Pimi (f) = tji=1Kf]vij : 10.7. MATRICES REALES DE JORDAN 243 Supongamos que P = Pi tiene grado n y pongamos m = mi , vj = vij y t = ti . Entonces, la busqueda de v1 : : :  vt se hace de forma similar a la explicada en el paso 4 cambiando en la tabla cada aparicion de un vi por vi  f(vi ) f 2 (vi ) : : :  f n;1(vi ): 5. Cada vij da lugar a una matriz de Jordan generalizada. 6. La base en la que se obtiene la forma de Jordan generalizada se obtiene \pegando" las bases Bf (vij ) de cada Kf]vij . Por tanto, las cajas de la forma de Jordan generalizada tienen tama~no hn donde h es la altura correspondiente al vector vi en la tabla. Ejemplo 10.6.2 Forma canonica de Jordan generalizada. Calculemos la forma canonica de Jordan generalizada de la matriz racional 0 1 ;1 ;3 0 1 B 1 ;1 ;2 0 CC 2 M ( ) A=B @ 0 0 ;2 ;1 A 4 0 ;1 3 0 Calculamos el polinomio caracterstico y obtenemos  X ; 1 1   ;1 X + 1 23 00  2  2 A =  0 0 X + 2 1  = (X + X + 1) :  0 1 ;3 X  Q C Q R Q R Como P = X 2 + X + 1 es indescomponible en X] y en X], A no tiene una forma de Jordan sobre ni sobre , aunque si la tendra sobre . Como 0 2 ;1 2 3 1 B 1 0 1 2 CC 6= 0 P(A) = A2 + A + 1 = B @ 0 1 0 1A ;1 0 ;1 ;2 el polinomio mnimo de A es P 2. Por tanto, la forma de Jordan generalizada de A es 0 1  B 01 ;;11  C(P ) CC : B = J= @ N C(P) 0 1 0 ;1 A 0 0 1 ;1 Para encontrar la matriz U tal que U ;1AU = J, buscamos v 62 Ker P(A), por ejemplo, v = (1 0 0 0). Entonces, las columnas de V estan formadas por v1 = v = (1 0 0 0) v2 = Av = (1 1 0 0) v3 = P (A)(v) = (2 1 0 ;1) v4 = (AP(A))(v) = (1 1 1 ;1) es decir, 01 1 2 11 B 0 1 1 1 CC U =B @0 0 0 1A 0 0 ;1 ;1 satisface U ;1 AU = J. 10.7 Matrices reales de Jordan Como hemos visto, para obtener la forma canonica de Jordan de un endomorsmo de un espacio vectorial (o una matriz) sobre un cuerpo K, a menudo tenemos que considerar un cuerpo mayor que el original. Esto no pasa si el cuerpo K es algebraicamente cerrado, pues el polinomio mnimo es completamente descomponible. Como no es algebraicamente cerrado, la forma canonica de Jordan de una matriz R  CAPITULO 10. FORMAS CANONICAS DE MATRICES RC 244 real es a menudo una matriz compleja, pero no real. Sin embargo, como y estan tan cercanos, es facil obtener una representacion matricial real de una matriz real (y por tanto de un endomorsmo de un espacio vectorial real) que es muy parecida a una matriz de Jordan. En efecto, sea f un endomorsmo de un espacio vectorial real V . Consideramos el espacio vectorial complejo V i] = fx + yi : x y 2 V g con la suma (x1 + y1 i) + (x2 + y2 i) = (x1 + x2) + (y1 + y2 )i y el producto por escalares (a + bi)(x + yi) = (ax ; by) + (ay + bx)i donde a b 2 . Dado v = x + yi 2 V i], pongamos Re (v) = x, Im (v) = y y v = x + yi. Consideramos V contenido en V i], identicando x 2 V con x+0i, y extendemos f a un endomorsmo del espacio vectorial complejo V i] poniendo f(x + yi) = f(x) + f(y)i: Sea W un subespacio de V i] invariante por f, de forma que la restriccion de f a W es indescomponible. Entonces, W = f]v y Anuf (v) = (X ; a)n para ciertos v 2 V i], a 2 y n 1. As que W = fv : v 2 W g = f]v y Anuf (v) = (X ; a)n  luego W tambien es un subespacio invariante por f y la restriccion de f a W es indescomponible. En consecuencia, por cada matriz elemental de Jordan de la forma Jn (a) con a 2 n que aparezca en la forma canonica de Jordan de f, existe otra de la forma Jn (a). Eso implica que V i] tiene una descomposicion primaria de la forma (10.7.5) V i] = f]u1   f]uk  f]v1  f]v1   f]vl  f]vl  donde uj 2 V Anuf (uj ) = (X ; aj )pj con aj 2 (j = 1 : : :  k) vj 2 V i] n V Anuf (vj ) = (X ; bj )qj con bj 2 n (j = 1 : : :  l) vj 2 V i] n V Anuf (vj ) = (X ; bj )qj con bj 2 n (j = 1 : : :  l) Fijemos un ndice j 2 f1 : : : lg, y pongamos v = vj  b = bj =  + i y q = qj : Entonces, v (f ; b)(v) : : :  (f ; b)q;1 (v) v (f ; b)(v) : : :  (f ; b)q;1(v) es una base de Tv = f]v  f]v. Pero tambien Bv = fRe (v) Im (v) Re ((f ; b)(v)) Im ((f ; b)(v)) : : :  Re ((f ; b)q;1 (v)) Im ((f ; b)q;1(v))g es una base de V i] como espacio vectorial sobre . Ademas, Bv  Bv i es una base de Tv como espacio vectorial sobre , de forma que Bv es una base de V \ Tv , y la matriz asociada a la restriccion de f a V \ Tv es 0  1 B CC ;  B B CC 1 0  B B CC 0 1 ;  B B CC  1 0  Jq ( ) = B B CC 0 1 ;  B B CC . . .. .. B B CA @ 1 0  0 1 ;  R C C C C R C C C C C C C C CCR RR CR 10.7. MATRICES REALES DE JORDAN 245 donde hay q  q bloques. Una matriz del tipo Jq ( ) se llama matriz elemental real de Jordan de tama~no q y valor propio  + i. Con los vectores ui de la descomposicion (10.7.5) se actua de igual modo, aunque en este caso solo hemos de quedarnos con las partes reales. La union, en el orden adecuado, de las bases as obtenidas nos da una base de V sobre en la que la matriz de f es 1 0 Jn1 (a1) CC BB ... CC BB Jnk (ak ) CC  BB Jm1 (b1  c1) CC BB ... A @ Jml (bl  cl ) con a1 : : :  ak  b1 c1 : : :  bl  cl 2 . Una matriz de este tipo se llama matriz real de Jordan. Como consecuencia, se tiene: Teorema 10.7.1 Sea f un endomorsmo de un espacio vectorial real V . Entonces f es representable R R por una matriz real de Jordan, unica salvo permutaciones de las matrices elementales reales de Jordan que la forman. Esta matriz se llama la forma canonica real de Jordan de f . Ejemplo 10.7.2 Formas canonicas de Jordan real y compleja. Veamos como podemos obtener la forma canonica real de Jordan de la matriz del Ejemplo 10.6.2 0 1 ;1 ;3 0 1 B 1 ;1 ;2 0 CC : A=B @ 0 0 ;2 ;1 A 0 ;1 3 0 Ya vimos que el polinomio mnimo de A es p;3 ; 1 + 2 2 2 2 (X + X + 1) = (X ; !) (X ; !) con ! = : 2 Por tanto la formas de Jordan compleja y real de A son 0! 0 1 0 ;1=2 p3=2 1 p B1 ! CC y J = BB ; 3=2 ;1=2 CC : p JC = B R @ A @ ! 0 1 0 ;p1=2 3=2 A 1 ! 0 1 ; 3=2 ;1=2 Para obtener una base del bloque con valor propio ! buscamos v 2 Ker (A ; !)2 n Ker (A ; !): Una forma rapida de obtener v es observar que, como (A ; !)2 (A ; !)2 = 0 entonces los elementos de la forma (A ; !)2 x pertenecen a Ker (A ; !)2 . En particular, las columnas de (A ; !)2 pertenecen a Ker (A ; !)2 . Calculando 1 0 1 + 3p;3 ;2 ; 2p;3 4 ; 6p;3 6 p p p B 2 + 2 ;3 ;3 ; ;3 2 ; 4 p;3 4p C C (A ; !)2 = 12 B @ 0 p2 ;3 ;;32 ;+ 36p;;33 ;2 7;+2 p;;33 A ;2 ;2 y 0 4 + 2p;3 ;9 ; p;3 4 + 2p;3 ;1 + 3p;3 1 p;3 p; 3 p;3 C B 1 3 + ; 6 3 + 2 2 C B p p (A ; !)(A ; !) = 2 @ 2p ;3 + ;3 2p 1 +p ;3 A  6 ;3 ; ;3 ;2 ;3 ;3 ; ;3 246  CAPITULO 10. FORMAS CANONICAS DE MATRICES deducimos que cualquiera de las columnas de (A ; !)2 pertenece a Ker (A ; !)2 n Ker (A ; !). Tomando p p v = (1 + 3 ;3 2 + 2 ;3 0 ;2) obtenemos que p p p v1 = v y v2 = (A ; !)v1 = (4 + 2 ;3 3 + ;3 2 ;3 ; ;3) forman una base del bloque correspondiente al valor propio !. As, p p p p p v1 = (1 ; 3 ;3 2 ; 2 ;3 0 ;2) y v2 = (A ; !)v1 = (4 ; 2 ;3 3 ; ;3 2 ;3 + ;3) forman una base correspondiente al bloque correspondiente al valor propio !. Por tanto, la matriz 0 1 + 3p;3 4 + 2p;3 1 ; 3p;3 4 ; 2p;3 1 B 2 + 2p;3 3 + p;3 2 ; 2p;3 3 ; p;3 CC U1 = B A @ 0 2 0 2 ;2 ;3 ; p;3 ;2 ;3 + p;3 satisface la ecuacion U1;1AU1 = JC . Tomando las partes reales e imaginarias de los vectores de las bases anteriores obtenemos la base u1 = (1p2 0p;2) u2 = (3 3 2 3 0 0) u3 = (4p3 2 p ;3) p u4 = (2 3 3 0 ; 3): Finalmente, vemos que la matriz 0 1 3p3 4 2p3 1 p p C B 2 2 3 3 3 C B U =@ 0 0 2 0p A ;2 0 ;3 ; 3 satisface la ecuacion U ;1AU = JR. 10.8 Aplicaciones Expresar un endomorsmo o una matriz en su forma canonica de Jordan tiene multiples aplicaciones y en esta seccion veremos algunas de ellas. Calculo de potencias La primera aplicacion es el calculo de potencias del endomorsmo o la matriz. Esto se basa en la sencilla formula (U ;1AU)n = U ;1 An U y en el hecho de que es facil calcular potencias de matrices de Jordan ya que 0 A1 1n 0 An 1 1 BB CC BB CC A2 An2 = B@ C B CA ... ... A @ Ak Ank y 0 an 1 0 0 0 ;  n an;1 n B CC 0 0 1  n;2 ;na n;1 B ; n n B CC : n a a a 0 Jm (a) = B ;n2an;3 ;n1an;2 ;nan;1 an B CA 2 1 @ 3 . . . . . .. .. .. .. . . El lector puede demostrar esta formula por induccion sobre n. 10.8. APLICACIONES 247 Ejemplo 10.8.1 Calculo de las potencias de una matriz. Vamos a calcular An , donde 01 BB 0 B0 A=B BB 0 @0 1 0 1 1 1 1 1 0 1 0 0C C 0 1 0 0 0C C: 0 1 1 0 0C C 0 0 0 0 1A 0 0 0 0 ;1 0 Para ello, calcularemos la forma canonica de Jordan de A. Empezamos calculando el polinomio caracterstico de A: A = (X ; 1)4 (X 2 + 1) = (X ; 1)4(X ; i)(X + i): As, el polinomio mnimo de A es de la forma (X ; 1)n (X 2 + 1) donde n 4. Como se tiene (A ; I)2 (A2 + I) 6= 0 y (A ; I)3 (A2 + I) = 0 el polinomio mnimo de A es (X ; 1)3 (X 2 + 1). De aqu, las formas canonicas compleja y real de Jordan de A son 01 0 0 B 1 1 0 B B 0 1 1 J =B B 1 B @ i ;i 1 01 0 0 CC B1 1 0 CC y BBB 0 1 1 CC BB 1 A @ Obtenemos la forma canonica compleja conjugando por la matriz 00 B 0 B B 1 U =B B 0 B @0 1 0 0 1 0 0 0 1 1 0 0 0 0 1 1+i 1;i 0 0 0 0 0 0 0 0 0 0 ;1 + i ;1 ; i 0 ;1 ; i ;1 + i es decir, U ;1AU = J. Por tanto, An = UJ nU ;1 donde 0 1 B B ;nn B 2 Jn = B B B@ 00 0 0 1 n 0 0 0 0 0 1 0 0 0 0 0 0 1 0 0 0 0 0 0 0 0 0 0 in 0 0 (;i)n 1 CC CC : CC 0 1A ;1 0 1 CC CC CC A 1 CC CC : CC A Termino general de una sucesion recurrente Otra aplicacion de la forma canonica de Jordan de una matriz es el calculo del termino general de una sucesion recurrente es decir, de una sucesion de numeros (an) de forma que cada termino an viene dado en funcion de los terminos anteriores. Ejemplo 10.8.2 Calculo del termino general de sucesiones recurrentes. Calculamos una formula para el termino general de la sucesion de Fibonacci 1 1 2 35 : ::  an = an;1 + an;2: Observese que se verica la siguiente formula a n;1 an   0 1  a  n;2 = 1 1 an;1 : 248  CAPITULO 10. FORMAS CANONICAS DE MATRICES Por tanto, si ponemos 0 1 A= a tendremos 1 1  a    = An;2 11 : p p Vamos a calcular An . El polinomio caracter stico dep A es X 2 ; X ; 1 = (X ; 1+2 5 )(X ; 1;2 5 ), luego A p es diagonalizable. Si ponemos  = 1+2 5 y = 1;2 5 entonces se comprueban facilmente las relaciones: n;1 an = An;2 1 a2 p  + = 1  ; = 5  = ;1 2 =  + 1  entonces U ;1AU = An = y, por tanto, a n;1 an  0 = J: 0  1 1  n 0  ; 1   ;1  n;1 ;5 n;1 n ; 0n n UJ nU ;1 = p1 = p15 Luego = + 1: 1 1 U= Si Luego 2  n ; n n+1 ; n+1 = An;2 1 = p15 n;2 ;  n1;3 ; n;3 n;2 n;2 ; n;2 n;1 ; n;1  1  1 an = p1 (n;2 ; n;2 + n;1 ; n;1 ) = p1 (n ; n ): 5 5 Descomposicion aditiva de Jordan N Teorema 10.8.3 Todo endomorsmo que tenga una forma de Jordan se puede escribir de forma unica como f = d + n, donde d es diagonalizable, n es nilpotente (es decir, existe k 2 tal que nk = 0) y dn = nd. Demostracion. Sea f un endomorsmo de un espacio vectorial V que tenga una forma de Jordan y supongamos que B es una base de V tal que 0 Jn (a1) 1 B fB = B B @ Jn2 (a2) ... 1 CC CA Jnk (ak ) es la forma canonica de Jordan de A. Cada matriz elemental de Jordan es de la forma Jni (ai ) = ai Ini + Jni (0): Claramente, la matriz Ni = Jni (0) es nilpotente. Sean d y n los endomorsmo de V tal que 1 0 N1 1 0 a1In 1 CC B CC B a2In2 N2 dB = B CA y nB = BB@ CA : B@ ... ... ak Ink Nk 10.8. APLICACIONES 249 Como dB es diagonal y nB es nilpotente, d es diagonalizable y n es nilpotente. Ademas, como fB = dB + nB y dB nB = nB dB , entonces f = d + n y dn = nd. Para ver la unicidad, observemos que d(v) = a1 1 (v) + + ak k (v) donde i : V ! tX ;ai (V ) es la proyeccion de acuerdo con la descomposicion de f de la Proposicion 10.3.11. Utilizando esta proposicion, deducimos que i = Pi (f) para algun polinomio Pi 2 KX] y por tanto d = P(f) para algun polinomio P 2 KX]. Sean ahora d0 diagonalizable y n0 nilpotente con f = d0 +n0 y tales que d0n0 = n0 d0. Entonces, tanto d0 como n0 conmutan con f y, por tanto, tambien conmutan con d = P(f) y con n = f ; d. De esto se deduce que n ; n0 es nilpotente y que d y d0 son simultaneamente diagonalizables (Corolario 10.3.18). Luego, d0 ; d = n ; n0 es diagonalizable y nilpotente. Esto implica que d0 ; d = n ; n0 = 0 (>por que?) y por tanto d = d0 y n = n0 . Corolario 10.8.4 Sea A 2 Mn(K) y K 0 un cuerpo que contiene a K como subcuerpo tal que A tiene una forma de Jordan en K 0. Entonces existen matrices unicas D N 2 Mn (K 0 ) tales que: A = D + N, DN = ND, D es diagonalizable en K 0. N es nilpotente es decir, existe un n 2 tal que N n = 0. Las descomposiciones del Teorema 10.8.3 y el Corolario 10.8.4 se llaman descomposiciones aditivas de Jordan. A pesar de que existen endomorsmos de espacios vectoriales reales para los que la forma de Jordan se obtiene sobre y no sobre , las descomposiciones aditivas de Jordan de matrices o endomorsmos sobre se pueden obtener sobre , como muestra la siguiente proposicion: Proposicion 10.8.5 Para todo A 2 Mn ( ) existen matrices unicas D N 2 Mn( ) tales que: 1. A = D + N , 2. DN = ND, 3. D es diagonalizable en . 4. N es nilpotente. Demostracion. Como A tiene una forma de Jordan sobre , A tiene una descomposicion de Jordan A = D + N sobre . Si denotamos por X la matriz formada por los conjugados de las entradas de X, entonces A = A = D + N, D es diagonalizable, N es nilpotente y D N = N D (>por que?). Luego A = D + N = D + N son dos descomposiciones aditivas de Jordan de A y, de la unicidad de estas descomposiciones deducimos que D = D y N = N es decir, D 2 Mn ( ) y N 2 Mn ( ). 1. 2. 3. 4. R C C C N RR R R C C R R Corolario 10.8.6 Todo endomorsmo de un espacio vectorial real de dimension nita se puede escribir de forma unica como suma de un endomorsmo diagonalizable (en ) y uno nilpotente. Ejemplo 10.8.7 Descomposicion aditiva de Jordan. Sea A la matriz del Ejemplo 10.8.1 y sean J y U como en dicho ejemplo. Entonces J = D1 + N1 es la descomposicion aditiva de Jordan de J donde 00 0 0 01 0 0 1 1 BB 1 0 0 BB 0 1 0 CC CC B B C CC : 0 0 1 0 1 0 CC y N1 = BB D1 = B BB CC 1 0 B@ CA @ i 0 A ;i 0  CAPITULO 10. FORMAS CANONICAS DE MATRICES 250 Por tanto la descomposicion aditiva de Jordan de A es A = D + N, con 01 0 0 0 1 11 0 BB 0 1 0 0 0 0 CC BB B C B 0 0 1 0 0 0 D = UD1 U ;1 = B BB 0 0 0 1 0 0 CCC y N = UN1U ;1 = BBB @0 0 0 0 0 1A @ 0 0 0 0 ;1 0 0 0 0 0 0 0 0 0 0 0 0 0 1 0 0 1 0 0 1 1 0 0 0 0 0 0 0 0 0 0 0 0 0 0 0 0 1 CC CC : CC A 10.9 Problemas 1. Demostrar que si una clase de semejanza de un endomorsmo o una matriz contiene un elemento invertible, entonces todos los elementos de dicha clase de semejanza son invertibles. 2. Demostrar que la matriz compa~nera de un polinomio P es invertible precisamente si P(0) 6= 0. Deducir que si P es el polinomio mnimo o el polinomio caracterstico de un endomorsmo f de un espacio vectorial de dimension nita, entonces f es invertible precisamente si P (0) 6= 0. En tal caso, obtener una formula para f ;1 en funcion de P. 3. Demostrar que si f es un endomorsmo de V , entonces el grado de min(f) es menor o igual que dimV . 4. Demostrar que si f es un endomorsmo de un espacio vectorial sobre tal que f n = 1 para algun n 1, entonces f es diagonalizable. 5. Demostrar que las siguientes condiciones son equivalentes para una matriz A 2 Mn (K). (a) A es nilpotente es decir, An = 0 para algun n 2 . (b) El polinomio caracterstico A es una potencia de X. (c) El polinomio mnimo min(A) es una potencia de X. (d) 0 es el unico valor propio de A en cualquier cuerpo que contenga a K como subcuerpo. (e) A tiene una forma canonica de Jordan formada por matrices elementales de Jordan con valor propio 0. 6. Sea f un endomorsmo idempotente (es decir, f 2 = f) de un espacio vectorial. Demostrar que f es diagonalizable. 7. Sea K un cuerpo. Demostrar que, si una matriz N 2 Mn (K) es nilpotente, entonces la traza tr(N) de N (es decir, la suma de los elementos de la diagonal) es 0. Recprocamente, demostrar que si tr(N k ) = 0 para todo k 0, entonces N es nilpotente. 8. Encontrar las formas canonicas de Jordan complejas y reales de las siguientes matrices y encontrar la matriz invertible U tal que U ;1 AU adquiere cada una de las formas canonicas mencionadas. 0 0 ;1 2 1 00 0 11 0 0 4 21 0 1 ;3 3 1 (a) @ 3 ;4 6 A (b) @ 1 0 ;1 A (c) @ ;1 ;4 ;1 A (d) @ 0 ;5 6 A 2 ;2 3 0 1 1 0 0 2 0 3 4 C N 02 0 0 01 B 3 2 0 ;2 C (e) B @0 0 2 0C A 0 0 2 2 0 0 1 B ;2 1 (h) B @ 2 ;3 2 ;3 0 1 ;2 ;1 0 1 B 1 0 ;3 0 CC (f) B @ ;1 ;2 1 0 A 1 2 1 2 0 1 0 1 BB 1 1 ;2 1 C ;6 3 C (i) B BB ;;11 ;10 0 1A B@ 0 0 ;2 3 0 0 0 ;1 ;1 ;1 1 B 1 2 1 1 CC (g) B @ 0 0 0 ;1 A 0 0 1 2 1 0 1 0 0 1 1 0 0C C 1 ;1 0 0 C C 0 1 0 0C C 0 ;2 0 1 A 0 0 2 ;4 ;4 R RR 10.9. PROBLEMAS 251 9. Sean a 2 y f : 3 ! 3 el endomorsmo dado por f(x y z) = (ax x + ay b + az). Calcular la forma canonica de Jordan de f y la descomposicion aditiva de Jordan de f. 10. Estudiar para que valores de los parametros a y b la siguiente matriz es diagonalizable 05 0 01 @ 0 ;1 a A 3 0 b 11. *] Sea f un endomorsmo de V tal que f = f1  f2 , y para i = 1 2 sean Vi el dominio de fi y Pi = min(fi ). Supongamos que mcd(P1 P2) = 1. Demostrar: (a) min(f) = P1P2. (b) Si V1 = Kf1]v1 y V2 = Kf2 ]v2, entonces V = Kf](v1 + v2). (c) Si V = Kf]v, entonces V1 = Kf1 ]P2(f)(v) y V2 = Kf2 ]P1(f)(v). (d) Todo endomorsmo de un espacio vectorial sobre K tiene una representacion matricial del tipo 1 0 C(P1) B CC C(P2) (P1  : : :  Pn) = B CA B@ ... C(Pk ) con los Pi polinomios monicos (no necesariamente potencias de irreducibles) de KX] y Pi+1 j Pi para todo i < n. (e) Si C(P1 : : :  Pn) y C(Q1 : : :  Qm ) son dos representaciones matriciales de un endomorsmo satisfaciendo las condiciones anteriores, entonces n = m y Pi = Qi para todo i. Dichos polinomios se llaman factores invariantes de f. (Indicacion: Observar la similitud con las descomposiciones invariantes de grupos abelianos nitamente generados.) 12. Sea f un endomorsmo de un espacio vectorial sobre un cuerpo K. Encontrar los factores invariantes de f, si sus divisores elementales son: (a) X ; 1 X ; 1 X ; 2 X ; 3 (X ; 2)2  X 2 + 1 X 2 + 1 X 2 + 1 (X ; 1)2 , con K = . (b) X ; i X + i (X ; 2)2 (X ; i)2  X + i, con K = . (c) (X + 3)6 X + i X ; i X ; i, con K = . (d) (X ; a) (X ; a) (X ; a) (X ; b) (X ; b) (X ; c). 13. Sea f un endomorsmo de un espacio vectorial sobre un cuerpo K. Encontrar los divisores elementales de f si sus factores invariantes son: (a) X 6 ; X 4 ; 2X 2  X 5 ; X ; 2X X 4 ; X 2 ; 2, con K = . (b) (X + 3)2(X 2 + 1) X + 3 X + 3, con K = . (c) (X ; a)2(X ; b)2 (X ; a). 14. Sea f un endomorsmo de un espacio vectorial V . Sean v1 v2 2 V tales que V = Kf]v1  Kf]v2  Anuf (v1 ) = (X ; 1)2 y Anuf (v2 ) = (X 3 ; 1): Calcular los divisores elementales de f. 15. Sea f un endomorsmo de un espacio vectorial V sobre un cuerpo K. Calcular todas las posibles combinaciones de divisores elementales de f en los siguientes casos: (a) dimK V = 6, min(f) = (X 2 + 1)(X + 3)2 , K = o K = . (b) dimK V = 6, min(f) = (X ; 1)2 (X 2 + X + 1)2 , K = o K = . Q C C Q Q R RC C  CAPITULO 10. FORMAS CANONICAS DE MATRICES 252 RQ RC Q 16. Sea f un endomorsmo de un espacio vectorial V sobre un cuerpo K. Encontrar todas las posibles formas canonicas generalizadas de Jordan de f en los siguientes casos: (a) K = , dimR(V ) = 5 y min(f) = (X ; 1)2. (b) K = , min(f) = (X 2 ; 2)(X ; 1) y f = (X 2 ; 2)2 (X ; 1)3 . (c) K = , dimR(V ) = 6 y min(f) = (X 2 + 1)(X ; 1) (d) K = , dimC (V ) = 6 y min(f) = (X 2 + 1)(X ; 1) (e) K = , f = (X 4 ; 1)(X 2 ; 1). 17. Encontrar todas las posibles formas canonicas de Jordan para matrices complejas de orden 4. >Existen dos matrices no semejantes sobre de orden 4 que tengan el mismo polinomio mnimo y el mismo polinomio caracterstico? 18. Encontrar todas las formas de Jordan que tienen como polinomio caracterstico (X ; 1)6 X 3 y como polinomio mnimo (X ; 1)3 X 2 . Hacer lo mismo para el caso en que el polinomio caracterstico es (x + 2)7 (x ; 1)5(x + 3) y el polinomio mnimo es (x + 2)3(x ; 1)2(x + 3). 19. Demostrar que si un endomorsmo f de un espacio vectorial V de dimension nita sobre 3 satisface f 3 = 1, entonces f tiene una forma de Jordan sobre 3. Suponiendo que V tiene dimension 3, dar la lista de todas las posibles formas de Jordan para endomorsmos de este tipo. 20. Sea f un endomorsmo de un espacio vectorial real con factores invariantes (X ; 2)3 (X 2 + 1)2 y (X ; 2)2(X 2 +1). Calcular el polinomio caracterstico, el polinomio mnimo y las formas canonica de Jordan real y compleja de f. 21. Sean a, b y c tres numeros reales (no necesariamente distintos). Escribir todas las posibles formas canonicas de Jordan de un endomorsmo que tenga uno de los siguientes polinomios caractersticos: (a) (X ; a)3(X ; b)2. (b) (X ; a)5(X ; b)3. (c) (X ; a)(X ; b)2 (X ; c)2 . 0 2 ;1 0 1 22. Calcular la forma de Jordan de la matriz A = @ 1 0 0 A y una expresion para An . 0 0 1 23. Calcular A100 para las matrices del Problema 8. 24. Calcular el termino general de una sucesion de numeros en la que cada numero es la media aritmetica de los dos inmediatamente precedentes y los dos primeros numeros son 0 y 1=2. 25. Se ha establecido experimentalmente que el gusano cabezon vive dos das, y cada da el numero de gusanos que nacen coincide con la mitad del numero de gusanos de la poblacion. Establecer el estado estacionario de una poblacion que empezo con 1000 gusanos cabezones recien nacidos. 26. Encontrar la forma aditiva de Jordan de las matrices del Problema 8. 27. *] Escribir la lista completa de las posibles formas canonicas de Jordan para matrices 2  2 y 3  3 con coecientes en el cuerpo 2. >Cuantas clases de semejanza tienen M2 ( 2) y M3 ( 2)? >Cuantas de estas clases de semejanza contienen una matriz invertible? Sea G = GL3 ( 2) el grupo de las matrices invertibles de orden 3 sobre 2. (a) Calcular el cardinal del centralizador en G de cada una las formas canonicas invertibles. (b) Calcular el numero de elementos de cada clase de conjugacion de G. (c) Demostrar que GL3 ( 2) es un grupo simple de orden 168. (Indicacion: Usar el Problema 56 del Captulo 5.) C Z Z Z Bibliografa del captulo Puerta 27], Spindler 33]. Z Z Z ZZ Bibliografa 1] Allenby, R.B.J.T.: Rings, Fields and Groups. An Introduction to Abstract Algebra. Arnold, 1983.  2] Anzola, M. { Caruncho, J.R. { Perez{Canales, G.: Problemas de Algebra 1: Conjuntos y Grupos. Autores, 1982.  3] Anzola, M. { Caruncho, J.R. { Perez-Canales, G.: Problemas de Algebra 2: Anillos, Polinomios y Ecuaciones. Autores, 1982. 4] Artin, M.: Algebra. Prentice Hall, 1991.  5] Atiyah, M. { Macdonald, I.G.: Introduccion al Algebra Conmutativa. Reverte, 1980. 6] Baumslag, B. { Chandler, B.: Teora de Grupos. McGraw-Hill, 1972. 7] Bujalance, E. { Etayo, J.J. { Gamboa, J.M.: Teora Elemental de Grupos. UNED, 1989. 8] Childs, L.: A Concerte Introduction to Higer Algebra. Springer, 1979.  9] Clark, A.: Elementos de Algebra Abstracta. Alhambra, 1974. 10] Cohn, P.M.: Algebra (vol. 1). Wiley, 1989.  11] Delgado, F. { Fuertes, C. { Xambo, S.: Introduccion al Algebra I. Ed. Complutense, 1993.  12] Delgado, F. { Fuertes, C. { Xambo, S.: Introduccion al Algebra II. Anillos, Factorizacion y Teora de Cuerpos. Universidad de Valladolid, 1999. 13] Dorronsoro, J. { Hernandez, E.: Numeros, Grupos, Anillos. Addison-Wesley/UAM, 1996.  14] Espada, E.: Problemas resueltos de Algebra (tomo 1). Edunsa, 1978.  15] Espada, E.: Problemas resueltos de Algebra (tomo 2). Edunsa, 1983. 16] Fraleigh, J.B.: A First Course in Abstract Algebra. Addison-Wesley, 1982.  Version en castellano: Algebra Abstracta. Addison-Wesley Iberoamericana, 1982. 17] Gamboa, J.M. { Ruiz, J.M.: Anillos y Cuerpos Conmutativos. UNED, 1989. 18] Gardiner, C.F.: A First Course in Group Theory. Springer, 1980. 19] Hartley, B. { Hawkes, T.O.: Rings, Modules and Linear Algebra. Chapman & Hall, 1976. 20] Herstein, I.N.: Topics in Algebra. Wiley, 1975.  Version en castellano: Algebra Moderna. Trillas, 1979.  21] Hilton, P. { Wu, Y.C.: Curso de Algebra Moderna. Reverte, 1977. 22] Hungerford, T.W.: Algebra. Springer, 1987. 23] Jacobson, N.: Basic Algebra I. Freeman, 1974. 24] Lang, S.: Undergraduate Algebra. Springer, 1987. 253 254 25] 26] 27] 28] 29] 30] 31] 32] 33] 34] 35] 36] BIBLIOGRAFIA  Lang, S.: Algebra. Aguilar, 1977. Macdonald, I.D.: The Theory of Groups. Oxford University Press, 1968.  Puerta, F.: Algebra Lineal. Universidad Politecnica de Barcelona - Marcombo, 1981. Robinson, D.J.: A Course in the Theory of Groups. Springer, 1982. Rose, J.S.: A Course on Group Theory. Cambridge University Press, 1978. Rotman, J.J.: The Theory of Groups (an Introduction). Allyn & Bacon, 1973. Rowen, L.: Algebra (Groups, Rings and Fields). Peters, 1994. Sharpe, D.: Rings and Factorization. Cambridge University Press, 1987. Spindler, K.: Abstract Algebra with Applications I. Vector Spaces and Groups. Dekker, 1994. Spindler, K.: Abstract Algebra with Applications II. Rings and Fields. Dekker, 1994. Suzuki, M.: Group Theory I. Springer, 1982. van der Waerden, B.L.: Algebra I. Springer, 1991. Indice Terminologico de un grupo abeliano, 37 de un polinomio, 106 ideal {, 42 cerrado para una operacion, 40 ciclo, 150 clase de conjugacion, 139 de semejanza, 224 lateral, 128 doble, 145 cociente (de una division), 13, 93 coeciente de grado n, 39, 88 de una combinacion lineal, 14, 42 independiente, 39, 88 principal, 39, 89 combinacion lineal, 14, 42 compatibilidad con operaciones de una relacion de equivalencia, 60 de una relacion de orden, 10 complemento directo, 163 completar de modo unico un diagrama, 56, 90 composicion de polinomios, 107 conjugacion clase de {, 139 compleja, 48 por un elemento de un grupo, 139 conjugado complejo, 48 de un elemento de un grupo, 139 de un subgrupo, 139 conjunto nito, 31 cardinal de un {, 31 G-conjunto, 204 inductivo, 54 conmutador, 145, 206 conservar identidades, 47 productos, 47 sumas, 47 coprimos, 17, 69 criterios de irreducibilidad de Eisenstein, 102, 109 de reduccion, 102 accion el, 188 por conjugacion en elementos, 189 por conjugacion en subgrupos, 191 por la derecha, 188 por la izquierda, 188 por traslaciones (en clases laterales), 189 por traslaciones (en elementos), 189 transitiva, 190 algoritmo de Euclides, 18 de Kronecker, 99 anillo, 38 cociente, 43 de fracciones, 58 de los enteros de Gauss, 41 de matrices triangulares, 60 de polinomios en n indeterminadas, 104 de polinomios en una indeterminada, 39, 89 de series de potencias, 40 local, 62 localizado, 63 noetheriano, 61, 85 anillos homomorsmo de {, 47 isomorsmo de {, 49 isomorfos, 49 antihomomorsmo, 189 anulador (de un subconjunto de un anillo), 60 automorsmo de un anillo, 49 de un grupo, 125, 134 axioma, 9 base, 164 buena ordenacion, 10 cadena, 54 cancelable, 36, 52 caracterstica (de un anillo), 51 cardinal, 31 cateto, 81 centralizador, 125 centro de un grupo, 124 n-esimo, 218 cero 255 256 R para polinomios sobre , 100 para polinomios sobre cuerpos, 100 cuaterniones, 127 cuerpo, 52 algebraicamente cerrado, 95 algebraico (sobre otro cuerpo), 108 de cocientes, 56 de fracciones, 56 de funciones racionales, 56 delta de Kronecker, 165 derivada (de un polinomio), 94 n-esima, 94 descomposicion aditiva de Jordan, 249 indescomponible de un endomorsmo, 225 de un grupo abeliano, 174 invariante (de un grupo abeliano), 175 por subespacios invariantes, 225 primaria, 233 de un endomorsmo, 225 de un grupo abeliano, 174 descomposiciones semejantes (grupos abelianos) invariante, 177 primaria, 177 DFU (dominio de factorizacion unica), 75 dicotoma, 10 diferencia simetrica, 58 DIP (dominio de ideales principales), 70 divide, 13, 66 divisor, 13, 66 divisor de cero, 24, 53 divisores elementales de un endomorsmo, 233 de un grupo abeliano, 178 doble clase lateral, 145 dominio, 52 de factorizacion, 75 de factorizacion unica, 75 de ideales principales, 70 de integridad, 52 eucldeo, 71 ecuacion bicubica, 111 de orbitas, 190 de clases, 140 de congruencias, 25 de Pell, 67 diofantica, 19 general de grado n, 208 resoluble por radicales, 208 resolvente de una {, 111 elemento cambiado por una permutacion, 150 INDICE TERMINOLOGICO  cancelable, 36, 52 cero (en un grupo abeliano), 37 divisor de cero, 24, 53 jado por una permutacion, 150 idempotente, 59 identidad (en un anillo), 38 inverso, 24, 38, 114 invertible, 24, 38 irreducible, 68 mnimo, 10 neutro, 10, 36, 37 nilpotente, 62 opuesto, 10, 37, 114 primer {, 10 primo, 20, 68 regular, 24, 52 simetrico, 36, 37 singular, 24 uno (en un anillo), 38 elementos asociados, 66 congruentes modulo un entero, 22 congruentes modulo un ideal, 43 conjugados, 139 que conmutan, 115 coprimos, 17, 69 equivalentes para una accion, 189 primos entre s, 17 endomorsmo de un anillo, 47 de un espacio vectorial, 222 indescomponible, 225 de un grupo, 134 nilpotente, 248 endomorsmos semejantes, 224 simultaneamente diagonalizables, 233 escalar, 223 estabilizador, 190 extension de cuerpos, 55 algebraica, 108 formula de Cardano, 111 factores de composicion (de un grupo), 216 de una serie, 209 invariantes (de un grupo abeliano), 178 factorizacion en irreducibles, 75, 76 prima, 21, 76 irredundante, 21, 76 factorizaciones equivalentes, 75 forma canonica, 223 de Jordan, 240 generalizada, 239 real, 245 INDICE TERMINOLOGICO  primaria, 235 formula de interpolacion de Lagrange, 108 de Leibnitz, 107 de Taylor, 108 funcion eucldea, 71 de Euler, 27 polinomica, 92 G-conjunto, 204 geometra afn, 106 algebraica, 106 grado, 39, 89 grupo, 114 abeliano, 37 de torsion, 168 indescomponible, 171 libre, 164 libre de torsion, 168 alternado, 154 innito, 159 cclico, 126 cociente, 131 de automorsmos, 125 de automorsmos internos, 139 de Klein, 117 de longitud nita, 210 de los cuaterniones, 127 diedrico, 121 diedrico innito, 198 nitamente generado, 126 lineal especial, 124 lineal general, 123 multiplicativo, 38 nilpotente, 218 p-grupo, 172, 192 resoluble, 208 simetrico, 117 simple, 144, 155 trivial, 38 grupos homomorsmo de {, 116, 134 isomorsmo de {, 116, 134 isomorfos, 116, 134 hipotenusa, 81 homogeneizado, 110 homomorsmo, 47 de anillos, 47 de cuerpos, 55 de evaluacion, 91 de grupos, 116, 134 de reduccion de coecientes, 92 de sustitucion, 49, 91 257 trivial (de anillos), 48 trivial (de grupos), 134 homotecia, 224 ideal, 14, 42 cero, 42 generado, 14, 42, 45 impropio, 42 maximal, 53 primario, 62 primo, 53 principal, 14, 42 propio, 42 trivial, 42 idempotente, 59 identidad de Bezout, 69 de Dedekind, 143 en un anillo, 38 imagen (de un homomorsmo), 49, 134 impar (permutacion), 154 independiente (familia de subgrupos), 163 indescomponible endomorsmo, 225 grupo abeliano, 171 indeterminada, 39, 88, 104 ndice de un subgrupo, 129 inversion (presentada por una permutacion), 153 inverso, 24, 38, 114 invertible, 24, 38 irreducible, 68 isometra, 120 isomorsmo de anillos, 49 de grupos, 116, 134 lema de Bezout, 17 de Gauss, 98 de Zorn, 54 ley de composicion interna, 36 libre (grupo abeliano), 164 libre de cuadrados, 59 linealmente independiente, 164 lista de divisores elementales, 178 de factores invariantes, 178 longitud de composicion (de un grupo), 210 de una serie, 209 matrices semejantes, 224 matriz compa~nera (de un polinomio), 229 de Jordan, 239 generalizada, 238 INDICE TERMINOLOGICO  258 real, 245 de un endomorsmo (en una base), 222 diagonal, 60, 223 diagonalizable, 223 elemental de Jordan, 239 generalizada, 238 real, 245 escalar, 224 traza de una {, 250 triangular superior, 60 maximo comun divisor, 16, 69 metodo de Kronecker, 110 mnimo, 10 mnimo comun multiplo, 16, 69 monomio, 88, 104 multiplicidad de un primo en una factorizacion, 21 de una raz en un polinomio, 93 multiplo, 13, 66 Z neutro, 10, 36, 37 nilpotente, 62,p248 norma (en  m]), 66 normalizador, 191 notacion aditiva, 114 multiplicativa, 114 nucleo de un homomorsmo de anillos, 49 de un homomorsmo de grupos, 134 de una accion, 188 numero de Fermat, 33 de Mersenne, 33 entero, 10 libre de cuadrados, 59 negativo, 11 positivo, 11 primo, 20 natural, 11 representacion binaria, 32 representacion decimal, 32 representacion en base n, 32 operacion, 36 asociativa, 36 bien denida, 23 conmutativa, 36 inducida, 40 opuesto, 10, 37, 114 orbita, 189 orden de un elemento, 136 de un grupo, 129 parcial, 30 total, 30 p-grupo, 172, 192 p-subgrupo, 172, 193 de Sylow, 193 par (permutacion), 154 para casi todo, 45 periodo (de un grupo), 168 permutacion, 75, 117 impar, 154 par, 154 signo de una {, 154 permutaciones disjuntas, 150 polinomio, 39 anulador (de un vector), 229 caracterstico de un endomorsmo, 236 de una matriz, 236 ciclotomico, 103 completamente descomponible, 95 constante, 42, 88 cuadratico, 89 cubico, 89 en n indeterminadas, 104 en una indeterminada, 88 homogeneo, 106 homogeneizado, 110 lineal, 89 mnimo de un endomorsmo, 228 de una matriz, 228 monico, 89 primitivo, 99 raz de un {, 93 simetrico, 158 elemental, 158 presentacion por generadores y relaciones, 180 primitiva (terna pitagorica), 81 primo elemento de un anillo, 68 ideal, 53 numero entero, 20 primos entre s, 17 principio de induccion, 12 producto directo (de grupos), 116 semidirecto (de grupos), 197 propiedad antisimetrica, 10 asociativa, 10, 36 asociativa generalizada, 37 buena ordenacion, 10 conmutativa, 10, 36 conmutativa generalizada, 37 dicotoma, 10 distributiva, 10, 38 reexiva, 10 regularidad, 10 INDICE TERMINOLOGICO  transitiva, 10 propiedad universal de las bases, 167 de los anillos de polinomios en una indeterminada, 90 en varias indeterminadas, 105 del cuerpo de fracciones, 56 proyeccion canonica, 48, 134 en una coordenada, 48 PUAP, 90, 105 punto jo (para una accion), 190 radical (de un ideal), 62 raz de un polinomio, 93 multiple, 93 simple, 93 n-esima de la unidad, 103 primitiva, 59 renamiento (de una serie), 210 regla de la cadena, 107 de los signos, 10 de Ru)ni, 107 regular, 24, 52 regularidad, 10 resolvente de una ecuacion, 111 resto (de una division), 13, 93 semejantes descomposiciones (de un grupo abeliano) invariante, 177 primaria, 177 endomorsmos, 224 matrices, 224 serie central, 218 de composicion, 210 de potencias, 40 derivada, 208 factor de una {, 209 longitud de una {, 209 normal, 209 renamiento de una {, 210 termino de una {, 209 series equivalentes, 215 signo (de una permutacion), 154 simetrico elemento, 36, 37 grupo, 117 singular, 24 sistema generador, 126 subanillo, 40 generado, 45 impropio, 41 259 primo, 41 propio, 41 subconjunto multiplicativo, 57 subcuerpo, 55 primo, 57 subespacio invariante cclico, 228 por un endomorsmo, 225 por una matriz, 225 subgrupo, 123 caracterstico, 145 centro, 124 corazon de un {, 191 de p-torsion, 172 de torsion, 168 derivado, 206 n-esimo, 208 generado, 125 impropio, 124 ndice de un {, 129 maximal, 186 normal, 131 normal maximal, 144 normalizador de un {, 191 propio, 124 p-subgrupo, 193 p-subgrupo de Sylow, 193 sistema generador de un {, 126 trivial, 124 sucesion casi nula, 88 de Fibonacci, 31, 247 recurrente, 247 suma de ideales, 46 directa de endomorsmos, 225 de grupos, 124 de subgrupos de un grupo abeliano, 163 sumando directo, 163 sustitucion, 117 teorema chino de los restos, 26, 52, 144 recproco del {, 61 de Abel, 156 de acotacion de races, 93 de Bolzano, 97 de Cauchy, 185, 192 de Cayley, 135 de Cayley-Hamilton, 236 de clasicacion de los grupos cclicos, 138 de d'Alembert-Gauss, 96 de estructura de los grupos abelianos nitamente generados, 178 de Euclides, 22 INDICE TERMINOLOGICO  260 de Euler, 27, 137 de Fermat peque~no, 27 sobre irreducibles de i], 79 ultimo (exponente 4), 82 de isomorfa para anillos primero, 50 segundo, 51 tercero, 51 de isomorfa para grupos primero, 135 segundo, 135 tercero, 136 de Jordan-H#older, 215 de Kronecker, 95 de la correspondencia para anillos, 44 para grupos, 132 de la division con resto en , 13 de Lagrange, 129 recproco del {, 133, 155, 179, 192 de Pitagoras, 81 de Ru)ni, 93 de Sylow primero, 193 segundo, 194 tercero, 195 de Weierstrass, 97 de Wilson, 78, 107 recproco del {, 86 del resto, 93 enorme, 156 fundamental de la aritmetica, 21 del algebra, 95 termino (de una serie), 209 terna pitagorica, 81 primitiva, 81 tipo de un monomio, 104 de una permutacion, 151 transitiva accion, 190 propiedad, 10 trasposicion, 152 traza (de una matriz), 250 Z Z unidad, 38 uno (en un anillo), 38 valor absoluto, 12 valor propio, 223, 239 vector propio, 223 Smbolos usados frecuentemente x2X XY XY X Y X \Y X nY jX j NZ QR CN n ajb (S) mcd(a b) mcm(a b) a  b mod n ZZ n n A Mn (A) AX]] Q(D) AS ;1 M=N Q M  N i2I Mi N H P M + N i2I Mi M  N i2I Mi MN M =N Ker f Im f i]p  pm]  m] N(x) AX] AX1  : : : Xn ] gr(P ) D(P ) = P 0 P (n) o ZZ Q conjunto vaco e es un elemento del conjunto X el conjunto X esta contenido en el conjunto Y el conjunto X esta contenido estrictamente en el conjunto Y union de los conjuntos X e Y interseccion de los conjuntos X e Y diferencia de los conjuntos X e Y (elementos de X que no pertenecen a Y ) cardinal de un conjunto / orden de un grupo numeros enteros numeros naturales numeros racionales numeros reales numeros complejos conjunto f1 2 : : : ng a divide a b ideal generado por el conjunto S maximo comun divisor de a y b mnimo comun multiplo de a y b a y b son congruentes modulo n clases de restos modulo n unidades de n Z grupo de las unidades del anillo A anillo de matrices cuadradas de tama~no n  n sobre el anillo A anillo de series de potencias sobre el anillo A cuerpo de fracciones del dominio D anillo de fracciones del anillo A por el subconjunto multiplicativo S anillo cociente o grupo cociente producto cartesiano de conjuntos / producto directo de anillos o grupos producto semidirecto de los grupos N y H con accion suma de ideales de un anillo o de subgrupos de un grupo abeliano suma directa de subgrupos de un grupo abeliano producto de dos ideales, subgrupos o subconjunto de un anillo o un grupo M y N son anillos o grupos isomorfos nucleo del homomorsmo f imagen del homomorsmo f anillo de los enteros de Gauss p subanillo de formado por los elementos de la forma a + bpm, con a b 2 subanillo de formado por los elementos de la forma a + b m, con a b 2 norma del numero complejo x anillo de polinomios en una indeterminada sobre el anillo A anillo de polinomios en n indeterminadas sobre A grado del polinomio P derivada del polinomio P derivada n-esima del polinomio P CC 261 ZQ INDICE TERMINOLOGICO  262 S(X) Sn An Dn Q8 Isom( 2) Aut(G) Inn(G) GLn(K) SLn (K) H G H EG hS i hai Z(G) CenG (x) NG (H) aH Ha G=H H nG G : H] o(g) ax aG (i1 i2 : : : in ) t1 t2 : : : tr ] sg() grupo de las permutaciones del conjunto X grupo simetrico sobre n elementos grupo alternado sobre n elementos grupo diedrico de los giros y simetras del n-agono regular grupo de los cuaterniones grupo de las isometras del plano 2 grupo de automorsmos del grupo G grupo de automorsmos internos del grupo G grupo lineal sobre el cuerpo K grupo lineal especial sobre el cuerpo K H es un subgrupo del grupo G H es un subgrupo normal del grupo G subgrupo generado por el subconjunto S o por el elemento a centro del grupo G centralizador en el grupo G del elemento x normalizador de H en G clases laterales modulo un subgrupo H conjunto de las clases laterales por la derecha de G modulo H conjunto de las clases laterales por la izquierda de G modulo H ndice de un subgrupo H en el grupo G orden de un elemento de un grupo conjugado de a por x clase de conjugacion de a en el grupo G ciclo en un grupo simetrico tipo de una permutacion signo de la permutacion sigma r(L) t(A) tp (A) p(A) ha1  : : : an j r1  : : : rm i G o G E() = EstabG () np a b] G0 G(t) `(G) rango del grupo abeliano libre L subgrupo de torsion del grupo abeliano A subgrupo de p-torsion del grupo abeliano A periodo del grupo abeliano A presentacion de un grupo abeliano por generadores y relaciones orbita de  en G estabilizador de  en G numero de p-subgrupos de Sylow en un grupo conmutador de a y b subgrupo derivado del grupo G t-esimo subgrupo derivado del grupo G longitud de composicion del grupo G E = EndK (V ) anillo de endomorsmos del K-espacio vectorial V matriz del endomorsmo f en la base B matriz del cambio de base de B a B 0 suma directa de endomorsmos endomorsmo consistente en multiplicar vectores columna por la matriz A producto de polinomio por vector subespacio invariante cclico generado por v polinomio mnimo del endomorsmo f o de la matriz A polinomio caracterstico del endomorsmo f o de la matriz A anulador del vector v por el endomorsmo f vectores de V anulados por potencias del polinomio P matriz compa~nera del polinomio P matriz elemental de Jordan generalizada asociada al polinomio P matriz elemental de Jordan de valor propio a matriz elemental real de Jordan de valor propio  + i R fB CB B f1   fn A P v Kf]v min(f) min(A) f  A Anuf (v) tP (V ) C(P) Jn (P) Jn (a) Jn ( ) 0 R